You are on page 1of 460

Prem K.

Kythe
Dongming Wei

An Introduction
to Linear and Nonlinear
Finite Element Analysis
A Computational Approach

With 152 illustrations

Springer Science+Business Media, LLC


Prem K. Kythe Dongming Wei
Department of Mathematics Department of Mathematics
University of New Orleans University of New Orleans
New Orleans, LA 70148-0001 New Orleans, LA 70148-0001
U.S.A. U.S.A.

Library of Congress Cataloging-in-Publication Data

Kythe, Prem K.
An introduction to linear and nonlinear finite element analysis : a computational approach
Iby Prem K. Kythe, Dongming Wei.
p. cm.
Inc\udes bibliographical references and index.
ISBN 978-1-4612-6466-8 ISBN 978-0-8176-8160-9 (eBook)
DOI 10.1007/978-0-8176-8160-9
1. Structural analysis (Engineering) 2. Finite element method. 1. Wei, Dongming, 1958-
II. Title

TA646.K98 2003
620'.0042-<ic22 2003057713
CIP

AMS Subject Classifications: 65N30,65-01

Printed on acid-free paper.


©2004 Springer Science+Business Media New York

Originally published by Birkhauser Boston in 2004
Softcover reprint of the hardcover 1st edition 2004
AII rights reserved. This work may not be translated or copied in whole or in part without the written
permission of the publisher (Springer Science+Business Media, LLC),
except for brief excerpts in connection with reviews or scholarly analysis. Use in connection with
any form of information storage and retrieval, electronic adaptation, computer software, or by similar
or dissimilar methodology now known or hereafter developed is forbidden.

The use of general descriptive names, trade names, trademarks, etc., in this publication, even ifthe
former are not especially identified, is not to be taken as a sign that such names, as understood by the
Trade Marks and Merchandise Marks Act, may accordingly be used freely by anyone.

ISBN 978-1-4612-6466-8 SPIN 10888337

Typeset by TechType Works, Inc., Gretna, LA.


Mathematica® is a registered Trademark ofWolfram Research, Inc.
MA TLA~ is a registered trademark of MathWorks, Inc.

9 8 7 6 543 2 1
Contents

Preface ix
Notation xvii

1 Introduction 1
1.1 Historical Sketch 1
1.2 Euler-Lagrange Equations 2
1.2.1 Functionals 2
1.2.2 Boundary Conditions 3
1.3 Weak Variational Form 3
1.4 Galerkin Method 9
1.5 Rayleigh-Ritz Method 11
1.6 Exercises 18

2 One-Dimensional Shape Functions 25


2.1 Local and Global Linear Shape Functions 25
2.2 Local and Global Quadratic Shape Functions 28
2.3 Parametric Coordinates 31
2.4 Hermite Shape Functions 31
2.5 Exercises 34

3 One-Dimensional Second-Order Equation 37


3.1 Galerkin Finite Element Method 37
3.1.1 Linear Elements for Second-Order Equation " 38
3.1.2 Local and Global Matrices 42
vi CONTENTS

3.1.3 Quadratic Elements for Second-Order Linear Problems 45


3.1.4 Mixing Two Different Elements 47
3.2 Two Dependent Variables 63
3.5 Exercises 66

4 One-Dimensional Fourth-Order Equation 75


4.1 Euler-Bernoulli Beam Equation 75
4.1.1 Finite Element Equation 75
4.1.2 Boundary Conditions 77
4.2 Exercises 86

5 Two-Dimensional Elements 89
5.1 Linear Three-Node Triangular Elements 89
5.2 Bilinear Four-Node Rectangular Elements 91
5.3 Global Shape Functions 94
5.4 Triangular Coordinates . . . . . . . . . . . . . . . . . . . . . . . . . . . . . . . . . . . . . . . . . 96
5.5 Shape Functions on the Sides of a Triangle 99
5.5.1 Linear Shape Functions on the Sides of a Triangle 99
5.5.2 Quadratic Shape Functions on the Sides of a Triangle 101
5.6 Exercises 105

6 Two-Dimensional Problems 107


6.1 Single Dependent Variable Problems 107
6.1.1 Local Weak Formulation 108
6.1.2 Finite Element Equation 109
6.1.3 Evaluation of Stiffness Matrix and Load Vector . . . . . . . . . . . . . 110
6.1.4 Evaluation of Boundary Integrals. . . . . . . . . . . . . . . . . . . . . . . . .. III
6.1.5 Assembly of Element Matrices 112
6.2 Exercises . . . . . . . . . . . . . . . . . . . . . 129

7 More Two-Dimensional Problems 137


7.1 Heat Transfer . . . . . . . . . . . . . . . . . . . . . . . . . . . . . . . . . . . . . . . . . . . 137
7.2 Torsion 144
7.3 Seepage 146
7.4 Fluid Flows 152
7.5 Exercises . .... . . . . . . . . . . . . ..... .. .... . . . . . . .. . ... . . ... . .. .. .. 164

8 Axisymmetric Heat Transfer 175


8.1 Radial Symmetry 175
8.2 Linear Elements 182
8.3 Linear Elements for Heat Transfer in Fluids 194
8.4 Nonlinear Heat Transfer 196
8.4.1. Gauss-Legendre Integration Method 198
8.5 Exercises 202
CONTENTS vii

9 Transient Problems 207


9.1 Classical Methods 207
9.2 One-Dimensional Transient Problems 212
9.3 Time-Dependent Heat Conduction 217
9.3.1 Derivation of Finite Element Equations 218
9.3.2 Numerical Time Integration 221
9.4 Two-Dimensional Transient Problems 228
9.5 Exercises 233

10 Single Nonlinear One-Dimensional Problems 241


10.1 Newton 's method 241
10.2 Radiation Heat Transfer 243
10.3 Stress Analysis of Plastic Rods 252
10.4 Power-Law Pressure Driven Flow between Two Plates 256
10.5 Mixing-Length Equation for Turbulent Flow in Pipes 259
10.6 Rayleigh-Ritz and Nonlinear Gradient Methods 261
10.7 Exercises 266

11 Plane Elasticity 269


11.1 Stress-Strain Relations 269
11.2 Constant-Strain Triangular Element 271
11.3 Virtual Displacement Finite Element Model . . . . . . . . . 272
11.4 Weak Form Finite Element Model 273
11.5 Stiffness Matrix and Load Vector 275
11.5.1 Linear CST Element 275
11.5.2 Assembly of CST Elements 276
11.5.3 Bilinear Rectangular Element 284
11.5.4 Two or More Bilinear Rectangular Elements 288
11.6 Exercises 291

12 Stokes Equations and Penalty Method 297


12.1 Equality-Constrained Programs and Lagrange Multipliers 297
12.2 Penalty Formulation for Linear Stokes Equation 299
12.3 Penalty Linear Triangular Stokes Element 301
12.4 Penalty Bilinear Rectangular Stokes Element 309
12.5 Penalty Linear Triangular Power-law Stokes Element 311
12.6 Solutions by Conjugate Gradient Methods 313
12.7 Exercises 321

13 Vibration Analysis 323


13.1 Hamiltonian Principle 323
13.2 Free Axial Vibrations of an Elastic Rod " 327
13.3 Free Vibrations of a Euler Elastic Beam 330
13.4 Free In-Plane Vibrations of an Elastic Plate 334
viii CONTENTS

13.5 Axial Vibrations of a Plastic Rod 342


13.6 Eigenvalue Problems 344
13.6.1 Hyperbolic Equations 344
13.6.2 Parabolic Equations 346
13.6.3 Helmholtz Equation 347
13.7 Exercises 347

14 Computer Codes 351


14.1 Mathematica Codes 351
14.2 Ansys Codes 363
14.3 Matlab Codes 368
14.4 Fortran Codes 374

A Integration Formulas 393

B Special Cases 397

C Temporal Approximations 405


C.l First-Order Derivative . . . . . . . 405
C.2 Second-Order Derivative 406

D Isoparametric Elements 409


D.l Introduction 409
D.Ll Local ~ -Syatem 409
D.l .2 Triangular Elements 411
D.1.3 Rectangular Elements 412
D.2 Curvilinear Coordinates 414
D.2.1 One-Dimensional Integrals 416
D.2.2 Two-Dimensional Integrals 417
D.3 Pascal Triangle 418

E Green's Identities 421

F Gaussian Quadrature 423

G Gradient-Based Methods 429


G.l Method of Steepest Descent 429
G.2 Conjugate Gradient Method 431

Bibliography 433

Index 439
Preface

This textbook on linear and nonlinear finite element methods has evolved out of
teaching of a sequence of two courses at the University of New Orleans for the past
12 years to a variety of students from the engineering, physics, geophysics, and
mathematics departments. The first of these courses is an introduction designed for
undergraduate and beginning graduate students, whereas the second is a follow-
up mainly for graduate students. In the past we used a number of textbooks and
selected material at different times on a trial basis, and we developed our own
notes and a repertoire of representative engineering and technological problems
that cater to the disciplines of our students.
As everyone knows the market is replete with textbooks on finite element
analysis. Then why do we need another textbook on this subject? To answer this
question, we next describe how we have used existing books and why we felt a
need to write our own textbook .
(i) Although there are many excellent books on the market, very few are de-
signed for students from different disciplines and use widely available modern
software such as Mathematica, Matlab, Fortran , and commercial finite element
codes such as Ansys.
(ii) In our view, the finite element technology oftoday has grown into a basic tool
for almost every field in engineering and applied sciences. Finite element courses
have become more and more popular in both undergraduate and graduate curricula
in colleges of engineering and sciences throughout the world. The new generation
of undergraduate and graduate students is more accustomed to using computers
for everyday homework assignments and research than ever before. There are
a few introductory finite element textbooks that provide a good combination of
x PREFACE

methodology, applications, computer implementation, and hands-on projects.


(iii) Of the books we have used, some incorporate commercial codes into
the text. A few books introduce applications of the finite element method in
electromagnetics together with other traditional applications in solid and fluid
mechanics. There are a few other books that deal exclusively with the finite
element analysis of electromegnetic problems. Such books are aimed at special
readers in specialized fields of interest and are not suitable for a general curriculum.
(iv) We have found only one specialized book on finite element analysis dealing
with non-Newtonian fluid flows, but it is designed for chemical engineers. Other
books do not consider this topic at all. Even in books with traditional applications,
only a few give details about vibrational finite element analysis, although there are
other books that deal exclusively with this topic.
(v) We believe that the basic theory of finite element analysis should be made
accessible to all undergraduate students in engineering and sciences with a min-
imum background in mathematics, computer science, physics, and any specific
engineering field. Our experience with the books we have used is that only a few
are designed to serve a broad range of engineering and science majors and to ex-
pose students to hands-on application projects using the computational software
tools that are now widely available on campuses.
(vi) We have found that in our introductory course it is very helpful to devote
sufficient time to cover equations in one space dimension and include problems
with applications in the following areas: structural, thermal , Newtonian and non-
Newtonian fluids, mechan ical vibration , electromagnetics, variational principles,
and thermal elastic coupled systems . The underlying mathematics for the first
course are kept at a minimum by using only basic techniques, such as the Gauss -
Jordan elimination method and the backward substitution method for solving linear
systems of algebraic equation, integration by parts formula s in one or two dimen-
sions , chain rule for differentiation, and change of variable formula for integration
and Newton 's method for solving nonlinear systems of algebraic equations.
(vii) In the second course we introduce advanced techniques for solving discrete
algebraic systems, viz., line search , method of steepest descent, linear and nonlinear
conjugate gradient methods, Newton's methods and penalty method.
During the past decade we have accumulated valuable experience and absorbed
many excellent ideas from other authors in teaching these two service courses. The
result is a general service applied finite element textbook that not only combines
all the above ideas and experience but is diverse enough in applications and simple
enough in theory for students in all engineering and science fields. We have kept the
mathematical theory to a minimum, just sufficient for understanding the beginner's
whys and hows of finite element methods.
PREFACE xi

Salient Features

This textbook presents basic theoretical material and practical applications for
solving problems of interest in engineering and technology. The book covers a
two-semester course and is designed for junior/senior undergraduate and first-year
graduate students from engineering, geophysics, physics, and applied mathematics
who have had a course in calculus of several variables, differential equations as
well as linear algebra and have some knowledge of computers.
This book
• Fulfills the need for a service course taught to students with diverse background.
• Provides thorough instructions and training to these diverse students, unlike
most textbooks that are designed with specific applications, e.g., for structural
engineers , mechanical engineers, electrical engineers, chemical engineers, geo-
physicists, or applied mathematicians.
• Involves students in the use of widely available modern software such as Mathe-
matica, Matlab, and the use of commercial finite element codes such as Ansys .
• Deals with problems of varied types, including linear and nonlinear problems
in radiation heat transfer, mechanics of elastic and plastic media, continuum
mechanics , non-Newtonian fluid flows, and electromagnetics.
• Provides accessibility to all students with diverse backgrounds with a minimum
of mathematical analysis .
• Serves a broad range of engineering and science students and exposes them to
applications from different disciplines , unlike books that deal with only one
specific fields.
• In the first-semester course, presents one-dimensional equations and their appli-
cations in the following areas: structural analysis, heat conduction, fluid flows,
mechanical vibration , electromagnetics, variational principles, and thermoe-
lastic coupled systems .
• In the second-semester course , presents advanced techniques for solving dis-
crete algebraic systems , viz., line search, method of steepest descent, conjugate
gradient methods, Newton's methods , and penalty method.
• Offers supporting and other interesting material via the Internet at the authors'
web sites .
• Provides service, accessibility, and applications toward the development of the
subject.
xii PREFACE

Overview

A general description of the topics covered in the book is as follows. After a brief
historical sketch of the development of the finite element method and its subsequent
development, Chapter 1 introduces the weak variational formulation of a boundary
value problem. Galerkin and Rayleigh-Ritz weighted residual methods. This in-
troduction lays the foundation of the Galerkin finite element method developed in
subsequent chapters. Chapter 2 is devoted to the development of one-dimensional
global and local linear and quadratic Legendre interpolation shape functions and
the Hermite interpolation shape functions that are used with fourth-order differen-
tial equations in the beam problems. The Galerkin method for a one-dimensional
second-order equation, also known as the potential or the bar equation, is first
developed in Chapter 3, using linear and quadratic elements. Problems are solved
by mixing these two types of elements, and a one-dimensional problem with two
dependent variables is solved in this chapter. Problems with a one-dimensional
fourth-order equation, also known as the beam equation, are covered in Chap-
ter 4. Two-dimensional problems start from Chapter 5, which is devoted to the
development of linear three-node triangular and bilinear four-node rectangular cle-
ments. Triangular coordinate system is introduced, and linear and quadratic shape
functions on the sides of a triangle are developed in this chapter.
Chapter 6 deals with two-dimensional problems involving a single space vari-
able; the finite element equation is developed, stiffness matr ix and the load vector
are evaluated, element matrices are assembled, and boundary integrals are eval-
uated for different types of boundary value problems taken from industrial and
technological areas . More two-dimensional boundary value problems, mainly
from heat transfer, torsion, seepage, and fluid flows, are presented in Chapter 7.
Axisymmetric linear and nonlinear heat transfer problems in solids and fluids
are presented in Chapter 8. Transient problem and numerical time integration
methods are presented in Chapter 9. Single nonlinear one-dimensional equations
and the related finite element method are discussed in Chapter 10. Plane elasticity
problems are introduced in Chapter 11; principle of virtual work, linear triangu-
lar elements and bilinear rectangular elements are introduced, the concept of two
degrees of freedom at each node is explained, and simple problems mostly involv-
ing two elements are presented. Chapter 12 deals with the Stokes equation and
the penalty method; linear triangular and bilinear rectangular Stokes elements are
derived, and solution by the conjugate gradient method is explained in detail. The
corresponding nonlinear Stokes elements for power-law non-Newtonian flows are
also introdu ced together with the penalty method and some nonlinear conjugate
gradient methods. Chapter 13 deals with the vibration analysis ; the Hamiltonian
principle is introduced, and the free axial vibrations of an elastic rod , free vibra-
PREFACE xiii

tions of an Euler elastic rod, free in-plane vibrations of an elastic plate, and axial
vibrations of a plastic rod are discussed. The chapter ends with solutions of eigen-
value problems that arise from hyperbolic and parabolic equations and that lead to
the Helmholtz equation. New codes in Mathematica, Ansys, Matlab, and Fortran
are given in Chapter 14.
There are six appendices, which contain useful supplemental information. Ap-
pendix A contains integration formulas that are used in the development of the
subject. Appendix B contains special cases of linear triangular and bilinear rectan-
gular stiffness matrix and load vector that arise in certa in special situations. For-
mulas for numerical solution of time-dependent problems are given in Appendix
C. Information about some basic isoparametric elements, together with curvilinear
coordinates and the Pascal triangle, is collected in Appendix D. Green 's identities
are available in Appendix E, Gaussian quadrature is presented in Appendix F, and
gradient-based methods, viz., the method of steepest descent and the linear and
nonlinear conjugate gradient methods, are discussed in Appendix G.
As mentioned earlier, this book contains both linear and nonlinear finite element
methods . The nonlinear development of two-dimensional heat transfer problems
and the related Gauss-Legendre integration method are presented in Chapter 8.
The finite element method for solving single nonlinear one-dimensional equations
is developed in Chapter 10. It introduces the Newton's method and discusses
problems involving radiational heat transfer, stress analysis of plastic rods, power-
law pressure driven flows, mixing-length equation for turbulent flows in pipes, and
the Rayleigh-Ritz and nonlinear gradient methods . Since the subject of nonlinear
finite element methods is advanced, we have taken care to include topics that can
be easily understood, especially by the students for whom this book is primarily
written.
The book is a bit heavy on heat transfer problems. It has been our experi-
ence that such problems are easy for instructors to explain and the students to
comprehend their elegance and simplicity.
There are 87 examples and 148 exercises in the book ; hints , answers, and
solutions for most of the exercises are available in the book. A liberal use of
illustrations is made to clarify the nature of the problems; as a result there are 152
figures and illustrations in the book.
The bibliography at the end of the book contains literature that has been con-
sulted as well as references that are cited . The book ends with a detailed subject
index.
This is an introductory textbook which provides a good combination of method-
ology, applications, computer implementation, and hands-on projects for students
with diverse background from engineering, technology, physics, geophysics, and
applied mathematics.
xiv PREFACE

To Instructors

As mentioned above, the chapters present balanced two-semester course material,


which can be tailored to the needs of different levels of instruction. Although the
following table outlines some suggested curricula at the two levels, the instructor is
free to adapt whatever chapters or sections in any order depending on the limitations
of the course.

First Semester Second Semester


Chapter 1 Chapter 8
Chapter 2 Chapter 9
Chapter 3 Chapter 10
Chapter 4 Chapter 11
Chapter 5 Chapter 12
Chapter 6 Chapter 13
Chapter?

Chapter 14 is to be used in both courses, as it pertains to the topics and sections


needed in each course.

Computational Aspects

Computer codes using Mathematica, Ansys, Matlab, and Fortran are presented in
Chapter 14. They cover linear as well as nonlinear problems, which are discussed
in detail. Our students have freely used and modified some well-known Fortran
programs, for example, by Taylor and Hughes (1981), Reddy (1984, 1993) and
Segerlind (1984), and certain other codes available in private domain. They have
generated these modifications for certain specific problems and used them to obtain
computational solutions. We have encouraged our students to study these and other
authors in order to use and modify their codes. It has helped them understand the
subject much better than using a single textbook.
We have provided some new codes in Mathematica, Ansys, Matlab, and For-
tran, as they arise during the course of the presentation of the textual material.
Mathematica 3.0 or higher is used to verify most of the solutions. For various
PREFACE xv

other useful items and suggestions, check either author's website :

www.math.uno.edu/fac/pkythe.html www.math .uno.edu/fac/dwei.html

Acknowledgments

The authors thank all the students who have participated in our finite element
courses since 1990. Although the list is very large, we will take liberty of mention-
ing names of certain students who, by their hard work and perseverance, excelled
in the study of this subject, benefited not only themselves but other students,
and kept the course running with full steam throughout these years . These dedi-
cated students are Antonis Antoniou, Pravan Kumar Bachu, Xiang Bian, Rambabu
Chennupati, Gordon C. Cuculla III, Shawn Davis, Andrea M. Dunham, Lang
Yan Fook , Karunakar R. Gaddam, Patrick Louis Garrity, Praveen Ghantasala,
Monikonda Shekar Goud, Wael M. Khairy, Gloria Ladnier, Haibiao Luo , Larry
Manalo, William A. Myers, Yashawant Mummaneni, Chakvartula 1. S. V. Nallani,
Vuong Hun Nguyen, Tien Szu Pan, Madahv K. Parimi, Srinivasa Rao Pasala, Srid-
har Rao Pasupuleti, Yezdi Parvez Pavri, Khoa Pham, Suresh K. Sivapuram, Atluri
Srinivas, Edupuganti Suneel, Jifeng Ru , Annette R. Schulte, Reddy P. Shashikanth,
Michael Sheaf, Ved P. Tandle, Nuthan K. Vajinapalli, Chandra Rao Valipe, Satya-
narayana Rao Valipe, Prakash Vyakaran, Bonnie Williams, Qin Xu, Zhidong Xu,
Xuan Yu, Keshu Zhang, and Peng Zang.
The authors thank Dr. Vivek Sahai of Cheng Fluid Systems, Mountain View,
CA , for some valuable advice. They thank Dr. Carroll F. Blakemore for help and
support, and some other colleagues for useful suggestions. The contribution of
the Senior Production Editor and the copy editor at Birkhauser Boston for the fine
work of editing the manus cript, and of the people at TechType Works , Inc. , Gretna,
Louisiana, for typesetting the manuscript is sincerely acknowledged.

New Orleans, Louisiana Prem K. Kythe


July 2003 Dongming Wei
Notation

Standard notation is used throughout the book. Since matrices and vectors occupy
most of the text, the usual convention is adopted. All matrices are printed in
uppercase boldface and all vectors in lowercase boldface letters , except U , F , Q,
and T, which represent the displacement vector, load (force) vector, flux vector,
temperature vector, and the error vector at the global nodal points, respectively,
and F (e) and R (e), which denote the load vector and the error vector for an element
n (e) , respectively. This practice does not lend to any confusion in the conte xt of
their use. Also, sometimes to save space on a page, a column vector is printed as
the transpose of its corresponding row vector.
A list of the notation , abbreviations and acronyms used in this book is given
below.

a elastic modulus in R"


ai , bi, c, coefficients in a linear triangular shape functions
a row vector for ai
A area; surface area of a fin; cross-sectional area
A (e) area of a two-dimensional clement
As total lateral surface area (of a fin)
AT, (Af transpo se of a matrix A
b(w, u) bilinear form
b, coefficients in a linear triangular shape functions
b row vector for bi
B matrix
B(e) matrix of the values of bi , c,
const constant
coefficients in a linear triangular element
xviii NOTATION

specific heat at constant pressure


Cv specific heat at constant volume
row vector for Ci
c.,
C
material (elastic) constants
C connectivity matrix ; matrix of the elastic constants Ci j
dof degree(s) of freedom
D dimension of the geometry in fluid flow
D matrix of first partial derivative of shape functions
e.g. for example
e finite element number e = 1, .. . ,NE
erfc complementary error function
E modulus of elasticity, Young's modulus
Ek elastic kinetic energy
Ep elastic potential energy
El flexural rigidity
Eq(s) equation(s) (when followed by an equation number)
f prescribed function; heat source; recharge (aquifer)
fee) value of f on an element nee)
fo uniform load
j axial body force
fx,fy body forces
Fo ,FL shear forces at x = 0 and x = L, repectively
P Froude number
FE finite element
f vector
F force or load vector (F = f + Q)
F (e) force or load vector of an element nee)
g prescribed function; shear modulus (torsion); gravity
g vector; body forces
h thickness of a fin or a plate
h vector
H(x,xo) Heaviside unit function (= 0 if x < xo, and = 1 if x > xo)
H square matrix
unit vector in the x direction
i.e. that is
iff if and only if
I moment of inert ia
l(u) functional ; total energy of an elastic mechanical system
1m modified Bessel function of the first kind and order m
Imn integral defined in Example 5.2
j unit vector in the y direction
J current
J Jacobian matrix
k thermal conductivity; permeability coefficient (aquifer)
NOTATION xix

kx ,ky thermal conductivity in the x and y direction


k unit vector in the z direction
k(e) value of k on an element e
K constant value of a metal property; consistency coefficient
K(e) stiffness matrix of an element n(e)
K global stiffness matrix
Kb matrix
l(w) linear functional
l( e) length of the interval [xie) , x~e)J
i.i« : length of nonuniformone-dimensional consecutive elements
L length of an interval; length unit; linear operator
i: Lagrange function; Laplace transform
c:' inverseof Laplace transform
L matrix
M bending moment; polar moment of a cross-sectional area
Mo, ML bending moment at x = a and x = L, respectively
M matrix
M(e) matrix (radially symmetric element)
n power-law index
n outward normal vector
direction cosines of n
a a a
n·\? = nx ax + n y oy + n z oz
NE number of elements
NL number of local nodes on an element
P
. I au
pressure; penmeter; a so = U x = ax
psi lbs/irr'
P vertical point load; vector
Pn(X) Legendre polynomials of degree n
Pi search direction
q heat source; also = uy =
au
oy
qn heat flux
q rate of heat generation
q temperature gradient
Ql shear force
Q2 bending moment
Q~ e) shear force at node i of an element n( e)
Q(e) vector of secondary degrees of freedom (boundary terms)
Q part of F corresponding to the natural boundary conditions
Qb vector
r radial distance
r(u) scalar residual (error) in the Galerkin method
(r,e ,z) cylindrical polar coordinates
xx NOTAT I ON

R radius
Rn Euclidean n-space
R+ set of positive real numbers
Rj errors (j = 1, . . . , n + 1)
Re Reynolds number
~ real part
R(e) residual or error vector
R global error vector
8 variable of the Laplace transform; arc length ; second(s) (time)
(8, t) nondimensional coordinates on the unit square
sym symmetric (matrix)
s(e) square matrix
t time
ix ,iy prescribed secondary variables
T temperature, temperature distr ibution
t; base temperature of a fin
Too amb ient temperature
T temperature vector
T( e) temperature vector for an element n( e)
U dependent variable; stress function; displacement; mean velocity
Uo prescribed velocit y
Uoo free stream velocity
(e)
Ui value of U at node i
(e )
Ua linear interpolation function for the interval [x ~e) , x~e) ]
U velocity vector, displacement vector
U vector of the first time derivatives of u
U vector of the second time derivatives of u
u( e) approximation of u on an element n (e)
u ,v velocity components of u or v , in x and y direction
ii , V approximate solution for u or v
Uo inlet velocity
Ue nodal value of U at a global node e, e = 1, . . . , NE
u (e) (e) (e) (e) (e)]
coIumn vector [u 1 ... uN vI .. . V N
U vector of global values of displacement U
Vr radial component of velocity v
V three-dimensional solid or volume
av boundary surface of a solid V
v velocity vector
W test function
Wi weights in Gaussian quadrature
Wl ,W2 ,W3 three different test functions
W work
global coordinate
end points of a line element
NOTATION xxi

local coordinate ( = x - x~e» )


initial guess (Newton 's method)
set of nonnegative integers
a thermal capacitance
fJ film coefficient, convective heat transfer coefficient
fJPY Dai-Yuan parameter

,fJr
s Hestenes-Stiefel parameter
specific heat ration (= cp / c,J
r boundary of a domain (= a n)
f 1,f z disjoint portions of boundary I' (f 1 U f z = an)
b tolerance
bi j Kronecker delta (= 1 if i = j ; -1 if i =1= j)
D.t time step
emissivity
strain ; penalty parameter
strain vector
shear strain
e strain ( = - ~~); angle of twist (torsion)
{) nondimensional temperature
= lu~e) -u~~ll , i = 1, 2, .. . , N
von Karman constant
eigenvalue
dynamic viscosity of a fluid
kinematic viscosity of a fluid (= J.L / p); Poisson's ratio (elasti city)
~ isoparametric one-dimensional coordinate, -1 < :::
~ 1
~, 'TJ isoparametric two-dimensional coordinates, -1 ::::: ~ , 'TJ ::::: 1
~i Gaussian points
6 ,6,6 trilinear coordinates
p density
a stress; Stefan-Boltzmann constant
aD uniform load
a stress vector
T shear stress
Txy , Tyz average shear stresses
¢ velocity potential of a flow
¢~ e) basis function ; interpolation shape function
<I>e global shape functions for e = 1, . . . , N + 1
lj>(e) vector of the shape functions for an element n (e)
X (e) characteristic function for the interval [x~e), x~e) ] or n (e)
'IjJ stream function
w radian frequency
n domain
n(e) general finite element
xxii NOTATION

In(e)1 area or volume of an element n(e)


an boundary of the domain n
an(e) boundary of an element n(e)
o null vector
o matrix of differential operators
I-D one-dimensional
2-D two-dimensional
3-D three-dimensional
.8 .8 k 8
\l gra d =1-+J-+ -
8x 8y 8z
I x] norm of a vector x
II\lull norm of gradient vector (= Jui + u~ + u~ )
\If gradient matrix of f
. (8P. Bp, P
\lp pressure gra dlent 8x 1 + 8yJ + 88z k)
\l·F divergence of a vector F (div F)
2 2
\l2 a2 a a )
Laplacian ( = 8x 2 + 8 y 2 + 8z 2
AlB set A minus set B
apprimately equal to
empty set
An Introduction
to Linear and Nonlinear
Finite Element Analysis
1
Introduction

In this chapter, after a brief historical sketch about the development of the finite
element methods, we discuss the weak variational formulation and present the
Galerkin and Rayleigh-Ritz methods, which belong to the class of weighted residual
methods . Some useful integration formulas are given in Appendix A, and Green's
identities are presented in Appendix E.

1.1. Historical Sketch

The pioneers in the development of the finite element method are Courant (1943),
Prager and Synge (1947), Schoenberg (1948), P6lya (1952, 1954), Hersch (1955),
and Weinberger (1956). Courant's work on the torsion problem is considered a
classic; it defined piecewise linear polynomials over a triangulated region . Prager
and Synge found approximate solutions for plane elasticity problems based on the
concept of function space. Schoenberg also developed the theory of splines and
used piecewise polynomials (interpolation functions) for approximation. Schoen-
berg also developed formulas for analytical approximation. P6lya, Hersch, and
Weinberger used a technique similar to that of Courant and the finite difference
methods to solve eigenvalue problems . Synge (1952) used a piecewise linear func-
tion defined on a triangulated region and the Ritz variational method to solve plane
problems . Greenstadt (1959) used a discretization technique to divide the doma in
into "cells," assigned a different function to each cell, and applied the variational
principle . White (1962) solved a plane thermoelasticity problem, and Friedrichs
(1962) developed finite difference schemes for the Dirichlet and Neumann prob-
lems. Both of them used triangular elements and the variational principle. In

P. K. Kythe et al., An Introduction to Linear and Nonlinear Finite Element Analysis


© Springer Science+Business Media New York 2004
2 1. INTRODUCTION

mathematical physics, Synge (1957) developed the hypercircle method in which


he provided a geometric interpretation for the minimum principle in plane elasticity.
A three -dimensional electrostatic problem was for the first time solved by
McMahon (1953) by using tetrahedral elements and linear interpolation functions .
The name "finite element method" was first used by Clough (1960). It was shown
by Melosh (1963), Jones (1964) , and de Veubeke (1964) that the finite element
method can be regarded as the Ritz variational method using piecewise interpolation
functions. The work of Zienkiewicw and Cheung (1965) extended the scope of the
finite element method to all types of problems that could be expressed in variational
form. Thus , the mid-1960s marked a transition from the early research into the
modern development of the subject. From mid-1960s through 1980 this method
developed from the earlier field of structural analysis into various other fields, with
more mathematical analysis and different computational methods and codes. The
development of mathematical theories provided a rigorous and firm foundation for
the finite element methods, their Galerkin or Ritz-based variational techniques,
questions of convergence, and error analysis. During this period Whiteman (1975)
published a bibliography for finite elements, and Clough (1980) published an
account of the development of this subject during the past 25 years.

1.2. Euler-Lagrange Equations

Let H" denote the n-dimensional Euclidean space, and Z+ the set of nonnega-
tive integers. A brief definition of functionals and the classification of boundary
conditions is given below before we discuss the weak variational formulation of
boundary value problems.
1.2.1. Functionals. A functional is an expression of the form

I(u) = fin F (x ,y,u,ux ,Uy ) dxdy+ i2 G(x ,y,u)ds, n E R2 , (1.1)

where F (x, y, u , u x, u y ) and G(x , y, u) are known functions, and f 2 is a part or


all of the boundary an of the domain n. Similar functionals can be defined in
R" with one or several variables . Although the value I (u) depends on u, yet for
a given u, the value of I(u) is a scalar quantity. The functional I(u) represents a
function defined by integrals whose arguments are themselves functions. In fact,
this functional is an operator I which maps u into a scalar value I(u) . Its domain
is the set of all functions u(x), whereas its range, which is a subset of the real field,
is the set of images of all functions u under the map I . Frequently the functional
I (u) represents the total potential energy of a mechanical system and a stationary
point of I is sought to satisfy the equation
d
dT I(u + TV) = 0 (1.2)
1.3. WEAK VARIATIONAL FORM 3

for all real numbers T and all v such that u + TV is admissible for I. The set of
admissible functions is defined to be those functions on n that give finite values of
ul r
I(u) and satisfy the condition r1 = fL, where 1 is the boundary ofn minus z . r
As a result of the calculations of the values ofEq (1.2), we obtain the corresponding
Euler-Lagrange equation

of
OU -
0 (OF) 0 (OF)
ox op - oy oq = 0 inn, (1.3)

subject to the boundary conditions

(1.4a)

(lAb)

where F = F(x, y, u, p, q), p = u x , q = u y , and n x , n y are the direction cosines


of the unit vector n normal to the boundary on := I' = I' 1ur z of a two-dimensional
region n such that r 1 n r z = 0. For details, see Axelsson and Barker (1984) .
1.2.2. Boundary Conditions. A partial differential equation is subject
to certain conditions in the form of initial or boundary conditions. The initial
conditions, also known as Cauchy conditions, are the values of the unknown
function u and an appropriate number of its derivatives at the initial point.
The homogeneous boundary conditions fall into the following three categories:
(i) Dirichlet boundary conditions (also known as boundary conditions ofthe first
kind, or essential boundary conditions), when values of the unknown function
u are prescribed at each point of the boundary r 1 of a given domain n.
(ii) Neumann boundary conditions (also known as boundary conditions of the
second kind), when the values of the normal derivatives of the unknown function
u are prescribed at each point of the boundary r z.
(iii) Robin boundary conditions (also known as boundary conditions of the third
kind, or mixed boundary conditions), when the values of a linear combination
of the unknown function u and its normal derivative are prescribed at each
point of the boundary r z.
The last two conditions are also known as the natural boundary conditions.

1.3. Weak Variational Form

The weak variation formulation of boundary value problems is derived from the
fact that variational methods for finding approximate solutions of boundary value
problems, viz., Galerkin, Rayleigh-Ritz, collocation, or other weighted residual
4 1. INTRODUCTION

methods, are based on the weak variational statements of the boundary value
problems.
For example, a special case of (1.3) is when F is defined as

8u ) 2 2 ( 8u ) 2]
F ="21 [ k1 ( 8x + k 8y - I u.

This equation arises in heat conduction problems in a two-dimensional region with


k1 , k2 as thermal conductivities in the x , y directions, and I being the heat source
(or sink) . Here

8F _ k 8u 8F
8p - 18x' 8u = -I,

and Eq (1.3) becomes

-:x (k ~~) :y (k ~~)


1 - 2 = I In n. (1.5)

If k 1 = k2 = 1, then we get the Poisson's equation - 'V2 u = I with appropriate


boundary conditions.
The weak variational formulation for the boundary value problem (1.3)-(1.4)
is obtained by the following three steps .
STEP 1. Multiply Eq (1.3) by a test function wand integrate the product over
the region n:

jJr [8F8u _ ~8x (8F)


e
0. 8p
_ ~ (8F)] w dxdy
8y Bq
= O. (1.6)

The test function w is arbitrary, but it must satisfy the homogeneous essential
boundary conditions (1.4a) on u.
STEP 2 . Use formula (A.8) componentwise to the second and third terms in
(1.6) for transferring the differentiation from the dependent variable u to the test
function w, and identify the type of the boundary conditions admissible by the
variational form:

j"rJn [8F r (8F 8F)


rt: + 8x 8p + 8y 8q dxdy- Jan x>: x> wds=O.
8w8F 8W8F]

(1.7)
Note that formula (A.8) does not apply to the first term in the integrand in (1.6) .
This step also yields boundary terms that determine the nature of the essential and
natural boundary conditions for the problem . The general rule to identify the
essential and natural boundary conditions for (1.3) is as follows . The essential
boundary condition is prescribed on the dependent variable (u in this case), i.e.,

u= u on f 1
1.3. WEAK VARIATIONAL FORM 5

is the essential boundary condition for (1.3). The test function w in the boundary
integral (1.7) satisfies the homogeneous form of the same boundary condition as
that prescribed on u. The natural boundary condition arises by specifying the
coefficients of wand its derivatives in the boundary integral in (1.7). Thus,
8F 8F 8G
8p n x + 8q n y + 8u = 0 on I' 2

is the natural boundary condition in a Neumann boundary value problem. In one-


dimensional problems, use integration by parts instead of formula (A.8) .
To equalize the continuity requirements on u and w, the differentiation in
formula (A.8) is transferred from F to w. It imparts weaker continuity requirements
on the solution u in the variational problem than in the original equation.
STEP 3 . Simplify the boundary terms by using the prescribed boundary con-
ditions. This affects the boundary integral in (1.7), which is split into two terms,
one on I' I and the other on I' 2 :

Jinr
r [8F
w 8u
8w 8F 8w 8F] J (8F 8F
+ ax 8p + ay aq dx dy- ! f'tUf' 2 ap n x + auyn y w ds =
)
O.
(1.8)
The integral on f l vanishes since w = 0 on fl . The natural boundary condition
is substituted in the integral on f 2 . Then (1.8) reduces to

r [aF r ec
J in
r aw 8F 8w 8F]
w au + 8x 8p + ay 8q dx dy + if'2W 8u ds = O. (1.9)

This is the weak variational form for the problem (1.3). We can write Eq (1.9) in
terms of the bilinear and linear differential forms as
b(w, u) = l(w) , (1.10)
where
b(w,u) =Jrrin [8W8F
8x 8p
+ 8W8F] dxdy,
8y 8q
(1.11)
l(w) = _Jrr w dxdy- r w
8F 8G
ds.
in 8u i r 8u 2

Formula (1.1 0) defines the weak variational form for Eq (1.3) subject to the bound-
ary conditions (1.4) . The quadratic functional associated with this variational form
is given by
1
I(u) = "2b(u , u) -l(u). (1.12)

EXAMPLE 1.1 . Derive the Euler-Lagrange equation and the natural boundary
condition for the functional

l
b
I(u) = a "21 [p(x)(u) 2 + q(x) u 2 - r(x) u] dx + "21 ap(a) [u(a)]2 , u EU,
I

(1.13)
6 1. INTRODUCTION

where U = {u E C 2 [a, b], u(b) = B} . We write the functional (1. 13) as

I (u) = l b
F (x ,u, u x ) dx + C(a,u(a)) . (1.14)

Let

g(T) = l b
F (x, u + TTJ ,Ux + TTJx) dx + C(a , u(a) + TTJ(a)) .

Now, since

~ =
dg(T) d
dT
I a
b
F (x , u + TTJ ,Ux + TTJx) dx
d
+ dTC (a, u(a) + TTJ(a))
b[
=l of d(U+TTJ )+ of d(UX+T TJx )] dx
a o(u + TTJ) dr o(u x + TTJx) dr
+ ---,----,-O_C_.,.......,...., d(U(a) + TTJ (a))
o(u(a) + TTJ(a)) dr
b
=
I a
[ OF 1] +
o(u + TTJ )
OF
o(u x + TTJx)
TJx] dx + OC
ou(a) + TTJ (a)
TJ (a),
(1.15)
we find that

dg(O )
~ =
I a
b
[OF OF] OC
au 1] + oUx 1]x dx + ou(a) TJ(a) = O.

Thus, after integration by parts we get

or

I o f I TJ(b) -~
of I TJ(a)+~
b
[OF
~ - !al ( ~
OF)] TJdx+~ oc( ) TJ (a) = O.
a uU ox uux uux x=b uux x=a uU a
(1.16)
Now, for U+TTJ to be inU, we have u(b) = B, U(b)+TTJ(b) = B, i.e., B+TTJ(b) =
B ; thus, TTJ (b ) = 0, or TJ (b ) = O. Then

dg(O)
- -=
dr
I
a
b
[OF
- - -a ( -OF)] n dx
au ax OU x
> -
of I TJ (a) + -
oUx x=a
ec- TJ (a) = o.
o(u(a)
(1.17)
For all TJ satisfying TJ(a) = TJ(b) = 0, the equation

I
b
[OF
au
_~( OF)]
ax oU
TJ dx=O
a x
1.3. WEAK VARIATIONAL FORM 7

implies that

-8F - -8 (8F)
- - -_ 0 in (a,b),
8u 8x 8u x
if it is continuous in (a, b) . Also,

- -88FI 8G
U x x=a1](a) + -U8
( ) 1](a) =
a
0 for all1](a)

implies that
8G
:~ Ix=a 8u(a)'
Then the Euler-Lagrange equation is

~~ - ~ (:~) = 0 in (a,b). (1.18)

Further, since

F(x ,u,ux ) = ~ [p( x) (ux)2 +q(x)u 2 - r (x ) u],


1 2
G(a,u(a)) = "2o: p(a) [u(a)] ,

8F 8F 8G
and 8u = q(x) u - r(x), 8u = p(x) ux , and 8u(a) = o:p(a) u(a), the Euler-
x
Lagrange equation (1.1 8) reduces to

d [p(x) dx
q(x) u - r(x) - dx dU] = 0, (1.19)

subject to the natural condition

8F I
-8 = p(x) Ux I = -8G
( ) = o: p(a)u(a).
8
U x x=a x =a U a

Finally, since U x Ix=a - 0: u(a) = 0, the Euler-Lagrange equation (1.19) becomes

d [p(x) dx
q(x) u - r(x) - dx dU] = 0,

subject to the natural condition

dul
-d = o:u(a).•
X x =a

In the next example we derive the bilinear and linear forms for a system of partial
differential equations in two variables with prescribed boundary conditions.
8 1. INTRODUCTION

EXAMPLE 1.2 . Consider the system of Navier-Stokes equations for a two-


dimensional flow of a viscous, incompressible fluid (pressure-velocity fields):

U ou + vou = _~ op + v (02 u2 + 02 U )
oX oy Pox ox oy2 '
OV ov lop (02 v 02 v)
U ox + voy = - poy + u ox2 + oy2 '
OU ov
ox + oy = 0,
in a region n, with boundary conditions u = Uo, v = Vo on f l , and
OU Ou)
v ( ox n x + oy n y - p1 pn x = t,« .
ov ov)
v ( ox n x + oy n y - p1 pn y = t,y ,
on I' 2, where (U ,v) denotes the velocity field, p the pressure, and ix ,i y the pre-
scribed values ofthe secondary variables. Let WI, W2, W 3 be the test functions, one
for each equation, such that WI and W2 satisfy the essential boundary conditions
on U and v, respectively, and W3 does not satisfy any essential condition. Then
1.4. GALERKIN METHOD 9

Note that the boundary integral in the linear form l (WI , W2, W3) has no term con-
taining W3 . •

1.4. Galerkin Method

We discuss two frequently used methods for obtaining approximate numerical solu-
tions of boundary value problems. They are Galerkin and Rayleigh-Ritz methods.
These methods give the same results for homogeneous boundary value problems.
Consider the boundary value problem

Lu=f inn, (1.20)

subject to the boundary conditions

u=g on I'i . (1.21 )

au
an + ku = h on f 2, (1.22)

where I' = I'j U I' 2 is the boundary of the region n . Let us choose an approximate
solution u of the form
N
U= :L Ui <Pi- (1.23)
i =l

An approximate solution does not, in general, satisfy the system (1.20)-(1 .22).
The residual (error) associated with an approximate solution is defined by

r(u) = Lu - f = L (tUi cPi) - f. (1.24)


1=1

Note that if Uo is an exact solution of (1.20)-( 1.22) , then r( uo) = O. The Galerkin
method requires that the residual be orthogonal with respect to the basis functions
cPi (also called the test functions) used in (1.23), i.e.,

fin r (u) cPi dx dy = 0 for 1, .. . ,N. (1.25)

Hence,

fin [L(u) - f] cPi dx dy = 0, i = 1, . . . , N , (1.26a)

or
10 1. INTRODUCTION

which in the matrix form is written as

KU=f, (1.26b)

where

Ii = Jl f¢i dxdy . (1.26c)

In the examples given below, we choose different values of N in (1.6) for the
trial function ii: There is some guidance from geometry for such choices . These
choices also satisfy the essential conditions and exhibit the nature of approximate
solutions. However, the larger the N is, the better the approximation becomes .
EXAMPLE 1.3 . Consider the Poisson's equation

2 2
_\72 U=_(88xU+88 yU) =C, O<x<a,
2 2
O<y<b,

such that U = 0 at x = 0, a and y = 0, b. First, we choose the first-order


approximate solution as

U~I) = axy(x - a)(y - b).

Note that this choice satisfies all four Dirichlet boundary conditions. The Galerkin
equation (1.26a) gives

ii b a
[ - 2a(y2 - by + x 2 - ax) - c] xy(x - a)(y - b) dx dy = 0

which simplifies to

Thus,
5c

Hence,
(1) 5c
UI = (2 2) xy(x - a)(y - b). •
2 a +b
EXAMPLE 1.4. Use the Galerkin method to solve the eigenvalue problem
\72u + AU = 0 in the polar coordinates for 0 < r < a.
1.5. RAYLEIG H-RI T Z M ET HOD 11

1-
We solve - d (rdU)
- + AU = 0, 0 < r < a. Take cPj(r ) = cos -j7rr . For
r ~ ~ ~

the first approximauon, '1'1 = cos -n r , and UI


• ' ,1..
= 0:1 cos -nr , W h'IChId
ea s to
2a 2a

l
a
27r d [r7r
{I--d - ( - sin -7rr)] O:I+ AO:ICOS -itr } rdr=O.
o r r 2a 2a 2a

This gives the equation for the eigenvalue Aas

Hence,
7r 2(7r2 + 4) 5.8304
Al = 4a2(7r2 _ 4) ~ ~ .

The exact value


. \
IS Al =
5.779
-2- '
For the second-or der approxim
. ation
. U2
a
ttr 37rr . . 5.792
0:1 cos 2a + 0:2 cos 2a .which gives A2 = ~. •

1.5. Rayleigh-Ritz Method

As a simple case, consider the Poisson's equation - V'2u = f, with the homoge-
neous boundary conditions u = 0 on f 1 and EJu / EJn = 0 on f 2 . Then , the weak
variational form leads to

(1.27)

where IV' u 1 = u ; + u~ . A generalization of the result in (1.27) for the case of


2

the system Lu = f with the above homogeneous boundary conditions, where L is


a linear self-adjoint and positive definite operator, leads to the functional

I( u ) = ~ JL {uLu - 2fu} dx dy. (1.28)

THEOREM 1.1. If the operator L is self-adj oin t and posit ive definit e,
then the unique solut ion of Lu = f with homogeneous boundary conditions
occurs at a mini mum value of I(u) .
An application of Theorem 1.1 is the Rayleigh -Ritz method, where we find the
direct solution of the variational problem for the system Lu = f by constructing
12 1. INTROD UCTION

minimizing sequences and securing the approximate solutions by a limiting process


based on such sequences. Thus, we choose a complete set of linearly independent
basis (test) functions (Pi, i = 1, . . " and then approximate the exact solution Uo by
taking the approximate solution ii in the form
n

ii = 2::>i4>i' (1.29)
i= 1

where the constants Ui are chosen such that the functional J(ii) is minimi zed at
each stage. If ii --+ Uo as n --+ 00, then the method yields a convergent solution.
At each stage the method reduces the problem to that of solving a set of linear
algebraic equations. The detail s for the boundary value problem - 'V2 u = I with
homogeneous boundary conditions arc as follows. Using (1.29) in the functional
(1.28) we get

fin {(::r + (:~r l


J(ii ) = - 2ii } dXdY

= fin {(L Ui~~ir + (L Ui~~ir - 2I L Ui 4>i }dXdY.

Thu s,

Hence,
8J
~ = 2Kii Ui + 2 ~ K ij Uj - 2Ji, (1.30)
ou,, ~
i i-j

J1 (84)i + 84>i
and
K '..J = J::)
84>j
J::) J::)
84>j) dx dY,
J::)
(1.31 )
!1 uX uX UY UY

Ii = fin 4>d dxdy. (1.32)

Now, if we choose Ui such that J(Ui) is a minimum (i.e., 8J/ 8Ui = 0), then from
(1.30) we get
n
L KijUi = Ii, i = 1, ' " ,n, (1.33)
j=1
1.5. RAYLEIGH-RITZ METHOD 13

which in the matrix notation is


Ku=f, (1.34)

where the matrix K has elements K ij given by (1.31), the vector f has elements
I. given by (1.32), and the vector u = [U1' . .. ,un]T. Note that (1.34) is a system
of linear algebraic equations to be solved for the unknown parameter Ui , and K is
nonsingular if L is positive definite.
The Rayleigh-Ritz method is alternatively developed by solving the equation
(1.10) for u, where we require that w satisfy the homogeneous essential conditions
only. Then this problem is equivalent to minimizing the functional (1.12) . In other
words , we will find an approximate solution of (1.10) in the form

n
Un = L UjrPj + rPo, (1.35 )
j=l

where the functions rPj, j = 1, .. . ,n, satisfy the homogeneous boundary condi-
tions while the function rPo satisfies the nonhomogeneous boundary condition , and
the coefficients Uj are chosen such that Eq (1.10) is true for w = rPi, i = 1, . . . ,n,
i.e., b(rPi' un) = l(rPi), or for i = 1"" ,n,

Thus,
n
LUjb(rPi,rPj) = l(rPi) - b(rPi, rPO) , i = 1, , " ,no (1.36)
j=l
This is a system of n linear algebraic equations in n unknowns Uj and has a uniqu e
solution if the coefficient matrix in (1.36) is nonsingular and thus has an inverse.
The functions rPi must satisfy the following requirements: (i) rPi be well defined
such that b(rPi, rPj ) =I- 0, (ii) rPi satisfy at least the essential homogeneous boundary
condition, (iii) the set {rPi}i=l be linearly independent, and (iv) the set {rPd i=l
be complete. The term rPo in the representation (1.35) is dropped if all boundary
conditions are homogeneous.
EXAMPLE 1.5 . Consider the Bessel's equation

X
2
U" + xu' + (x 2 - l)u = 0, u(l) = 1, u(2) = 2.

Set U = v + x. Then the given equation and the boundary conditions become

211
X V + xv / + (x 2 - l)v + x 3 = 0, v(l) = °= v (2).
14 1. INTRODUCTION

In the self-adjoint form this equation is written as

x 2 -1
xv" + v' + - - v + x 2 = O.
X

For the first approximation, we take

VI = al¢1 = al(x -1)(x - 2).

Then using (1.25) we get f12(LvI - J)¢I dx = 0, which gives

JIr [2a Ix -
2 2
x
1)(x - 2)al + x 2] (x - 1)(x - 2) dx= 0,
1
(3 - 2x)al + ~(x -

which, on integration, yields al = -0.811, and thus,

UI = VI +X = -0.811(x - 1)(x - 2) + x.

Theexactsolutionisu = CI J I(X)+C2 YI(X), where c, = 3.60756, C2 = 0.75229.


A comparison with the exact solution in the following table shows that UI is a good
approximation.

Table 1.1.

x UI Uexact

1.3 1.4703 1.4706


1.5 1.7027 1.7026
1.8 1.9297 1.9294.

EXAMPLE 1.6. Consider the fourth-order equation

[(x + 2l)u"]" + bu - kx = 0, 0 < x < l,


with the boundary conditions: u(l) = 0 = u'(l), (x + 2l)u"(0) = 0, [(x +
2l)u"]' (0) = O. We choose the test functions

¢I(X) = (x _l)2(x 2 + 2lx + 3l2) ,


(h(x) = (x -l)3(3x 2 + 4lx + 3z2).

For the first approximation, we have UI = al¢1 (x) . Then f~ (LUI - J)¢I (x) dx =
0, which gives
1.5. RAYLEIGH-RIT Z METH OD 15

If, for example, we take I = 1= band k = 3, then a1 = 0.0119174, and thus ,

U1 = 0.0119174 (x - 1)2(x 2 + 2x + 3).


For the second approx imation, we take U2 = a1<P1(x) + a2<P2(x) . Then

and 1(I
LU2 - f) <P2(X) dx = 0,

which , with I = b= 1, k = 3, yield

3776/45
[ -21172/315
- 21172/ 315 ]
63674/693
{aa2 1 }
=
{ 1 }
-5/7 .

Thus , at = 0.013743, a2 = 0.002279, and


U2 = 0.013743(x - 1)2(x 2 + 2x + 3) + 0.002279(x - 1)3(3x 2 + 4x + 3).
Instead of determining the exact solution , we can compare U1 and U2 . Thus , e.g.,
U1(0.5) = 0.012662, and u2(0.5) = 0.012964, which give good results.•
EXAMPLE 1.7 . Use the Rayleigh-Ritz method to derive the matrix equation
of the type (1.33) for the Poisson's equat ion - 'V2 u = f on a two-dimensional
region D, subject to the nonhomogeneous boundary conditions U = U1 on r 1 , and
~~ + k(s) u = h(s) on r 2 , where r 1 U r 2 = r is the boundar y of the region D.
The functional I (u) is given by

We choose a linearly independent set of basis function s <Pi that satisfy the Dirichlet
boundary condition <Pi = 0 on r 1 . In view of (1.35), the solution u is assumed to
be of the form

n n+1
U = <Po + LUi <Pi = LUi <Pi, with Un+1 = 1 and <Pn+1 = <Po.
i =l i= l

Then
16 1. INTRODUCTION

which yields

where K i j are given by (1.31), Ii by (1.32), and

Let K +M = A , and f +g = h . Then ~I = 0 implies that Au = h , where


UUi

A is an n x (n + 1) matrix, and u = [UI, . . . ,Un+l] T . Thus, the system of the


Rayleigh-Ritz equations is given by

All UI + A I 2 U2 + + A l n Un + AI,n+1 Un+1 = hI ,

A 2 I UI + A 22 U2 + + A 2n Un + A 2,n+l Un+l = liz ,

or

All UI + A I 2 U2 + + A l n Un = hI - AI ,n+l ,

A 2l UI + A 22 U2 + + A 2n Un = h2 - A 2 ,n+l ,

AnI UI + A n 2 U2 + ... + Ann Un = hn - An,n+l,

which , in matrix form , can be written as

A'u=h',

where A~j = A i j, and h~ = hi - Ai ,n+l for i, j = 1, . . . ,n.•


EXAMPLE 1.8. Consider the Stokes flow of an ideal fluid in a channel of
width b with zero vertical velocity component (v = 0). Thus , U = u(y) is the
velocity component in the y direction (see Fig. 1.1 on page 18).
The equation governing the Stokes flow is

(1.37)
1.5. RAYLEIGH-RITZ METHOD 17

with the boundary conditions u(O) = 0 = u(b). Since ~~ = 0, Eq (1.37) implies


2
d u . ap .
that f-L d IS constant. Hence, ax = const, say, fo. Then, this problem has the
y2
exact solution
u(y) = - fo (by _ y2) .
2f-L
The exact solution suggests that we consider an approximate solution of the form
U = 0: sin 7r: with ¢(y) = sin 7rt Then, using the Galerkin method (§1.4),
d2 u 7r 2 7ry
the residual r (u) = f-L d - fo -f-L 0: /;2 sin b - f o. Hence, solving

l
y2
b
r (u) ¢(u ) dy = 0, we find that

which gives
4fob 2
0: -
- ---
f-L7r3 '
Note that the Rayleigh-Ritz method also gives the same result (see §1.5). The

l
detail s are as follows . The weak form of this problem with w as the test function
b
is w [f-L ~:~ - fo] dy = 0, which gives the bilinear and linear forms

b(u,w) =- i
r f-L dwdy dudy dy,
o
b
l(w) = l b
fo w dy,

and the quadratic functional J(u) is given by (see (1.12»

J(u) = "21 [b (u, u) -l(u) ] = -"21 io r [f-L (dU)


dy 2
+ 2fo u] dy. (1.38)

We take the above approximate solution u ~ 0: sin 7r: . Then from (1.38) we get

J(u) = -"21 r [f-L7r2


b
Jo
( 2 7 r Y) 7rY]
2b2 0: 1 + cos -b- + 2fo 0: sin b dy
2

7r2 2
= _ ~ [f-L 0: + 2fob 0:]
2 4b 7r'
which , when minimized, leads to

aJ 7r2
1 [f-L 0: + 2f ob] = O.
ao: 2 2b 7r
18 1. INTRODUCTION

4fob 2
Thus, we obtain 0: = - - -3- ' We take fo = 1 = /1 = 1, and b = 1. Then
/17r
0: = -4/7r3 . Since the flow is symmetric about the line x = 0.5, the values ofthe
exact and approximate solutions of u are compared in Table 1.2.

Table 1.2. u
0
0.1 0.2 0.3 0.4 0.5
Y u ex act uapprox - 0.02 ", ,
-0. 04 "-, Exact Solution: Dashed Line
0.0 0.0 0.0 ,
- 0.06 -,
,
0.1 -0.045 -0.039865 -,
-,
- 0.08
0.2 -0.08 -0.075827
- 0.1
0.3 - 0.105 -0.104368
- 0.12
0.4 -0.12 -0.122692 - Y
0.5 -0 .125 -0.129006. Fig. 1.1. Stokes Flow.

Note that the Rayleigh-Ritz method can be similarly applied to the Poisson's
and other more general equations.

1.6. Exercises
Derive the weak variational forms for the boundary value problem s 1.1-1.7. Note
that a , b, f, 9 are functions of z, and uo, ho, mo , qo, Too , Uoo are constants:

1.1. -
d ( a dx
dx dU) - f = 0, u(O) = uo, a du
dx (l) = qo, 0 < x < l,

ANS .
r
(one-dimensional heat conduction).
dw du
b(w,u) = io a dx dx dx,

l(w) = I' wf dx - w(O) [a~u] + qo w(l).


io x x=o

1.2. - d~ (a~~) - cu + x 2 = 0, u(O ) = 0, a~~ (1) = 1, 0 < x < 1


(one-dimensional deformation of a bar).

ANS . b(w,u) =1 (a~:~~


1
-cwu) dx , l(w) =-1 1
2
wx dx +w (I) .

1.3. -~ ( Cll au + C12 au) _ ~ ( C21 au + C22 au ) + f = 0 in n with


ax ay ax ay axay
.. u =
boundary conditions Uo on r 1, and qn == ( Cll ax
au + C12 au
ay ) nx +

( C21 ~~ + C22 ~~) n y = qo , on f 2 , where Cij , Uo, and qo are prescribed .


1.6. EXERCISES 19

ANS . b(w,u) = Jk [~: (Cll ~~ +C12~~ ) + ~: (C21 ~~ +C22~~)


+w f ] dxdy, l(w) = fan wqn ds,
where a« = (Cll ~~ + C12~~ ) n x + (C21 ~~ + C22~~ ) n y .

1.4. - k ( ~:~ + ~:~) =f in the region n with bound ary conditions as shown
in Fig. 1.2. The following bound ary conditions are prescribed: kTx = qo(Y) on
H A ; kTx = -(3 (T - Too) on B C ; T = To(x) on A B, and aT/an = qo = 0
on C DE F GH (insulated), where k is the thermal conductivity of the mater ial
of the regi on n, (3 and Too are ambient quantities, and aT/an = -aT/ax =
- T x on H A (two-dimensional heat conduction).

A NS. 0 = - fL k ( ~~ + ~:~) w dx dy,


=
J'rI n
k
( dW dT
dx dx
dw dT )
+ dy
(dT
dy dxdy- Jan kw dx n x
r dT )
+ dy n y ds.
The bound ary conditions on C 1 = A B (prescribed temperature To): n x = 0,
n y = -1 ; on C 2 = BC (convective boundary, Too): n x = 1, n y = 0; on
C 3 = C DE FG H (insulated boundary): q = aT/an = 0; and on C 4 = H A
(prescribed conduction qo(Y) : n x = - 1, n y = O. Thu s,

'r r b
b(w,T) =
J Ink (dW dT dw dT)
dx dx + dy dy dx dy + (3 J w(a, y)T (a, y ) dy ,
o
l(w) = -l b
w(O , y)qo(Y) dy + er; l b
w(a, y) dy.

E
insulated d
qo=O
H i-------'G D '---''------, C

n b

Fig. 1.2.

1.5. - -d {a[ -dU + -1 (dV)2


- ]} + 9 = 0,
dx dx 2 dx
2 2
d (d V) d { dv [dU 1 (dV)2]}
dx2 bdx2 - dx a dx dx +"2 dx + f = 0;
20 1. INTRODUCTION

2V]
u = V = dVI
0 at x = 0, l; -d = 0, [d
b -2 = mo (large-deflection
x x =O dX x =1
bending of a beam).
ANS . Let WI and W2 be the two test functions, one for each equation, such
that they satisfy the essential boundary conditions on u and v. Then

rl
0= io a dx
[ dWI {dU (dV) 2
dx + 2" dx
}+ WIg]dx ,
1

0--1 1

o
[bd2W2
dx dx
2v
- -2-d-2+ adW2-dv
- {dU
dx dx dx
- +-1 (dV)2}
-
2 dx
+ W2 f] dx

dW2
- mo dx (l) .

Thus,
2
[ dWI {dU 1 (dv)2} d2W2 d v
b((Wl' W2) , (u, v )) = i(o a dx a dx + 2" dx + b dx 2 dx 2

+adW2 dV{dU + !(dV)2}] dx ,


dx dx dx 2 dx

l((Wl ' W2)) = -1 1


(WIg + w2f ) dx + mo d::X (I) ,
2

[1{
1
2
I[(u,v)] = 1
- a (dU)
- +b ( -d v2 ) 2 +a-
2 du ( -dV ) 2 a (dV) 4 }
+--
o 2 dx dx dx dx 4 dx
dW2
+ WIg + w2f ] dx - mo dx (l) .

1.6. Con sider the Poisson's boundary value problem: - \72u = fin n, with the
boundary conditions U = 0 on 0 1 and au/an = 0 on 02 . Show that

A NS. b(w, u) = fl (~: ~~ + ~; ~~) dxdy ,

l(w) = fl ~~
fwdxdy - [ W ds.

1.7. Consider

o < x < L, EI > 0, f = const,

where EI is called the flexural rigidity of the beam, subject to the boundary
conditions

dx (El dx 2 ) Ix =L =
du d2U
u(O) = 0 = dx (0) ,
d2UI
E I -2
dX x= L
= M o, .!!- 0.
1.6. EXERCISES 21

ANS.
r
d2w d2 u
L

b(w,u) = Jo EI dx 2 dx 2 dx,

r wf dx + w(O) [ddx ( EI dw
l(w) = - Jo
L

dx dU)]
dx x= o

_ [dW]
dx x=o
[El du
dx
]
x=o
_ fow(l) + mo [dw]
dx x= L
.

1.8. Use the Galerkin or Rayleigh-Ritz method to solve - \J2 U = 1 in S1 =


{(x, y) : 0 < x , Y < I} such that

u(l , y) = 0 = u(x , 1),


au au
an (0, y) = 0 = an (x ,0),

by choosing (a) w = ¢i = (1 - x i )(l


- yi) for i = 1, . .. ,N, or
(2i - 1)1TX (2i - 1 )1TY .
(b) w = ¢i = cos 2 cos 2 for z = 1, . . . ,N.
ANS. (a) This choice satisfies the essential boundary conditions, but not the
natural boundary conditions. Hence, we assume the first approximate solution
as U1 = a¢l, ¢1 = (1 - x 2)(1 - y2). The exact solution is

u(x ,y) = ~{(1- y2)


+ 32
1T 3
f ( _l)k cos[(2k - 1)1Ty/2] cosh[(2k - 1)1Tx/2] }
(2k-1) 3cosh[(2k-1)1T/2] .
k=l

The second choice can be dealt similarly .


1.9. Find the approximate solution by the Galerkin method for the nonlinear
problem Ut = U x x + €U 2 on 0 < x < 1, subject to the boundary conditions
u(O ,t) = °
= u(l,t) and the initial condition u(x ,O) = 1, by choosing
¢j(x) = sin jrrz .
N
{I-Uj(t) 1 1T2 Uj (t ) - [2(1 -
L + _j2 COSj1T)
ANS. € . u;(t)
2 2 3]1T
j ,k=l

+ 21T
1
L um(t)f(m, n,j)J} , m, n = 1,2 , · ·· ,N,
m#n
where

.) 1-cos(m-n+j)1T 1-cos(m-n-j)1T
f( m,n,] = . - .
m-n+] m-n-]
1 - cos(m + n + j)1T 1 - cos( m + n - j)1T
- m+n+]
. + m+n-]
. '
N
1
To find Uj(O), solve fo ¢j Rd x = 0, where R = L Uj(O) ¢j(x) - 1.
j ,k= l
22 1. INTRODUCTION

1.10. Usethe Galerkinmethodto solvethe Poisson's equation \7zu = 2 subjectto


theDirichletboundary conditionu = 0 alongtheboundaryofthesquare{-a :::;
x , y :::; a} by choosing (a) the basis functions ¢(x, y) = (a 2 - x 2)(a2 _ y2),
and consideringthe approximate solution

ANS. For N = 1, we have

This yields

We must have A z = A 3 . Then for N = 3, take

where
A _ 1295
1 - 1416az '
A2 =~=A
4432a4 3·

This gives

U2 (X, y) = ~2 (a2 -
4432a
x 2)(a2 - y2) [74 + a15(x 2 + y2)] .
2

. . j1l'X k7l'y .
(b) The baS1S functions ¢jk = cos - cos - , where), k are odd, and
2a a

-
UN =
L j1l'X
Cl! 'kCOS- C O S - ,
k7l'y
J a 2a
j,k=l
j ,k odd

which leads to

f afa [L.J
(j 21l'2 k 21l' 2)
4a2 J
+ -4a-
' " Cl!'k
Z -- COS -
j1l'X
a
k7l'Xy]
COS--
2a
-a -a j ,k

m1l'X k1l'y
X cos - - cos - dxdy = O.
2a 2a
Hence, for j = m and k = n ,
128a2(_1)(j+k-2) /2
Cl!jk = jk(P + P)1l'4
1.6. EXERCISES 23

1.11. Use the Galerkin method to determine the lowest frequency (fundamental
tone) of the vibration of a homogeneous circular plate n of radius a and center
at the origin of cylindrical polar coordinates, clamped at the entire edge, i.e.,
solve V'4 u = AU subject to the conditions u(a) = 0 = ur(a).

HINT . Change to polar coordinates, and take UN = L (Xj ( 1 -


N r2
a2 )
j+1

ANS. Solve j=l

Then for N = 2, we have


4) 4)
(Xl (192 _ Aa + (l2 (144 _ Aa = 0
9 5 9 6 '
4) 4)
(ll (144 _ Aa +(X2 (96 _ Aa =0
9 6 5 7 '
and the equation for A is

(Aa4)2 _ 97;2 Aa4 + 435456 = 0,

. 104.387654 .. .
which has the smaller root as A = 4 . Using this value of A in the
a

r r],
above system of two equations, we find (l2 = 0.325 (l l, and

U2 = (ll [ (1 - :~ + 0.325 ( 1 - :~
where (Xl can be found from the above system of two equations.
1.12. Use the Galerkin method to solve the boundary value problem of Example
1.3 by taking the first-order approximate solution as
N . k
. J1rX. 1ry
Ul = s:
~
(Xjksm~sm-b-'
j ,k=l

which is an orthogonal trigonometric series with a finite number of terms, such


that

l
a
. m1rX . n1rX { 0, m f; n
sm--sm--dx =
o a a a/2 , m = n.
SOLUTION . Note that Ul satisfies the boundary conditions. Then the
Galerkin equation (l .26a) gives

fJ[ (7
b a
j 21r2 k 21r2 ) j 1rX k1ry ] jtt« k1ry
(Xjk +~ sin ~ sin -b- + c sin ~sin -b- dx dy = O.
o 0
24 1. INTRODUCTION

Hence,

(j2- 2
+ -kb2 ) = -.
2
7r C .
0: "k - -(1 - cos)7r)(1 - cos k7l"),
J 4 a2 )k7r 2

which yields

Thus, this approximate solution is

Note that Ul becomes the exact solution Uo as N -> 00 . At the center point
(a/2, b/2), we have

If a = b, then at the center point (a/2 ,a/2)

_",,,,4a 2c(1-cosj7r)(1- cosk7l") . j 7r . k7l"


Ut ,center - L L "kTr4( "2 + k2) sin 2 sm 2
j k J J

= a2c[8+~+~+~+
4
" '] =uo~ 36.64
4
c(~) 2 .
7r 15 15 81 7r 2
1.13. Let

I(u) =
in 2
r ~ [p(x ,Y) IVuI 2
+ q(x, y) u2 - r (x, y) u] dxdy

r2
u E V = {v E C )n ), r l 2
+
1vl 1
"2O:(x ,y)p(x,y)u ds,

= o:(x , y)} ,
2

where r 1 ur 2 = an,
r 1 nr 2 = 0. Derive the corresponding Euler-Lagrange
equation and the natural boundary condition.
2
One-Dimensional Shape Functions

The Galerkin finite element method requires the use of the test functions w in
polynomial form . We will first define the local and global linear and quadratic
Lagrange and Hermite interpolation shape functions. These interpolation shape
functions are used in the next two chapters to solve one-dimensional stead y-state
second-order and fourth-order boundary value problem s by finite element method s.

2.1. Local and Global Linear Shape Functions

Before we proceed with the Galerkin finite element method , we introduce some
basic Lagrange finite element interpolation shape function s, which are used in
solving the second-order bar (or potential) equation s.
We discretize a finite interval [0, L] by dividing it into N subintervals by a
partition
0= Xl < X2 < ... < XN < XN+ l = L,

and denote a typical subinterval by [xi el, x~e)], where e = 1, . .. , N . This sub-
interval denotes a linear finite element nee) of length lee) = x~e) - xi el. The end
points of this subinterval are called the global nodes of the element, which are
labeled e and e + 1 in bold face for e = 1, . . . , N for an N -element mesh of linear
elements. For example, a 4-element mesh of the interval [0, L] is schematically
presented in Fig. 2.1(a), where the global nodes are marked by 1, 2, 3, 4 (in bold) .
The local nodes for each element are marked over the global nodes by 1, 2.

P. K. Kythe et al., An Introduction to Linear and Nonlinear Finite Element Analysis


© Springer Science+Business Media New York 2004
26 2. ONE-DIMENSIONAL SHAPE FUN CTIONS

The point-slope form of the equation of the straight line passing through the
two points (x(e)
I , u(e»)
I and (x 2(e), u(e»
2 ) is

( e) U2(e ) - U(le) ( )
Y - uI = (e ) (e )
X - X(le) ,
X 2 - Xl

which can be rewritten as

where
(e)
(e) X2 - X
¢l (X) = (e) (e) ' (2.1)
X2 - Xl

are called the local linear interpolation shape fun ction s, or simply the local
shape fun ction s. These shape functions are defined in the global sense (with
respect to the variable X).

0 I (e)

o~-- - - - tXl""1
I

rxl
d e)
e
X Xl (e) @ )
Global I I
[( e)

1 2. 1 2 I 2
•1 • • x=! (e)
CD 2 @ 3 CD 4 x = o
u (e)= U
u1 U2 u3 U4
u (eL u
1 - e d e) 2 e+ l
1 2
• •
1 n (1) 1 n (2) 2 I n (3 ) X =Xl (e) ® X =x (e)
2
2 2
•1 2• • • 3• •
CD 2 @ 3 CD 4 (b)
(a)

Fig. 2.1. A Typical Finite Element Mesh Scheme.

However, in the local sense, i.e., with respect to the variable ii , where X =
x + x ~e) (see Fig. 2.1(b)), these functions become
(e) (_) _ X (e)( _ X o::; x < l (e ) . (2.2)
¢l X -1-~, ¢2 x) =~ ,

Let
X E [x(e) x (e)] ,
I , 2 (2.3)

denote this linear interpolation function for the interval [x~e) , x~e) ].
2.1. LOCAL AND GLOBAL LINEAR SHAPE F UNCTIONS 27

e
The characteristic function associated with the interval [x i ), x~e) ] is defined
by
x E [xie) , x~e)],
x(e)(x) ={ 1, (2.4)
0, x rf [x (e) x (e) ]
'F l ' 2 ,

for e = 1, .. . , N . The global piecewise linear interpolation function associated


with the above partition 0 = Xl < X2 < .. . < XN < XN+! = L is defined by
N
ua(x ) = LX (e )(x)u~e ) (x), X E [O ,L].
e= I

( 1) (1) ( N) ( N)
LetUI = UI , U2 =U 2 "", UN =U 1 , and UN+I =u2 . Then
N N+I
e
ua(x) = L x(e)(x )[¢ie)(x)ui ) + ¢~e) (x) u~e)] = <I>i( X)Ui , L X E [O, L],
e= I i=l
(2.5)
where

<I> I(X) = X(l )(x) ¢i1) (x),


<I>2(X) = x (l )(x ) ¢~l) (x) + X(2l(x) ¢i2) (x),

<I>N(X) = x (N)(x) ¢~N- l) (x) + X(N)(x) ¢iN)(x ),


<I> N+!(X) = X( N)(x) ¢~N)(x), x E [O ,L] .

,,
,,
/
/

,,
/
/

,,
-,
- - - - - - -'F---------'.,:- - - - - __- - -
e-2 e -1 e e+1

Fig. 2.2. Linear Global Shape Functions.

The tent-shaped functions <I>1(x), . .. , <I> N+l (x), defined on [0, L] and shown
in Fig . 2.2, are called the global piecewise li near shape fun ctions associated with
the partition 0 = Xl < X2 < .. . < XN < XN+! = L , or simply, the lin ear global
shape functions associated with the above partition . They can be represented as
d,(e -l)
'1'2
'f Xe - l ::; X ::; Xe'
1
<I> e -- d,(e)
'1'1
'f Xe <
1 _ X ::; Xe+I, (2.6)
{
o elsewhere.
28 2. ONE-DIMENSIONAL SHAPE FUNCTIONS

This result will be useful in defining the function u continuously for any value of
x E (0, l) in terms of the nodal values Ui, i = 1, .. . ,N.

2.2. Local and Global Quadratic Shape Functions

e)
Consider a finite element n( e) that consists of the end points xi and x~e) and
the midpoint with x~e) such that x~e) = xie) + l (e) /2, where l (e) = x~e) - xi e)
denotes the length of the element. * The unique quadratic function of the form
u (e)( x) = ax 2 + bx + c passing through the three given points (x ~e), uie )),
(x~e) , u~e)), and (x~e) , u~e)) can be determined by solving for the undetermined
coefficients a, b. and c from the following system :

1 (2.7)

We will use Cramer's rule to solve this system. For the sake of simplicity we use
the following notation:

(xie)f xl(e) 1 (e)


UI Xl
(e)
1
IDI= (x~e)f (e )
x2 1 , IDI I =
(e)
u2
(e)
x2 1 ,
(e ) (e)
U3 x3 1
(x~e)) 2 (e )
x3 1

(x~e)) 2 (e )
UI 1 (xie)f (e)
Xl
(e)
UI

ID2 1 = ( x~e)f (e )
U2 1 , ID3 1 = ( x~e) f (e)
X2
(e)
U2

(x~e)f (e)
U3 1 ( x~e)f (e)
X3
(e)
u3

Then
IDII = ID2 1 jD3 1
a = lDT' b
IDI' c= lDT'
*The len gth z( e) of a n element n( e) is always the d ist ance between the two end
poi nt s of the element , i.e., z( e) = x~e) - x~e) . For a n N-n od e eleme nt ( N ~ 3) , there
are two end points x ~e) and x~e ) , a nd (N - 2) int erior points whi ch are den ot ed by x~e),
k = 3, . . . , N . For unifo rmly spaced nod es the inter ior points ar e defined succe ssively by
(e) _ (k - 2) z( e)
xk - for k= 3, . . . ,N.
N -1
2.2. LOCA L AND GLOBAL QUADRATIC SHAPE F UNCTIONS 29

Thus, the quadratic interpolation function is given by

(2.8)

Rearranging the terms, this can be written as

where

I X(e) (x~e) ) 2 1 (x~e)) 2 X2(e)


X
2 2
(e) ~1 + X(-l) + (e)
x3 ( x~e) f 1 (x~e)f x3
rPi ) (X)
e
=
IDI
x2 X 1
(x~e)) 2 x 2(e) 1
(e)
( x~e) f x3 1
IDI
I (e ) ( x ie)f 1 (x ie))2 x 2(e)
x 2( - 1) x te)
x3 ~ I+x ( x~e) f 1
+ (- 1)
(x~e)f x 3(e)
rP~e ) (x ) =
IDI
( x ie)f (e)
Xl 1
X2 X 1
(e)
(x~e) f x3 1
IDI
( xie)) 2 ( x ie)) 2 (e)
I Xl(e) 1 Xl
X
2
(e) ~ 1+ X(-l) + (e)
x2 (x~e)f 1 (x~e) ) 2 x2
rP~e) (x) =
IDI
( xie)) 2 Xl(e) 1
(e)
( x~e)f x 2 1
x2 X 1
IDI
These quadratic functions rPi ) (x) , rP~e) (x), and rP~e) (x) are called the local quad-
e

rati c shape func tions associa ted with the partition 0 = Xl < X2 < .. . <
30 2. ONE-DIMENSIONAL SHAPE FUN CTIONS

XN < XN+ l = L. By evaluating the determinants, they can be also written in the
following form:

(X~e) _ X) (X~e) - X)
e) (e) ) ( X e) - (e) ) ,
(X2 - Xl 3 Xl

( x~e) _ X) (x~e) - X)
(2.9)
e) - X(e)) ( X3e) - X2(e )) ,
(Xl 2
(x~e) _ X) (x~e) - X)
e) - X(e) ) ( X e) - X3(e) ) .
(Xl 3 2

In terms of the local coordinates ii , where X = x~e ) + x, the local cubic shape
functions (2.9) reduce to

¢ (e) _
l (x) = ( X ) (1 - ~
1- ~ 2x ) ,

(e) _ _ X ( X ) (2.10)
¢2 (X) - 4 ~ 1 - ~ ,
(e) _ X ( 2X)
¢3 (x ) = -~ 1 - ~ ,

where l(e) denotes the length of the element, i.e., l(e) = x~e) - x ~e) , and x~e) =
l(e)/2. Note thatthese interpolation functions have the properties: (i) ¢~e) ( x )e)) =
n n
Oij, and (ii) L ¢~e) (x) = 1, which implies that L d::
(~

= O. The graphs of
i =l i =l
these shape functions are presented in Fig. 2.3. Similarly, we can define the corre-
sponding global and local cubic shape functions .

------- ",,"---~'7----~-:---_-----':~-------,,"' ---


x

2 3
I

Fig. 2.3. Local Quadratic Shape Functions.


2.4. HERMITE SHAPE FUNCTIONS 31

2.3. Parametric Coordinates

Consider the linear transformation

e
The inverse of e(x) is x(e) = xie) + (x~e) -xie))e. Thus, we have cPi ) (x(e)) =e
and cP~e) (x(e)) = 1 - e. Since it is dimensionless, e is called a parametric
coordinate. The functions

are called the local linear shape functions in the parametric coordinate associated
e e
e
with cPi ) (x) and cP~e)(x). We can verify that cPi ) (x(e)) = cPI(e), cP~e) (x (e)) =
e e
cP2(e), and cPI(e(X)) = cPi ) (x), cP2(e(X)) = cP~e) (x) . We also have e(xi )) = 0,
e(x2e)) = 1, 0 :::; e:: ; 1, ddex = (e)
x2
1
-
(e)' X 0
xl
( )
= Xl(e ), X()
1 =
(e)
x 2 , and
dx _ ( e) (e)
de - x 2 - Xl .

Similarly, for the the three quadratic shape functions defined in §2.2, let

_
(e)
X3
+ Xl(e) (X e)
3 - Xl
(e) ) c
x- 2 + 2 <,

(e) + (e)
be the linear transformation and for simplicity alsoletx~e) = x3 2 Xl . Then we
have the corresponding local quadratic shape functions in the parametric coordinate
e:

cP3 (e) = e(1 + e). (2.12)


2

2.4. Hermite Shape Functions

These shape functions are used in solving problems that involve the beam equa-
tion. To derive the interpolation functions for a fourth-order (beam) equation, note
32 2. ONE-DIMENSIONAL SHAPE FUN CTIONS

that since the approximated primary variables must satisfy the essential boundary
conditions

where e= - du (strain) , there are four conditions in an element, two at each node.
dx
Hence, we choose a four-parameter polynomial of the type

(2.13)

We will determine Ci, i = 1,2,3,4 , such that the above four conditions are satis-
fied. This leads to a set of the following four simultaneous equations in the four
unknowns c.:

ul
(e)
=W ( (e) )
Xl = CI + C2 X (e)
I + C3 X l
( (e) ) 2
+ C4 ( X (e)
l
)3
,

(e)
u2 =
[ d W]
- -d _ [e ) = - C2 - 2C3 x (e)
I -
3C4 ( X l e)) 2 ,
X X -Xl

(e )
u3 =W ( (e ))
X2 = CI + C2X2(e) + C3 ( X 2(e) )2 + C4 ( X 2(e) )3 ,
(e )
U4 = [ - -d W] (c) = -C2 - (e) ( (e) )2
2 C3X2 - 3C4 X 2 ,
dx x =x 2

or in the matr ix form

1 Xl
(e)
(x~e)f (x~e)) 3
l

{~ }~ .
CI UI
(x~e)f
~l::
0 - 1 2 (e)
-3
- Xl { (e }
(2.14)
1
(e)
X2 (x~e)f ( x~e) f
C4 ute)
4
0 - 1 -
2X (e)
2 -3 (x~e)f

Solving this system for Ci in terms of u~ e) , i = 1,2 , 3,4, and substituting the values
of c, into (2.13), we obtain

L u~e ) <p~e)
4
W(e) ( X) ~ ( X) , (2.15)
i= l
2.4. HERMITE SHAPE FUNCTI ONS 33

whe re

¢(e) =1_ 3 X - Xl
e)) 2 + 2 ( X - Xl
(e)) 3
I ( I (e) I(e) ,

(e) (e) X - Xl
e)) 2
¢2 = - (X- XI ) ( 1- I (e) ,
(2.16)
¢(e) =3 X - Xl
e)) 2 _ 2 (
X -
(e)) 3
Xl
3 ( I(e) I(e) ,

¢(e) = _ (X_ x(e)) X- Xl


e)) 2 _ X- X (e)
l
]
4 I [( I (e) I (c ) ,

wher e I(e) = x~e) - x~e) . The polynomials ¢~e) (x) derived in (2.16) belong to
the Hermite family of interpolation functions , and are called the Hermit e cubic
interpolation ju nctions represented in the global mode. By taking X = x + xie ) ,
these interpolation fun ctions take the following form in the local mode:

¢l
(e)
= 1- 3
( X ) 2 + 2 ( 10
10 X) 3 ' ¢2
(e) _
= - (X) (
1-
X )2
10 '
(2.17)
(e)
¢3 = 3
( X)2- 2 (10
10 X)3' (e)
¢4 = - (X)
_ X
-
(10) - 10
2 - ]
X
.
[

These functions are useful in solving problems involving the Eu ler-B ern oull i beam
problems by finite elem ent meth ods. The first , seco nd, and the third deri vatives of
the functions ¢~e) with respect to X are given by

d¢ ~e ) 6x X
dx =-(I (e))2 ( 1- T0 ) ,

d¢~e ) _ d¢~e )
dx X '
¢i
d2 e ) 6 X
dx 2 = - (l (e)) 2 (1- 2~ ) ,
d2 ¢~e) d2 ¢~e )
dx 2 -~
d3 ¢ie
) 12
dx 3 = (I (e)) 3'

d3 ¢~e ) 12
dx 3 - (I (e)) 3'
34 2. ONE-DIMENSIO NAL SHAPE FUN CTIONS

These functions satisfy the following properties:

¢~~~ l (x) = Oij, ¢~~) = 0, L ¢~~~l = 1,


i=l
dA,( e)
d¢~~)]
[ ~]
dx X=Xi
=0, [- - -
dx X= Xi
= Oij,

where Xl = 0 and X2 = l(e) denote the local coordinates of nodes at the points
x~e) and x~e) of the element n( e). Their graphs are presented in Fig. 2.4. Notice
that the first derivatives of these shape functions are continuous. The properties
of these shape functions are different from those of the Lagrange shape functions
derived in the previous sections.

0 .2

$1 0 .1 $4

0.5 0.5
-0 .1 ~

-0 .2
0.5 xii xii

1. 5 1.5
d~3
dx

0.5

-1 ~
dx
- 1. 5 L -_ _-----"==_---::----'
xii

Fig. 2.4. Hermite Cubic Functions and Their First Derivatives.

2.5. Exercises

2.1. Derive the local quadratic shape functions ¢ie)(x), i = 1,2,3, for the case
when the global node at x~e) is not the midpoint of the element n( e), i.e., when
x~e) = x i e ) + a l( e), where 0 < a < 1, and l (e) = x~e) - xie) .
SO LUTION. Since the coordinates of the nodes satisfy the equation u(x) =
a x2 + b x + C, we get
2.5. EXERCISES 35

u~e) == u(x~e») = a (X~e)) 2 + b x~e) + C,


U~e) == U(X~e) ) = a (X~e») 2 + b X~e) + C,
which in the matrix notation become s

{('~te)l}
(e)
( x ie)f Xl 1

(e)
U3
= ( x~e) f

(x~e)f
(e)
x2
(e )
x3
1

1
{O
Solving this system we find that

1 ~ (e) (e)
a i(e) -_ Xj(e) ( Xk(e)) 2 - Xk(e) (Xj )2 ,
a = D (e) ~ a i Ui ,
i= l
3
b = _1_ "
o» Z:: ,
(3( e) (e)
U, , (3i(e) -_ ( X j(e)) 2 _ ( (e)) 2
Xk '
i =l

Lai,
3 3
1 , , (e) (e) (e) _ ( (e») 2 _ ( .)2 D (e) =
C = D (e) ~ I i Ui ' I i - Xk X )'
i= l i =l

where the indices i, i. k are taken in the cyclic order as follows :

if i = 1, then j = 2 and k = 3,
{ if i
if i
= 2, then j = 3 and k = 1,
= 3, then j = 1 and k = 3.
Hence ,

L ¢je) uje),
3
u (x ) ~
j=l
where
A,(e)(x) = _1_
'1', D (e)
( a ,(e) + (3(e)
,
+ 'V(e) )
I, ,
. = 1, 2, 3 ,
~

which define the global interpolation functions for the quadratic element. In
e)
terms of the local coordinates x, where x = x i + X, the local interpolation
functions ¢~e)(x) are defined by

(e)
¢ 1 (x)
_
= ( X ) (1 - ;1 ~
1- ~ X ),
(e) _ 1 X ( X )
¢ 2 (x) = a (1- a ) ~ 1- ~ ,
(e) _ a X ( 1 X)
¢3 (x ) = -1 _ a ~ 1 - ; ~ ,
36 2. ONE-DIMENSIONAL SHAPE FUN CTIONS

where 0 < Q: < 1, and x~e) = x~e) + Q: l (e) .


2 .2. Derive the cubic shape functions ¢~e) (x), i = 1,2 ,3 ,4 , in terms of the local
variable x= x~e), where the element nee) consists of four equall y spaced
x -

nodes at the points x~e) , i = 1,2,3,4 with its end points at x~e) and x~e) .
A NS. The shape functions are

2.3. Let X l = 0, X2 = 1/3, X3 = 2/3, X4 = 1, and let U l = 1.0, U2 = 2.0,


U3 = 3.0, U4 = O. Sketch the global piecewise linear interpolation function
U= L Ui¢i for the three linear elements in the interval [0,1].
4

a
i= l

2.4. Let Xl < X2 < X 3 < X4 be four nodes in a finite element mesh, where X3 =
X2 +2 X4 . Let [X (1l ) 'X 2(1 )] = [ Xl,X2 ] , [X (2l ) ,X2(2) ] = [ X2,X4 ] , and x 3(2) =
X3 . Let ¢ (1)(x ) and ¢(2) (X ) be the two corresponding linear shape functions
for [X ~l) , X~l )] , and let ¢ (1) (x ) , ¢(2) (x) , and ¢(3) (X) be the quadratic shape
functions for [x~2), x~2) ] . Write the global shape functions <Pl (x ) , <P2(X) ,
<P3(x) and <P4(x) in terms of the local shape functions and the characteristi c
function s. Also for X l = 0, X2 = 1, X3 = 2, X4 = 3, derive <I>2( X ) explicitly
and sketch its graph .
3
One-Dimensional Second-Order Equation

One-dimensional steady -state problems are encountered in boundary value prob-


lems involving displacement of prismatic rods, heat transfer in a cooling fin,
pressure-driven laminar ideal and viscous fluid flows between two parallel infi-
nite walls and the corresponding heat transfer problems, and in general , potential
problems as well as engineering problems involving the bar equation.
The Galerkin finite element method presented in this chapter requires a knowl-
edge of the Lagrange and the Hermite interpolation shape function s for linear,
quadratic, and cubic elements. This material is presented in the previous chapter.

3.1. Galerkin Finite Element Method

One-dimensional steady-state problems are governed by an equation of the type

2 2U)
d ( a(x) dU)
- dx dx d ( b(x) ddx
+ dx + c(x) U = f( x ), 0 < x < L , (3.1)
2 2

which reduces to the second-order bar equation for b = 0, and to the fourth- order
beam equation for a = O. First, we will study the second-order linear equation and
present the finite element solution of some representative boundary value problems.
In Chapter 4 we will discuss the fourth-order linear equation, which is also known
as the Euler-Bernoulli beam equation, with some examples.
A simple application of the Galerkin method is provided by Exampl e 1.8.

P. K. Kythe et al., An Introduction to Linear and Nonlinear Finite Element Analysis


© Springer Science+Business Media New York 2004
38 3. ONE-DIMENSIONAL SECOND-ORDER EQUATION

3.1.1. Linear Elements for Second-Order Equation. These problems


are governed by the equation

d ( a(x) dx
- dx dU) + c(x) U = f(x), 0< x < L, (3.2)

where a, c, and f are given functions of x, and U is the unknown function of x.


The two-point boundary value problem associated with this equation is to find a
particular solution U ofEq (3.2) that satisfies some additional boundary conditions.
Typically, we prescribe u(O) or ~~ (0) and u(L) or ~~ (L) among other boundary
conditions. If a, b, and f are constants, then an explicit analytic solution can be
found. However, in general , such a formula-type solution is not always available.
In such a case, an approximate solution is frequently sought. There are many
methods that are effective in finding an approximate solution of u. The Galerkin
method is one of the basic methods in finite element analysis for numerically
approximating solutions of such boundary value problems.
We introduce the Galerkin method for an approximation of solution u of Eq
(3.2) in the finite interval [0, L]. For simplicity, we begin with the tent-shaped linear
shape functions <I> 1 (x), . .. , <I> N +1 (x), presented in Fig. 2.2, and the corresponding
piecewise linear function ua(x) defined in (2.5). In order for ua(x) to be a good
approximation of u, it is first required that U a (x) satisfy Eq (3.2) in some sense ,
and second that U a should also satisfy the prescribed boundary conditions. Since
U a (x) does not possess second-order derivatives, it can not satisfy Eq (3.2) directly.
Instead of substituting U a (x) into this equation, we first multiply both sides of Eq
(3.2) by an arbitrary shape function, say, <I>j(x), where j can be any integer in
1, .. . , N + 1, and then integrate the resulting equation over the interval [0, L]. We
have

r r
L L
_i t' ~
dx
(a
dU)
dx
<I>dx+
io cu<I>· dx=
i o f<I> J·dx.
o J J

By using integration by parts formula (A.6), which is valid for piecewise continuous
functions on [0, L], the first term in the above equation can be replaced by

l du ·IL
L
a du
_d<I>
_J dx-a-<I>
o dxdx dxJo

which yields

(3.3)

Note that Eq (3.3) holds only if u is a solution of Eq (3.2). The next step in the
Galerkin method is to replace u in the first two terms of Eq (3.3) by U a which
3.1. GALERKIN FINITE ELEMENT MET HOD 39

is defined in (2.3), take the sum over all elements, and derive a closed system of
linear equations. As a result , we have

(r
N+ 1
~ J
L
dCPi dcpj
a dx dx dx+ J
rL
C <1>i<1>j
)
dx Ui =
r L
du L
J f e, dx + a dx <1>j 10 + u, l
o o o
(3.4)
where Rj , j = 1, . . . , N + 1, denote errors caused by replacing the exact solution
U by the piecewise linear approximation U a . For j = 1, . . . , N + 1, the N + 1
linear equations defined by (3.4) can be written in the following matrix form :

l
Kll
K 21

KN~l'l (3.5a)

or

KU =F, where F =f +Q +R, (3.5b)

where

(3.6)

Since <1>N+1(L) = 1, <1>j(L) = 0 if j i= N + 1 and <1>1 (0) = 1, <1> j (O) = 0 if


j i= 1, we get

The coefficient matrix in (3.5), denoted by K, is called the global stiffn ess ma-
trix and the first vector on the right side, denoted by f , is called the global load
(force) vector. The third step is to drop the unknown column vector (errors)
40 3. ONE-DIMENSIONAL SECOND-ORDER EQUAT ION

R = [R 1 .. . R N +1 f in (3.5), and consider the following N + 1 linear equa-


tions in N + 3 unknowns , namely U 1,.· · , U N+l , Ql, QN +l:
K ll K 12 Kl ,N K1 ,N+l U1
K 21 K 22 K 2,N K 2,N+l U2

K N,l K N ,2 KN ,N K N ,N +1 UN
KN+l ,l K N+l ,2 KN+l ,N KN+1 ,N +l U N +1

~UU+ J, .
(3.7)

Q, )

The fourth step is to assign the prescribed values to two of these unknowns ac-
cording to the boundary conditions . For example , if u(O) = A and ~~ (L) = B
are the given boundary conditions, then we set U l = A and QN+1 = a(L) B and
obtain the following (N + 1) x (N + 1) linear system of equations:

llJ
n K 12 K1 ,N Kl ,N+1
K 21 K 22 K 2,N K 2,N+l
[ K

K NI K N ,2 K N ,N K N.N+l ]
K N+1 ,1 K N+1 ,2 K N+1 ,N K N+1 ,N+l

~ll)+LIJ
which can be solved for the remaining unknowns U 2 ,· ·· l U N +1. In general,
the quantities K i j and !J are evaluated locally and numericall y in each relevant
subinterval in terms of local shape functions. Due to the particular choice of these
linear tent-shape functions , <I>i<I>j == 0 on [0, L], if Ii - j l ~ 2. Hence , the matrix
K is tridiagonal. For example ,
3.1. GALERKIN FINITE ELEMENT METHOD 41

(e )
I K ij
For e= l , . . . , N ,et = 12
X« )

«)
Xl
(d¢i
a -d-
X
(e) d ,.l,(e)
'Pj
-d-
X
+ C'P,.I,(e),.I,(e
i 'Pj
») d
x, and f j(e ) =
x &<)

1Xl
(0)
f ¢;e) dx, where i = 1,2 and j = 1,2. The 2 x 2 matrix

is called the local stiffness matrix assoc iated with the e-th element [x ~e), x~e) ], or
simp ly the local stiffness matrix . Similarly,

is called the local load (force) vector . From the above calc ulatio ns, we have

K ll K I2 KI ,N KI ,N +l
K21 K 22 K 2,N K 2,N+l

KN ,I K N ,2 KN ,N KN,N+I
KN+l ,1 K N + I ,2 KN+I ,N KN+I ,N+l
K (I ) K (I ) (3.8)
II 12 0 0 0
K (I )
21
K (l )
22
+K (2)
II
K (2)
12 0 0
0 K (2)
21
K (2)
22
+ K I(3)I 0 0

K (N ) K (N )
0 0 21 22
42 3. ONE-DIMENSIONAL SECOND-ORDER EQUATION

Similarly,

(3.9)

Notice that the entries in the local matrix K(e) and the corresponding entries in the
global matrix K are defined similarly in terms oflocal and global shape functions,
respectively. The global nodes are numbered for the linear elements according to
the entries in Table 3.1.
Table 3.1.
Linear Global Global
Element Node I Node2
1st 2
2nd 2 3

N-th N N+I

3.1.2. Local and Global Matrices. We discuss the global and lo-
cal matrices K(e), F( e), K~e), F~e) of the linear elements for the second order
problems.
The local matrix K(e) and F(e) defined in the previous section can be derived
in the exact same manner as the corresponding matrices. We simply multiply both

sides of (3.2) by ¢(e) = {:1:: } and integrate by parts over the element interval

[xie) , x~e) ] . This gives

(3.10)

If we replace the exact solution u in (3.10) by the linear interpolation function


3.1. GALERKIN FINITE ELEMENT METHOD 43

then the two terms on the left can be written as

d¢(e)
d}
1 } { U(e) }

1x\e)
x~e) d<jJ(e) du 1x~e) - [ d,l..(e) ,I..(e) ]
a - - - dx = a x dx
dx dx x\e){ d¢~e ) ~
'1'1 '1'2
dx
1
u~e )

{ l1
{
dx
x ~e)
d,l..(e) d,l..(e)
_'1'_1_ _'1'_1_
d,l..(e) d,l..(e)]
_'1'_1_ _'1'
_2_
} (e)

- l\e)
- a
d¢~e) d¢~e) d¢~e) d¢~e)
dx dx dx dx dx
[u~e) ] ,
u1

dx dx dx dx
(e) } [¢(e ) } { U(e) }
1
x <e) { x 2(e) {
2 c<jJ(e)udx = C ¢1 ¢(e) ] dx 1
,I..(e) 1 2 ute)
X,<e)
X, (e)
'1'2 2

{l
x~e) [¢(e) ¢ (e) ¢(e)¢(e) ] } { u(e)}
- c 111 2 dx 1
- <e) ,I..(e) ,I..(e) ,I..(e) ,I..(e) U(e) '
X, '1'1 '1'2 '1'2 '1'2 2

Let R (e) = { Rt
R (e) }
e) denote the corresponding error vector. Then we have the
local system

(3.ll a)
that is, in matrix form,
(3.ll b)
where

(e ) = { - a (x~e») ~~ (x~e») } + R1
(3.llc)
Q ( (e ») du ( (e ») { R2 } .
a x2 dx x 2
Suppose that a, c, and j are equal to constants a(e), c(e), and j (e), respectively, in
the element interval [x~e), x~e) ]. Then the local system takes the following simple
form :
a(e)
( l(e)
[1-1 -1]
1 + 6 [212 1]) { u~e)}
c(e) l(e)
u (e)
2

{1} + { _a(e)~~ (x~e»)


= j (e)l(e) } + {R~e)} .
(3.12)

2 1 a (e)du
- ( x 2(e» ) R(e)
2
dx
44 3. ONE-DIMENSIONAL SECOND-ORDER EQ UATION

Note that if a = a( e) x, c= c( e), and j = r». then


K( e) = a
(e) ((e)
Xl
2l(e)
+Xz
(e))
[1 -1] +
-1 1
c (e)l(e)
6

F
(e) _
- --
j(e)l( e) {I} + {Q~e)
1 (e)
}
'
(3.13)

2 Qz

EXAMPLE 3.1. Consider a prismatic rod made of 304 stainless steel rigidly
fixed at the top point subject to a vertical load of 1000 lb/in. Determine the
deflections, reactions, and stresses for the additional properties data: L = 100 in,
A = 1 inz, E = 29 x 106Ib/in z, and p = 0.29Ib/in 3 .
f =1000lb/in f = 1000lb/in

1 I 1 I

Linear
CD Element

2; Quadratic
I
CD Element

Linear
CD Element

3 2

(a) (b)

Fig. 3.1. A Mesh of (a) 2 Linear Elements; (b) 1 Quadratic Element.

As a simple case, we use only two linear finite elements of equal length to
approximate the solution (FIg. 3.1a). In this case a = EA = 29 x 106 , c = 0,
and j = 1000 + pA(L - x) = 1000 + 0.29(100 - x). For simplicity, we will
ignore the gravitational force 0.29(100 - x) in the following calculations. Since
a(e) = 29 x 106 lb/in z c (e) = 0 L = 100 in l( e) = !:.. = 50 in, j (e) = 1000
, , , 2
lb/in, we get from (3.12)

1 -1] 5[1 -1]


[ -1 1 = 5.8 x 10 -1 1 '

and

r(e) __ 1000l(e)
2
{I}
1 = 25000
{I} 1 for e=1 ,2.
3.1. GALE RKIN FINITE ELEME NT MET HOD 45

Let Ul1) = U1, u~l) = U2 = u12), and u~2) = U3 , we have the global system

Applying the boundary conditions U1 = u(O) = 0, Q 3 = a ~~ (L ) = 0, and


uncoupling the first equation from the rest, we have

5.8 X
5
10 [-1 0] { ~~} = 25000+Ql ,

and

5.8 x 10
5 [ 2
-1
-1 ]
1
{U
U
2 }
3
=
{ 50000 }
25000 .

The second linear system can be solved explicitly for the displacements U2 and U3 ,
and the result is U2 = 0.1293 in, and U3 = 0.1724 in. The reaction force at the
top Ql can now be solved by substituting the displacements into the first equation
and it equals - 90000 lb.•
3.1.3. Quadratic Element for Second-Order Linear Problems.
For the quadratic shape functions, the corresponding local element matrices are
given by
46 3. ONE-DIMENSIO NAL SECOND-ORDER EQUATION

!
and

{~i:; } ~~ :;:::~l~i ) {~i:; }


Q~e)
=a
cjJ(e) I X~<)
3 x\<)
+
R~e)

= {-a (x~e))o~~ (x~e)) } + {~t:;}


a (x~e)) ~~ (x~e)) R~e)'
When a, c, and j are constant, equal to a(e), c(e), and j(e), respectively, in the
element interval [x~e) , x~e) J , the corresponding local system is

ate) 7
- 8
( 3l(e) [ 1
-8
16
-8
~8] + c(e) l(e) [~
7 30 -1
126 ~1]) {~t:;fe) }
2 4
u3

j (e)l(e) {I} +4
d]
-a(e) d~ (x~e))
0 +
{R~e) }
R(e)
(3.14)

6 1 [ ate) ~~ (x~e)) R~e)


for quadratic elements with the second node at the midpoint of the element, where
the shape functions cjJ~e) (x), i = 1,2, 3, are defined by (2.10).
EXAMPLE 3.2. Consider a prismatic rod made of 304 stainless steel rigidly
fixed at the top point subject to a vertical load of 1,000 lb/in. Determine the
deflections, reaction and stresses for the additional properties data: L = 100 in,
A = 1 in2 , E = 29 X 106 lb/in", and p = 0.29 lb/in". For simplicity, we will use
only one quadratic finite element to approximate the solution (Fig. 3.I(b)). Again
we ignore the gravitational force pA(L - x) in the following calculation s. Since
ate) = 29 x 106 lb/in", c(e) = 0, l(e) = L = 100 in, and j (e) = 1000 lb/in, and
e = 1, we have
7 - 8
- 8 16
[ 1 -8

9.6667 X 10
4
[ ~8 -8 1] {Ul}
16 -8 U2 = 1.6667 X 104 {1
4 }+ {a.
0 }.
- 8 7 U3 1 Q3
3.1. GALERKIN FINITE ELEMENT METHOD 47

Applying the boundary conditions U1 = u(O) = 0, Q 3 = a du (L) = 0 and


dx
uncoupling the first equation, we have

9.6667 X 10
4
[-S 1] {~~} = 1.6667 X 10
4
+ Ql ,

and

9.6667 x 104 [16


-S -S]{U2} =
7 U
3
1.6667 x 10 4 {4}
1 .

The second linear system can be solved explicitly for the displacements U2 , U3 and
the result is again U2 = 0.1293 in and U3 = 0.1724 in, which is almost identical
to those obtained by using two linear elements . The reaction force at the top Q1
can now be solved by substituting the displacements into the first equation and it
equals -90000 lb. •
REMARK . Although the two finite element solutions in Examples 3.1 and 3.2
give identical displacement solutions at the nodes x = 50 in and x = 100 in.
respectively, the quadratic element gives better approximation between these two
nodes. In fact, we can verify that the exact solution to the given problem, that is
29 x 10 6 u" = 100, u(O) = 0 = u' (100 ), is

10- 3
u(x ) = 29 x(100 - 0.5x) , 0 < x < 100 ,

which equals the finite element solution of the quadratic element identically.
3.1.4. Mixing Two Different Elements. One of the advantages of
the finite element meth od is its ability to adapt and mix different element types
for better approximation of the soluti ons of cert ain problems locally. Consid er the
following example.
EXAMPLE 3 .3. An elastic rod is 100 in long rigidly fixed at one end . The first
half of the rod from 0 in to 50 in is made 0[304 stainless steel with Young's modulus
E = 29 X 106 lb/in'', density p = 0.29Ib/in 3 , cross-sectional area A = 1 in 2 , and
is free of loading. The other half of the rod is made of aluminum with Young 's
modulus E = 4.46 X 10 6 lb/in" , density p = 0.0979 lb/in", cros s-sectional area
A = 1.5 in2 , and is subject to a distributed body load of 1000 lb/in. Determine the
deflections, react ions, and stresses for the given data .
We will combine one linear element with one quadratic element of equal length
to approximate the solution (Fig . 3.2). Ignoring the small gravitational force, we
have a (1) = 29 x 10 6 lb, c(1) = 0, [ (1) = ~ = 50 in, f (1 ) = 0 lb/in , and

K
( 1) = a (l) [ 1
[ (1) - 1
-1]
1 1
= 5.S x 10 5 [ -1
- 1]
1 '
48 3. ONE-DIMENSIONAL SECOND-ORDER EQUATION

a(2 ) = 6.69 x 106 lb, C(2) = 0, l (2 ) = ~ = 50 in, f (2 ) = 1000 lb/in, and

K (2 ) = a (2)
3l(2 )
[~81 -8
~: ! 8] = 4.46 x 104
7
[~81 -~:8 18
-7 ]'

and

F (2) = 10006 l (e ) {I}


: = 8.3333 x 103 { 1}
: .

(1) (1)
Let u 1 = U1 , u 2 = U2 = u (2) (2)
1 ' U2 = U3, and u3(2) = U4 . Then we have the
global system

[
-
580000

r
58 OO
- 580000 0
892200 -356800
0]
44600
- 356800 713600 -356800
44600 - 356800 312200
{UU
U3
U4
1 }
2

1 =0

Fig. 3.2. A Mesh of 1 Linear and 1 Quadratic Element.

Applying the boundary conditions U1 = u(O ) = 0, Q 4 = a ~~ (L) = 0, and


uncoupling the first equation from the rest, we have - 5.8 x 105 U2 = Qb and

102
8922
-3568
-3568 446]
7126 -3568
{U2}
U 3= 8333.3
{41 } .
[ 446 -3568 3122 U4 1

The second linear system can be solved explicitly for the deflections U2 , U3, U4 ,
and the result is U2 = 0.0865988 in, U3 = 0.227477 in, U4 = 0.274294 in. The
reaction force at the top Q1 can now be solved by substituting the deflections into
the first equation and it equals - 50227.3 lb.•
EXAMPLE 3.4. Consider the second-order equation (3.2). We will discuss
the following three cases.
3.1. GALERKIN FINITE ELEMENT METHOD 49

CASE 1. We will solve the following system:

- d~ (a ~~) = f 0 x
< < L,
(3.15)
U=Uo, [a~~ + f3(U-uoo)L=L =0,

using a 2-element mesh with nodes at x = 0, L /2 , L (Fig. 3.3(a». The weak


variational formulation of this problem on an element [x ~e) , x~e) ] is given by

l
0=
X~I ) ( dw du ) [ (dU) ] X~l)
a - - - wf dx+ w(x ) - a- ,
(I) dx dx dx ( I)
XI XI (3.16)

0= l X2

X( 2)
I
(2 )
w
(a dd ddu - wf) dx + w(x~2»)addu (X~2»)
X X X
- w (x~2» ) ( f3 Uoo - f3 u (x~2 » ) ) , (3.17)

where w denotes a test function . We use a 2-element mesh to compute u at the


nodes .

UI u2 u3
• • q(4e) )=-Q/e) q (xt ) )=~(e)
1 CD 2 CD 3
I

o •

0 I (e) U2 I (e) L
I 2 (b)
(a)

Fig. 3.3. (a) 2-Element Mesh ; (b) Flux at the Nodes.

The fluxes at the nodes 2 and 3 (Fig. 3.3(b» are given by

- ( a-dU) I_ (I) = Q (11) , (3.18 a )


dX X -x I

( addU)
X
I_
X -X 2
(2) = - [f3 (u - Uoo)] _
X -X2
(2) = Q~2) ,
(3.18b)

where Q~2) = Q~1) are unknown, and Q~2) corresponds to the mixed boundary
condition at node 3. Then the finite element equations for [X~l) , X~2) ] are given
50 3. ONE-DIMENSIONAL SECOND-ORDER EQUATION

by

2
= ""
L
K (2) U(2)
'J J
_ p (2 )
J
= 0' for i = 1, 2, (3.19b)
j= l

which in matrix notation is again of the form

K (e) u (e) = F(e) for e = 1 , 2, (3.20)

where

K
(2) _
-
a (2)
1(2)
[1 -1] + [0
- 1 1 a(3 0
(3.22)
F( 2) = a (3uoo { ~ + { Qr)},
}

where (3 and Q~e) denote the film coefficient and the heat flux, respecti vely, at the
n
node i of the element (e) •
3.1. GALERKIN FINITE ELEMENT METHOD 51

CASE 2. We will solve the steady heat conduction in an insulated rod, which
is governed by the system (3.15). We assume that a = kA , where k denotes the
thermal conductivity, and A is a constant. As in Case 1,using a 2-element mesh with
nodes atx = 0, L /2, L, with L = lOcm, kA = 1 (W· cm)/K, ,6 = 25 W/(cm 2 . K),
To = 50° C, Too = 5° C, and f = 0, we compute the temperatures at x = 5 and
x = 10. Since there exists no convection at interelement nodes , the film coefficient
e
,6 between any two elements satisfies the relation Too ,6~e) + Too,6i +1 ) = O. An
evaluation of the stiffness matrix and the force vector from (3.21) for each element
yields

K(l)=~[l -1] [0 0]_[1 / 5 -1 /5]


5 -1 1 + a 25 - -1 /5 126/5 '

F (l ) =5 { 0 } + { Q~l) }
25 Q ~l ) '

K (2) = ~
5
[1 -1] +
-1 1
[-25
a 25
a ] = [-124/5
-1 /5
-1 / 5 ]
126/5 '

F (2) = 5 { -25} + {Q~2) }


25 Q~2) '

After the assembly of these matrices , we get

- 1/ 5
126/ 5 - 124/5
0] {Ul U= lOa}
-1 /5 2
-1 /5 126/5 U3

0 _ dT I _
_ ql (X l e)) - - Q(1l) }
dx x=o
={ 125 - 125 } + Q~l ) + Qi2 ) = a '
{
125 Q~2) = - [25 (U3 - 10)]
or

Solving for U2 and U3, we find from the second and third rows of the above system
that
2 1 1 126
- U2 - - U3 = 10 - - U2 + - U3 = 125
5 5 ' 5 5 '
which gives U2 = 27.5896° C, and U3 = 5.17928° C. Then from the first row,
we get Q~l ) = 10 - ~ U2 = 4.48208 watts/cm'' . The exact solution of this
52 3. ON E-DIMENSIONAL SEC OND-ORDER EQUATION

problem is u(x) =
50 - 1 x, which gives u(5) == U2 = 27.5896° C, and
215215
u( 10) == U3 = 5.1792 8° C.

C ASE 3 . Assume that A = A o (1 + I)' where A o is a constant. Then from


(3.2 1) we get

K (e) = kA o ( 1 +
x~e) + x~e» ) _ 1 + x (e)
( 1 + x(e»)
2 1 (e)
+
lee) [ (

x (e) + x (e» )

x(e) + x (e)) [0
-Pl
_ 1+ 1
2l
2
(1 + I
2l
2

F (e) _ { p ie) } { _Q~e) }


- u oo (3~e) + _ Q~e) ,
(3.23)

where the relations between the fluxes at the nodes are given in Fig. 3.3(b). Again
for a 2-element model, with kA o = 1 and the same data as in Case 2, we obtain
the assembled matrix

- 1/ 4
101/4-493/4
0]
- 7/ 20
{UU= 50 }
I
2
- 7/20 507/20 U3

0 _ qi (X (e»)
l -_ dT
dx x=o
I _Q (l) }
- I

=
{
125 - 125
}
+ Q ~l ) _ Qi2) = 0 '
{
125 Q~2) =0
or,

[ ~10/;4 ;~:: -7~20] { U~~} = { ~


- 7/20 507/20 3 125
}+ {Q
r }.
0

Solv ing this system, we get U2 = 23.902237 °C , U3 = 5.260978°C, and Q i l ) =


6.52444075 W/cm 2 . The exact solution is given by

u(x) = 199.05667 - 64.7345 In(lO + x ),


which yields u(5) == U2 = 23.7524 , u(lO) == U3 = 5.1295, and QP ) = 6.47345.
This matches with the finite element solutions.
For any x E [0, 10], we have

UI if>~I)(X) + U2 if>~2) (x) = (1 - ~) UI + ~ U2, O S; x S; 5,


u(x) -
- { U2 if>~I) (X) + U3 if>~2) (x)=(2-~) U2 -(1 - ~) U3 ' 5S;xS; 10.

Thus, for example, u(2) = '35 UI + '25 U2 = 39.56°C, and u(8) = '25 U2 + '35 U3 =
12.7165°C. •
3.1. GALERKIN FI NITE ELEMENT METHOD 53

EXAMPLE 3 .5 . We will consider the problem of Example 1.8. The weak


variational form ofEq (3.1) over an element n (e) leads to the finite element equati on
K (e)u (e) = F (e) , where

l
Y~r. ) d A-.(e) d¢ (e )
(e)
K ij - 'l'i J d
- /l-d--d- Y,
, y l<) Y Y
(3.24)

l
y~< )
F (e )
J
=
(r.)
f A-.~e ) d
J O'l'J Y
+ Q (e) A-.{e ) ( (e» ) + Q (e) A-.~e) ( (e» )
1 'l'J Yl 2 'l'J Y2 .
Yl

With the choice of the test function s ¢i, i = 1,2 , as in (2.1) or (2.2), we find that

K (e) = i:
l (e)
[1 -1]
-1 1 '
F (e) _
-
l ot (e)
2
{I} + {
1
Q 1(e ) }
Q ~e) '

We will consider a lO-element model with l (e) = 0.1, /l = 1 = b, and io = 1.


Then , since Uo = 0 = UlO , we solve the system

0.01
2 -1 0 0 o 0 o 0
0.01
-1 2 -1 0 o 0 o 0
0.01
o -1 2 -1 o 0 o 0
0.01
o 0 -1 2 - 1 0 o 0
0.01
o 0 0 -1 2 -1 o 0
0.01
o 0 0 0 -1 2 -1 0
0.01
o 0 0 0 o -1 2 - 1
0.01
o 0 0 0 o 0 -1 2
0.01

and find that U2 = - 0.04 = U«, U3 = - 0.07 = Us, U4 - 0.09 = U7 ,


U5 = -0.1 = U6 because of symmetry about y = 0.5. Using the value of U2 , we
find that - 10 U2 = 3/20 + Q~l ), which gives Q ~1) = 0.25.
If we consider an 8-element mesh of nonuni form elements and take different
values of l(e), for example, as lr = 0.1, l2 = 0.15, l3 = 0.125 = l4, l5 = 0.15,
l6 = 0.2, and h = 0.15, and keep the above values of p; b, and 10, then we solve
the system

50/ 3 -20/3 o o o 0 1/ 8
-20/3 44/3 -8 o o 0 11/ 80
o -8 16 -8 o 0 1/ 8
o o -8 44/3 - 20/3 0 11/ 80
o o o - 20/3 35/3 -5 7/40
o o o o - 5 35/3 7/ 40

which gives U2 = - 0.045 , U3 = - 0.09375, U4 = - 0.117188, U5 = - 0 .11375,


U7 = -0.06375 , and Q~l ) = 0.3. These values can be compared with those in
Example 1.8.•
54 3. ONE-DIMENSIONAL SECOND-ORDER EQUATION

EXAMPLE 3.6 . The axial deformation u of an elastic bar of length L in the


presence of external forces P and f and temperature change T is governed by

- : x [EA( x) (~~ - (3T)] = f , 0 < x < L, (3.25)

where (3 is the thermal expansion coefficient (= 12 x 10- 6 in/(in·oF)) , E the


modulus of elasticity (= 30 x 106 lb/in''), A(x) the area of the variable cross-
section of the bar (= 6 - x / IO in 2 ) , P = 400 lbs, T the uniform temperature
change uniform (= 60° F) throughout the length of the bar L = 30 in. It is
assumed that the bar is fixed at the left end and is subject to an axial force of
f = 10000 lb/in uniformly throughout its length. The weak variational form of
Eq (3.25) with the test function w over an element n ee) is

0=1 x~e)
x(1e) w { -
d [
-
dx EA (
du
-
dx - (3T )
]
- f} dx

l
x~e) [ dw du dW]
= EA(x) - - - (3 ET A(x) - - w f dx
X(e)
1
dx dx dx

- EA[W(X) { d~~) - (3T(X)}] x=x~e)+EA [W( X){ d~~) - (3T(x) }] x=X\e)


2
which, with u = I>~e ) (x )u~e) and w = ¢je) (x ) that are defined in (2.1) or (2.2),
i= I
we obtain the finite element equation
n
"
L...K(e)ute)
1))
= F)(e), J. = 1, 2 ,. . . , n ,
i= I

where

Note that for the above linear element n (e) , we have set

EA du ( (e) ) _ _ p ee) EA(3TI X-X


_ (e) = T 1(e),
dx Xl - l ' l

EA du ( (e) ) _ p ee)
dx X2 - 2 , EA(3T Ix-x2
_ (e) = TJe),
(e) _ p ee) + T (e) (e) _ p ee) _ T (e)
Q1 - I I ' Q2 - 2 2 '
3.1. GALERKIN FINITE ELEMENT METHOD 55

Thus, for e = 1, ... , NE,

K( c ) = _E (6 _ (c )
xl + X2
(c)
)
[
1 -1 ]
L(c) 20 -1 1 '

F
(c ) _
- (JE T 6
(_ xi c
) + x~C) ) {-1 }
0 1 +
l (c)L( c) {
2
1}
1 +
{Qi C

(e)'
) }

2 Q 2

For a 3-element model, with 1(0)) = 104 and L(c ) = 10, we find that
K (l) = 3 106 [5.5 -5.5]
x -5.5 5.5 '
l
F (l ) = 216 x 102 { -5 .5 } + 104 {5} + { Qi ) }
5.5 5 Q ~l) '

K (2) = 3 106 [4.5 -4.5]


x -4.5 4.5 '
2
F (2) = 216 x 102 {-4.5} + 104 {5} + { Qi ) }
4.5 5 Q~2 ) '

K (3) = 3 106 [3.5 - 3.5 ]


x -3.5 3.5 '
3
F(3) = 216 x 102 {-3.5} + 10(4) {5} + { Qi )}
3.5 5 Q~3) '

Hence, the assembled equation K U =F becomes

5.5
-5.5
-5.5
10.0 o
-4.5 0] {UI}
0 U2
[ o - 4.5 8.0 - 3.5 U3
o o -3.5 3.5 U4
l (3.27)
_
-0.0229333 }
0.0405333
{ Qi
Q~l) + Qi
) 2
)
}

- { 0.0405333 + Q~2) + Q P ) .
0.0418667 Q ~3 )

From the boundary conditions, we have UI =


2
) = 0 (essential), and Q~l ) + ui l

Qi2) = 0 = Q ~2) + Qi3), and Q~l) + Q i ) = 0 = Q~2) + Qi3); also, Q~3) =


p~3) _ T? ) = P - T~3) = 400 - {JE T (6 - 30/10) = - 64400, which yields
(10- 6/ 3) Q~3) = -0.0214667. Thus, the system (3.27) reduces to

6 l
5.5
-5.5
-5.5
10.0 o
-4.5 00 J{UO} 2 _
{-0.0229333}
0.0405333 + { 10- 0 )/3 } Qi
[ o - 4.5 8.0 -3.5 U3 - 0.0405333 0 '
o o -3.5 3.5 U4 0.0418667 -0.0214667
56 3. ONE-DIMENSIONAL SECOND-ORDER EQ UATION

which, after solving for U2 , U3, U4 , gives

U2 = 0.0184475 , U3 = 0.03199 , U4 = 0.37817 ,

and then the first equation in (3.27) gives Qi l


) = -2.3558475 X 105 . The value
of u at any point x E [0,30] is given by

U2¢~l) (x) = 0.00184475 x , 0::; x ::; 10,


u2¢i2)(x) + U3 ¢~2)(x) = 0.00186675 (20 - x ) + 0.003199 (x - 10) ,
u(x) = 10 ::; x ::; 20,
3
u3¢i )(x) + U4 ¢~3) (x) = 0.003199 (30 - x ) + 0.0037817 (x - 20) ,
20 ::; x ::; 30.

The exact solution is determ ined as follows: since E A (~: - (JT) = - [ x + ci ,


and EA ddU - (JT I = 400, we find that c i = 235600. Also, using A(x) =
x x =30
6 - x/ lO,

du _ (JT _ I x 235600
dx - EA + EA
101 235600
= (JT - E [x + 60 In(60 - x )] - - E - In(60 - x ) + C2·

. 1247 In 60
Since u(O) = 0, we find that C2 = 60 X 10- 2 , and thus finally,

u(x) = 4.053 X 10- 3 x + 0.121447 In(60 - x) - 0.497246.

This yields u(O) = 0, u (lO) = 0.0183874 = U2, u(20) = 0.03182 ~ U3, and
u(30) = 0.03741 ~ U4. Note that the finite element solution does not quite match
at points other than the nodes. This is because of the 3-node mesh. Since the
function A(x) is linear, we can get better results by taking more elements. •

EX AMPLE 3.7 . The temperature distribution T in a tapered fin is governed by

- dxd (dT)
x dx +N (T-T 2
oo ) =0, 0 < x < L,

where N 2 = ~ J+ ;2
1 (for notation , see Fig. 3.4, where a cross section of the
fin in the xz-plane is shown) . Since the thickness of the fin at the base x = L = 4
is small as compared to its length, the above equation is valid under the following
assumptions: The temperature distribution throughout the width of the fin (in the y
direction) remains uniform; the heat transfer from the edge of the fin is negligible
3.1. GALERKIN FINIT E ELEMENT METHOD 57

as compared to that from the top surface of the fin; and the temperature variatio n
in the z directio n can be disregarded. The boundary conditions are

[x~~] x=o = 0, and T( L) = To = 250° F.

Fig. 3.4. Cross Section of the Fin with Four Linear Elements.

We use the following data : k = 120 (BTUlhdt)OF, (3 = 15 BTU/(hd t2 )0F,


Y = 0.1 in. Then N 2 = 5.00156. The stiffness matrix and the force vector for an
element n( e) are defined by

l
x~e) ( d¢(e) d¢(e) )
K~) = . x - i -d J
+ N 2 ¢i¢j dx ,
( e)
Xl
dX X

l
x ~e )
l e) = N 2 T, ¢(e) dx .
J (e) 00 J
Xl

Taking a linear four-eleme nt mesh, and using formula (3.13) with a = ale) x and
c = N2, we obtain

r;
- 1 0 0 1 0 0

~]
~]
4 -3 0 4 1 0
K =~ 8 -5 o + -N' 0

rt
-3 1 4 1
2 - 5 12 -7 72 0
0 0 1 42

r
0 0 -7 7 0 0 0 1
-0.430534 0 0 0
0 .638932
- 0.430534 2.27786 0 0
0 - 1.43053 4.27786 - 2.43053 0
0 0 -2.43053 6.27786 -3.43053
0 0 0 - 3.43053 3.63893

r
Qi 1) 1
6299 Qi ) = 0

F ~ N'1~ { ~}
24
2
+ Qi2)
Q~l)

+ Q~l) + Qi2)
Q~l) + Qi2)
=
31.2597

31.2597
Q~l) + Qi2) = 0
31.2597 + Q~l) + Qi2) = 0
Q~l ) + Qi2) = 0
}
1 Q~4) 15.6299 Q~4)
58 3. ONE-DIMENSIONAL SECOND-ORDER EQUATION

Solving the system KT = F with Ts = 250, we find that T l = 123.85, T 2


= 147.496, T 3 = 175.734, T4 = 209.629, T s = 250, and Q~4) = 174.964
BTUlhr. •
EXAMPLE 3 .8 . For each J..L > 0, cons ider the functional

I(u) = 1~ 1
[ (U,)2 + 2U] dx + J..L [u(O ) - If, (3 .28)

which is a particular case of Eq (2.1). We find min lover the finite element space
generated with two linear elements of equal length by using the two methods:
(1) the method of steepest descent, and (2) the conjugate gradient methods (see
Appendix G). We will derive the local and global gradient vectors. We will also
discuss the limiting case lim u(J..L) by taking J..L = 10 2 , 10 3 , .. . •
J-L---+ OO

U2

1

2 3

0.5 0.5

Fig. 3.5 . 2-Element Mesh.

From (1.18) , we find that

~~ -:x(:~) =0
for all 1](a) = 1](b) = 0, which yields oOF I
Ux x = a
= OOC
( )
U a
for 1](b) = 0 and

OF I
OU x x = b
( ) for 1](a) = O. Since in this example a = 0 and b = 1, we
= OOC
U a
of of OC
get ou = 1, oU = u x , and ou(a) = 2J..L [u(O) - 1]. Then the Euler-Lagrange
x
equation (1.19) becomes
d2u
dx 2 - 1 = 0, 0 ~ x ~ 1, (3 .29)

subject to the boundary conditions

d~~O) = 2J..L[u(0) - 1], and du(l) =0 (3 .30)


dx .
The weak form of Eq (3.29) is

[d 2 ]
o = 1~e2) dX~ + 1
x(e)
W - dx

_l
(3.31 )
x2
( eJ

- [d
dW d
du - W] dx - W ( X (e»
l
) Q(e) ( (e») Q(e)
1 - W X2 2'
x~e) X X
3.1. GALERKIN FINITE ELEMENT METHOD 59

where
Q(e) = _ dU I and Q(e) = dUI .
dx xi') ' 2 dx x ~e )
2
We assume that U = L<Pje)uje). Then Eq (3.31) reduces to the finite element
j =l
equation
2
L k~j) u je) = Fi(e) == f ile) + Q~e) , i = 1,2 , (3.32)
j=l

where

K (e) =
tJ
l x 2

<e)
Xl
<,) (e)
d<Pi
dx
d<pj dx
dx '
fie) =
t
_l x2
H

(e )
Xl
<Pi dx. (3.33)

Using the interpolation functions (2.1), we obtain the matrices

K( e) =_1 [1
lie) -1
-1]
1 '
r (e) = _~
2
{I} l '
which , on assembly for the two elements, gives

K =-
1[1 -1 0] -1 2 -1 , F =--
lie) {1} +
2
Qt
+ Qi }
Q~ Q§ .
lie) 0 -1 1 2 1 { 2
1
Thus , the equation KU =F becomes

[ ~2o ~2 ~2] {~~} = {-0~~.~5Q~


-2 2 U3 -0.25 + Q§
}.
Since h1 = 0.5 = h 2 , U1 = u(O) = 1, and Q§ = 0 as J.L ---4 00, the above equation
reduces to
4
[ -2
-2]
-2
{UU 2
3
}
-
{1.5}
-0.25 '
(3.34)

which we write as Au = b . We solve Eq (3.34) by the following two methods.


M ETHOD 1. METHOD OF STEEPEST DESCENT. (See Appendix G) The
quadratic function associated with Eq (3.34) is
1
f(u) = 2" u TA u - bTu

= ~ [U 2 U3 ] [ !2 ~2 ] { ~~ } - [1.5 - 0.25 ] { ~~ }
= 2 (U2)2 - 2U2U3 + (U3)2 - 1.5 U2 + 0.25 U3 .
60 3. ONE-DIMENSIONAL SECOND-ORDER EQUATION

Also, \l f = Au - b = 0 yields

u
( 1) = {0.7} =
0.6
{U2
U
}
3 '

Then

\l f( (1)) ={ 4(0.7) - 2(0.6) - 1.5 } ={ 0.1 }


u -2(0.7) + 2(0.6) + 0.25 0.05'
g(t) = f(0 .7 - 0.1 t, 0.6 - 0.05 t)
= 2(0.7 - 0.1)2 - 2(0.7 - 0.1 t)(0.6 - 0.05 t) + (0.6 - 0.05 t)2
- 1.5 (0.7 - 0.1 t) + 0.25 (0.6 - 0.05 t)
2
= 0.0125 t - 0.0125 t - 0.4 ,
g'(t) = 0.025t - 0.0125.

Thus, g'(t(1)) = 0 yields 0.025 t(1) - 0.0125 = 0, i.e., t(l) = 0.5, and

u(2) = u(l) - t(1) \l f(u(l)) = {~:~} - 0.5 { ~o~ }


= { 0~56:5 } = { ~~ } .
For the second iteration we use the above technique and get

v f(u(l)) = { 4(0.65) - 2(0 .575) - 1.5 } = { -0 .05 }


-2(0.65) + 2(0 .575) + 0.25 0.1'
g(t) = f(0 .65 - 0.05 t, 0.575 - 0.1 t)
= 0.025 e - 0.0125 t - 0.403125 ,
g'(t) = 0.025t - 0.0125 .

Thus , g'(t(2)) = 0 yields 0.05t(2) - 0.0125 = 0, i.e., t(2) = 0.25, and

u(3) = u(2) - t(2) \If(u(2 )) = {0~56;5} - 0.25 { -g.~5}

= { °o~~;5 }= { ~~ } .
3.1. GALERKIN FINITE E LEMENT METHOD 61

After a few more iteration s, Eq (3.34) converges to U2 = 0.625, and U3 = 0.5.


Math emat ica also gives the same solution.
Now, we cons ider the case when J.l f+ 00 . Then , since Qt = 2J.l (1 - Ul ) , the
finite element equation Ku = F for Eq (3.29) is given by

2 + 2J.l -2 0 ] { Ul } { - 0.25 + 2J.l (1 - Ul ) }


-2 4 - 2 U2 = - 0.5 . (3.35)
[ o -2 2 U3 -0.25
If we take J.l = 10000, then the solution of Eq (3.35) is Ui = 0.99995, U2 =
0.62495, and U3 = 0.49995, which converge to Ul = 1, U2 = 0.625, and
U3 = 0.5 as J.l -+ 00 . However, if we set J.l = 10000 in Eq (3.35), we get

20002 -2 0 ] { Ul } { 19999.75 }
-2 4 -2 U2 = -0.5 (3. 36)
[ o -2 2 U3 - 0.25
which we write as Au = b . Then , we have
1
f (u ) = "2 u T Au - b Tu

=~ lUI U2 U3 ] [2~~2 ~2 ~2] { ~~}


o -2 2 U3

- [19999.75 - 0.5 - 0.25 1 {g:}


= 10001 Uf - 2Ul U2 + 2ui - 2U2U3 + ul - 19999.75 U,
+ 0.5 U2 - 0.25 U3 ,
20002 o. - 2 U2 - 19999.75 }
\7 f = Au - b = - 2 Ul + 4 U2 - 2 U3 + 0.5
{
-2 U2 + 2 U3 + 0.25
af
eo,
at = o.
aU2
at
aU3
Aga in, sinceg(t ) = f (u (l ) - t \7 f( u(l))), and u (2 ) = u (l ) - tCl ) \7f(u(1)) , let
0.95}
u {1) = 0.7 . Then
{ 0.6

20002(0.95) - 2(0.7) - 19999.75 } { - 999.25 }


\7 f (u (1 )) = - 2(0.95) + 4(0.7) - 2(0.6) + 0.5 = 0.2 ,
{
-2(0.7) + 2(0.6) + 0.25 0.05
62 3. ONE-DIMENSIONAL SECOND-ORDER EQUATION

g(t) = j(0.95 + 999.25 t, 0.7 - 0.02 t , 0.6 - 0.05 t)


= 10001(0.95 + 999.25 t) 2 - 2(0.95 + 999.25 tHO. 7 - 0.2 t)
+ 2(0.7 - 0.2t)2 - 2(0.7 - 0.2t)(0.6 - 0.05t) + (0.6 - 0.05t) 2
-19999.75(0.95 + 999.25t) + 0.5(0.7 - 0.2t) + 0.25(0.6 - 0.05t)
= 9.986 x 109 e - 9.9850041 x 105 t + canst,
g'(t) = 1.9972 X 1010 t - 9.9850041 X 105 .

Thus, g'(t (2») = 0 yields 1.9972 x 1010 t(2) - 9.9850042 x 105 = 0, i.e., t (2) =
5 x 10- 5 , and

u (2) = u (1) - t (2) 'Vj(u(1 ») = 0.95}


~:~ - 5 x 10- 5 { -9~.~.25
{ 0.05 }

0.9999625 } { U1 }
= 0.69999 = U2 .
{
0.599976 U3
METHOD 2. CONJUGATE GRADIENT METHOD . (See Appendix G) We
start from Eq (3.36). For the first iteration , note that r (O) = b - Au(O) . Let

0.95}
u(O ) = 0.7 .
{ 0.6

Then

19999.75} [20002 - 2
r(O) = -0.5 - -2 4 o ] {0.0.7
-2 95}
{ -0.25 0-2 2 0.6

= { 9~~..~5 } == v (O) .
-0.05

(r(O») T r (O)
Since a (O) = T ' we find that
(Av (O») v(O)

(r (O»)T r(O ) = [999.25 -0.2 -0.05] { 9~~..~5 } = 998500.605.


-0.05

Since
3.2. TWO DEPENDENT VARIABLES 63

we get

(Ay (O))T y (O) = [19998699.9 -1999.2 0.3] {9~~..~5}


-0.05
= 1.9972009 X 10 10
.

(0) 998500.605 5 0- 5 d
Thus,O: = 1.9972009 X 1010 = x l , an

0.95 } { 999.25 }
u(1 ) = u(O ) + 0:( 0) y (O) = 0.7 +5 x 10- 5 - 0.2
{ 0.6 - 0.05

0.9999625 } { U1 }
= 0.69999 = U2 •
{ 0.5999975 U3

After more iterations, we get

U1 } {0.99995}
U2 = 0.692495 . •
{U 0.49995
3

3.2. Two Dependent Variables

Two one-dimensional second-order equations in two dependent variable s are some-


times encountered in physical problems . These equation s may be coupled or un-
coupled. We present the finite element method to solve such problems by an
example.
EXAMPLE 3.9. We discuss the case of two unknown functions of x, which
are two dependent variables u and T, representing, for example , the displacement
and temperature distribution, respectively, in a rod of length L shown in Fig. 3.6.
The governing equations for this problem are

(3.37)

which are subjected to the boundary conditions u(O) = 0 = u'(L) and T( O) = To,
T(L) = 0, where E denotes the Young's modulus , k the thermal conductivity, and
64 3. ONE-DIMENSIONAL SECOND-ORDER EQUATION

f3 the film coefficie nt.


We will use a 2-element mes h. Then, starting with the weak
e
formu lation, as in §1.3, and the interpo latio n funct ions rt>i ) (x), i = 1,2, defined
in (2.2). Since there are two degrees of freedom, each node has two unknowns
U;e) and T;e), e = 1,2 and j = 1, 2. The final finite element equations for an
element n( e) are

which leads to the local system K(e )u(e) = F (e) + G (e) , i.e,

fl( e)
E
- -f3
E f3 u1
(e)
-Eu'(xie))
TW 2 -TW 2 2
gl(e)
k
-kT'(xi e))
k T( e)
0 0 1
~ -~ 2 +
E f3 E f3 (e) fl( e)
-~ Uz Eu'(x~e))
2 ~ 2 2
k k gl(e)
0 0 r.(e) kT'(x~e))
-~ ~ z 2
(3 .29)

I U)' T 1

L

U2 , T
2 U 3'T3
f-. X

• 1 2.1 2•
CD CD
Fig. 3.6. 2-Element Mesh for the Rod .

We take f = 0 = g. Since l(e) = L/2 for each element, Eq (3.29) reduces to


the following:
3.2. T WO DEP ENDENT VARIABLES 65

r}
For element 1: K (l)u(1 ) = F(l ) + G ( l ), i .e,
2E (3 2E (3
- -- -
L 2 L 2
2k 2k
0
(3
-
L
0 --
L
(3
T1 rEU'(OJ}
- k T ' (O)
2E 2E U2 - Eu'(L/2) ,
-- -- -
L 2 L 2 T2 kT'(L /2)
2k 2k
0 -- 0
L L

r} r
and for element 2: K( 2)u (2) = F(2) + G (2), i.e,
2E (3 2E (3
L
0 -
2k
2 L
0
2
2k EU
L L T2 ' (L/ 2) }
-kT'(L/2)
2E (3 2E (3 U3 = Eu' (L) .
--
L 2 L 2 T3 kT' (L )
2k 2k
0 -- 0
L L
Using the boundary conditions, the assembly of the above equations gives
2E (3 2E (3
L 2 L 2
o o
2k 2k
o L
o L
o o
2E (3 4E 2E (3
L 2 L
o L 2
4k 2k
o o o L
o
L
2E (3 2E (3
o o
L 2 L 2
2k 2k
o o o L
o L
or

I~X]E:}~2ft ~ E}
L 2 L 2
. . 3f3T oL To f3T oL
Solving this system, we get U2 = -----ge' T 2 = 2' and U3 = ~. The exact
solution of the problem with f = 0 = g is given by

u(x ) = ::~ (2Lx - x


2
) , T( x) = To (1 - ~) ,
which for x = L / 2 and L yields the above results. •
66 3. ONE-DIMENSIONAL SECOND-ORDER EQUATION

3.3. Exercises

3.1. Obtain the weak formulation for the second-order equation - d~ (a ~~) +
cu = f, and show that the stiffness matrix and force vector for an element nee)
are given by

3.2 . Obtain the stiffness matrix and force vector for a 4-node cubic element nee)
.
for the second-order equation - dx d (a dU)
dx + cu - f = O.

A NS. Using the shape functions (A.lO), we get

148 -189 54 - 13 ]
K( e) =~ -189 432 -297 54
40l(e) [ 54 -297 432 -189
-13 54 -189 148
128 99 - 36
c(e)l(e) 99 648 -81 19 ]
-36
+ 1680 [ - 36 - 81 648 99 '
19 -36 99 128

Q(e)}
F
(e) _
-
f ee)lee)
8
{~} Qte
)
3 + Q~e) .
{
1 Q~e)

For details, see the Mathematica Notebook FourNodeCubic. nb in §14.1.


3.3. Consider the problem of the transverse deflection of a nonuniform cable
which is fixed at both ends and subjected to a distributed transver se force.
The governing equation is (3.2), with a(x) = 1 + 2x, c(x) = 0, and f( x ) =
1 + 4x + x 2 , 0 < x < 1, subject to the boundary conditions u(O) = 0 = u(l) .
Use a mesh of 4 linear elements of uniform length lee) = 1/4, and compute
the transverse deflection u .
Hint. Use

K (e) = _1_([
lee)
1
-1
-1] + (
1 Xl
e)+ x 2(e») [1 -1])
-1 1 '
3.3. EXERCISES 67

pee) _ _lee ) { 1 } + __ 1 + x 2(e)


2l(e) {2X(e) }
- 2 1 3 x (e)
1
+ 2x(e)
2

~ { 3 (xi e») 2 + 2xie)x~e) + (x~e») 2 } {Qie ) }


+ 2 2 + (e) '
12 (xi e)) + 2xi e)x~e) + 3 (x~e») Q2

0, Q~2) +Qi3) = 0,
2
The boundary conditions are U 1 = U5 = 0, Q~l ) + Q i ) =

andQ~3)+Qi4) = 0; the secondary variables are Qi


1
) = [-(1 +2X)ddu] =
x x=o
_ du(O) ,an d Q 2(4) = [(1 + 2x )dU] = 3du(l)
d .
dx dX x=l x

ANS . K [~5o ~~
=
0
i: ~9
- 9 20
~] ,
-11
and solve the system

o 0 0 -11 11

[o
~~ ~; o } { U3
2
U } 199/384 }
-9 ={ 313/384 .
-9 20 U4 439 /384

59519 18799 54559


Then U2 = 362496 ~ 0.164192, U3 = 90624 ~ 0.207439, U4 = 362496 ~

0.150509, Qi
1
) = - ~:~:~: ~ -0.988929, and Q~4) = - ~:~:~~ ~
358483
+ fo f( x) dx = - 362496 -
. (1) (4) 1
-2.344403. It can be venfied that Q1 + Q2
~:~:~~ + (1 + 2 + 1/3) = O. The value of u at any point x E [0,1] is given
by

59519x
90624
for 0 :::; x < 1/4,
59519(1 - 2x) 18799(4x - 1)
for 1/4:::; x :::; 1/2,
181248 + 90624
u(x) = 18799(3 - 4x) 54559(2x - 1)
for 1/2:::; x < 3/4,
90624 + 181248
54559(1 - x)
for 3/4 :::; x :::; 1.
90624
3 2
· · ( ) 5In(I+2x) x 11x x du
T he exact so Iuuon 1S u x = 9 In 3 - - - - - - - and - =
3 12 24' dx
10 2 11x 1
...,--------,-----x - - - -
(1 + 2x) 9 In 3 6 24 .
3.4. A cylindrical sleeve that insulates a cylinder is constructed of four homoge-
neous layers in contact with one another. Assuming that there is no internal
68 3. ONE-DIMENSIONAL SECOND-ORDER EQUATION

heat generation in the sleeve and the heat conduction is steady-state with one-
directional heat flux (dTldy = 0), determine the temperatures T 1 , T2, T3 ,
T4 , and T 5 at the outer and inner surfaces and at the interfaces of the layers,
subject to the data given in Fig . 3.7, where (30 and (36 are the film coefficients
and k l = 60 W/(cm.C), k2 = 25 W/(cm.C), k3 = 55 W/(cm.C), k4 = 30
W/(cm.C) are the respective thermal conductivities of the layers .

Sleeve Cylinder
Ambie nt Ambien t
Temperature Tempera ture
TO= 35°C T6 = 12°C
1 5 X
130= 15 W/(cm.C) It, =30 W/(cm.C)
T) T2
I I
2 mm 8 mm 6mm 4mm

Fig. 3.7. 4-Element Mesh for the Sleeve.

ANS. T 1 = 26.2999°e, T2 = 25.8649°e, T3 = 21.6888°e, T4


20.2651°e, and T 5 = 18.5251°C.
HINT . The stiffness matrix and the force vector are

315 -300 o o
-30 331.25 -31.25 o o
K= 0 -31.25 122.917 -91.6667 oo ] ,
[ o o -91.6667 166.667 -75
o o o -75 95
f = [525 0 o 0 240(.

3 .5. The temperature distribution T in a rectangular cooling fin of length Land


thickness a is given by

where (3 is the film coefficient, k the thermal conductivity, and Too the ambient
temperature of the air surrounding the fin (Fig . 3.8). The boundary conditions
are T(O) = To, and [kA ddT] = 0, where A is the surface area. Using
x x=L
(a) a uniform 3-element linear mesh, (b) a uniform 2-element quadratic mesh ,
and (c) a l-element cubic mesh, compute the temperature distribution T for
the following data: L = 0.3 m, a = 0.02 m, and compare the finite element
solutions with the exact solution.

HINT . Use the nondimensional quantities {) = ~ -=. ~:' and e = x] L .


2
. equation
Th en the govermng . an d the boun d ary con dimons
. b ecome - d de{)2 +
3.3. EXERCISES 69

N 2'19 = 0, '19(0) = 1, and


d'Atl9 l = 0, where N
2 (3 L
= -k . Use N 2 =
~ E=l a
4(3j(ka) = 400.
ANS. (a) l(e) = 0.1. Then solving

2l
2l(e) 2
1 [2 -1 0] [4 1 0]) { 'I9 }
- -1 2 -1 +-
N
- 1 4 1 '19 3 = { l (:) _ N
6
(e) }
( l(e) 0 -1 1 6 0 1 2 '19 4 ~ ,

we get '19 2 = 0.0717987, '19 3 = 0.00518135, '19 4 = 0.000740192, and Q~l) =


16.2225.

Fig . 3.8.
(b) l(e) = 0.15. Then solving

16
1 -8 ~1 ~8 ~] N 2 z(e) [~6
( 3l(e) [ ~ -8 16 -8 + 30 0
1 -8 7 0
8 N 2 l (e )
3l(e) -15
1 N 2 l (e )
= -3l( e) +-W-
o
o
we get '192 = 0.216767, '193 = 0.0628569, '19 4 = 0.0144185, '19 5 = 0.0078405, and
QP) = 20.583.
(c) l( e) = 0.3. Then using the results from Exercise 2.2, and solving

1 [432
- - -297
-297
432
54]
-189 +- -
2
N l (e ) [648
-81
-81
648
-9
396]
) { ~32 }
u
( 40l(e) 54 -189 148 1680 -36 99 128 19 4
70 3. ONE-DIMENSIONAL SECOND-ORDER EQUATION

189 99N 2z(e)


40Z(e) - 1680
54 36N 2z(e)
- 40z(e) + 1680
13 19N 2z(e)
40Z(e) - 1680

we get '19 2 = 0.0034136, '19 3 = 0.003000833, '19 4 = 0.00242254, and Qi l


)
21.453.
The exact solution, given by '19(0 = coshN~ - (tanhN) sinhN~, is com-
puted for ~ = 0(0.025)0.3, and the results are compared in Table 3.2.
Table 3.2.

(a) (b) (c) 'Igexact

O. 1.0 1.0 1.0 1.0


0.025 0.606531
0.05 0.367879
0.075 0.216767 0.22313
0.1 0.07179871 0.0034136 0.135335
0.125 0.082085
0.15 0.0628569 0.0497871
0.175 0.0301974
0.2 0.00518135 0.00300833 0.0183156
0.225 0.0144185 0.011109
0.25 0.00673795
0.275 0.00408677
0.3 0.000740192 0.0078405 0.00242254 0.00247875 •

3.6. The fuel element in a nuclear reactor consists ofa 0.25 m thick plate of length
L composed of an alloy of Uranium (U-235) and Zirconium (Zr-95) with a 0.05
m protective cladding of Zirconium on each side as shown in Fig. 3.9.

T6= 26(fC
5 0.05 m
4

3
I025m
2
1 0.05 m
To= 26(fC cladding

Fig. 3.9.

A coolant flows over the outside surface of the cladding at 260°C. The heat
transfer coefficient f3 = 2500 W/(m 2 .K), and the thermal conductivities of the
3.3. EXERCISES 71

fuel plate and the cladding are ku = 25 W/(m.K) and kZ r = 21 W/(m.K),


respectively. Determine the maximum temperature within the fuel element
and the temperature at the interface of the fuel element and cladding when the
reactor is operating with a uniform heat generation of ij = 8 X 10 8 W1m 3 .
ANS . Maximum temperature within the fuel element is 710°C, and the
interface temperature is 660°C.
HINT . Use the symmetry and determine T I , T 2, and T 3 .
3.7. A fin of length L has a parabolic profile defined by f(x) = fo (L - X)2 as
shown in Fig . 3.10, where fo is a given constant. The fluid surrounding the
fin has the heat transfer coefficient It and is maintained at a given temperature
Too. The base of the fin is kept at a prescribed temperature To. Assuming that
the side surfaces of the fin are insulated and the thermal conductivity k of the
material of the fin has a constant value, determine the temperature distribution
T(x) in the fin.

f(x)=!0(L-x)2

T(:d

Fig . 3.10.

HINT. Solve ~:~ - N 2 {) = 0, where N 2 = ~~,


where P is the perimeter
of the extended surface, and A the cross-sectional area of the extended surfaces.
Take L = 8 em, fo = 100 em, k = 50 W/(m.C), Too = 28°C, f3 = 15
W/(cm.C), and To = 10° C.
3.8. Solve the one-dimensional nonlinear heat conduction problem in a rod of
length L and diameter 0.2 m:

d~ (k(T) ~~) + ij = 0, 0 < x < L, ddT I


x x=o
=O ,TL=Tw ,

I
by using a 4-e1ement mesh .
T
1 (X)
HINT. Set {)(x) = kw k(s)ds,wherek w = k(Tw ) . The problem
Tw

becomes linear: k w dd2~


x
+ ij < x < i. such that dd{) I
= 0, 0
x x=o
= 0 and
{)(L) = O. Use a 4-elementmesh with L = 4, k w = 50W/(m .C), T w = 20°C,
72 3. ONE-DIMENSIONAL SECOND-ORDER EQUATION

and q = 1.59 X 10- 6 W/m 3 • where q denotes the rate of internal energy
generation in the rod per unit volume.

ANS. UI = 25.44, U2 = 23.85, U3 = 19.08, U4 = 11.13. and Q~4) =


9.54.
3.9. Use the Galerkin method to solve the boundary value problem (3.37) subject
to the same boundary conditions. and show that the results are the same as in
§3.2.
3.10. The flow of a thin-film lubrication is governed by

d [(1 +x)2 3 -dP]


-dx
- = f, 0< x < 1,
dx

where P denotes the pressure at the end of the bearing. Compute the pressure
distribution subject to boundary conditions p(O) = 0 = p(l) by using (a) a 4
element mesh. and (b) an 8 element mesh of uniform length. with f = O.
ANS. (a) U2 = 0.018558, U3 = 0.021593, U4 = 0.012415.
The exact solution is U2 = u(0.25) = 0.019295. U3 = u(0.5) = 0.022544.
U4 = u(0.75) = 0.012984.
(b) U2 = 0.010745. U3 = 0.018798, U4 = 0.022630. Us = 0.022213,
U6 = 0.018538, U7 = 0.012912, Us = 0.00647.
Exact solution is U2 = u(0.125) = 0.010848, U3 = u(0.25) = 0.019295,
U4 = u(0.375) = 0.02287, Us = u(0.5) = 0.022544, U6 = u(0.625) =
0.018749, U7 = u(0 .75) = 0.012984, Us = u(875) = 0.006544.
3.11. Solve the problem of Exercise 3.10 for the case when f(x) = x.
3.12. Solve - d~ (kA ~~) + (3pT = qoA, 0 < x < l, subject to the boundary
conditions T(O) = To and T(l) = 0 for a circular bar whose radius varies linear
from ro at x = 0 to ro/4 at x = l (Fig. 3.11), where A(x)
3X)2 .
= A o (1 - 41
Ao = 1rr5, and p(x) = Po (1 - ~7). Po = 21rro , by taking a mesh of
uniform (a) 4 linear elements, and (b) 8 linear elements.

Fig . 3.11.
3.3. EXERCISES 73

HINT. Take u = T /To, 8 = xll. Then the problem reduces to

d
- d8 [(1 - 4 )2 ]+ (cl) (
38 du
dx1- )
4
2 38
u = QI
2
()2
38
1- 4 '
u(O) = 1, u(l) = 0,

2 jJpo qo
where c = kA o' and Q = k'
ANS . (a) U1 = 1, U2 = 0.907817, U3 = 0.769871, U4 = 0.53192,
U5 = O.
Exact solution: Ul = u(O) = 1, U2 = u(0 .25) = 0.909432, U3 = u(0.5) =
0.773902, U4 = u(0.75) = 0.531441 U5 = u(l) = O.
(b) U1 = 1, U2 = 0.959098, U3 = 0.909674, U4 = 0.849517 , U5 =
0.774604, U6 = 0.677732 , U7 = 0.544836 , Us = 0.345241, Ug = O.
Exact solution: U2 = u(0 .125) = 0.0.958956, U3 = u(0.25) = 0.909432,
U4 = u(0.375) = 0.849081, U5 = u(0.5) = 0.773902, U6 = u(0.625) =
0.676678, U7 = u(0 .75) = 0.531441, Us = u(875) = 0.34357.
3.13. The torsional vibrations of a prismatic bar are governed by

d(d¢)
dxa dx + W
2
p¢ = 0, 0 < x < I,

where a = MG, p = PM, w denotes the radian frequency of the vibrations,


P the mass density, M the polar moment of the cross-sectional area, and G
the shear modulus. Solve the eigenvalue problem if ¢(O) = 0 = ¢' (l ) for a
uniform circular bar.
HINT. Since a and p are constant, the problem reduces to ¢" + A ¢ = 0,
¢(O) = 0 = ¢/(l), i.e., (A - )"B) ¢ = 0, where for a 2-element mesh

1 -1 0 210
A = ~ -1 2 -1 B=~ 1 4 1
l 0 -1 1 12 0 1 2

)"l2
or, 2 (1 - 2b)2 - (1 + b)2 = 0, where b = 24' The roots are b1 = 0.1082,
and b2 = 1.3204 . Hence, Ai = 2.5968/12, and A2 = 31.69/12. The exact

( rr ) 2, 2 (3rr )
solution is Ai = 21 2 ;:::: 2.4674/1 and X, = 21 2 ;: : 22.207 /1 . Solve
for a 4-element mesh and get Al12 = 2.4993, )..212 = 24.872, A312 = 82.073,
A412 = 171.63 .
3.14. Let
74 3. ONE-DIMENSIONAL SECOND-ORDER EQUATION

be an energy functional, where p, q, r, o, and U oo are known functions of x.


Derive the corresponding Euler-Lagrange equation and the natural boundary
conditions.
HINT . Suppose that u is replaced by the linear finite element interpolation
2
u(e)(x) = L. ¢~e) Ui(e) in the interval [xlI) , x~e)]. Write the corresponding
i=l
local energy /(e) = I (u(e)) in the form

where u(e) = [ule) , u~e)f. Let u(1) = Us, u~l) = ul 2) = U2, and uf) = U3 .
Suppose that p, q, r, o , and U oo are all constants in [a, b1. Calculate K( e) and
F(e) for e = 1,2 explicitly, and write I = /(1) + /(2) in the quadratic form
1 T KU-U T F,whereU= [Ul,U2,U3 ]T .
'2U

ANS .

ql2 ql2
p+-
3 -P+6 0
ql2 2ql2 ql2
-p+- 2p+ -3-
6
-p+ 6 U
ql2 ql2
o -P+6 P+T
rl
+-2 [1 2 1] T + '21 np(a) [u(a) - U oo
]2,

where 1(1) = 1(2) = l.

1
11
3.15. Consider the functional /(u) = - [(U')2 + 2u] dx + 10 [u(l) -11 2 .
o 2
Approximate the minimum of I over the finite element interpolation functions
generated by one quadratic element by using the method of steepest descent
with initial guess u(O) = [0 0 0lT for the finite element nodal solution
U = [Ul U2 u3 f . Perform only two iterations, i.e., compute the values
of u(1) and u(2).
ANS . u(l) = lUll) u~1) u~l)lT = [0 0 O.793650793jT,
u (2) = [Ul2) u~2) u~2)jT
= [-0.024599668 0.233696847 0.793650793]T.
4
One-Dimensional Fourth-Order Equation

4.1. Euler-Bernoulli Beam Equation

For problems involving beams with different types of supports and boundary con-
ditions we use the Euler-Bernoulli beam theory . The governing equation for the
tran sverse deflection u of a beam of length L is

0 < x < L, (4.1)

where b(x) and f( x) are known functions. For a constant value of b we often use
the flexural rigidity El, where E is the modulus of elasticity and I the moment
of inertia of the beam. The boundary conditions depend on the type of support
system used for beams and frames and will be discussed in §4.2.
4.1.1. Finite Element Equation. For a beam we will first deri ve the
finite element equation for a two-element mesh. The weak variational form of Eq
(4.1) is

l
x ~e )

0= W
x,( e)
2
u) ]
-
- l x~e)[dW
-- d (d
b -2 -wf dx+ [
xle) dx dx dx
w-d (b
dx
d2U)]
- X~e)
dx 2 xle)

( d2w d2u [ d (d 2U)


=
l x~e)
xle) dx? dx
)
dx dx
dw d2U] x~e)
b - - -2w f dx + w- b -2 - - b -2
dx dx xle)

P. K. Kythe et al., An Introduction to Linear and Nonlinear Finite Element Analysis


© Springer Science+Business Media New York 2004
76 4. ONE-DIMENSIONAL FOURTH-ORDER EQUATION

= lx~e) (b d2 w2 d2u2 _ Wi) dx _ W (x(e») Q(e) _ [_ dW] Q(e)


Xl
(c) dx dx 1 1 dx Xl(e ) 2

- W ( X 2e») Q(e)
3 -
[dW]
--d
X <c )
Q(e)
4
x2

= b(w ,u) -l(w) , (4.2)

where w(x) E C 2(O , L) denotes the test function , and

2
Q(e)
1
= [~
dx
(b d
dx
U) ] 2 (e)
,
X=X 1

2 (4.3)
Q(e) = _ [~ (b dU) ] ,
3 dx dx? X= X 2
<c )

The bilinear and linear forms are

b(w, U) = lX~C) (b ~2~ ~2~) dx,


xi c
) x x

l(w) = lXl
(e)
x 2

<e)
ui] dx + w ( x (e»)
1
Q(e) + [_ dW]
1 dx (c)
Xl
Q(e)
2
(4.4)

+ W (X (e») Q(e) + [_ dW] Q(e).


2 3 dx X2
<e) 4

For a uniform notation, let ~~ = -8 denote the strain, Q~e) and Q~e) the shear
force and the bending moment, respectively, at the node x ~e), and Q~e) and Q~e) the
shear force and the bending moment, respectively, at the node x~e) . It is obvious
from the variational form (4.2) that the interpolation functions are of the class C 3
so that the shear forces Q~e) and Q~e) are nonzero, and they are such that u satisfies
the end conditions on an element n (e); thus,

(4.5)

Since there are two essential boundary conditions , we take u and du f d» as pri-
mary variables at each node. Thus, the essential conditions are included in the
interpolation functions (2.16). The natural boundary conditions are included in
the weak variational form (4.2); they are present in the load vector F(e), defined
below in (4.7). We choose a four-parameter polynomi al of the type (2.13), and
use the Hermite interpolation functions (Pi. i = 1,2, 3,4, defined by (2.16) in the
global mode (or by (2.17) in the local mode). Then, taking the approximation
n
u( x ) ;::::: L u~e) ¢~e) (x) , and replacing in Eq (4.3) the deflection function u by
i= l
4.1. EULER-BERNOULLI BEAM EQUAT ION 77

¢~e)(x) and the weight w by ¢;e)(x), i , j = 1,2,3,4, we obtain the finite element
equation for an element n (e) as

(4.6a)

or in matrix form as

4
~ K (e ) u(e ) - F (e) = 0 . 1 2 34
L..J tJ t J ' J = , , , , (4.6b)
i= l

where

(4.6c)

If we take b = const, and f = const over the element n (e) , then the matrix K (e)
and the vector F( e) become

(4.7)

4.1.2. Boundary Conditions. The different types of commonly used


support systems for a beam are given below in Table 4.1. They determine the
essential boundary conditions at the support point of a beam . Recall that if u is
specified at a boundary, the boundary condition is essent ial, whereas if u is not so
specified , then a natural boundary condition is satisfied at equilibrium. Sometimes
a boundary condition formed as a linear combination of the function and its de-
rivative is used. If a first-derivative term appears in a boundary condition with the
second-order Sturm-Liouville problem, such a boundary condition is always natu-
ral. If only the dependent variable u appears, the boundary condit ion is essential.
78 4. ONE-DIMENSIONAL FOURTH-ORDER EQUATION

Table 4.1. Support Conditions for a Beam.

Essential conditions Natural conditions


Type of Support (Primaryvariables) (Secondary variables)

( Free
None All prescribed
I ·X

.' Z
Clamped

Roller
'x
u =0
w =0
8=0
None prescribed

Horizontal Horizontal force and


w =0

JIll 'x bendingmoment


prescribed

( Roller
Vertical u =0
Transverse force and
moment prescribed
~ 'x

( Pinned u =0 Moment prescribed

1>+ ·X W =0

To explain the nature of displacement and force boundary conditions in various


type of beams we will discuss some examples.
EXAMPLE 4.1. Let b = El, and 1 = 10 const. We consider a two-element
model presented in Fig. 4.1. There are three global nodes and a total of six global
primary and six secondary variables in this model. Then the local U i and the global
U, are connected by the following relations, which are obvious from Fig. 4.1.

U 1( l ) --
U u(l ) - e (l ) - U u( l) - u (l ) - U (2) - U
1, 2 - 1 - 2, 3 - 2 - 1 - 3,

U~l) = e~l ) = ei ) = U4 ,
2
u~2) = u~2) = Us, u~2) = e~2) = U6 .

Since there are three nodes and two primary unknowns at each node, the global
4.1. EULER-BER NOULLI BEAM EQUATION 79

stiffness matrix for this model is given by

K(l) K(l) K(l) K(1)


11 12 13 14 0 0
K(1) K(1) K(1) K(1)
21 22 23 24 0 0
K(1) K(1) K (1) + K (2) K( 1) +K(2) K (2) K(2)
31 32 33 11 34 12 13 14 (4.8)
K= K(1) K(1) K(1) +K(2) K(1) + K(2) K (2) K (2)
41 42 43 21 44 22 23 24
K(2) K(2) K(2) K(2)
0 0 31 32 33 34
K(2) K(2) K(2) K(2)
0 0 41 42 43 44

and the global force vector by

1)
f?) Qi
f J1) Q~1)
f~1) + f?) Q~1) + Qi2)
F = + (4.9)
f~ 1 ) + fJ2) Q~1) + Q~2)
f~2) Q~2)
fF) Q~2)

/0

Fig. 4. 1. Linear Two-Element Model.

The connectivity matrix C for this two-element mesh is

(4 .10)

Hence, the finite eleme nt equation for this model is

6 - 31 -6 -31 0 0 U1
-31 2 12 31 12 0 0 U2
2EI -6 31 6 +6 3 1 - 31 -6 - 31 U3
13 -31 12 31- 31 212 + 212 31 12 U4
0 0 -6 31 6 31 Us
0 0 -3l l2 3l 212 U6
80 4. ONE-DIMENSIONAL FOURTH-ORDER EQUAT ION

6
-I
fol 6+6
12 I -I + (4.11)
6
I

where I = l (l ) = 1(2) . Now, suppose that the beam is clamped at the node 1. Then
the boundary conditions are vi I ) = u~l ) = Ul = 0, and V~l ) = ep ) = U2 = O.
Also, QP ) and Q~l) at node 1, which correspond to the shear force Fo and the
bending moment M o, are unknown. At node 2 there are no shear forces and bending
mom ents , and therefore, we take Q~1) + Q~2) = 0 and Q~1) + Q~2) = O. At node
3, the shear force FL and the bending moment ML are given as Q~2) = -FL and
Q~2) = -ML. Hence, using these boundary conditions in Eq (4.11) , we solve the
system and obtain

As a numerical example, we take EI = 375 x 10 5 lb ft 2, FL = 9000 lb,


M L = 0, fo = 10 lb ft, and L = 10 ft, so that I = 5 ft. Substituting this dat a into
(4.12) we obtain

U3 = -0.0251181, U4 = 0.00903889,
U5 = -0.803333, U6 = 0.0120444,
Fo = 9100, M o = - 90500.

It can be easily seen that at node 1 the shear force Fo and the bending moment Mo
satisfy the equilibrium conditions of the beam:

Fo - FL - 2fl = 0, and M o + 2lFL + 2f l2 - M L = O.

The finite element solution at any point on the beam is given by


4.1. EULER-BERNOU LLI BEAM EQUAT ION 81

where, from (2.17) we have

¢~l) = 3(y) 2 _ 2(y) 3, ¢~l) = l [(yf -(yf] ,


¢ (2)
l = 1 - 3(1 - T
X) 2 - 2 ( 1 - T
X)3 ' ¢2(2) X) ( 2 - T
=l (1- T X) 2'

¢~2) = 3 (1 - T) 2 + 2( 1 - T) 3, ¢ ~2) = l [( 1 - T) + (1 - T)
3 2] .

The exact solution for 0 :s :s L is given by


X

f FL ML 2 (f L3 h L2)
El u(x) =-24(L- x ) - 6 (x-L) +T X - - 6- +-2- x
4 3

fL 4 hL3
+2"4+-6- '
f F f L3 FLL2
"2L (X - L ) - MLx + -6- + -2- '
3 2
EIB(x) = - 6 (x - L) -

M(x) = -f (x - L)2 + h(x - L) + M L· (4.14 )

The finite element and the exact solutions for u(x) are compared in Table 4.2 ,
which also contains exact solutions of B(x) and M( x).

Table 4.2 . Values of the Deflection and Other Quantiti es .

x UFE (X) (4.13) uexact (x) (4.14) Bexact (x) Mexact(x )


0.0 0.0 0.0 0.0 -90500.
1.0 -0.001166 - 0.000972 0.00191 - 81405.
2.0 -0.004502 -0 .003752 0.00361 - 72320.
3.0 -0.009768 -0 .008141 0.00512 - 63245.
4.0 -0.016721 -0.01 3934 0.00642 - 54180.
5.0 -0.025118 -0 .020932 0.00753 -45125.
6.0 -0.034718 - 0.028932 0.00843 - 36080.
7.0 - 0.045281 - 0.037734 0.00913 -27045 .
8.0 -0.056564 -0.047137 0.00963 -1 8020.
9.0 -0.068328 -0 .056941 0.00993 -9005 .
10.0 -0.080333 -0.066944 0.01003 O.•

EXAMPLE 4. 2. Consider the problem of deflection of a beam of length 10 m


supported in the middle by a horizontal roller, presented in Fig . 4.2 . The beam is
subjected to a uniform load of fa = 5000 N.m., and boundary conditions u(O) = 0,
2 2u
u(5) = O, u'(O) = 0, -dd [EI d U]
2 = FL = O,and [EdI -2 ] = ML = O.
X dX x = l O dX x=l O
82 4. ONE-DIMENSIONAL FO URTH-ORDER EQUATION

Using the data EI = 2.5 x 106 N.m 2 , and I = 5 m, we solve the system (4.11)
which in this case gives

2
where Q = Q~l) + Qi ). The solution of this system is U4 = 0.0260 417 , U5 =
-0.286458 , U6 = 0.0677083, and h = -0.703125 , M L = 0.911458, Q =
0.078125.
To determine the exact solution, we use Macauly 's method for an overhanging
beam . The bending momen t is given by

subject to the above essential conditions, where R is the reaction from the support.
Thus, we have
du lox 3 R 2
EI dx = - 6- +"2 (x - 5) + cr .
'( )
Since, u 10 = 0, we get Cl
lOla
= - 6- -
25R
2 .
' Integrating .
again , wc have

lox 4 R 3
EI u = 24 + 6" (x - 5) + C1X + C2 · (4 .15)

2 3
I I
Sm Sm

Fig . 4.2. 2-Element Mesh .


4.1. EULER-BERNOULLI BEAM EQUATION 83

To determine C2 we use the conditions u(5) = 0 = u(10), which gives two


equal values of C2:

10410 _ 53R -10 (10 10 _ 25R) 410


C2 = _ = _ 5 _ 5 (1010 _ 25R) .
24 6 6 2 24 6 2

Solvi
vmg theng
. htt sid . we find that R = ---;W-'
SI e equation 38310 whiIC h m
. turn grves
. C2.

566510 902510
With our data we find that CI = -----:is' and C2 = ----U;-. Then from (4.15)
the exact solution for u(x), x = 0(1)10, is
u(x) = {-14.8302 , -7.27867, -2.79229, - 0.60808, 0.03495, -0.104167,
-0.26845, 0.29708 , 2.34546, 6.62767, 13.8927} . •
EXAMPLE 4.3. Consider the problem of deflection of a beam of length 16 m
and flexural rigidity E1 = 18.5 x 107 N.m, which is loaded by a constant weight
of 600 N/m from x = 0 to x = 8, and a point load 2500 N/m acts at x = 12; the
end point x = 16 undergoes a bending moment of 9000 N.m. We will solve this
problem by two methods.

2500 N
9000 .m
1
I ) I X
()
U 1,U2
(a) +l.-- -=-- -t
I
..J

1 12m
1----------+----------1
8m 8m

U 1,U2 U 3,U4 U7 ,U8


U5 ,U 6
(b) .'-
1

-=-
2
---=•
3
----.J

4
1 - - - - - - - - - - +I - - - - - - I+ - - - - -I
8m 4 m 4 m

Fig. 4.3. (a) 2-Element Mesh, (b) 3-Element Mesh.

METHOD 1. The 2-element mesh is presented in Fig. 4.3(a). The load on the
element 2 is defined by I(x) = 10 {; (x - xo), where Xo = 12, {; (x - xo) is the
Dirac delta function, and 10 = - 2500. This load vector is represented by l i(2 ) =
- 10 cPi (xo) with Xo = 12, or in the local coordinates, by IF) = - 10cP (io) with
io = 4. Thus, 1(2) = -10 ui
2) IJ2 ) 1~2) 1~2)r = -2500 [0.5, -1,0.5 , l r =
[-1250,2500, -1250, -2500]T. Using the formulas in (4.7), the global stiffness
84 4. ONE-DIMENSIONAL FOURTH-ORDER EQUATION

matrix and force vector are given by


6 -24 -6 -24 o 0
- 24 128 24 64 o 0
7
K = 37 X10 -6 24 12 0 -6 -24
512 -24 64 o 256 24 64
o o -6 24 6 24
o o - 24 64 24 128

-2400
QF)
3200 Q~1)
-2400 - 1250 Q~1) + Q~2)
F= -3200 + 2500 + Q~I) + Q~2)
-1250 Q~2)
2500 Q~2)

= 0, Us = 0, Q~I) = 0, Q~I ) + Q~2) = 0, Q~I) +


The boundary conditions are UI
Q~2) = 0, and Q~2) = 9000. Thus, solving the system K U = F, we find that
U2 = 0.000316757, U3 = -0.0013982, U4 = -0.00004, U6 = -0 .000171892,
Q~I ) = 0.0000197973, and Q~2) = 0.0000196622 .

METHOD 2. A 3-elementmesh is presented in Fig. 4.3(b). Using the formulas


(4.7), the stiffness matrices and the force vectors for the three elements are given
by
3/128 -3/ 32 - 3/128 -3/32]
(1) _ - 3/32 1/2 3/32 1/4
K - EI [ - 3/128 3/32 3/128 3/32 '
-3/32 1/4 3/32 1/2
3/ 16 -3/8 -3/16 -3/8]
( 2) _ -3/8 1 3/ 8 1/2 (3)
K - EI -3/16 3/ 8 3/16 3/8 = K ,
[
-3/8 1/2 3/8 1

F(I) = -600 { ~8 Q\I~E\2) } ,


8
} +{
Q~I )

(2) _
Q~2)
Q~2)
}
F (3) _
{ Q~3)
Q~3)
}
F -
{ Q3(2 ) ' -
Q3(3) '
Q~2) Q ~3)

Then, using the boundary conditions UI = 0, U7 = 0, Q~I) = 0, Q~1) + Q~2) = 0,


Q~1) + Q~2) = 0, Q~2) + QP) = 0, and Q~3) = 9000, and solving the system
4.1. EULER-BERNOULLI BEAM EQUATION 85

K U = F, we find that U2 = 0.000316757, U3 = -0.0013982, U4 = -0.00004,


Us = -0.000867027, U6 = - 0.000209189, Us = -0.000171892, Qi 1) =
0.0000197972, and Q~3) = 0.0000196622. •
EXAMPLE 4.4 . Consider the problem of the deflection of a beam of length
14 m, which is clamped at x = 0 and subjected to a variable load of the form
i(x) = a + box 2 for 0 ::; x ::; 8 m, where a = io, and bo = - io /(l(1))2, and a
point load of 2000 N at x = 14 m. We will consider a 2-element mesh, as shown
in Fig. 4.4.

2400

8m 6m

Fig. 4.4. 2-Element Mesh.

For the element 1, the matrix K(1) is given by (4.7) with l(1) = 8. Let a = io
and b = - io/ (l(1))2. Then the components ofthe force vector f(1) are determined
by computing the values of the integral

where (/Ji(x), i = 1,2,3, are defined in (2.10). We take io = 1000. This gives

-10400/3 }
f (1) = 12800/3
{ -5600/3 .
-3200

For the element 2, f(2) = {O}, and the matrix K(2) is defined by (4.7) with l(2 ) = 6.
86 4. ONE-DIMENSIONAL FOURTH-ORDER EQUATIO N

Thu s, the stiffness matrix and the force vector are


3/1 28 - 3/ 32 - 3/1 28 - 3/ 32 o o
- 3/ 32 1/2 3/ 32 1/4 o o
-3/128 3/ 32 91/11 52 - 7/96 - 1/1 8 -1 / 6
K =EI
-3/32 1/ 4 - 7/ 96 7/ 6 1/6 1/3
o o -1 / 18 1/6 1/1 8 1/ 6
o o - 1/6 1/3 1/6 2/3

-10400/ 3
QP)
12800/ 3 Q ~1)
-5600/3 Q~l ) + Q ~l )
F=
-3200 + Q~l ) + Q ~2)
o Q ~2)
o Q~2)

We use the boundary conditio ns: U1 = 0 = U2 , Q~1)+Qi2) = O ,Q~1 )+Q~2) = 0,


and Qf ) = -2400. Then, using EI = 18.5 x 106 N.m and solving the system
K U = F , we obtain U3 = - 0.0587339, U4 = 0.0122234, U5 = - 0.141415,
and U6 = 0.0145586. The shear force and the bending moment at x = 0 are
Fo = 3466.67, and M o = - 4266.63, respectively. •

4.2. E x ercises

4. 1. Obtain the weak formu lation for the fourth-order equation d~2 (b~:~) -
~ (adU) + cu = f and show that the stiffness matrix and force vector for
dx dx
an elem ent n (e) are given by
-3l(e) -6
~ [-3~(')
-31(' ) ]
K (' ) 2b (o)
2 (l(e))2 3l(e) (l(e)) 2
(l(e))3 -6 3l(e) 6 3l(e)
- 3l(e) (l(e))2 3l(e) 2 (l (e))2
-3l(e) - 36 - 31(0) ]
a(e) [ - 36 3l(e) 4 (l(e)) 2 3l(e) _ (l(e))2
+- - -36
30l(e) 3l(e) 36 3l(e)
-3l(e) _ (l(e)) 2 3l(e) 4 (l(e))2
- 22l(e) 54 131(' ) ]
c(e)l(e) [ -22l156(e) 4 (l(e)) 2 - 13l(e) -3 (l(e)) 2
+ 42(j 54 -13l(e) 156 22l(e) ,
13l(e) - 3 (l(e))2 22l(e) 4 (l (e)) 2
4.2. EXERCISES 87

f (e)l(e)
r (e) = -- [ - 6 lee) - 6 - l(e)f .
12

4.2. Do Example 4.4 when the load is defined by

forO ~ x ~ 8,
for 8 ~ x ~ 14.

For other data , see Fig. 4.5.

600
fix) =/0 r i - xt l } 2 2000

3
Us' u 6
I
8m 6m

Fig. 4.5.
A NS. U3 = 0.0373622, U4 = -0.00837189, U5 = 0.09537 73, U6 =
- 0.0103178, Q ~I ) = 1279.9 999827, and Q~l) = -1280.00003458.

HI NT. r (I ) = - 2l 4 lr .
_ fol [16
60
4 .3. Solve the beam problem for the system shown in Fig. 4.6 and compute the
unknown displacements and moments. The load f is defined by f (x ) =
600 + H (x - 8) (600 - 75x ), where H (x ) denotes the Heaviside unit function .

/ 0= 600 (b/ft

2EI

8 fl 8ft

Fig. 4.6.

HINT. Use (4.7) for K (e) and r (e) , e = 1,2, with b = 2E I for the element
1 and b = EI for the element 2. Take EI = 107 lb.ft" .
A NS. U3 = -0.4608, U4 = 0.896, U5 = - 1.25952, U6 = 0.1024.
88 4. ONE-DIMENSIONAL FOURTH-ORDER EQUATION

4.4. Solve the beam problem for the system shown in Fig. 4.7, and compute the
unknown displacements and moments . The load I is defined by

10 x 2 for 0 < x ::; 6,


I(x) ={ x2
10 (48 - 3) for 6 ::; x ::; 12.

Take 10 = 10, l = 6 m, and EI = 2 x 106 N.m 2 .

!(x)=!ox 2

x
2 3

6m 6m

Fig. 4.7.

HINT. Because of symmetry at node 2, consider the first element only. The

force vector is f(l) = - 1~~3 [4 - l 16 2lf. The boundary conditions


are U1 = 0 = U2 , and the components of Q are zero at node 2.
ANS. U3 = -0.016848, U4 = 0.003888, QP) = 1440, and Q~2) =
-3240.
5
Two-Dimensional Elements

To solve two-dimensional boundary value problems in a domain n c R2, we


partition the domain n into N disjoint simplexes, such as triangles, rectangles and
quadrilaterals. A typical simplex is denoted by n(e), where e = 1, . . . ,N. In this
chapter we will present interpolation shape functions for the linear triangular and
bilinear rectangular elements.

5.1. Linear Three-Node Triangular Elements

We choose the linear function

u = (}:1 + (}:2 X + (}:3 Y, (5.1)

as the interpolation function in a triangular element n( e). Let the coordinates ofthe
nodes of this triangular element be (Xl , Y1), (X2, Y2), and (X3' Y3) (see Fig. 5.1).
We will solve for (}:1 , (}:2 , and (}:3 in the linear function (5.1), satisfying the nodal
conditions U (Xi, Yi) = u~e ) at each node for i = 1,2 ,3 at the three global nodes
of the linear triangular element n( e ). Then
At node 1: u~e) = (}:1 + (}:2 Xl + (}: 3 Y1; At node 2: u~e) = (}:1 + (}:2 X2 + (}: 3 Y2;
At node 3: u~e) = (}:1 + (}:2 X3 + (}:3 Y3 .

Thus, we solve

P. K. Kythe et al., An Introduction to Linear and Nonlinear Finite Element Analysis


© Springer Science+Business Media New York 2004
90 5. TW O-DIMENSIONAL ELEMENTS

and obtain

(}:l =
1 1[(
2!n(e) Ul
e) (
X2Y3 - X3 Y2
)
+ U 2(e) (
X3Y l - XI Y3
)
+ U 3(e) (
XIY2 - X2Y I
)]
,

1
n(e)1 U l Y2
[ (e ) ( ) (e) ( ) (e) ( )]
(}:2= 2I -Y3 +U 2 Y 3-YI + U 3 YI-Y2 ,

(}:3 = 1 )! [U (e) ( X 3 -
2!n(e X2
)
+ U 2(e ) ( X l - X3
)
+ U 3(e ) ( X2 - Xl) ] ,
I

where

In(e)I = area of the element nee) =


1[1
2" 1
Xl
X2
Yl ]
Y2 .
1 X3 Y3

The area of a triangul ar element n(e) is also denoted by A (e) .

Q I£)

1
(xl ' Yl )

Fig . 5.1. A 3-Node Triangul ar Element.

Hence, since U = (}:l + (}:2 X + (}:3 y, we get


3
U = " u (e )", ( e ) (5.2)
L...J t "Pt ,
i =l

where
-i,(e)
'+'",
= art e) + b ee ) X + de)
t t'
Y (5.3)

and
(e) _ XjYk - XkYj
ai - 2In<e)1 '
i = 1, 2,3
(e) Yj - Yk
bi = 2In<e)I ' j = 2, 3, 1 (5.4)
(e) Xk - Xj
k = 3,1 , 2.
Ci = 2In<e)I '
5.2. BILINEAR FO UR-NODE RECTA NGULAR ELEMENTS 91

The functions rP~e) satisfy the conditions rP~e) (x Je)) = Oij, and L rP~e\x) = 1,
i= l N
where Oij denotes the Kronecker delta , which is equal to 1 if i = j and zero
otherwise. Note that a~e) + a~e) + a~e) = l.

EXAMPLE5. 1. Consider the linear triangular element S1(e) shown in Fig. 5.1.
Let (X l,yt} = (2, 2), (X2,Y2) = (5,3), and (X3,Y3) = (3,6) . Then we have
21S1(e)1 = 11, and from (5.4) we get

(e) X2Y3 - X3Y2 21


a1 = 21S1(e)1 = 11'
(e) X3Yl - XIY3 6
a2 = 21S1(e)1 = - 11'
a
(e)
-
XIY2 - X2Yl
---
4
3 - 21S1(e)I - n'
(e) Y2 - Y3 3 b(e) _ Y3 - Yl -±- b(e) = Yl - Y2 1 = _ ~
bl = 21 S1 (e)I = -11' 2 - 21S1(e)j 11 ' 3 21S1(e) I 11 '
c(e) _ X3 - X2 __ ~ (e) Xl - X3 1 c(e) - X2 - Xl - -3
1 - 21S1(e)1 - 11' c2 = 21S1(e)! = -11' 3 - 21S1(e)1 - 11

Then from (5.3) we obtain the interpolation functions

rP~e) = 1\ (21 - 3x - 2y), rP~e) = III (-6 + 4x - y), rP~e) = 1\ ( - 4-x+ 3y).

The values of a~e) , b~e) and c~e) , i = 1,2 ,3, can also be written in matrix form as

a (e) = [X2Y3 - X3Y2 x3Y l - Xl Y3 Xl Y2 - X2Yl] /(2I n (e) I)


= [21 - 6 -4 ] / 11,
b (e) = [Y2-Y3 Y3 -Yl Yl -Y2 ]/(21S1(e)I) = [-3 4 -1 ] /11 ,
c (e)=[ X3- X2 Xl -X3 X2 -Xl ]/(21S1(e)I) = [-2 - 1 3 ] /11.

These matrix forms will be useful later in Chapter 11. The graphs of the local
shape functions rP;e), i = 1,2 ,3, are presented in Fig . 5.2 .

5.2. Bilinear Four-Node Rectangular Elements

A bilinear rectangular element n (e); {O < X < a,O < Y :::; b} is shown in Fig . 5.3.
We choose a bilinear function

U(X, y) = Q:l + Q:2X + Q:3 Y + Q:4 X Y, (5.5)


92 5. TWO-DIMENSIONAL ELEMENTS

and satisfy the nodal condition at each global node of a bilinear rectangular element
n(e) . Thus, using the coordinates of each node, we have

At node 1: u~e) = u(O, 0) = 0'1,


At node 2: u~e) = u(a, 0) = 0'1 + 0'2 a,
At node 3: u~e) = u(a , b) = 0'1 + 0'2 a + 0'3 b + 0'4 a b,
At node 4: u~e) = u(O, b) = 0'1 + 0'3 b. Hence, we solve

and obtain

0'1 }
0'2 1
o
~: = ab ~a -1
[ ab
-b ~ ~ ~] {:1:;}
0 (e)'
~1 :~e)
{
1

ell (c)
2

6 6

Fig. 5.2. Plots of the Local Shape Functions.


5.2. BILINEAR FOUR-NODE RECTAN GULAR ELEMENT S 93

Substituting these values of a i into (5.5), we get

u(x , y) = [ 4>~e ) 4>~e) 4>~e) (5.6)

where

4>~e) = ~ (1 - *) ,
*.
(5.7)
e)
4>i = (1- ~)
Note that the functions 4>;e) satisfy the two conditions mentioned at the end of §5.1.

4 3
4 3

n(e)
b

2
1 2

a
Fig. 5.3. A 4-Node Rectangular Element.

To derive dimens ionless shape functions for a 4-node linear rectangular element
n (e),which are useful in solving elliptic boundary value problems , we proceed as
follows. The geometry of such an element is shown in Fig. 5.4. Since there are a
e)
total of four degrees of freedom, one at each node, we denote them as ui , u~e),
u~e), and ui e). Using a bilinear interpolation of the form (5.5), which is linear in
x and y and bilinear in xy, we determine the four unknowns ai, i = 1, 2, 3,4, by
requiring that

After substituting the values of ai determined from (5.8), we find that

4
u(e)(x , y) = L u~ e) 4>~eJcx , y), (5.9)
i =l
94 5. TWO-DIMENSIONAL ELEMENTS

where

-I,(e) ( (X2 - x ) (Y3 -


) _ y) -I,(e) () _ (Xl - X}( Y3 ~ y)
X,Y
- (X2 - Xl ) (Y3 - Y - (Xl - X2 ) (Y3 -
'f' l
Yl) , 'f' 2 X,
Y2) '
(5.10)
-I,(e) ( ) _ (Xl - X) (Yl - y) -I,(e) ( ) _ (X - X2)(Yl - y)
va X, Y - ( )( , 'f' 4 X, Y - .
Xl - X3 Yl - Y3) (X4 - X2 ) (Yl - Y4)

The graphs of these four functions are shown in Fig. 5.5. It can be easily verified
8u( e)
7iY is a linear function of x . The
8u(e)
that ~ is a linear function of y, whereas
shape functions (5.10) can be represented in terms of the local coordinates ~ and
1], shown in Fig. 5.4, by

-I,(e ) ( c ) = (a - ~)(b -1])


vi <" ,1] 4ab '
««
'f' 3 <" ,1]
)= (a + ~) (b + 1])
4ab '
(5.11 )

If we introduce the nondimensional coordinates s = ~ /a, t = 1]/b, the shape


functions (5.11) become

-I,(e) ( ) _ (1 - s)(l - t) -I,(e) ( t)= (l+ s)(l-t)


'f' l S, t - 4 ' 'f'2 s, 4 '
(5.12)
A,(e) ( (1 + s)(l
) _ + t) A,(e ) ( (1 - s)(l
) _ + t)
'f'3 s, t - 4 ' 'f' 4 S, t - 4 .

The se are also known as the isoparam etri c form of the shape functions, which
are defined on the unit square -1 ::; S ::; 1, -1 ::; t ::; 1. For more information
about isoparametric elements , see Appendix D.

5.3. Global Shape Functions

The flobal shape functions for the elements n (e) , e = 1, .. . , N , can be defined
similar to the one-dimensional case as presented in §2.2. Thus , a global shape
function at a global node i is denoted by <Pi' It is identical to a local shape function
¢je) only if the local node j of n (e) is the global node i , and <Pi = 0 elsewhere.

EXAMPLE 5 .2 . Consider the mesh of one linear triangular and one bilinear
rectangular element, as shown in Fig. 5.6. For the triangular element n (l ) we have
In (l) I = 6, and

1
2
- ~]
2 '
b (l ) = [_ ~ ~
3 6
~]
2 '
c(1 ) = [0 - ~ ~]
4 4'
5.3. GLOBAL SHAPE FU NCTIONS 95

which yields the shape function s

¢/l ) - 1 _:: (1) 1 x Y


1 - 3' cP3 = - -2 + -6+4- .

For the rectangular element n (2) , we use formulas (5. 10) and obtain

1
cP~2) = 11 (6 - x )( 4 - y), cP2
(2)
= - 12 (3 - x) (4 - y),
2
cP~2) = -11 (3 - x) y , cP~2) = -11 (x - 6) y .
2 2
11
4 (x 4 ·Y4 ) (x 3'Y3) 3

4 3

(xo,Yo)
. a
.
I 2

Fig. 5.4. Geometry of a 4-Node Linear Rectangular Element.

Fig. 5.5. Shape Functions cP~e) (x, y), i = 1,2, 3, 4.


96 5. TWO-DIMENSIONAL ELEMENTS

The global shape functions <1>2 and <1>3 are defined by

for (x,y) E n (1), for (x, y) E n(1 ),


for (x,y) E n(2) , for (x ,y) E n( 2) ,

elsewhere; elsewhere.

(3,4) (6,4)
3 . . . - - - - - - - - - -.. 5
4 3
3

1
(0,2)

2
1 2
2 (3,0) (6, 0)

Fig. 5.6. A Mesh of Two Elements.

5.4. Triangular Coordinates

We will discuss the relationship between rectangular cartesian and triangular coor-
dinates. An oblique coordinate system simplifies the generation of shape functions
for a triangular element (see Fig. 5.7). ~2

Fig. 5.7. Line Element.


5.4. TRIANGULAR COORDINATES 97

In Fig. 5.7, note that ~1 = ( 1/l31, 6 = ( 2/l 32, thus the (6 ,6)-system
is dimensionless. Also note that the coordinates of the point 1 which are (1, 0)
in the (6 , 6)-system will be (l31,0) in the ((1, ( 2)-system. The point P, with
coordinates (x, y) in the rectangular cartesian system, has new coordinates (6, 6),
which implies that

x = X3 + (Xl - X3) 6 + (X2 - X3) 6


(5.13)
y = Y3 + (Y1 - Y3) 6 + (Y2 - Y3) 6 ,

where (Xl , yI) , (X2,Y2), and (X3,Y3) are the rectangular cartesian coordinates of
the points marked 1, 2, and 3, respectively, in Fig. 5.7. Eqs (5. 13), after solving
for 6 and 6 give
~1 = a~e ) + b~e) x + c~e) y ,
(5.14)
6 = a~e ) + b~e)x + c~e) y,

where

a~e) = (XjYk - XkYj ) /( 2A(e)), b~e) = (Yj - Yk) / (2A(e)),


c~e) = (Xk - Xj) /( 2A (e)), i = 1,2 ; j = 2, 3; k = 3, 1; (5.15)
A (e) = area of the triangle (123).

Note that Ai = area of the triangle bounded by side i (i = 1, 2, 3) and P . Also,


l16 is the distance from side 1 along the normal toward node 1. This implies that
area of the triangle (32P ) Al
6 = area of the triangle (312)
= -
A (e) .

Similarly, 6 = :~) '


and 6 = :~)'
Thus, 6 + 6 + 6 = 1, which shows that
the triangular coordinates ~ b 6, and 6 are inter-dependent.

-,
-,
,,
, -,

1
( 1,0,0)
----;;1--------------------+ x
Fig. 5.8. Area Coordinates .
98 5, TWO-DIMENSIONAL ELEMENTS

(6,6,6) are linearly dependent since 6 + 6 +


The three area coordinates
6 = 1. Thus, although 6, 6, 6 are not linearly independent, it is convenient
to use all three coordinates (6 ,6, 6) while generating shape functions, where
6 = 1 - 6 - 6 . Hence, the relation between the rectangular cartesian coordinates
(x, y) and the triangular coordinates (6,6, 6) is given by
x = x16 +x26 +x36,
y = Y16 + Y26 + Y36 ·
Alternatively, after solving for 6,6, and 6, we obtain

{6}~23
1
= 2A( e)
[X2Y3- X3Y2
x 3Y1 - X1Y3
Y3-Y2
Y1 - Y3
X2- X3]
X3 - Xl
" x1Y2 - X2Y1 Y2 - Y1 Xl - X2

= 1
2A(e)
[:1:; :1:; ~1:;] {~},
(e) b(e) (e) Y
a3 3 c3
where a~ e), b~e), and c~e) are defined in (5.15) . Also, for an element n(e), integrals
of polynomials in 6, 6 , and 6 are given by

J
i j _ t".).
'I
(e)
1-D Case : ~1~2ds- (i+j+1),(l ), (5.16a)

J
" 'I k l
2-D Case : cicickdA = t .) . . (2A(e))
"1"2"3
t+)+ k + 2)1.
(" ' (5,16b)

J ci ci c k (:1 dV = " kl l'


'f
t .),. . (6v(e ))
3-D Case:
"1"2"3"4 ("
t +) + k + l + 3)1. ' (5,16c)

where l (e) is the length, A (e) the area, and V( e) the volume of the element n(e) •
One should be careful in transforming the coordinates in the new system since ~i
are not linearly independent. For example, in 2-D transformations we should use
the oblique system of Fig. 5.7, where 6,6 are independent. For this system

JJ f(6,6) dA = 2A(e) 1[1 1 1


-
6
f(6 ,6) d6] d6 · (5.17)

5.3. Consider the integral

L
EXAMPLE

lmn = xmy n dxdy.

We will use formula (5.16b) . Recall that 6 + 6 + 6 = 1, and ~i = Ai/A(e) ,


i = 1,2,3, are dimensionless area coordinates (see Fig. 5.8), where A(e) denotes
the area of the triangle 1:::.. Then

10 0 = L dxdy = A(e),
5.5. SHAPE FUNCTIONS ON THE SIDES OF A TRIANGLE 99

(5.18)

where
_ Xl + X 2 + X 3 + Y2 + Y 3 •
X = --=----=--....::.
3
_
y--
Yl
3 .

5.5. Shape Functions on the Sides of a Triangle

We derive linear shape functions for each ofthe sides of a triangle (Fig. 5.9). These
functions are useful in computing the line integrals of the type In «) qn (s) ¢~e) ds,
which arise in the determination of the quantit ies Q~e).
5.5.1. Linear Shape Functions on the Sides of a Triangle. Let
(s, t) denote a coordinate system with origin at the node 1 and the s-axis along the
side joining the nodes 1 and 2 (side 1). Then the two coordinate systems (x, y)
and (s, t) are related by

x= a l + (31 s + ')'1 t, Y= a 2 + (32 s + ')'2 t, (5.19)

where the coefficients ai, (3i , and ')'i , i = 1,2 ,3 , are to be determ ined from the
100 5. T WO-DIMENSIONAL ELEMENTS

following conditions:
At Node 1 (s = 0, t = 0): X = Xl , Y = Yl,
At Node 2 (s = a, t = 0): X = X2, Y = Y2, (5.20)
At Node 3 (s = C, t = b): X = X3, Y = Y3.

Fig. 5.9. Geometry ofa Triangle.

The first equation in (5.20), when substituted in Eq (5.19), gives X l = 0:'1 , Yl =


. X2 - 0:'1
0:'2 , and the second leads to X2 = 0:'1 + (3 1 a, Y2 = 0:'2 + (32 a, i.e., (31 = =
a
X2 - X l Y2 - 0:'2 Y2 - Y1
- - - , and (32 = = - - - . Then the third equation in (5.20), after
a a a
substituting in Eq (5.19), yields X3 = 0:'1 + (31 C + ')' 1 b, and Y3 = 0:'2 + (32 C + ')' 2 b,
or using the above results, we get

')' 1 = X3 - 0:'1 - (3 1 C
= -b1 [ X3 - Xl -
X2 - Xl J
C
b

i
a

(~ -I) Xl - ~ Xl] '


= [X3 +

C2 = i
[Y3 + (~ - 1) Yl - ~ Yl] .

Hence, substituting these values into (5.19), we get

x (s,t) = Xl + (X2 - Xl) ~ + [(~ -1) Xl - ~ X2 + X3] ~ ,


(5.21)
y(s, t ) = Yl + (Y2 - yr) ~+ [G - 1) Yl - ~ Y2 + Y3] ~.
Now, we derive ¢~e) for the side 1. At nodes 1 and 2 we have

¢~e) (s) == ¢~e) (s, 0) = ¢~e) (x( s, 0), y(t , 0)) ,


5.5. SHAPE FUNCTIONS ON THE SIDES OF A TRIA NGLE 101

where from (5.21)

S
X(s,O) == X(s ) = Xl + (X2 - xI) - ,
a
s
y(s,O) == y(s) = Yl + (Y2 - Yl) - .
a

For example, by using (5.4) we get

¢~e) (s) = a~e) +b~e) [(1-~) Xl + ~X2] + c~e) [(1 -~) Yl + ~Y2]
= (X2Y3 - X3Y2) + (Y2 - Y3) [(1 - ~) Xl + ~ X2]
+ (X3 - X2) [(1 - DYl + ~ Y2]

= (X2Y3 - X3Y2) + [(X3Yl - XIY3) + (XIY2 - x2yI) ] (1 - ~)


s
+ (X2Y3 - X3Y2) -
a
= { (X2Y3 - X3Y2) + (X3Yl - XIY3 ) + (XIY2 - X2Yl ) } (1 - ~)
= (a~e) +a~e) +a~e )) (1-~)
= 1 -~ , because a~e) + a~e) + a~e) = 1,
a
s
= 1 - -, (5.22)
h2
where h2 = a is the length of the side 1. Similarly,
¢~e) (s) = a~e) + b~e) X + c~e) Y = ~ = ~ ,

°
a l1 2
", (e ) -_
'1'3 •

Note that these shape functions ¢~e) (s) and ¢~e) (s) are similar to the linear
one-dimensional shape function s in the local mode, defined by (2.2) . We can find
the shape functions for the sides 2 and 3 in an analogous manner. To summarize,
the linear shape functions for the three sides of the triangular element of Fig. 5.9
are:

For side I . ¢(e)(s) =1- ~ ¢(e)(s) =~ ¢3(S) = 0;


. 1 h 2' 2 lvz '
For side 2: ¢~e)(s) = 0, ¢~e) (s) = 1- -i
s , ¢~e\ s ) =
23
i:;
23
(5.23)

For side 3: ¢~e) (s) = -is, ¢~e) (s) = 0, ¢~e) (s ) = 1 - -i


s.
31 31

5.5.2. Quadratic Shape Functions on the Sides of a Triangle.


For the triangular element n (e) (Fig. 5.10) the quadratic shape functions are similar
102 5. TWO-DIMENSIONAL ELEMENTS

to those in (2.10). Thus, the functions ¢~e), i = 1,2, ... , 6, are given by

(S.24a)

(S.24b)

(S.24c)

where lt3, l35 , and l51> respectively, are the length of the sides, and the nodes 2,
4, and 6 are at the midpoint of the sides of the triangular element.

Fig . 5.10 . A Quadratic Triangular Element.


5.5. SHAPE FUNCTIONS ON THE SIDES OF A TRIANGLE 103

EXAMPLE 5.4. Consider the four cases of boundary integrals for the side of
triangular elements shown in Fig. 5.11(a)-(d). We evaluate the boundary integral
Q~e) = r
Jao« )
¢~e\s) qn(s) ds for each case.

3 3
(a) (b)

5
(c) (d)

Fig. 5.11.

CASE 1. See Fig. 5. 11(a). In this case

Since qn(s) = qo = canst on the side 1-2, we get

Then using the shape functions ¢~e) for side 1 from (5.23), we get
104 5. TWO-DIMENS IONA L ELEMENTS

CASE 2 . See Fig. 5.11 (b). In this case

-1
Q i(e) -
aO( e)
S-I,(e)(S ) dS -- -qo
qO -l( ) 'f'i
e
( l)
e
1 0
lee)
-I,(e) dS
S'f'i i = 1,2 , 3.

Th en usin g (5.23), we get

(e) _ qo
Q 1 - [(e) J
r (1 - lee)S) ds -_ -qol6-
l ee)
S
( )
e (_ (e) )
- Qn ,
o

r .s.
l ee) (e)
Q 2(e) = ~
lee) J lee)
d
S S
=~
3
(= Q(e))
21 '
o
(e)
Q3 - _ 0 ( _
-
Q(e))
31 .

C ASE 3. See Fig. 5.11(c). In this case since qn = qo= con st, we have
Q~e) = r
i- :
qo ¢~e)(s) ds , i = 1,2 , .. . , 6.

Th en using (5.23), we get

(e)
Q 1 = qo J
r (1 - [(e) 1 - [(e)2s ) ds = -qol6- = Q (e))
l ee)
S ) (
n ,
( )
e (
o

= qo Jor LW 1 - LW ds = -2q3l - = Q21


lee) ( )
Q2
(e) 48 o ( (e) ) , 8 ) e (

(e) _
Q 3 - -qo J
r l ee)
8 ( 28 )
[(e) 1 - [(e) ds - - 6- - Q 31 .
_ qol e
( )
(_ (e))

o
CASE 4 . See Fig . 5.11(d). In this case we have

Th en using (5.23) , we get

Q (e) _ q1
1 - -
1 (1_ 8) S -
dS_- q1 l12 ( _- Q(e))
Ll. ,

1 + 1(
lt2 1-2 lvz 6

Q 2(e) -- -ql-2 82 dS qo S ) d8
1 - [(e)
(l12) 1- 2 2 -3

_ ql l12 + qOlt2 (_ Q(e) + Q(e))

1
- 3 2 - 21 22 '

Q 3(e) -_ qo 8
dS + 0 -_ qo b ( -_ Q(e))
32 .•
l23 2-3 2
5.6. EXERCISES 105

5.6. Exercises

5.1. Compute the shape functions ¢~e) for e = 1,2 for the mesh of two triangular
elements shown in Fig. 5.12.

2
(0,0)
1 (1,0) 2

Fig. 5.12. A Mesh of Two Triangular Elements.

ANS . Elementl: (xI,yd = (0,0), (X2 ,Y2) = (1,0), (X3,Y3) = (l,b /a) ;
2A(1) = b[o: Then
a (1) = ~ [ap) a~l) a~l)] = ~ [X2Y3 - X3Y2 X3Yl - XIY3 XlY2 - X2Yl ]
a
=b[b/a 0 0]=[1 00];
btl) = ~ [b~l) b~l) b~l)] = ~ [Y2 - Y3 Y3 - YI YI - Y2]
a
=b[ -b/a b/a 0]= [-1 1 0] ;
c(1) = ~ [ C(l ) C(l) C( l ) ] = ~ [X3 - X2 Xl - X3 X2 - Xl ] =
b I 2 3 b
~ [0 - 1 1] = [0 - ~ ~ ]. Thus,
¢~l) (x, y) = a~1} + b~l ) X + c~1} Y = 1 - X, ¢~l ) (x , y) = x - a y/b, and
¢~l) (x , y) = ay/b.

°
Element 2: (Xl ,Yl) = (0,0) , (X2,Y2) = (l ,b/a) , (X3,Y3) = (O ,b/a);
A(2)=b/a;a(2 ) =[1 0];b(2)=[0 1 -1];c(2) =[-~ 0 ~] ;
Thus , ¢ ~2) (X, y) = 1- ay/b, ¢~2) (x, y ) = x, and ¢~2)(x ,y) = ay /b - x .
5.2. Compute the shape functions for the rectangle in Fig. 5.4, with (Xl , YI) =
(0,0), (X2, Y2) = (1,0), (X3' Y3) = (1, b/a), (X4 , Y4) = (0, b/a).
ANS . ¢~e) (x, y) = (1 - x )(l - ay /b) , ¢~e) (x , y) = x (l - ay /b),
¢~e) (x , y) = a x y/b, ¢~e) (x , y) = (1 - x ) a y/b .
106 5. TWO-DIMENSIONAL ELEMENTS

5.3. Compute the global shape functions <Pi, i = 1,2,3,4, for the rectangle
defined in Exercise 5.2.
ANS. <PI = <p~e), <P2 = <p~e), <P3 = ¢~e), <P4 = <p~e) .
5.4. Compute the global shape functions <Pi, i = 1,2,3,4, for the mesh of two
triangular elements shown in Fig. 5.12.
(1)
ANS . <PI = XO (1)<Pl + XO (2)<Pl(2) ' <P2 =
(1)
X0(1 )<P2 ,<P3 =
(2)
X0(2 ) <P3 '
(1) (2)
<P4 = XO(1) <P3 +XO( 2) ¢2 ' where

I if (x, y) E n( e),
Xo<e) (z, y) = { 0
elsewhere.
5.5. Compute:

(a) r <PI <P4 dx dy ,


i.:
», a<P4 dxdy, and
(b)
1Ole)
-a-a
x x

(c) r
Jo(e)
a<Pl a<P4 dxdy .
ay ay
A NS. (a) r
i-.
<PI <P4 dx dy = r
JO (I )
<pF) <p~I) dx dy + r
J O(2)
<p~2) <p~2) dx dy,

and complete the integration by using the shape functions (5.3).


6
Two-Dimensional Problems

We consider second-order partial differential equations which involve the scalar-


valued dependent variable u = u(x, y). A simple example of the equations of this
type is the Poisson 's equation. We will present some examples of solving Poisson 's
and Laplace equations using the linear triangular and bilinear rectangular element.

6.1. Single Dependent Variable Problems

The finite element analysis of two-dimensional boundary value problems involves


the following steps :
1. The boundary value problem is defined in a given domain n by a second-order
partial differential equation that is subject to prescribed boundary and initial
values, and
2. The boundary an of the domain n is a closed curve in most problems.
Thus, the finite elements for the domain n are two-d imensional figures, such as
triangles, rectangles, or quadrilaterals. A mesh of these elements covers the given
domain, and the solution of the boundary value problem is approximated over this
finite element mesh. Obviously, such a solution contains the discretization as well
as approximation errors; the former error is because of the approximation of the
domain, and the latter because of the approximation of the numerical solution.
We consider the general second-order equation
aC l acz
-- -- +cu- f = o, inn, (6.1)
ax ay

P. K. Kythe et al., An Introduction to Linear and Nonlinear Finite Element Analysis


© Springer Science+Business Media New York 2004
108 6. TWO-DIMENSIONAL PROBLEMS

where c and I are known functions of x and y, subject to the prescribed boundary
conditions: u = uonr l , and -(G I n x+G 2 n y) = qn on r 2 , where I', ur 2 = an
and r I n r 2 = 0, and

(6.2)

with aij (i, j = 1,2), are known functions of x and y. Note that if all = a = a22,
al2 °
= = a2b and c = 0, then Eq (6.1) reduces to the Poisson's equation

(6.3)

A mesh of quadrilateral elements in the region 0. is shown in Fig. 6.1 . This mesh
consists of different geometric figures of triangular, rectangular, or quadrilateral
shapes. A typical element is denoted by n( e), and the discretization error is
represented as the portions of the region (shaded in Fig. 6.1) between its boundary
r == an and the boundaries of the elements that lie toward the boundary r.

Discretization Error

Fig. 6.1. Finite Elements.

6.1.1. Local Weak Formulation. Let the essential boundary conditions


u
be prescribed as u = on I' I, and the natural boundary condition as - ( G I n x +
G 2 n y) = qn on r 2, as mentioned above. Let w denote a test function which
vanishes on rl' Then the weak variational form for an element n (e) is

°= /1O ( e) [ax (auax +


-OW all -
au) + -ow (au au)
ay ay a21 -ax + a22 -ay
al2 -
(6.4)
+ cwu - WI] dx dy + ( wqn ds == b(w,u) - l(w) ,
i:
6.1. SINGLE DEPENDENT VARIABLE PROBLEMS 109

where r( e) = an is the boundary of n(e ). The bilinear and the linear forms are
given by

b(w, u) = J~(e) (~~ G 1 + ~~ G 2 + cwu) dxdy ,


(6.5)
l(w) = f'rJo(e) wfdxdy- r
i- wqnds .

6.1.2. Finite Element Equation. Let the function u be approximated


bY ",n h
(e) A,(e )
were
L...-i= l U i 'f/i 'u(e ) ( ) d A,(e) h 1" .
= U Xi, Yi , an 'f/i are t e mear interpolation
i
functions such that ¢;e) ( Xj , y j) = Oi j. The functions ¢;e) are derived in Chapter
5 for triangular and rectangular elements. Note that Eq (6.4) holds for any test
function w . The natural choice for w is the n interpolation shape function s ¢~e) ,
i = 1, ... , n, and each choice of w yields an algebraic relation between all u~ e) .
The i-th algebraic equation in the finite element equation is obtained by substituting
",n
L...-i=I Ui 'f/i lor U and 'f/j
(e) A,(e ) ~ A,(e) ~ . E (64) .
lor w 10 q . , I.e.,

(6.6)

which we write as
n
" K (e ) u (e ) = p( e ) or K (e)u( e) - F (e) (6.7)
~ tJ t J' -,
i= l

where F (e) = f (e) + Q (e), and

(6.8)

Frequently, qn on r (e) nr 2 depends also on the unknown u (e) and it can be split
into two parts , one of which depends on u (e) and is denoted by _K~e)u(e), while
110 6. TWO-DIMENSIONAL PROBLEMS

the other is denoted by f~e) . For example, in heat transfer problems we have
qn = (3 (u - u oo ) on reel n r 2 , which yields Q(e) = f~e) - Kie)u(e) , where
f~e) = r (3u oo 4>(e) ds, and Ki e) =
Jr<elnr2
r
(3u oo 4>(e) (4)(e)) T ds. Note
Jr(e)nr2
(e)
that K ij = K(e) lv if
ji on Y I a12 = a21 ·

Before we solve the system (6.7), we will derive formulas for the evaluation of
the stiffness matrix and the force vector for triangular and rectangular elements,
impose boundary conditions, compute the boundary integrals, and discuss the
assembly of local matrices .
6.1.3. Evaluation of Stiffness Matrix and Load Vector. We
assume that the coefficients aij and c and the function f are constant. Then the
matrix K and the vector f are evaluated as described below.
FOR A TRIANGULAR ELEMENT n( e), the matrix K( e) is composed of four
double integrals

11
H ij =
f1 84>~e) 84>je)
-J::l--J::l-dxdy,

f1
n(el uX uX

22 _
H ij -
84>~e)
8¢;e)
-J::l-
J::l dx dy,
n (e) uy uy

Thus,

(6.10)

The vector fee) is defined by

(6.11)

Note that the integrals in (6.10) and (6.11) are of the type

(6.12)

Using formula (5.18) the integrals I m n for m , n = 0,1,2, have the following
values:

1 3
110 = A(e)i , i="3 LXk'
k=l

10 1 = A (e) Y, III = ~~) (t XkYk + 9iY) ,


k=l
6.1. SINGLE DEPENDENT VARIABLE PROBL EMS I II

(6.13)

Then, using the results in (5.3)-(5.4) we have


a rjJ (e )
a~ =
e
b~ ), and a;
arjJ( e)
= c~ ),
e

which, in view of formulas (6.9), yield

H I1 = A (e ) bee )b ee )
lJ t J'
H 12 = A (e) b ee )c(e )
lJ l J '
H2~ = A (e ) c (e ) c~e)
lJ l J '

H lJ.. = A (e) [ (e) (e )


a l aJ
+ ( a l(e) b(e)
J
+ a (e) b(e» )
l X
A+ (a (e) C(e)
l
+ ct(e) a (e») YA]
J J J

+ b (e) bJ(e ) I 20 + (b t(e) CJ(e ) + bee)


t J Ct
(e ») I 11 + C(e) C(e) 102 ,
t J
T

(e) f ee) A (e) (e) qn A (e)


fj = 3 ' Qi = -3- '
(6.14)
The values of K (e) and f ee) are then evaluated for each element n ee) from the data
(coordinates) of the nodes.

. F OR A R ECTANGULAR ELEM ENTn (e) = { ( x , y ) :O ~ X ~ a , O ~ y ~ b} ,


let a m n , c, and f have the constant values a~~ , c (e), and f (e) , respe ctively, for
m , n = 1, 2. Then, using (5.7) and evaluating the double integrals, we get

H 11 = ~
6a
[~2 -1 1
-2
2
-1
1
2 -2
!!] '
HI' ~ ~4 [!!
- 1
1
- 1
-1 1 -1 '
- 1
1
~1
1 -1 -2 2 1 1 -1 -1
1 -1
-2] 2 1
H 22 ~ -,,-
6b
[ ;
2
-1 -2
-2 -1
-2 - 1
2
1
1
2
H= ab
36
[1
4 2
2 4
1 2
~] ,
b j (e )
f ee) = _a _ _ [1 1 1 1(. (6.15)
4

6.1.4. Evaluation of Boundary Integrals . This is an important aspect


in the process of the finite element method. Consider the boundary integra l

(6.16)

where s is meas ured along the boundary an (e). Note that in the case of two
adjacent triangular elements e and e' (see Fig . 6.2) the function q~e) cancels q~e' )
112 6. TWO-DIMENSIONAL PROBLEMS

on the interface of these two elements . Also, in Fig. 6.2 the function q~e) along the
side k l of the element e cancels q~el) along the side m n of the element e', where
the sides k land m n represent the same interface between the elements e and e'.
This situation can be regarded as the equilibrium state of the internal forces, known
as interface continuity.

(c)

Fig. 6.2. Interface Continuity and Boundary Elements .

Now, if an element nee) falls on the boundary of the region, then the function
q~e) (s) is either known, or it can be computed if not prescribed. In the latter case
the primary variable must be prescribed on that side of the element n( e). Again,
this boundary consists of linear one-dimensional elements. Hence, to evaluate
the boundary integrals we compute the line integrals (6.16). This topic is already
discussed in detail in Example 5.3 . In the case when tl« = {3 (u - u oo ) , see
formulas (B.22)-(B.25) for
e
Ki
) anf f~e) .

6.1.5. Assembly of Element Matrices. This is carried out in the same


way as in one-dimensional problems (Chapter 3). For example, consider a mesh
of two elements shown in Fig. 6.3.

2
3
2

Fig. 6.3. A Mesh of Two Elements.

Let K i j and Kt) denote the global and local coefficient matrices, respectively.
6.1. SINGLE DEPENDENT VARIABLE PROBLEMS 113

Then the following relations hold for the stiffness matrix K:


Global ~ Local
K (1)
K ll 11
K(1 ) - K(1 )
K 12 = K 21 12 - 21
K 13 = K 31 o
K (1) - K(1 )
K 14 = K 41 13 - 31
K (1 ) - K(1)
K 15 = K 51 14 - 41
K (1 ) - K(2 )
K 22 22 - II
K (2 ) - K(2 )
K 23 = K 32 12 - 21
K 24 = K 42 K (1 )
23
+ K 13(2 ) -- K(1)
32
+ K 3(21)
K (1 ) - K(1)
K 25 = K 52 24 - 42
K (2)
K 33 22
K (2 ) - K (2 )
K 34 = K 43 23 - 32
K 35 = K 53 o
K 44 K (1)
33
+ K 33(2 )
K 45 = K 54 K (1) -
34 -
K (1 )
43
K (1 )
K 55 44

The above relations between the global and local nodes can also be obtained
from the connectivity matrix C for the mesh shown in Fig. 6.3. This matrix is

C [1 2 4 5]
=
2 3 4 '
where bold face numbers refer to the global nodes. The vector F = f + Q can be
similarly written. Thus , we have

[
~~~ ~~~ ~~: ~~:
K = K 31 K 32 K 33 K 34
K 41 K 42 K 43 K 44
K 51 K 52 K 53 K 54

F = [F1 F2 F3 F4 F5 f
f F) Q~1 )
f~ 1) + fi 2 ) Q~1 ) + Q ~2)
f~2) + Q ~2)
f ?) +fi 2
) Q~l) + Q~2)
f~1) Q ~1)

The extended form of the matrix K in terms of the local function s Ki~) can be
written by replacing the global forms by the respective local forms given in the
above relations. This is left as an exercise.
114 6. TWO-DIMENSIONAL PROBLEMS

The primary variables are givenby: U1 = u~l), U2 = U~1) = U~2), U3 = u~2),


U4 = U~1) = U~2) , US = u~l) . This enables us to write the finiteelementequation
KU=F.

EXAMPLE 6.1. Consider the mesh of elements shownin Fig. 6.4.

6
3

2 5

Fig. 6.4. A Mesh of Triangular and Rectangular Elements.

The connectivity matrix C is given by

The correspondence betweenthe global and local nodes is as follows:


Global ---; Local

K l1 K(1)
11
K(1) - K (1)
K 12 = K 21 12 - 21
K 13 = K 31 o
K 14 = K41 o
K 1S = K S 1 o
K 16 = K61 o
K 17 = K 71 o
K(l) - K(l)
K 18 = K 8 1 13 - 31
K(l) - K(l)
K 19 = K 9 1 14 - 41
K22 K(l) -
22 -
K(2)
11
K(2) - K(2)
K 23 = K 32 12 - 21
K 24 = K42 o
. .. continuedon next page
6.1. SINGLE DEP ENDENT VARIABLE PROBLEMS 115

Global -... Local


K 25 = K 52 0
K 26 = K 62 0
K 27 =Kn 0
K 28 = K 82 K(2) -
12 -
K(l)
32
+ K 31(2)
K(l) _ K(l)
K 29 = K 92 24 - 42
K 33 K(2)
22
+ K(3)
11
+ K(4)
11
K (4) _ K(4)
K 34 = K 43 12 - 21
K 35 = K S3 0
K 36 = K 63 0
K 37 = K 73 K( 3)
12
+ K(4)
13 -
_ K(3)
21
+ K(4)
31
K 38 = K 83 K(2)
23
+ K 13
(3) -
-
K(2)
32
+ K(3)
31
K 39 = K 93 0
K 44 K(4)
22
+ K(S)
11
K(S) _ K (S)
K 45 = K 54 12 - 21
K(5) - K(S)
K 46 = K 64 13 - 31
K 47 = K 74
K(4)
23
+ K(S)
14
-
-
K(4)
32
+ K 41(S)
K 48 = K 84 0
K 49 = K 94 0
K(S)
K 5S 22
K(S) _ K(S)
K 56 = K 65 23 - 32
K(S) _ K( S)
K S7 = K 7S 24 - 42
K 58 = K 85 0
K S9 = K 9S 0
K(S)
K 66 33
K( 5) - K(5)
K 67 = K 76 34 - 43
K 68 = K 86 0
K 69 = K 96 0
K 77 K( 3)
22
+ K 33(4) + K 44(S)
K 78 + K 87 K(3) -
23 -
K(3)
32
K 79 = K 97 0
K 88 K(I)
33
+ K 33
(2) + K( 3)
33
K 89 + K 98 K(I) -
34 -
K(I)
43
K(I)
K 99 44

Th us, we have

[KlI K19
K 29 ]
K 12 K 13 K 18

K = ~2.1 K 22 K 23 K28
,
K91 K92 K 93 K 98 K 99
116 6. T WO-DIMENSIONAL PROBLEMS

The extended form of the matrix K in terms of the local functions can be KG)
written by replacing the global forms by the respective local forms given in the
above relations. This is left as an exercise to the reader since the size of the matrix
is too large to present here.
. . . (1) (1)
The pnmary vanables are given by U1 = u 1 ,U2 = u 2 = U (2) (2)
l , U3 = u 2 =
U (3) - u(4) U - u(4) - U(5) U - u (5) U - u (5) U - u(5) - u(4) - U(3)
1 - l' 4 - 2 - l ' 5 - 2 ' 6 - 3' 7- 4 - 3 - 2 '
Us = u~3) = u~2) = U~l), Ug = u~1}. This enables us to write the finite element
equation KU = F.
EXA MPLE 6.2. We will solve the Poisson's equation - \72u = 2 over the
triangular region shown in Fig. 6.5, subject to the boundary conditi ons

u(x , y) = 5 -1.5 y + 2.5 y 2 on the boundary joining the nodes 1 and 6 ,

- ay
au
= ° for y = 0, and
au
-ax = ° for x = 0.
We will use a uniform mesh of four equivalent triangular elements. The local
nodes are chosen such that all four triangles are identical in both geome try and
orientation. This reduces the numerical computation significantly in that we com-
pute the required quantities only for one (the first) element. The numbering of
the six global nodes is arbitrary. Note that there are no discretization errors in the
problem.
Using formulas (5.3)-(5.4) we find forthe element n (1) that A(1 ) = 3/16, and

b1 = -4/3 , b2 = 4/3 , b3 = 0,
Cl = 0, C2 = -2 , C3 = 2.
Then , after using the formul as (6.9) and (6.14), we have

K (~ ) = A (e) [b~e) b ~e ) + c( e) c( e)]


tJ t J t J '

(e ) f A(e)
fj = - 3- '
6.1. SINGLE DEPENDENT VARIABLE PROBLEMS 117

This yields

1/ 3 - 1/3
K(1) = -~3 13/12
[ -3/4

where Q(1) _ Q (1 )
j - 11
+ Q 21
(1 ) + Q (1)
13 .

6
3 (0,1)
'\.

,:> /
,,,4 "'),,,4

~-jo
3 1 2
(-0 .75. 0.5) 'A"-2---'------~ 5 (0.0.5 )
3
au =0
ax

2 3 1
2
au -0 (-0.75, 0)
i)y-

Fig. 6,5. Mesh of 4 Triangular Elements.

The connectivity matrix of the finite element domain is given by

Thus, the connectivity matrix K and the load vector F are defined by

K=

sym

o 0
K (2) + K(3 ) 0
31 13
K (2)
21
+ K (4)
12
K(4)
13
K~~) 0
K (2)
11
+ K(333 ) + K 22(4) K (4)
32
K (4 )
33
118 6. TWO-DIMENSIONAL PROBLEMS

F=

Note that U1 = 5, U3 = 39/8, and U6 = 6.

O 2
6
3
Q (e)
21
Load Distribution for an Element
~~4) ~1(2)
(3)
rJz;2) Q32 Q (4)
22
3 Q (2)
Q (4) 13
21

~~I) (3)
Q21

Fig. 6.6. Resolution of the Vector Q.

Since

( I ) _ Q(I)
Q1 - 13'
Q(I)
3
+ Q(2)
2
+ Q(4)
1
_ Q(I)
- 33
+ Q(4)
13 ,
Q(4) _ Q4)
3 - 33 '

(see Fig. 6.6 for the resolution of the vector Q), we use the values from (6.17),
which holds for all four elements , i.e., K(1) = K(2) = K(3) = K(4) and F(I) =
F(2) = F(3) = F( 4) ,* and using the prescribed boundary conditions solve the

*If the local nodes are numbered counterclockwis e in a manner different from that
in Fig. 6.5, the stiffness matrix and the force vecto r must be computed for each element
separately before their assembly and the solution of the system (6.7) .
6.1. SINGLE DEPENDENT VARIABLE PROBLEMS 119

system

1/3 -1/3 0 0 0 0 U1 = 5
-1/3 13/6 -3/2 -1/3 0 0 U2
0 -3/2 13/6 0 -2/3 0 U3 = 39/8
0 0 0 13/12 -3/4 0 U4
0 0 -2/3 -3/4 13/6 -3/4 Us
0 0 0 0 -3/4 3/4 U6 = 6
1
Qi1)
3 Q~l) + Q~2) + QP) = 0
8 3 Q~l) + Q~2) + Qi4)
-
9 1 + Q~2) =0
3 Qi 2
) + Q~3) + Q~4) = 0
1 Q~4)

The values of the unknown quantities are

U2 = 4.67601, U4 = 5.45655, Us = 6.6965.

These values then give

Q~~ + Qi~
(1) (4)
Q13 = -2.55867, = -12.5112, Q 33 = -1.41126. •

EXAMPLE 6 .3 . We use two linear triangular elements over the unit square
(Fig. 6.7) and solve the Poisson's equation

[Pu
Bx 2 +
aB2 u2 = 0, (x , Y) E n = [0,1] x [0.1]' (6.18a)
y

subject to the boundary conditions

au
u(l , y) = 1, ax (0, y) = 0, 0 ~ y ~ 1,
au Bu
-B (x, 1) = 1 - u(x, y), 0 ~ x ~ 1.
(6.18b)
By (x, 0) = 0
Y .

For a linear triangular element n(e) the stiffness matrix is defined by

(6.19)

The connectivity matrix is


120 6. TWO -DIMENSIONAL PROBLEMS

where we have dropped the element numbers. For the element 51(1) , we have
A (I ) = 05 b(l ) = 0 b(l) = 2 b (l ) = -2 C( I) = -2 C( I) = 0 C(I) - 2
"1 ' 2 ' 3 '1 ' 2 ' 3- '
0 = K 21 , K 13 = -0.5 =
(1) (1) (1) (1) (1) (1)
Then K ll =0.5, K 12 = K 31 , K 22 =
(1) (1) (1) "
0.5, K 23 = -0.5 = K 32 ' and K 33 = 1, WhICh yield

0.5 0 -0.5]
K (I ) = 0 0.5 -0.5 .
[ - 0.5 -0.5 1

Similarly , for the element 51(2) , we have A (2) = 0 . 5' b (2


1 ) = 2' b(2)
2 = 0' b3(2) -- -2 ,
(2)
C1 -
-
-2 c
, 2
(2) -
-
2 C(2)
, 3
-
-
0 .
Then K (2)
II -
- 1 ,
K (2)
12
-
-
-0 . 5 -- K 21(2) -
-
(2) (2) (2 ) (2) (2) (2) . .
K 13 = K 31 , K 22 = 0.5, K 23 = 0= K 32 ' and K 33 = 0.5, which yield

~ [ -~5
-0.5
-05]
K(2) 0.5 o .
-0.5 0 0.5

Y
au -1 u
ay--
2 4
3 2
2
CD
au
ax =0 u =1

0
3 I x
1
t =0
3

Fig. 6.7. Two Triangular Elements of the Unit Square.

Hence, after assembly the stiffness matrix is given by

1 -0.5
-0.5 0]
K = -0.5 1 o -0.5
[ -0.5 0 1 -0.5 '
o -0.5 -0.5 1
6.1. SINGLE DEPENDENT VARIABLE PROBLEMS 121

and the force vector f = O.

3 2

II

"------+x

(a) (b) (c)

Fig. 6.8.

In this case Q (e) = f~e) - K~e)u(e) , where f~e) and K~e) can be obtained from
(B.22) and (B.23) directly. Since <PI = 0 on hand 13, and <P i = 1:0~ = ton 1
2

. (h - s) h/2 11- S
(see Fig . 6.8b), and <P2 = OonI2andI3,and <pl = O, <P2 = I h/ = - -,
1 2 i,
sh /2 S
<P3 = hh/ 2 =L-lonll,weget

(i ) }
=t -[O l~ ~] ~~1)
6 { (1)
.

u3
122 6. TWO-DIMENSIONAL PROBLEMS

· '1ar1y, Since
Smu . A,
'1'1 = 0,'1'2
A, h l-,
=- - s A,
'1'3 = S
-l on
l I, and '1'1
A, sh/2 = -,
= -/- s
1 1 l2h 2 l2
(l2-s)h/2 l2- S .
¢2 = 0, ¢3 = l2
h/2
= - l2- (Fig. 6.8c) , we get

2 2

of

(a) (b)

Fig. 6.9.

Hence ,

o]{
ld6
(I) }
~~I)
ld3 u (l )
3
(6.20)
o
1/3 1/6
o]{ (I) },
~~1)
1/6 1/3 u (l)
3

where h = l2 = 1.
6.1. SINGLE DEPENDENT VARIABLE PROBLEMS 123

Now, for the element 2, refer to Fig. 6.9(a)-(c), and note that (PI = 0 on l i : (PI =
shl 2 s (l2-S)hI2 l2-S .
-h i
l2 2
= -,
l2
(h = 0, and ¢3 = l hi
2 2
= - l-
2
on l2 (Fig. 6.9(c )); and
(l3 -s)hI2 l3 - S shl 2 s .
¢1 = l3h l 2 = - b-' ¢2 = bh/2 = G' and ¢3 = 0 on b (Fig. 6.9(b)) .
Thus, we have

Thus,

-
0
0
0
0
0]
0
{U(I)}
utI)
[o 0 0 U(I ) 3

O
o. 0]
0
{U~I) }U(l )

o 0
2
U(l )
3
'

(6.21)
where l2 = l3 = 1. Then , combining (6.20) and (6.21) we obtain

0 0
_ 0 1/3
Q=
[o 0
o 1/6

(6.22)
Thus, since KD = Q = Fb - K, D, we get

(6.23)
124 6. TWO-DIMENSIONAL PROBLEMS

where

K+Kb =
[ -0.5
1
-~ .5
- 0.5
1
a 1
-0.5 -0.5
-0.5
a
-0.5 + a a
1
-~.5 [~
a 1/6
j 1/3
a
a
a
a
+j
1/3

[-~5
-0.5
- 0.5 -0.5
4/3
a
a
1 -0.5 .
-~/3 j
a -1 /3 -0.5 4/ 3
Then, from (6.23) we obtain

1 -0.5
-0.5 4/3
[ -0.5 a
a - 1/ 3

After applying the essential boundary conditions U1 = U4 = 1, this system


simplifies to

1 -0.5 } { U2 } { 0.5 }
[ -0.5 4/3 U3 = 5/ 6 '

which yields U2 = 1 = U3 , and qi 2


) = O.•

EXAMPLE 6.4 . Solve the Laplace equation \72u = a to compute the two-
dimensional electrostatic field due to the electrodes shown in Fig. 6.10.

(0 , b)
6 n
9
4 3 4 3

CD (2) U= V
I 2 I 2 8 (a, 0)
4 3
1
4 3
G) 8)
I 2 I 2 7
4
(0 , -b)

(b)

Fig. 6.10. Electrostatic Field.


6.1. SINGLE DEPENDE NT VARIABLE PROBLEMS 125

PART (A). FOUR IDENTICAL TRIANGULAR ELEMENTS . We consider the


four identical triangular elements (Fig. 6.1Oa), and compute the stiffness matrix and
the force vector for the element ~W ) . Thus, from (5.3)-(5.4) we have A ( 1) = a 2 / 4,
and
b~l ) = -Vi/a , b~l ) = Vi/a, b~l ) = 0,
C~l) = 0, c~l) = -Vi/a, c~l) = Vi/a ,
which leads to

K (l ) = ~ [~1 -;1 ~1] '


2 0 -1 1
Q(l)} (6.24)
F (l ) = {O} + Q~1) ,
{ Q ~l)
(see (8.1) for K(1) . Note that by taking the numbering of the local nodes as in
Fig. 6.1O(a), the above results hold for all four K (e ) and F (e) for e = 1,2,3, 4.
Since the connectivity matrix is

154] 354
c= [ 1 5 2 .
352
The stiffness matrix K and the force vector F are given by
Kg ) + Ki~) KW 0 K g )

K~~) + K~~) K~~) 0


K = K (2)
11
+ K 11
(4) K (2)
13
sym Kg ) + Kg )

K (l ) +K (3)
12 12
K (3) +K (4)
32 32
K (2)
12
+ K (4)
12
K (l ) +K (2)
32 32
K (l )
22
+ K (2)
22
+ K (3)
22
+ K 22(4)
F = [- v - v v v O]T.
Hence, we solve the system (taking a = 1)
1 0

[
o
o
o
1
0
0
~ ~ =:] {g;} ~~
o 1 -1 U4
= {
v
} ,

-1 -1 -1 -1 4 U5 0
126 6. TWO-DIMENSIONAL PROBLEMS

which gives U1 = -v, Uz = -v, U3 = v, U4 = v, and U5 = O. The exact


solution is given by (see Lebedev et al. (1965, Problem 271)

U
vV2
(x , y ) = - ( )
- x +y -
4v ~ {(
- ~ -1
)n . h ml' [a -
sm
V2 (y - x)]
a 1l' n=1 2a
. n1l'[a-V2(y+x) ]
n1l'[a + V2 (y - x)] } sm
. 2a
+smh ,
2a nsinh n1l'

where u(x, y) denotes the electrostatic potential. The above results match with
those obtained from this exact solution .
PART (B). FOUR IDENTICAL RECTANGULAR ELEMENTS. We consider
the four identical triangular elements (Fig. 6.1 Ob),and compute the stiffness matrix
and the force vector for the element 1. This is left as an exercise (Exercise 6.6).
The exact solution is given by (see Lebedev et al. (1965, Problem 271)

2 ~ cosh (n1l'y/a) sin(n1l'x/a)]


-+-
X
uxy
( ) =V [ ~
, a 1l' n=l cosh (n1l'b /a) n '

where u(x, y) denotes the electrostatic potential. The above results match with
those obtained from this exact solution . •
EXAMPLE 6.5 . Find the distribution of de current in a thin rectangular sheet,
if the current is applied by electrodes at the points x = -a, y = 0 and x = a,
y - 0 (see Fig. 6.11). This problem is equivalent to solving the Laplace equation

- (~:~ + ~:~) = 0 in n = {(x,y) : -a::; x ::; a, -b::; y::; b}" (6.25)

subject to the boundary conditions

aUI =0 (6 .26)
ax y =±b '
J
aUI
- _ = f(y) = - - = A if IYI < e,
2c:kh - (6.27)
ax y-±a { 0 If IYI > c.

We will use the following values: Total current J = 1.198 X 103 A; conductivity
k = 0.599 X 108 n- l m- l ; thickness of the sheet h = 0.1 em = 10- 3 m; e = 0.1
6.1. SIN GLE DEPENDENT VARI ABLE PROBLEM S 127

em = 10- 3 m; a = 5 em, b = 3 em, and c = 0.1 em. Then A = -10 A·n·m- 1 .

f------f------H--+ x

7
au -0
ay- 8
Q!! - 0
i)y- 9
(o,b)

CD 8)
t =0
au o
ax =

(o.c)
au 4 (2)
6
ax=A - (1) x
1 2 3
(0,0) (a/2 ,0 ) (a,O)

A Mesh of Four Rectangular Elements

Fig . 6.11 . Distribution of de Current.

Sinee there exists a biaxial symmetry, we will model the first quadrant of the
domain for finite element analysis. To discretize this quadrant we use a 2 x 2 mesh
of four rectangular elements, of which the element 1 is identical to the element 2,
and the element 3 is identical to the element 4. There is no discretization erro r in
this model.
Using formu las (5.7) we find that for the element 1 and 2, each with sides 0.025
m and 0.001 m, the stiffness matrix is given by

m9 ]
623 313
75 150 -75 - 150
[ 6U
623 626 1249 313
K (1 ) - 150 75 - 150 -75 - K (2)
- 313 1249 626 623 - .
-75 - 150 75 150
1249 313 623 626
- 150 -75 150 75
Similarly, for the elements 3 and 4, each with sides 2.5 em and 2.9 em, we have
1057 733

K( 3) _
[
2175
'466
1057
- 4350
- 4350
1466
2175
- 2175
409
- 4350
409 ] -
- 4350
733
-2175 K (4)
- 733 409 1466 1057 - .
-21 75 - 4350 2175 - 4350
409 733 1057 1466
- 4350 - 2175 - 4350 2175
128 6. TWO-DIMENSIONAL PROBLEMS

The connectivity matrix is

12 23 56 4]5
c= [4 5 8 7 .
5 6 9 8

Thus, after assembly the stiffness matrix is given by

626
75'
623
150 a -
1249
150
313
-75' a
623 1252 313 313 1249 313
150 ---rs -75' -75' ----rs -75'
a 313
- 75'
626
"75 a 313
- 75'
1249
- 150
1249
- 150
313
-"75 a 1308
145
567
145 a
313 1249 313 567 2616 567
K= -75' ----rs -"75 145 145 145
a -"75
313
-
1249
150 a 567
145
1308
145
a a a 409
- 4350
733
-21 75 a
a a a 733
-2175
409
-2175
733
- 2175
a a a a 733
- 2175
409
-2175
a a
a a
a a
409
- 4350 o
737 737
-2175 - 2175
o 409
- 2175
1466
2175 o
1057 1057
- 4350 - 4350
a 1466
2175

The vector f = 0, and thus,

·
SInce
Q(I ) _ a Q (I ) + Q( 2) _ a Q(2) _ 0.001 A 0.001 x (-10)
1 - ' 2 1 - , 2 - 2 2 =
-0.005, Q~l ) + Qi3 ) = 0, Q~l ) + Q~2) + Q~3) + Qi4) = 0, Q ~2) + Q~4) = 0,
6.2. EXERCIS ES 129

Q~3) = 0, Q~3) + Q~4) = 0, and Q~4) = 0, we solve the system KU = F, and


obtain Ui = 4.31058 X 10- 4 , U2 = 5.64686 X 10- 4 , U3 = 2.99468 X 10- 5 ,
U4 = 4.45957 X 10- 4 , U5 = 5.34319 X 10- 4 , U6 = 7.96766 X 10- 5 , U7 =
4.60917 X 10- 4 , Us = 3.649 X 10- 4 , and Ug = 4.20937 X 10- 4 • The exact
solution, given by

J [x 2 ~ sinh(mrxjb) mr y]
u(x , y) = - 2kh b +:; ~ n cosh(mrajb) cos -b- + const
(see Lebedev et al. 1965, Problem 190) yields results that differ from the finite
element solution, because of the unknown constant in the exact solution . •

6.2. Exercises

6.1. Consider the generalized Poisson's equation - div (a grad u) = ! on a


domain n, subject to the Dirichlet boundary conditions and obtain the stiffnes s
matrix by using the corresponding functional

I(u) = fl[gr adu .(a gradu)-2U!]dXdY.


6.2. Use axial symmetry to develop formulas for the stiffness matrix and the force
vector for the three-d imensional Poisson 's equation in the cylindrical polar
coordinates
a2u 1 au a2u
- ar2 - ~ ar - az2 = !(r, z) ,
by using triangular elements.
6 .3. Write the correspondence between the global and local nodes, and obtain the
assembled stiffness coefficient matrix and the global force vector for the finite
elements shown in Fig. 6.12.

8
7 f';:--,--------,...-,---------;----;>I' 9
2 2

CD CD
0 (j)
I
I
3 3
Q) @

CD (})
2 2

Fig. 6.12.
130 6. TWO-DIMENSIONAL PROBLEMS

6.4. Write the correspondence between the global and local nodes, and obtain the
assembled stiffness coefficient matrix and the global force vector for the finite
elements shown in Fig. 6.13.

11 12
10 13
4 3 4 3 4

CD 0 (0

2 I 2
6
2 4

Q) (4)
CD
CD CD G)
2 I 2 2
1 5
2 3 4

Fig. 6.13.

6.5. The values at the nodes of a triangular element in the finite element solution
of the Poisson's equation - 'V2u = f are: Us = 270.2, U6 = 235.5, and
Ug = 296.4 (Fig. 6.14). Compute the gradient of the solution, i.e., 'Vu. Also,
determine where the isocline 282 intersects the boundary of this element.

(4,3)
9
3 U9=296.4

---
--- Yo

2
5eL---------'-----------..6
(2, 1.5) (4, 1.5)
U 5= 270.2 U6 = 235.5

Fig . 6.14.

A NS. '"
vU = U6 - Us.1+ Ug
- U6 J• = - 17 .35'1+ 40 . 6'J. Thee iISOCI'me
2 1.5
u = 282 intersects the slant line 19-s at a distance So from node 9. Since 19-s =
296.4_- 270.2
2.5, we get So = (2.5) 296.4 282 = 1. 374 . Al so, thiIS ISOC
rme .intersects
6.2. EXERCISES 131

. II'me at a diistance Yo from node 9; so Yo = ( 1.5) 296.4


the vertica 296.4_- 235.5
282
0.3547. Note that Xo = 0.677. Hence, the global coordinates where the
isocline u = 282 intersects the line 15 - 9 are (2.9,2.177) and the line 16 - 9 are
(4,2.645) .
6 .6. Solve part (b) of Example 6.4 by using a mesh of 4 rectangular elements.
6.7. Solve the Laplace equation - V'2 u = 0 in a region n which is a square with
vertices at (0,0), (1,0) , (1, 1), (0, 1), and is subject to the boundary conditions

au
-ax = 0 on x = 0 and x = 1'
au
ay + u = 2 on y = 1,

u = 1 on y = O.

Use a mesh of one rectangular element and a mesh of 2 triangular elements .


ANS. U1 = U2 = 1, U3 = U4 = 1.5 in both cases . The exact solution is
u(x , y) = 1 + y/2.
6.8. Solve the Poisson's equation - V'2U = 2(x + y) - 4 in a square with vertices
at (0,0) , (1,0) , (1, 1) , (0, 1), and is subject to the boundary conditions

u(O,y) = y2, u(x, 0) = x 2, u(l ,y) =1- y, u(x , 1) = 1- x ,


by using a mesh of (a) four rectangular elements , and (b) four triangular cle-
ments, using the diagonal symmetry.
A NS. (a) U5 = 0.3555 . Exact solution u(1 /2, 1/2) = 0.356.
(b) U4 = 0.25. Exact solution u(1/2 , 1/2) = 0.25.
6.9. Solve the Laplace equation V'2 u = 0 in the first quadrant bounded by the
parabola y = 1 - x 2 and the coordinate axes, and subject to the boundary
conditions u = - 1 + 3x - x 2 on the parabola ; u = 3x - 2 on the x-axis;
and ~~ + 2u = 2y - 7 on the y-axis. Use a mesh of four triangular elements
shown in Fig. 6.15.
ANS. U4 = -1.36. Exact solution is u(x , y) = 3x +Y - 2, which gives
u(0 ,0.64) = -1.36.
6.10. Solve the Poisson 's equation - V'2u = 2 in the domain of Fig. 6.15,
subject to the boundary conditions u = -1 + 3x - 2x 2 on the parabola;
u = -2 + 3x - x 2 on the x-axis; and ~~ + 2u = 2y - 7 on the y-axis.
ANS. U4 = -1.35
6.11 . Solve -V'2u = 2 in the rectangular domain n = {(x ,y) : 0 ~ x ~
a, 0 ~ y ~ b}, subject to the boundary conditions u(a,y ) = 0 = u(x, b),
and ~~ (0 , y) = 0 = :~ (x, 0), by considering the right half triangular region
132 6. TWO-DIMENSIONAL PROBLEMS

because of the symmetry about the line y = ~x and taking a mesh of 4


a
triangular elements as in Fig . 6.4, with a = 3 and b = 4 cm.
7 8 9

y
CD 0
(0, 0.64) 4 ik--""'---""""" 5
4 6

CD CD
-..---~-__- - + x
2 3
(0.6, 0) (1, 0)
1 2 3

Fig . 6.15. 4 Triangular Elements. Fig. 6.16. 4 Rectangular Elements

ANS . U1 = 5.93023, U2 = 4.43023, U3 = 3.03488; Qg) = -3.95348,


Q~~) + Q~~ = -7.0465, and Q~~) = -1.0.

6.12. Solve Exercise 6.11 for a mesh of 4 rectangular elements for a = 3 and
b = 4 em (Fig. 6.16) .
ANS. U1 = 5.68911, U2 = 4.87968, U4 = 5.05078, Us = 3.72635;
Q~2) = -4.27627, Q~2) + Q~4) = -6.24698, Qi3) = -3.39996, Q~3) +
4)
Qi = -4.96076, Q~4) = -2.79387.
6.13. Solve the Laplace equation - V'2 u = 0 in the region {(x , y) : y = 4 -
x 2 , 0 :S x :S 2}, which is in the first quadrant bounded by the parabola y =
4 - x 2 and the coordinate axes. The boundary conditions are: u = 2 + 3x - x 2
. au au
on the parabola, u = 3x - 2 on the z-axis, and an + 2u = - ax + 2u = 2y - 7
on the y-axis . Use the mesh of 4 triangular elements with node 1 at the origin,
node 2 at the point (1,0), node 3 at (2,0), node 4 at (0,3), node 5 at (1,3),
and node 6 at (0,4).
ANS. U4 = 1.
6.14. Solve the Laplace equation - V'2 U = 0 in the region {(, y) : y = 1 -
x 2 , 0 :S x :S I}, which is in the first quadrant, and subject to the conditions:
u = -1 + 3x - x2, u = 3x - x-aX1S, and au
2 on thee z-axi an + 2u = 2y - 7 on the
y-axis. Use the mesh of 4 triangular elements with node 1 at the origin , node 2
at the point (0.6, 0), node 3 at (1,0), node 4 at (0, 30.64), node 5 at (0.6,0.64),
and node 6 at (0, 1).
ANS. U4 = -1.72272.
Exact solution is u(x) = y + 3x - 2, which gives U4 (exact)= -1.36.
6.2. EXERCISES 133

6.15. Solve the Laplace equation _\72u = 0 on a rectangle n {( x,y):


o < X < it , 0 < Y < I} such that u(x ,O) = sin x, u(x , 1) = 1 + sin 2x ,
u(O, y) = y = u(7r, y), by considering a mesh of 2 x 2 linear triangular
elements , as shown in Fig. 6.17.

(0. l) 7 9

u =y 4 ~-=-----~<--'----'"

(0, 0) 2 3
u =sinx (TC ,O)

Fig. 6.17. Heat Conduction in a Rectangle.

H INT. Using (B.3), we find that

Note that U1 = 1, U2 = 1//2, U3 = 0, U6 = 1/2, U7 = 1, Us = 0, and


Ug = 1. Also, f = O. Then, solve

(e) K 45
44 (e)] [U]
4 _ {K41
(e)U1 +K(e)U
47 7 }
[K
K (e) K (e) U5 - K (e)U + K( e)U .
54 55 52 2 56 6

ANS. U4 = 0.324782, U5 = 0.252299. The exact solution is given by

cosh 1 . ) sinh 2y
u(x , y) = y + ( cosh y - - '-h- sinh y sin x + -'-h- sin 2x ,
sin 1 sin 2

which gives U4 = u(7r / 2, 1/2) = 0.943409, U5 = u(7r , 1/ 2) = 0.489511.


6.16. Solve the Laplace equation -\7 2u = 0 on a rectangle n = { (x,y) :
o < X < 7r,0 < Y < I} such that u(x ,O) = cosx, u(x, l) = sin 2 x,
ux(O , y) = 0 = ux (7r, y), by considering a mesh of 2 x 2 linear triangul ar
elements, as shown in Fig. 6.17.
HI NT. Use the matrix K (e) in Exercise 6.15, and note that U1 = 0,
U2 = -1 //2, U3 = -1 , U7 = 1, Us = 1/2, and Ug = o.
134 6. TWO-DIMENSIONAL PROBLEMS

ANS . U4 = 0.140261, U5 = -0.0273159, and U6 = - 0.30838. The


exact solution is given by (see Kythe et al. 2002, pp. 132-133)

cosh 1 ) sinh 2y
u(x , y) = y + ( cosh y - - '-h- sinh y cos x - - . - - cos2x ,
sin 1 smh 2

which yields u(-rr/2 , 0.5) = U4 = 0.574027,u(31T/4.0.5) = U5 = - 0.0635378,


u(1T ,0.5) = U6 = -0.517437.
6.17. Solve Exercises 6.15 and 6.16 by using a mesh of2 x 2 bilinear rectangular
elements shown in Fig. 6.18.

7 8 9 4 5 6
4 3 I 3

CD 8) CD CD
2
5 I 2
4 6 2
I 3
4 3
CD au _0
CD CD an -
2
I 2
2 3

Fig. 6.18. Fig. 6.19.

6.18. Solve - V' 2U = 2 over a unit square n = {( x , y) : 0 < x , Y < I} , subject


to the boundary condition s u(x ,O) = x 2 , u( x ,l) = 2 - x, u(O , y) = y2,
u( 1, y) = 2 - y2, by taking a mesh of 2 x 2 linear rectangular elements.
HINT . Use (B.5) .
ANS. U5 = 1.125.

6.19. Solve the boundary value problem of Exercise 6.18 by considering the mesh
of elements shown in Fig. 6.19.
6.20. A rectangular membrane of dimensions 20 x 12 ern? is subjected to a
uniform load of f o N/cm 2, and the boundary conditions shown in Fig. 6.20.
Take fo = 2, and use the symmetry to compute the stress function u at the
global nodes.
HINT . To compute the stiffness matrices and the force vectors, use (B.5)
for the four elements which are all similar. Note that K (e) = H ll + H 22 . This
leads to

41/ 10
[ -7/40
- 7/40 ]
41/20
{UU 5 }
6
= {395 /2}
539/4 '
6.2. EXERCISES 135

ANS. U5 = 51.1628, U6 = 70.0993.

II = 2x 2-x + I
7
(0. I
4 3 4 3

Q) @
0 0
II 1 2S 1 2 II
:: (0. 6 ::
"0
Il)
4 3 4 3 "0
Il)
.§ .§
:::l :::l
Vl
.5 CD al Vl
.5

(0. .0)
(5. 0) (10. 0)
Insulated II = 0

Fig. 6.20.

6 .21. (Example 5.9, Kythe et al. 2002, p.137) Consider the Laplace equation
- 'V2 u = 0 in a half-strip (see Fig. 6.21) subject to the boundary cnditions

u(O, y) = f(y), lim u(x,y) = U oo ,


x->+ oo
uy(x,O) = 0, uy(x,b) + ,8[u(x,b) - u oo ] = 0,

where ,8 is the film coefficient. Take f (y) = 50 (1 + y), and U oo = 20° C.

y
~, U oo
b
u = f (y) ----+ /loo

0 x

I
10 1112 13 14 15 17

~:
1 2 3 4 5 6
@
:r
7
(J)

8
®
'f
9

Fig. 6.21.
136 6. TWO-DIMENSIONAL PROBLEMS

6.22. Let

I(u) = r [~k(x,Y)IV'uI2_q(X,y)u]dXdy+
Jll 2
r ~ ;3(u- uoo)2ds,
Jr2 2
where f 1 and f 2 are two disjoint parts of the boundary an
in R 2 , u is ap-
propriately chosen such that ul = a (x, y), and both u and V'u are square-
r1
integrable over n. Derive the corresponding Euler-Lagrange equation and the
natural boundary condition.

ANS. -kV'2u = q in n; -k ~~ = ;3(u - u oo ) on f 2•


7
More Two-Dimensional Problems

We will consider some two-dimensional steady-state boundary value problems


from the areas of heat transfer problems (with and without convection), torsion ,
seepage, and fluid flows, and solve them by the finite element method .

7.1. Heat Transfer

The steady-state heat transfer in the (x, y)-plane is governed by Eq (1.5), which is

a ( kx aT)
- ax a ( ky aT)
ax - ay ay = f (x , y), (x ,y) E n, (7.1)

with the boundary an= f 1 U f 2 , where T denotes the temperature, f( x , y) the


internal heat generation per unit volume , and k x and k y the thermal conductivities
along the x and y axis, respectively . For heat conduction due to conduction or
convection through the boundary f 2 , the natural boundary condition , in view of
Newton's law of cooling, is

where (3 denotes the convective heat transfer coefficient, TcX) is the ambient tem-
perature of the surrounding medium, fin the prescribed heat flow, and on f 1 the
temperature is prescribed as T. The weak form ofEqs (7.1) and (7.2) on an element

P. K. Kythe et al., An Introduction to Linear and Nonlinear Finite Element Analysis


© Springer Science+Business Media New York 2004
138 7. MORE TWO-DIMENSIONAL PROBLEMS

ole) with the test function w is

o= J1 ( O( c )
aw er
k x -a -a
x x
awaT
+ kY -a y -ay - wf
)
dx dy

- Jr(
1 c) w (k x aaT
x
nx + k y ara n y)
y
ds

Jr
e (awaT awaT ) (7.3)
= JO ( c ) k x ax ax + k y ay ay - w f dxdy

+1 w [qn - ,6 (T - Too) ] ds + 1 w qn ds
Jr<C)nr2 Jr<C)j r 2
= b(w,T) -l(w) .

As mentioned in §6.l .l , sinceEq (7.3) holds for any test function w, we choose this
function in the form of n interpolation shape functions
e
¢J
) , e = 1, .. . , n. Thus,

T by LTje) ¢Je)(x, y), and


n
to derive the finte element equation we approximate
j=l

replace w by ¢~e). Then Eq (7.3) yields

(7.4)

where

dxdy ,

(7.5)

If we set ,6 = 0, we get the finite element model for heat conduction without
convection. The boundary integrals for the triangular elements are discussed in
Examples 6.2 and 6.3.
EXAMPLE 7.1 . Calculate the temperature distribution under steady-state heat
conduction in an isotropic rectangular domain of length 4a and width 2a such
that the boundaries x = 0 and y = 0 are insulated , the boundary x = 4a is
kept at zero temperature, and the boundary y = 2a is maintained at a temperature
7.1. HEAT TRANSFER 139

T = To cos 7rX (Fig. 7.1).


8a

T=O

Fig. 7.1. Heat Conduction in a Rectangle.

11 12
2

6~'----~--':'-_--;1I'-'---~--'------~

2
5

Fig . 7.2. A Mesh of 16 Triangular Elements.

METHOD 1. We use a mesh of 16 triangular elements, as shown in Fig. 7.2.


In view of (B .l), the stiffness matrix for a typical triangular element is given by

K(') ~ ~ [~I 2 0
-1
2
-1
~I ]
The connectivity matrix is
2 3 I 3 4 12 14

C~ [i
9 6 7 8 9
I
I:1
6 1 9 3 8 12 11 6 14 13
3 8 I 4 5 10 7 8 13 9 10
7 2 I 10 9 4 13 12 7 15 14
f3 in (7.5); also, f = O. The boundary
i,
Since there is no convection, we take = 0
conditions give T 5 = TlO = TI5 = 0, Tn = To, T I 2 = To cos TI3
140 7. MORE TWO-DIMENSIONAL PROBLEMS

rr 3rr
To cos 4' and T14 = To cos 8 ; moreover, F1 = Fz = F3 = F4 = F5 = F6 = 0
(becauseof the insulatedboundary). Then after assembly we solve the system
2 -1 0 0 -1 0 0 0 T1 0
-1 4 -1 0 0 -2 0 Tz 0
0 -1 4 -1 0 0 -2 0 T3 0
k 0 0 -1 4 0 0 0 -2 T4 k 0
- -
2 -1 0 0 0 4 -2 0 0 T6 2 To
0 -2 0 0 -2 8 -2 0 T7 2To cos ~
0 0 -2 0 0 -2 8 - 2 T8 2To cos i
0 0 0 -2 0 0 -2 8 Tg 2To cos 3;
and obtain
T 1 = 0.759838 To, T6 = 0.817677To,
T z = 0.701999To, T 7 = 0.755435To,
T3 = 0.537286 To, T8 = 0.578185 To,
T4 = 0.290777To, Tg = 0.312911 To.
Wecan computethe heat generated at a node. Thus, for example, the heat at node
. (7) k
5 IS F5 = Qz = K51Tl + K 5Z TZ +...+ K 5,15T1 5 = - 2' T4 = -0.145389To
W.
METHOD 2. We use a mesh of 8 rectangular elements (Fig. 7.3).

11 12 13 14 S
4 3 4 34 3 4 3

CD @ (j) ®
I 2 I 2 I 2 I 2
6 10
4 3 4 3 4 3 4 3

CD CD 0) 8)

I 2 I Z I 2 I 2

1 2 3 4 5

Fig. 7.3. A Mesh of 8 Rectangular Elements.

In view of (B.lO), the stiffness matrix for a typical triangularelement is given


by

K (e) = ~
4
-1
-1
4
-2-1]
-1
4
-2
-1 .
6 [ -2 -1
-1 -2 -1 4
7.1. HEAT TRANSFER 141

The connectivity matrix is

1 2 4 6
2 3 4 7
3 4 9 8
4 5 10 9
c= 6 7 12 11
7 8 13 12
8 9 14 13
9 10 15 14

Using the same boundary conditions as above, the assembled system is

4 -1 0 0 -1 -2 0 0
-1 8 -1 0 -2 -2 -2 0
0 -1 8 -1 0 -2 -2 -2
k 0 0 - 1 8 0 0 - 2 -2
-
6 -1 -2 0 0 8 -2 0 0
-2 -2 -2 0 -2 16 -2 0
0 -2 -2 -2 0 -2 16 -2
0 0 -2 -2 0 0 -2 16
T1 0
T2 0
T3 0
T4 k 0
T6 = -6 To
T7 2To + 2To cos 7r/ 8 + 2To cos 7r / 4
Ts 2To cos 7r / 8 + 2To cos 7r/ 4 + 2To cos 37r/8
Tg 2To cos 7r/ 4 + 2To cos 37r/8

After solving this system, we obtain

T 1 = 0.749839 To, T 6 = 0.809968 To,


T 2 = 0.692761 To , T 7 = 0.748313To ,
T3 = 0.530216To, Ts = 0.572734 To,
T4 = 0.286951 To, Tg = 0.309961 To.
We can compute the heat generated at a node. Thus, for example, the heat at
. (4) k k
node 5 is F 5 = Q 2 = K 5 1T1 + K 52T2 + ... + K 5 ,15T1 5 = --6 T 4 + -3 T g =
- 0.15114 To W. The exact solution is given by

T( ) = T, cosh(7ry/8a) cos(7rx / 8a)


x, y 0 cos h 7r / 4 '
142 7. MORE TWO-DIMENSIONAL PROBLEMS

which yields the following values:

T(O, 0) = T 1 = 0.75494 To, T(O, a) = T6 = 0.813902 To,


T(a ,O) = T2 = 0.694373 To, T(a , a) = T7 = 0.751948 To,
T(2a , 0) = T3 = 0.533823 To, T(2a ,2a) = Ts = 0.575516 To ,
T(3a ,0) = T4 = 0.288903 To, T(3a ,2a) = Tg = 0.311467 To.•

EXAMPLE 7.2 . Consider the steady-state heat transfer with convection in a


rectangular region of dimensions 4a x 2a m 2 , as shown in Fig. 7.4, where the top
side is the convection boundary with a prescribed value of (3 = 60 W/(m 20C), and
the ambient temperature Too = 20°C; the left side has a heat supply qo = 2 X 105
W/m 2 ; the bottom side is insulated (i.e., T = 0); and the right side is maintained at
a prescribed temperature To = 100°C. The conductivity of the medium is uniform
with k = 30 W/(m°C) and uniform heat generation fo = 10 7 W/m 3 , and we take
a = 0.01 m.

Co nvec tio n ~. T_

,.,~
lL, Q.
c.. T=O
a
;:;
<>
:I:

insulated

4a

Fig. 7.4. Heat Conduction in a Rectangle with Convection.

We use the 8 element mesh of rectangular elements (Fig. 7.3) . We assume that
kx = ky = k. In view of (5.7) and (B. 1O)-(B. I 1) and assuming that all values of
(3s are equal to (3, all values of T oos are equal to Too, allZ(e) = a, and f( e) = fo,
the stiffness matrices and the load vectors for an element n(e) are given by

[
~2 ~2 ~1
-1 1 2
~1] + 'k6 [~- 1 -2~ =~2 =~]
-2 1

[!1 ~1 =~ -1]
1 -1 -2 2 -2 -1 1 2

k -2
6 -2 -1 4 -1 '
-1 -2 1- 4
7.1. HEAT TRANSFER 143

K( e) = f3a
b 6014
[~ ~ ~ ~] '
1 0 1 41

f(e) = Jot [1 1 1 1 1( ,

f~e)= f3Tooa[1 1 1 1 1(.

2
f = Joa [1 2 2 2 1 2 4 4 4 2 1 2 2 2 I]T ,
4
a
fb = '2 [qO 0 0 0 0 qo 0 0 0 0 qo 0 0 0 O]T

+ f3 T oo a [1 2 2 2 1 2 4 4 4 2 1 2 2 2 I]T ,

we solve the linear system (K + Kb) T = f + f b, which reduces to


40.8 -9.8 0 0 -4.9 -10 o o
-4.9 81.6 - 4.9 0 -10 -9.8 - 10 o
o -9.8 81.6 -9.8 0 -10 -9 .8 - 10
o o - 9.8 81.6 0 0 -10 -9.8
x
-4.9 -10 0 0 20.4 -4.9 o o
-10 -9.8 -10 0 -4.9 40.8 -4.9 o
o -10 -9.8 -10 0 -4.9 40.8 -4.9
o o -10 -9.8 0 0 - 4.9 40.8
T6 1524
T7 1048
Ts 1048
Tg 3088
x
Tn 1262
T 12 524
T 13 524
T 14 2014

After solving this system, we obtain

T6 = 89.127474, Tn = 133.076907,
T7 = 58.286775, T 12 = 88.503372,
T s = 58.8575, T 13 = 78.55114,
T g = 65.211716, T 14 = 88.883588. •
144 7. MORE TWO-DIMENSIONAL PROBLEMS

7.2. Torsion

We study two-dimensional steady-state problems of torsion of bars in this section .


The Prandtl theory of torsion is governed by the Poisson's equation. According to
Prandtl's theory for computing shear stresses in a solid noncircular shaft which is
subjected to torsion, the governing equation is

(7.6)

where u(x, y) denotes the stress function, g is the shear modulus (Nrcrrr'), and B
the angle of twist per unit length (rad/cm).
The shear stress components are related to u by

au (7.7)
T zx = ay'
The gradient vector for a triangular element is given by

(7.8)

and for a rectangular element of dimensions a x b by

1 [-(2a-t) (2a - t)
\7u-- (7.9)
- 4ab -(2b - s) -s

where b~e) and c~e) are defined in (5.4). We study the following example.

EXAMPLE 7.3 . Consider the problem of twisting of a square bar, a cross


section of which has dimensions 1 x 1 cm 2 (Fig. 7.5(a» . The square has four axes
of symmetry. Thus, we analyse only the one-eighth of the cross section (shaded
region). We discretize this region into four triangular elements (Fig.7.5(b». We
take g = 8 X 106 N/cm 2 , and B = 0.01 deg/cm= 0.011r/180 rad/cm. There is no
discretization error involved in the solution . We consider a mesh of 4 triangular
elements of length a = b = 0.25 em. Since all four elements have the same
element matrices, we compute the stiffness matrix and the force vector for a typical
triangular element n(e) by using formulas (B.t )-{B.2) . Thus,

{1} +
-1
{Q(e) }
K(e) = -1 [ -1
1 2
F(e) = 2gBA(e)
3
1 Q~e) ,
2 0 -1 1 Q~e)
7.2. TORSION 145

where A( e) = 1/ 32. The connectivity matrix is

6
3

0)
2
4

(a) (b)

Fig. 7.5. Torsion of a Square Bar.

After assembly and noting that K , = 0 , the finite element equation becomes
1 - 1 0 0 0 0 U1 1
-1 4 -2 -1 0 0 Uz 3
1
-
0 - 2 4 0 - 2 0 U3 2gBA (e) 3
2 0 -1 0 2 -1 0 U4 3 1
0 0 - 2 -1 4 -1 U5 3
0 0 0 0 -1 1 U6 1
Since U3 = U4 = U5 = 0, we solve the system

~2 [!!
0
- 1
4
- 2
~2 { ~: ~
] } 2908882087 { ~ } •

and obtain U1 = 94.5387, Uz = 36.361, and U3 = 61.8137. To compute the


shear stresses, note that the gradient vector for the element n(l) is given by (7.8),
i.e.,

- 1 1 0 ] { 94.5387} { - 232.711}
Vu =4 [ 0 - 1 1 36.361 = 101811 '
61.8137 .

which gives T z x = 101.811, and T z y = 232.711. Similarly, for elements n (Z)

1 0 - 1 ] { 94.5387} { 130.9 }
Vu =4 [ -1 1 0 36.361 = - 232 711 '
. 61.8137 .
146 7. MORE TWO-DIMENSIONAL PROBLEMS

with T zx = 232.711, and Tz y = 130.9; and for the element n( 3)

0 -1 1 ] {94.5387} _ { 101.811 }
V'u = 4 [ 1 0 -1 36.361 - 1309 '
61.8137 .
with Tzx = 130.9, and T z y = -101.811. •

7.3. Seepage

The seepage flow problem is basically a potential flow problem, which is governed
by the Laplace equation, where the unknown function ¢(x , y) denotes the velocity
potential of the flow. We study the following example.
EXAMPLE 7.4 . (Groundwater Seepage Flow) Formulate and solve the ground-
water flow problem for the domain shown in Fig . 7.6. A well is located at
(850,400). The flow is governed by the Laplace equation

_~ (all
8x
8¢)
8x
_~8y 8¢)
8y
0,
(a 22 = (7.10)

all , a22 are the coefficients of permeability. The boundary conditions are as fol-
lows: ~~ = 0 on the impermeable boundary, a constant head of 100 m on the
right boundary, and the river infiltrates at the rate of 0.5 m3day- 1m- 1 . We will
determine the velocity potential ¢ for all = a22 = 1, and 15 m day-I, and find
the lines of constant potential ¢ in the rectangular domain of 1050 x 600 m 2 • The
weak form for Eq (7.10) is given by

(7.11)

L uje) ¢je) (x, y),


n
¢(x, y) ~
j=l

and substituting ¢i (x , y) for w in Eq (7.11), and noting that f = 0, we obtain the


finite element model equation as

(7.12)
7.3. SEEPAGE 147

where

K(e) =
ij
Jr{ [ow (
In <c ) ox all
oct»
ox +
ow
oy (a22 oct»]
oy dxdy
Q~e) = j wqn ds .
Jr<C)

Eq (7.12) can be written in matrix form as

(7.13)

~:----o~~,FIF-::---,-" 13
3 (600. 400)

--.,:;:-.-<~--=---.. 12
(600.200)

(0. 0)

(400. 0)
450
All coordinates marked above are in the local mode .
Well at Global Coordinates (850,400), and Local Coordinates (400, 400)

Fig. 7.6. Seepage Flow.

Now, in view of Fig. 7.6 and formula (B.1), we find that for the elements 1
through 9 (see Fig. 7.7(a»

Again, by formula (B.1) for the elements 10 through 15, we get (see Fig. 7.7(b»)
148 .7. MORE TWO-DIMENSIONAL PROBLEMS

The connectivity matrix is given by


3 1 9 48 12 8

~ [~ 1~
5 2 4 95 8 12
5 3 10 59 13 9 8] .
C 3 6 5 6 10 9 13 14
8 4 7 11 12 14 10 9

Thus, the assembled stiffness matrix and the force vector are
3 3
4 0 - 4 0 0 0 0 0 0 0 0 0 0 0
13 2 3
0 6" - 3 0 - "2 0 0 0 0 0 0 0 0 0
3 2 13 3
- 4 -3 6" 0 0 -4 0 0 0 0 0 0 0 0
13 1 3 1
0 0 0 6" - 3 0 0 -"2 -3 0 0 0 0 0
3 1 2 3
0 - "2 0 - 3 4 -3 0 0 0 - 4 0 0 0 0
3 2 13 3
0 0 -4 0 -3 6" 0 0 0 -4 0 0 0 0
4 5 1
0 0 0 0 0 0 3 -6 0 0 - "2 0 0 0
K= 3 5 25 5 ,
0 0 0 -"2 0 0 -6 6" -6 0 0 -1 0 0
1 3 5 9 5
0 0 0 - 3 -"2 0 0 -6 "2 -6 0 0 -1 0
3 5 25 1
0 0 0 0 0 -"4 0 0 - 6 12 0 0 0 -"2
1 1
0 0 0 0 0 0 - "2 0 0 0 1 - "2 0 0
1 1
0 0 0 0 0 0 0 -1 0 0 - "2 2 - "2 0
1 1
0 0 0 0 0 0 0 0 1 0 0 - "2 2 - "2
1 1
0 0 0 0 0 0 0 0 0 -"2 0 0 -"2 1

Q (e ) = [45.0 7 90.14 0 90.14 0 0 45 .07 0 -200 0 Q l1 Q12


Q13 Q14f ·

(200,200)
(200, 400/3) 3
3

CD @

1 2 2
(0, 0) (200,0) (0, 0) (200,0)

a = 200, b = 400/3 a =b =200


(a) (b)

Fig. 7.7.
7.3. SEEPAGE 149

The secondary variables at the river nodes 1, 2, 4, and 7 are defined by

where ¢i
e
) = 1- T' ¢~e) = y. and l = ~ )(600)2 + (400)2 = 180.27756 m
(see Example 6.2). Then

Q1 = 110
.5 (1 - 1) ds = 45.06939 = Q7,
Q2 = 1/ 0.5 (1) ds + 1/
0.5 (1 - 1) ds = 90.138787 = Q4.

For the point source (well) at node 9, we have Qg = 200 m3/day/m3 . Of the
remaining Q3 = 0 = Qs = Q6 = Qs = QlO, and Ql1 ,Q12,Q13,Q14 are
unknown. Since Ul1 = 100 = U12 = U13 = U14, the unknowns U1 through U10
are determined by computing Eq (7.13). Thus, for al1 = a22 = 1 m per day, we
have

U1 = 455.4, U2 = 398.9, U3 = 395.3, U4 = 302.0, Us = 340.4 ,

U6 = 332.0 , U7 = 218.3 , Us = 235.3, Ug = 294.4 , UlO = 261.3 .


But if al1 = a22 = 15 m per day, the results are

U1 = 112.60 , U2 = 108.38 , U3 = 108.59, U4 = 106.04, Us = 104.28,

U6 = 104.77, U7 = 104.01 , Us = 102.81, Ug = 99.18 , UlO = 101.39 .


Hence , we conclude that
(i) In the case of permeability coefficient all = a22 = 1 m per day, low hydraulic
conductivity exists, and water flow through the porous medium faces high
resistance. Meanwhile, high values of the velocity potentials are found , ranging
from 100 m to 450 m.
(ii) In the case of al1 = a22 = 15 m per day, high hydraulic conductivity exists,
and water flow through the porous medium occurs due to the weak resistance.
Meanwhile, low values of the velocity potentials are found, ranging from 100
m to 112 m.
150 7. MORE TWO-DIMENSIONAL PROBLEMS

The potential lines (lines of constant potential </» and velocity vectors for
all = a22 = 1 are shown in Fig. 7.8 and Fig. 7.9, and those for all = a2 2 = 15
are presented in Fig. 7.10 and Fig. 7.11. •

1 10 200

13

...-..l--+tI'--+--+-"12

L...----------------- ..L.-.-J'"---..11
7 150

Fig. 7.8. Potential Lines for all = a 22 = 1.

1 3 6 10
. - - - - - - - - - - - - 4 . . . - - - - - t t - - - - 4 I I I - - - - 4 I I I - - - - -... 14

__ -.:..::.t~= : L - __ 12

L...------------------....- - -..... ll
7

Fig. 7.9. Velocity Vectors for all = a22 = 1.


7.3. SEEPAGE 151

100
.-------------....,....-~t-.....:.::.:,.....-- .....-~"-- 14

---.....qt_-f~=------___413

L...------------------__---'___en
100

Fig. 7.10. Potential Lines for a ll = a2 2 = 15.

1 3 6 10
.------------il...----..--...--~t__------e 14

lJ_,:.---==4IJ----....- - -.... 13

L...------------------__---__en
7

Fig. 7.11. Velocity Vectors for a n = a22 = 15.


152 7. MORE TWO-DIMENSIONAL PROBLEMS

7.4. Fluid Flows

We will study two-dimensional steady-state fluid flows of ideal fluids. These flows
are also known as the potential flows, which are governed by the Poisson's equation.
We will solve the flow problem around an elliptic cylinder and that of a partially
filled circular cylinder.
EXAMPLE 7.5. (Flow around an elliptical cylinder) Formulate and solve
the problem of irrotational flow of an ideal (inviscid) fluid around an elliptical
cylinder using (a) the stream function 'Ij; and (b) the velocity potential ¢. The
flow in the case (a) is governed by the Laplace equation "i1 2 '1j; = 0, and in the
case (b) by - "i1 2 ¢ = O. We will, therefore, solve the equation - "i1 2 u = 0 in
a rectangular region n which is 8 x 4 m Z , where U is either the stream function
'Ij; or the velocity potential ¢ (Fig. 7.12). Thus, in the former case the velocity
. b a'lj; a'lj; h
components U = (Ul , Uz ) are glven y Ul = ay' Uz = - ax' w ereas In t e
. h

a¢ a¢ I . h h . h
latter case bY Ul = - ax , U2 = - ay. n eit er case t e constant term In t e
solution U does not affect the velocity field.

Insulated Boundary

/I = ( I--- - - - - - - -j .-- - - - - - - -1 /I = I
Semi-major Axis = I m
Semi-minor axis = 0.5 m

Insulated Boundary

Fig. 7.12. Flow Around an Elliptical Cylinder.

Since the problem is symmetric about both horizontal and vertical centerlines,
we will consider only the top left quadrant. A mesh of 32 triangular elements is
chosen (Fig. 7.13) . Note that in the case of the stream function formulation the
velocity component orthogonal to the horizontal line of symmetry is zero. Thus ,
we use this line as a stream line and take the value of the stream function on this
horizontal line of symmetry to be zero. Then we determine the value of 'Ij; on
7.4. FLUID FLOWS 153

the upper wall by using the condition that ~~ = Ua, where Ua denotes the inlet
horizontal velocity.
The mesh of elements in Fig. 7.13 consists of three sets of similar triangular
elements :
SET 1. All elements 1 through 24 and 29 through 32 are like the element 1 (see
Fig.7.14(a));
SET 2. The elements 25,26, and 27 are all like the element 25 (see Fig. 7.l4(b));
and
SET 3 . The element 28 is only one of its kind (see Fig. 7.14(c)).

10 20
5.-----~~----_1_-----__::*_-----_:::II 25

4~----~~----='--""'*c:......----~"------~24

3 ~----~~---='--_...£-----"'"*"------_il23

2.-=:-----~~----""'*e::::....-----*-=--~-=-- 22

1 6 11

Fig. 7.13. Mesh of 32 Triangular Elements .

We use formul a (B.l) to compute the stiffness matrices for the first two sets of
triangular elements, and formula (5.3)-(5.4) for the third set. They are given by

~I] ,
-1 /4
K(e ) [ 1/4
Set 1 = -~4 5/4
-1

-~/+
-1 /2
K( e) [ 1/2
Set 2 = -~2 1
-1 / 2 1/2

Ki~" ~ [-{/2
-1 /2
-1/2]
o .
1/2
-1 /2 0 1/2
154 7. MORE TWO -DIMENSIONAL PROBLEMS

The connectivity matrix for this mesh is

e i j k e i j k e j k
1 1 6 7 12 13 8 7 23 14 19 20
2 7 2 1 13 8 13 14 24 20 15 14
3 2 7 8 14 14 9 8 25 21 17 16
4 8 3 2 15 9 14 15 26 17 21 18
5 3 8 9 16 15 10 9 27 21 22 23
c= 6 9 4 3 17 11 16 17 28 21 23 18
7 4 9 10 18 17 12 11 29 18 23 24
8 10 5 4 19 12 17 18 30 24 19 18
9 6 11 12 20 18 13 12 31 19 24 23
10 12 7 6 21 13 18 19 32 25 21 19
11 7 12 13 22 19 14 13

;4
(1,0.5)

(0, 0)
2
(1, 0)
~
(3.5,0.5) (4,0.5) (3.5, 0.5)

(a)

(0, 0) (0.5,0) (0.5,0)

(b) (c)

Fig. 7.14 . Three Sets of Triangular Elements.

Then the assembled stiffness matrix is given by K , which is presented on the


next page. Since U1 = U6 = Un = U16 = U5 = UlO = U15 = U21 = U22 =
U25 = 0, and U21 = U3 = U4 = 1, the assembled equation KU = Q simplifies
to the following 11 x 11 system
Continue on page 156
5 1
4 -1 0 0 0 -4 0 0 0 0 0 0 0 0 0 0 0 0 0 0 0 0 0 0 0
5 1
-1 2" - 1 0 0 0 -2" 0 0 0 0 0 0 0 0 0 0 0 0 0 0 0 0 0 0
5 1
0 -1 2" -1 0 0 0 - 2" 0 0 0 0 0 0 0 0 0 0 0 0 0 0 0 0 0
5 1
0 0 -1 2" -1 0 0 0 - 2" 0 0 0 0 0 0 0 0 0 0 0 0 0 0 0 0
5 1
0 0 0 -1 4 0 0 0 0 - 4 0 0 0 0 0 0 0 0 0 0 0 0 0 0 0
1 5 1
-4 0 0 0 0 2" -2 0 0 0 - 4 0 0 0 0 0 0 0 0 0 0 0 0 0 0
1 1
0 -2" 0 0 0 -2 5 -2 0 0 0 -2" 0 0 0 0 0 0 0 0 0 0 0 0 0
1 1
0 0 -2" 0 0 0 -2 5 -2 0 0 0 -2" 0 0 0 0 0 0 0 0 0 0 0 0
1 1 ~
0 0 0 -2" 0 0 0 -2 5 - 2 0 0 0 - 2" 0 0 0 0 0 0 0 0 0 0 0
1 5 1
~
0 0 0 0 -4 0 0 0 -2 2" 0 0 0 0 - 4 0 0 0 0 0 0 0 0 0 0
1 5 1 "'l
0 0 0 0 0 -4 0 0 0 0 2" -2 0 0 - 4 0 0 0 0 0 0 0 0 0 0 e-
1 1 c:
0 0 0 0 0 0 - 2" 0 0 0 -2 5 -2 0 0 0 -2" 0 0 0 0 0 0 0 0
K =I 0 0 0 0 0 0 0 1
0 0 0 -2 5 -2 0 0 0 1
0 0 0 0 0 0 0
8
- 2" - 2" "'l
0 0 0 1 1 e-
0 0 0 0 0 -2" 0 0 0 -2 5 -2 0 0 0 -2" 0 0 0 0 0 0
1 5 1 0
0 0 0 0 0 0 0 0 0 - 4 0 0 0 - 2 2" 0 0 0 0 -4 0 0 0 0 0
1 7 3
~
0 0 0 0 0 0 0 0 0 0 0 0 0 0 0 0 0 0 0 0 0 0 ill
-4 4 - 2"
1 3 9 3 1
0 0 0 0 0 0 0 0 0 0 0 - 2" 0 0 0 -2" 2" - 2" 0 0 - 2" 0 0 0 0
1 3 19 1 1
0 0 0 0 0 0 0 0 0 0 0 0 - 2" 0 0 0 - 2" "4 -2 0 -2" 0 -4 0 0
1 1
0 0 0 0 0 0 0 0 0 0 0 0 0 -2" 0 0 0 - 2 5 - 2 0 0 0 -2" 0
1 5 1
0 0 0 0 0 0 0 0 0 0 0 0 0 0 -4 0 0 0 -2 2" 0 0 0 0 -4
1 1 5 1 1
0 0 0 0 0 0 0 0 0 0 0 0 0 0 0 0 - 2" -2" 0 0 2" -2" -2" 0 0
1 1
0 0 0 0 0 0 0 0 0 0 0 0 0 0 0 0 0 0 0 0 -4 1 -2" 0 0
0 1 1 1 9
0 0 0 0 0 0 0 0 0 0 0 0 0 0 0 0 -4 0 0 -2" - 2" 4 - 1 0
1 5
0 0 0 0 0 0 0 0 0 0 0 0 0 0 0 0 0 0 -2" 0 0 0 -1 2" -1
0 0 0 0 0 1 5
0 0 0 0 0 0 0 0 0 0 0 0 0 0 -4 0 0 0 - 1 4

......
'"
'"
156 7. MORE TWO-DIMENSIONAL PROBLEMS

. .. Continued from page 154

1
5 -2 0 - "2 0 0 0 0 0 0 0
1
-2 5 -2 0 -"2 0 0 0 0 0 0
1
0 -2 5 0 0 -"2 0 0 0 0 0
1 1
-"2 0 0 5 -2 0 -"2 0 0 0 0
1 1
0 -"2 0 -2 5 -2 0 - "20 0 0
1 1
0 0 -"2 0 -2 5 -2 0 - "2 0 0 x
1 9 3
0 0 0 - "2 0 0 "2 -"2 0 0 0
1 3 19 1
0 0 0 0 - "2 0 "2 "4 - 2 - 4 0
1 1
0 0 0 0 0 - 4 0 -2 5 0 - 4"
0 0 0 0 0 0 0 -41 0 9
4 -1
1 5
0 0 0 0 0 0 0 0 - "2 -1 "2
x [U7 U8 U9 U12 U13 U14 U17 Ul 8 U19 U23 u24 f
= [1 1
"2
1
"2 0 0 0 0 0 0 0 of ·
By using the Gauss elimination method, this system reduces to the following
system, which is in the upper echelon form, and thus the results are obtai ned by
starting at the value of U24 and working upwards all the way to the value of U7 .

1 -0.4 0 -0.1 0 0 0
0 1 - 0.47619 - 0.47619 - 0.119 0 0
0 0 1 - 0.0235 -0.0588 -0.1235 0
0 0 0 1 -0.4109 - 0.00239 - 0.10125
0 0 0 0 1 -0.497 -0.0502
0 0 0 0 0 1 - 0.0263
0 0 0 0 0 0 1
0 0 0 0 0 0 0
0 0 0 0 0 0 0
0 0 0 0 0 0 0
0 0 0 0 0 0 0
U7 0.1
0 0 0 0
U8 0.16667
0 0 0 0
U9 0.20588
0 0 0 0
U12 0.02084
-0. 122 0 0 0
U13 0.04267
-0 .063 -0.1273 0 0
U14 0.04838
-0.345 - 0.0029 0 0
U17 0.00545
1 -0.4913 -0.0603 0
U18 0.01006
0 1 - 0.0312 - 0.127
U19 0.01137
0 0 1 -0.455
U23 0.00175
0 0 0 1
U24 0.00378
7.4. FLUID FLOWS 157

Solving this upper echelon form, we get

U7 = 0.217 56, Us = 0.2811185, U9 = 0.217569, U12 = 0.0511291 ,


U13 = 0.070667 , U14 = 0.05122 , U17 = 0.011065 , UI S = 0.01615 2,
U19 = 0.011961, U23 = 0.003476 , U24 = 003783.

For the stream function 'l/J, we find that 'l/J7 = 0.824600 8, 'l/Js = 0.773 5452,
'l/J9 = 0.824699, 'l/J12 = 0.8059803 , 'l/J 13 = 0.7942585 , 'l/J 14 = 0.8090609, 'l/J 17 =
0.7835873, 'l/J 18 = 0.8113408, 'l/J 19 = 0.8275934, 'l/J2 3 = 0.784807 3, and 'l/J2 4 =
0.6837628 .
The contours of the velocity potenti al and the stream function are presented in
Fig. 7.15 and Fig . 7.16, respectively.•

10 15 20
5.-------..-----_-------1~----_lI25

4t--------+--'c-----......".----~l------____.l24

0.003
3t--------+-t-----+--+----~H_----____<I 3

2.-------..-+----..-f----~~-----=~-

1 6 11 16

Fig. 7.15 . Contours of the Velocity Potential.

10 15 20

---
5 25

4
<. :---
0.8247
~
0.8276
<, 24
0.8091
-
'/
3 23
0.7735 0.794

V
/'"
22
2
-> 0.8246 0.8

1 6 11 16
.
Fig. 7.16 . Contours of the Stream Function.
158 7. MORE TWO-DIMENSIONAL PROBLEMS

EXAMPLE 7.6. (Partially Filled Pipe Flow) Formulate and solve the steady-
state problem of flow of an ideal fluid flow in a partially filled circular pipe
(Fig. 7.17). The governing equation is

_~
ax (au ax
au) _ ~ (a 22au)
ay ay = 2. (7.14)

The weak formulation of this equation with au = a22 = 1 on an element n( e) is

o= r
Jnw
W a2~ _ a2~ _ 2] dx dy
[_
ax ay
= r
In< e)
[aaWaa
x x
u w u
+ aa aa - 2W] dx dy + 1 Wqn ds
y y free)
= b(w, u) -l(w) ,
au au
where qn = n x ax + n y ay'
y

au -0
an-
(0.0) I ' - - - - - - - t - - - X

Fig. 7.17. Geometry of the Partially Filled Circular Pipe.

I>je)¢je)(x, y), and w = ¢~e), then the above


n
Now, if we take u(x, y) ~
j=l
weak form leads to
n
'"' K( e)u(e) = p(e) = j(e) + Q(e) (7.15)
~ tJ J t t t'
j=l
7.4. FLUID FLOWS 159

where

(e) _] [o¢~e) O¢)e) o¢~e ) o¢)e)]


K ij - !:l !:l +!:l !:l dx dy ,
11« ) ox ox uy uy

f?) = 2 r
}11( e)
¢~e) dx dy, Q~e) = J ¢~e) qn ds ,
t.;
which is written in matrix form as

(7.16)
We consider the case when the pipe is 95% full (Fig. 7.18). Because of the
symmetry about the vertical axis (y-axis), we consider only the right half of the
circular domain .
y

1l~""*~""--c:¥---Y

7t"--::*'--::oIl"--Y

o
100 20

Fig. 7.18 . A Mesh of 154 Triangular Elements in a 95% Filled Pipe.

The boundary conditions are (see Fig. 7.17)

Natural Boundary Conditions: ~~ == a on the line of symm etry ,

OU == a on the free surface , (7.17)


oy
Essential Boundary Condition: u =a on the pipe boundary.
160 7. MORE TWO-DIMENSIONAL PROBLEMS

We create a mesh of linear triangular elements as follows. First divide the


right half of the domain by taking central rays of angle 10° each, and then draw
horizontal and vertical lines from each intersection of the circle and the rays , and
finally divide each rectangle into two triangular elements by joining its diagonal
(Fig . 7. I). This produces a mesh with 95 nodes, 154 elements, and 0:: = 36.94°.
Note that the half-angle 0:: varies with the area (percentage) occupied by the fluid.
This is presented in Table 7.1 for future reference, where the values of the half-angle
0:: are computed for different percentages of the filled pipe .

Table 7.1. Area vs. Half-Angl e 0:: for Pipe Radius = 1.0 m.

Area Half-Angle 0:: Number of Nodes

95% 36.94° 95
94% 38.88° 95
93% 41.50° 91
92% 44.67° 91
91% 46.00° 91
90% 47.33° 91
85% 55.17° 90

The coordinates (x,y) of the nodes in the discret ized domain are given below
in Table 7.2.

Table 7.2. Coord inates of the Nodes.

Node# x y Node# x y

1 0.000 -1.000 49 0.500 0.000


2 0.000 - 0.985 50 0.643 0.000
3 0.174 -0.985 51 0.766 0.000
4 0.000 - 0.940 52 0.866 0.000
5 0.174 -0.940 53 0.940 0.000
6 0.342 -0.940 54 0.985 0.000
7 0.000 -0.866 55 1.000 0.000
8 0.174 -0. 866 56 0.000 0.174
9 0.342 -0.866 57 0.174 0.174
10 0.500 -0. 866 58 0.342 0.174
11 0.000 -0.766 59 0.500 0.174
12 0.174 -0.766 60 0.643 0.174
13 0.342 -0.766 61 0.766 0.174
14 0.500 -0.766 62 0.866 0.174
7.4. FLUID FLOWS 16 1

15 0.643 - 0.766 63 0.940 0.174


16 0.000 -0.643 64 0.985 0.174
17 0.174 - 0.643 65 0.000 0.342
18 0.342 -0.643 66 0.174 0.342
19 0.500 -0.643 67 0.342 0.342
20 0.643 - 0.643 68 0.500 0.342
21 0.766 -0.500 69 0.643 0.342
22 0.000 - 0.500 70 0.766 0.342
23 0.174 - 0.500 71 0.866 0.342
24 0.342 - 0.500 72 0.940 0.342
25 0.500 - 0.500 73 0.000 0.500
26 0.643 - 0.500 74 0.174 0.500
27 0.766 - 0.500 75 0.342 0.500
28 0.866 -0.500 76 0.500 0.500
29 0.000 - 0.342 77 0.643 0.500
30 0.174 -0.342 78 0.766 0.500
31 0.342 - 0.342 79 0.866 0.500
32 0.500 - 0.342 80 0.000 0.643
33 0.643 -0.342 81 0.174 0.643
34 0.766 -0.342 82 0.342 0.643
35 0.866 - 0.342 83 0.500 0.643
36 0.940 -0 .342 84 0.643 0.643
37 0.000 -0.174 85 0.766 0.643
38 0.174 - 0.174 86 0.000 0.766
39 0.342 - 0.174 87 0.174 0.766
40 0.500 - 0.174 88 0.342 0.766
41 0.643 - 0.174 89 0.500 0.766
42 0.766 - 0.174 90 0.643 0.766
43 0.866 - 0.174 91 0.000 0.766
44 0.940 -0.174 92 0.174 0.799
45 0.985 -0.174 93 0.342 0.799
46 0.000 0.000 94 0.500 0.799
47 0.174 0.000 95 0.601 0.799
48 0.342 - 0.000

Using the boundary conditions, the problem is solved, and the following results
obtained for the case of 95% filled pipe are given in Table 7.3 on the next page.
162 7. MORE TWO-DIMENSIONAL PROBLEMS

Table 7.3. Coordinates of the Nodes.

Node# U Node# U Node# U

1 0.14724 2 0.15709 3 0.00000


4 0.15661 5 0.05608 6 0.00000
7 0.19198 8 0.13324 9 0.07439
10 0.00000 11 0.25526 12 0.22077
13 0.16399 14 0.08850 15 0.00000
16 0.33599 17 0.31223 18 0.25854
19 0.18284 20 0.09332 21 0.00000
22 0.42069 23 0.40088 24 0.34909
25 0.27316 26 0.18242 27 0.08793
28 0.00000 29 0.49705 30 0.47833
31 0.42675 32 0.34976 33 0.25738
34 0.16130 35 0.07216 36 0.00000
37 0.55678 38 0.53786 39 0.48484
40 0.40536 41 0.31018 42 0.21164
43 0.12065 44 0.04718 45 0.00000
46 0.59597 47 0.57601 48 0.51972
. 49 0.43559 50 0.33570 51 0.23337
52 0.13970 53 0.06451 54 0.01638
55 0.00000 56 0.61452 57 0.59265
58 0.53070 59 0.43868 60 0.33105
61 0.22302 62 0.12576 63 0.04855
64 0.00000 65 0.61681 66 0.59207
67 0.52148 68 0.41668 69 0.29620
70 0.17987 71 0.07810 72 0.00000
73 0.61008 74 0.58188 75 0.49971
76 0.37513 77 0.23188 78 0.10430
79 0.00000 80 0.60174 81 0.57077
82 0.47648 83 0.32495 84 0.13630
85 0.00000 86 0.59615 87 0.56513
88 0.46367 89 0.28818 90 0.00000
91 0.59576 92 0.56486 93 0.46306
94 0.28630 95 0.09993

Similar results can be obtained for the cases of 90%, 91%, 92%, 93% and 94%
filled pipes. The three plots on the next page show (i) the area of the partially
filed pipe (Fig. 7.19), (ii) the area of the partially filled pipe vs. the flow velocity
(Fig. 7.20), and (iii) the area of the partially filled pipe vs. the flow discharge
(Fig. 7.21) . It is seen from Fig. 7.21 that the maximum flow discharge occurs
when the pipe is 92.89% filled (Khairy, 1998).•
7.4. FLUID FLOWS 163

3.0 , - - - - - - - - - - - , - - -
I
----------,----------,
I
2.97
I
__________ I ~
1I Il _

I I I I
I I I
I I I
I , I

29 - - - - - - - - - - ~ - - - - - - - - - :- - - - - - __ 1-_-- _ _ - - - - ,- - - - - - - - - -
1 I
I I
: 2.~2
g
1 I
I 1 I
_ _ _ 1 1 1 _
I 1 I

~
1 I I
I I I
__________ I ~ II II IL _
2.8 I I I I
I I I I
I I I I
I I t I
_________ L L L L _
I I I I
l I t I
I I I I
2.7 L- - - 'I --'-I I
-'- - 'I- --1

90% 91% 92% 93% 94% 95%

Area(%)

Fig. 7.19. Geometry of the Partially Filled Circular Pipe.


0.31r - - - - - : - - - - - - : - - - - - - : - -
I
----:--------,
I I I I

_________ ~--_---_ --~ ----_---JlW4-------~-_-------


I I I
I I I

0.30 --- -- -- -O!"298' - Q2~ ------ -~ - ----- - -~ --- ---- --


0.297 "I
S
~
fJ') I ~ L
I ~
I L _
........ I I I I
I I I I
~ _ _ _ _ _ _ _ _ _L
I IL IL I
~ _
] 0.29 I I I I

>" I
I
I
I
I
I
I
I

o
~ ---------~---------~---------~------
I I I
--+-------- -
I
ti: I I I I
0.279
------Qi7
I , I
0.28 ----- ----~-- -------~------ ---i------ --
I I ,

---------.---------.---------.---------.---------
I I I I

I I I I

0.27 L- I
--'- I
..L- --'-
I
-'-
I
--'

90% 91% 92% 93% 94% 95%


Area (%)

Fig. 7.20. Area of the PartiaIIy Filled Pipe vs. Flow Velocity.
0 . 9 0 , - - - - - - - - , - - - -i - , - - - - - - - , - - - - - , .I - - - - - - - ;
: I 0.884 :
_________ 1 1 .J1 1 _
0.88 I I II I
I I II I
~
'1 0.86
I I II I

---------~------1},~- - -----~~---- ----~---------


0.845 " I
~ I II I
] 0.84 -~---------~--------~~------
I I II
--~---------
~ I I 11 9.822 0.828
is 0.82
1
- --------~---------~---
I
-----r~---------r---------
II

~ "
~ I I
I
I
I 0\ II I
fi: 0.80
---------~---------r-------~+r---------~---------
I I 0\ II I
0.78'-- -'-
I '--
I .........
II '--
I --1

90% 91 % 93% 94% 95%

Fig. 7.21. Area of the PartiaIIy Filled Pipe vs. Flow Discharge.
164 7. MORE TWO-DIMENSIONAL PROBLEMS

7.5. Exercises

7.1. Determine the temperature distribution for the heat transfer problem in Ex-
ample 7.3 for the case when the ambient temperature Too = 0° C.
ANS. T6 = 87.138015, T7 = 56.60317, Ts = 57.095015, Tn =
130.619864, T 12 = 86.048119, T 13 = 76.26186, T 14 = 87.224933 .
7.2. The governing equation for a two-dimensional fin in the form of a circular
pipe is

cPT cPT
k x h 8x 2 + ky h 8y2 - 2f3T + 2f3Too = 0,

where k x and k y are the thermal conductivities in the x- and y-direction, re-
spectively, h is the thickness of the fin, f3 the convection coefficient, and Too
the ambient temperature. The boundary conditions are T(8D.) = T; on the
pipe boundary, and kx h a;: n x + ky h ~: n y = 0 along the outer edge of the
fin. This condition can be regarded as the insulated boundary condition. The
fin is two-dimensional because it is too thin to develop a temperature gradient
along the z-axis (Fig. 7.22). Determine the finite element model equation for
this problem .

Outer Edge

Fig. 7.22 . Two-Dimensional Circular Fin.

7.3. Solve the heat conduction problem discussed in Example 7.1 for a rectangle
of dimensions 3 x 2 cm 2 , subject to the following boundary conditions: The
boundaries x = 0 and y = 0 are insulated; the boundary x = 3 is kept at
zero temperature, and the boundary y = 2 is maintained at a temperature
T = e- x / 3 . Use (a) a mesh of 12 triangular elements, each of unit base and
unit altitude; and (b) a mesh of 6 rectangular elements, each of unit base and
unit height.
7.5. EXERC ISES 165

ANS . (a) T, = 0.588745, T2 = 0.500 791, T3 = 0.296294, Ts =


0.676718, T 6 = 0.559058, T 7 = 0.342192.
(b) T, = 0.563439, T2 = 0.48 6402, T 3 = 0.282152, Ts = 0.63740 6,
T 6 = 0.565973, T 7 = 0.320434. The exact solution is: T, = T(O,O) =
0.62488, T2 = T(I ,O) = 0.44775, T3 = T(2 ,0) = 0.296294, Ts =
T(O, 1) = 0.637406, T 6 = T(I , 1) = 0.510544 , T7 = T (2, 1) = 0.365 821.
7.4. (Lebedev et al. 1965, Problem 164) A rectangular bar of length a and
width 2b units is constructed by joining two sections of lengths a l and a2 with
different condu ctivities k l and ka, respectively , where a = al + a2 . Determine
the temperature distribution in the bar if the two opposite faces at y = ±b are
maintained at temperature To, while the other two ends are at zero temperature.
Use the symmetry about the x -axis, and consider the mesh shown in Fig . 7.23,
where al = a/3, a = 30 m, b = 10 m, k l = 20 W/(m °C) , k2 = 30 W/(m °C) ,
and To = 100° C.

y
9 10 11 12

0 ® @
CD 7
b=1O 5 8

0 ®
x
2 3 4
I I
10 20
a = 30
Fig. 7.23. Rectangular Bar with Two Different Conductivities.

HINT . Note that the elements 1 through 4 are similar with thermal condu c-
tivity k 1 , and the elements 5 through 12 are similar with thermal conductivity
k2 • Use (B.l) for each element.
A NS. U2 = 69.219, U3 = 66. 248, U6 = 76.568, U7 = 74.158. The exact
solution is given by (see Lebedev et al. 1965)

T (x , y) = 200 f (~o~ 'Yn - cos 2'Yn) cosh(O.1 Y'Yn) T (x ),


tan 'Yn
2 sin (2'Yn )
n=l 'Yn [ 3 + 2 sin2 'Yn ] cosh 'Yn
where
sin (O.1 x'Yn ) sin (2'Yn ) if 0 :::; x :::; 10,
T (x ) ={
sin (O.1 (30 - x),n ) sin 'Yn if 10 :::; x:::; 30,
and 'Yn are the consecutive roots of 3 tan 'Y + 2 tan (2'Y) = O.
166 7. MORE TWO-DIMENSIONAL PROBLEMS

7.5. Consider the steady-state heat transfer problem with a prescribed convection
coefficient {3, thermal conductivity k, and an internal heat generation io, in
a square region of side a. This region is insulated both at the top and the
bottom ; a uniform heat flux qo acts on the left side; and the right side is kept at
a prescribed temperature To. The boundary conditions are shown in Fig. 7.24,
where Too denotes the ambient temperature . Take a mesh of 2 x 2 linear
rectangular (square) elements, and compute the temperature distribution at the
global nodes. Use thefollowingdata: a = 0.002 m; k = 30W/(m·°C), {3 = 80
W/(m 2 . 0C); Too = 10°C; To = 100°C, io = 107 W/m 3 , and qo = 3 X 105
W/m 2 •

Insulated
Convection p, Too
(O,a) 7 8 9 (a, a)
4 3 4 3

CD @
I 25 I 2
(0, a12) 4 6 T =To
4 3 4 3
Heat Flux %- -
CD 0
I 2 1 2
(O,Oh 2 3 (a, 0)
(aI2 ,0)
Insulated

Fig. 7.24.

HINT. Use (3.10) , (3.11), (3.15), (B.8), and (B.9) to obtain

K( e) =~
6
[:2 -1
-2
2
1
-1
1
2
!1] +"6 [J
-2
k
1
1
2
-2
-1
-2
2
-2]
-1
1

-I]
1 - 1 -2 2 -2 -1 1 2

~~6
[!I - 2
-1
-1

-1
-2
4
- 2
-1
4
-1
-2
-1
4
'

2
r( e) = ioa [1 1 1 l]T ,
16

H(') ~ pa
12
[i° °~]
1
4 1
1 4 ,
1
° 1
7.5. EXERCISES 167

and solve (K + K, + H) T = f + fb, where T3 = T6 = T g = 100.


ANS . T 1 = 450 .357 , T 2 = 326 .768, T 4 = 416.402, T 5 = 311. 457,
T7 = 441.355 , Ts = 288.786.
7.6. Assuming that heat is generated in a rectangular bar I] = {(x,y) : 0 < X <
a, O < y < b} at a constant rate q per unit volume, that there is no temperature
gradient in the z-direction, and that the therm al conductivity k of the bar is
constant, solve the Pois son's equ ation k (u xx + uyy) + q = 0, subject to the
boundary cond itions:

ux(O, y) = 0, u(a, y)= 0,


uy(x ,O) = 0, u(x ,b)= O.

The exact solution is (Kythe et al. 2002 , p. 140)

__ 2q ~ (_l)n cos Anx cosh Any qa 2 ( _ x2 )


( )
U x , y - ka ~ A~ cosh Anb + 2k 1 a2 '

(2n + 1)7l"
where An = , n = 0,1 , . .. .
2a
7.7. The torsion of a hollow membrane of square cross section of inner and outer
dimensions 2a and 6a, respectively, is governed by the Poisson 's equation
- 'V2 u - 2, where u denotes the stress function. The boundary conditions are
u = 2r on the outer boundary and u = 3r 2 on the inner boundary, where r
denotes the ratio of the outer and inner dimensions of the squares. Take a = 1,
and compute the stress function at the global nodes of the mesh of elements
shown in Fig . 7.25.

/
9
3
0)
7 I 2
8
4 3

8)
I 2
4 6
3 4 3

CD @
2 2

1 2 3

Fig. 7.25 .
168 7. MORE TWO-DIMENSIONAL PROBLEMS

HINT. For the triangular elements, use (B.I )-(B .2), which give

1 -1 0]
K(e) = ~
2 [-1
0
2
-1
-1
1
,

For the rectangular elements (a = b = 1), using (B.5) we have

K(e) _ ~
4
-1
-1
4
-2
-I
-11
-2
- 6 [ -2 -1 4 -1 '
-1 -2 -1 4
Use the boundary conditions U1 = U4 = 27, U3 = U6 = Us = Ug = 6.
ANS. U2 = 15.8651, Us = 13.4852, U7 = 5.91974.
7.8. Use the Prandtl theory of torsion, governed by Eq (7.6), where n is the
cross section of an elliptical membrane being twisted, and compute the stress
function u at the global nodes marked in Fig. 7.26, where the semi-major axis
is a = 3 in and the semi-minor axis b = 2 in, and 9 8 = 2.

Fig. 7.26.

HINT . To compute the stiffness matrices and the force vectors, use (B.3)-
(Bo4) for the elements 1,2 ,3,5,6,7,8, and (5.3)-(504) for the element 4.
ANS. U1 = 4.32553, U2 = 2.11027, U3 = 4.24104, Us = 3.18775,
U6 = 4.36778. Compare it with the exact solution

C8a2b2 ( x2 y2)
u( x,y) = a2+b2 1- a2 - b2 .
7.5. EXERCISES 169

7.9. Find the steady-state temperature in a solid circular cylinder of radius 1 and
height 1 under the conditions that the flat faces are kept at 0° and the curved
surface at 10 . The exact solution is (Kythe et al. 2002, p. 154)

~ Io(mr r) sin mrz


u(
r, )
z = 4 L....- '
n = l 10 (mr) sin mr
n odd

where 10 are the modified Bessel functions of the first kind and zero order.

7.10. A grid of heating cables is embedded in a thin concrete slab to help melt the
snow on the surface of the slab, which is exposed to the surrounding medium
maintained at a temperature Too . The cables are 5 em apart and are 2.5 em
below the surface . The slab, which is 10 em thick, is laid on a thick insulated
bound ary with a negligibl e heat loss. The top surface of the slab is subject to the
convection boundary condition (Fig. 7.27). Take Too = -10°C , k x = k y = 36
W/(m 0C), and f3 = 42 X 105 W/(m 2 0 C) (which amounts to about 34 kmlhr
wind velocity). Compute the temperature distribution on the surface of the slab
when the cables generate 5 x 104 W1m2 of heat.

Convection Boundary

2.5 em

- -..- ---- ---- .:g.


.!:::
------.. - ----- ---- e--- ----
E
u
5 em
'"
is
~ IOem
.9

Legend:
e Heating Cables

Insulated Boundary

Fig. 7.27. Heating Cables Embedded in a Concrete Slab.

HINT. The region to be discretized is a vertical rectangle, shown shaded


in Fig . 7.27. Discret ize this rectangle into a mesh of 12 bilinear rectangular
elements shown in Fig. 7.28. Notice that the size of the elements is smaller
toward the top of this rectangle.
A NS. The temperature distribution at the global nodes is given below on
the next page.
170 7. MORE TWO-DIMENSIONAL PROBLEMS

2
1 3
4 3 4 3
TI = 2.4, CD (]
T2 = 2.4
4 4I
2 I 'I

3 4 6
T3 = 2.3 3
T4 = 3.9 CD 0
Ts = 3.7 I
7 4
2 1 2
3 9

«
T6 = 3.6 3 4

T7 = 7.3 (])
Ts = 4.8 I
10 4
2 1 2
12
3 4 3
T9 = 4.5
T lO = 5.3
1
CD 'I1
® 2
Tu = 5.1 13 15
4 3 4 3
T I 2 = 5.0
T I 3 = 5.2
T I 4 = 5.1 G) @
T I 5 = 5.1
T I 6 = 5.1
T I 7 = 5.1 16 I 2 1 2
18
4 3 4 3
TI S = 5.1
T I 9 = 5.1
T 20 = 5.1
@ @
T2I = 5.1

1 2 1 2
19 21
20
Fig. 7.28 .

7.11. Solve the steady-state problem of temperature distribution in a half-cylinder


o:::; r :::; 1, 0 :::; e:::; 11", 0 :::; z :::; 1, where the flat faces are kept at 0° and the
curved surface at 10. The exact solution is (Kythe et al. 2002, p. 154)

00
16 I m (n1l"r ) sin n1l"z
u(r ,e,z) = -
11" 1m (n1l") sin n1l" .
m ,n=1
m ,nodd

where 1m denote the modified Bessel functions of the first kind and order m .
7.12. A furnace with no heat source and heat convection in its wall and with
asbestos insulation is shown in Fig . 7.29. The temperature distribution is
governed by Eq (7.1) , subject to the natural boundary condition (7.2) with
(3 = O. The following data are used: k x = k y = k = 0.1 Btu/(ft. °F hr.), and
f = 0; other data are marked in the figure. Use a mesh of 9 linear triangular
elements, shown in Fig. 7.29(a) and compute the temperature distribution.
Compare the result with the mesh of 5 bilinear quadrilateral elements shown
7.5. EXERCISES 171

in Fig. 7.29(b).

3" 3"

insulated I
I
insulated
I
I
I

3'

6"

6" 6"

3.------Jr 3 4

CD

2 4 3 11

CD CD CD
2 1 2 1 2
1 1 4 7 10
(b)

Fig. 7.29.

HINT. To solve KT = 0, note that

Also, T 1 = T2 = T3 = T 4 = T7 = Tg = 100, and T6 = Ts = 600.


ANS . T 5 = 405.5556.
172 7. MORE TWO-DIMENSIONAL PROB LEMS

7.13 . Solve the Poisson 's equation U x x + U y y = -1 ,0 < x, y < 1, subject to


the Dirichlet boundary conditions u(O , y) = 0 = u(1 , y) = u(x, 0) = u(x ,1).
The exact solution is (Kythe et al. 2002 , p. 158)

_ ~ ~ sinj7l"x sin k7ry


u (x , y ) - 7l"4 6 ·3 k 2 + ·2k 3 .
j ,k = l J J
j ,k odd

7 .14. Solve the two-dimensional steady heat conduction problem for the quadrant
x,y > 0

where k is the thermal conductivity, if the side y = 0 is maintained at zero


temperature, while the other side x = 0 is thermally insulated except for the
region 0 < y < b through which heat flows with constant dens ity q (Fig. 7.30),
that is, the boundary conditions are

T (x,O) = 0, aTI = f (y) = { - ; ' 0 < y < b,


ax x =o 0, b < y < 00.

y (0, 3)10

"0

~ --- (0, 2) 8
i;l
.s

b (0, I) 5

q
x 1 2
(0, 0)
0 T=O 1 4
(3, 0)

Fig . 7.30.

The exact solution is (Kythe et al. 2002, p. 280)

T (x ,y) = -
2q
7l"a
1 0
00
1 - cosab
a
2 e- a x sinay da. (7.18)
7.5. EXERCISES 173

7.15. Consider the normalized Navier-Stokes equation in a domain n

8u 8u gx 8p 1 (8
2u 8 U)
2
U 8x + v 8y = F 2 - 8x + R e 8x 2 + 8 y2 '
(7.19a)
2 2
8v av gy 8p 1 (8 V 8 v)
u ax + v ay = F 2 - ay + Re 8x 2 + 8 y2 '
(7.19b)
au av
- +-=0 (7.19c)
ax 8y

where u and v denote the velocity componentsalong the x and y directions; P is


the pressure; F = uo/ vfiL denotes the Froude number, where Uo is velocity, L
the length, and 9 the accelerationdue to gravity; and R e the Reynolds number.
Use the weak formulation and derive the expressions for K (e) , r (e) , and Q (e)
for an element n (e) , and obtain the finite element equation.
HINT. Write the weak form as in Example 1.1.
SOLUTIO N . Approximate the velocity distribution and pressure by u ::::;

L ¢~e) L ¢~e) L ~~e)


N N M
Ui , V ::::; Vi, and P ::::; P i. Then, from the weak form
i= 1 i=1 i =1
of the first equation, we obtain (suppressing the superscript e)

NE J'r [
~ JO(' ) ¢i ~ ¢kUk
N
t; a;
N 8¢ ·
Uj
N
+ ~ ¢kVk t; 8;
N 8¢ ·
Uj

+ ""
LJ 8x
1=1
M a~
_ I PI _ ~x
F 2
__
1
Re
o: LJ
N

j =1
_
82¢ .
ax 2
J u· + ""
J
82¢ .
N
LJ a y
j =1
u.)
_
]
dx dy = 0,
2
J
J

(7.20)
where the first summation refers to each element in the domain. By using the
Green's first identity (E.2), the second order terms on the right side ofEq (7.20)
become
174 7. MORE TWO-DIMENSIONAL PROBLEMS

Then Eq (7.20) reduces to

"[J1
NE
L..-
.
t= l
n«)
(¢i¢k
BUk¢_ J. Uj
Bx
+ ¢i¢k Vk _
B¢J· Uj + ¢i-B'l/Jl PI
By Bx
1 (B¢i B¢j
gx
- ¢i -=- + -Re -- -- u · + -B¢i -B¢j u ·)) dx dy
1 1
p2 Bx Bx J By By J

- -1 ¢i -B¢·J ujds - - 1 ¢i-Bu:J ds = 0,


Re r«1 ) Dn Re r«)
2
Bn

where riel u r~e) = Bn (e) . A similar expression is obtained for Eq (7.19b).


Finally, for Eq (7.19c), we obtain

L J1n«) 'l/Ji (B¢Bx . Uj + B¢BY.Vj) dx dy = O.


NE
i= l
J J

Combiningthe three equations we obtain KU = f + Q, where each element


K ij of the matrix K is given by

q1 = - 1
Re
1 ant e)
¢i ( _BUJ') ds,
Bn.
q2 = - 1
Re
1 ant e)
¢i (BV.)
_J
Bn
ds, q3 = O.
8
Axisymmetric Heat Transfer

In this chapter we introdu ce axisymmetric elements for heat transfer problems in


axisymmetric domains. First, we discuss the radially symmetric finite element that
reduces to the one-dimensional case , similar to the one studied in Chapter 3, and
present examples of heat conduction in axisymmetric solids . Next we derive the
linear triangular and bilinear rectangular elements for standard linear heat transfer
problems in both solid and fluid medium , and then we present an application of
the element for a nonlinear heat transfer problem. We also provide a finite element
Fortran code that uses the axisymmetric linear triangular element and a Newton 's
iterative solver as well as numerical solutions of an industrial example.

8.1. Radial Symmetry

Radially symmetric problems in cylindrical polar coordinates (r, e,z) are governed
by the second-order equation

- -1 -d [a(r)-
dU] = f(r), (8.1)
r dr dr

where T denotes the radial distance, U is the dependent variable, and a and f are
known functions of T. This equation is encountered in linear problems of radial
heat conduction in long cylinders of inner radius R} and outer radius R 2 . Since the
cylinders are assumed to be long, we can assume that the temperature distribution
remains uniform away from the ends. The weak variational form of Eq (8.1) with
weight w(r) over the volume of the cylinder of unit length for a linear element

P. K. Kythe et al., An Introduction to Linear and Nonlinear Finite Element Analysis


© Springer Science+Business Media New York 2004
176 8. AXISYMMET RIC HEAT T RANS FER

n (e) of length z(e) = r~e) - ri el is given by

o= JJ~(e) w [ - ~ : (a ~~) - f(r)] rdrdBdz


t' r lr~e) W [- ;1 drd ( a du
= i l«
21r

ric)
]
dr ) - f( r ) rdrdBdz
o
r~e) W [-
lr(e)
1 d ( dU) ]
= 27r - - a- - f (r) r dr
r dr dr
1

l
r~c) ( dw du ) [ dU] r~e)
= 27r a - - - rwf dr - 27raw-
r (c ) dr dr dr r 1( e )

l
1
( e)

= 27r
r2
( a dw du _ rWf) dr _ w (ri e)) Qi e) _ w (r~e)) Q~e) ,
ric ) dr dr (8.2)

where

2
Using the approximation u(r) = L u~e) ¢~e) (r) in (8.2), we obtain the finite
i= l
element equation
(8.3)
where
dA,(e) dA,(e)

l
r~ C )
K ij(e) = 27r 'l"'i 'l"'J d (8.4)
(e)
a -d- r -d- r r,
r1

(e) (e)
A,(e) ( ) = r2 - r A,(e) ( ) = r - r 1 (8.5)
vi r l(e) '1"'2 r [(e)

If a = a(e) and f = f (e), where a(e) and f (e) are constant, then

K
(e ) _ 2na(e)
- l(e) [1 -1]
-1 1 '
(8.6)

If a = a(e)r and f = f( e) , then for a linear element we have

(e) _
na (e) ( r(e)
1
+ riel
2
) [ 1-1 ]
K - z(e) -1 1 '
(8.7)
r (e) _
nf(e) l(e) { z(e) + 3r(e)
1
}
- 3 e
2l(e) + 3ri ) .
8.1. RADIAL SYMMETRY 177

For a quadratic element, we have

(8.8)

EXAMPLE 8.1. The radially symmetric potential problems in the polar coor-
dinates (r, B) are governed by

_~ ~
r dr
(kr dU)
dr
= f(r) , 0< r < L. (8.9)

Such problems include the transverse deflection of a cable, axial deformation of a


bar, heat conduction through a circular region, pipe flows, laminar incompressible
flows through a channel under constant pressure gradient, flows through porous
media, and certain electrostatic problems where the domain is circular or annular
and radially symmetric. To derive the finite element model for Eq (8.9) subject to
the mixed boundary conditions

U (0) = Un , (8.10)

where n r denotes the radial component of the outward normal n, and un, U OCH k,
f3, and ij are constants, we find that the weak variational
formulation of Eqs (8.9)
and (8.10) over the interval (ri e
) , r~e)) is given by

which yields the bilinear and linear forms

(8.11)
178 8. AXISYMMETRIC HEAT TRAN SFER

Sub stituting ¢~e) for u and ¢;e) for w, where i, j = 1, . .. , n , and assuming that k
and f are constant on an element n (e) , we obtain

K (e) = 27T
'J
1 r
r (c)
2

(c)
(e) d¢(e)
keel d¢i zu: r dr
dr dr
+ 27T [fJr¢~e) ¢ (e) ]
' J
( c)

r2
r (c) ,

1
1 1
(8.12)
r~c ) ( r.)

f~e) = 27T f (e)¢ (e)rdr + 27T [fJr ¢(e) (Uoo _ q) ] r 2


..
J (c ) J J r ( r.)
~ 1

(e)
For a 2-element model we take the linear shape functions ¢~e) = r2 l (e~ r, ¢~e) =
( e)
r ~(~1 , where lee) = r~e) - r ~e) . Then the stiffness matrix and the force vector
are, respe ctively, given by

(8.13)

where we have used the simplifications r~e) + r~e) = r~e) + t(e)/2, and
(l (e)) 2 (r~e) _ 2r~e) ) = (r~e) ) 3 _ 3r~e) (r~e)) 2 + 2 (r~e) ) 3
= ( l(e)) 2 (3r~e) + l(e)) .•

EXAMPLE 8 .2. The flow problem of an unconfined aquifer in the radial direc-
tion is given by Eq (8.9), where u denote s the piezometric head, k the permeability
coefficient, and f the recharge. Note that (- J) denote s pumping .

Fig. 8.1. Radial View with 7 Linear Elemen ts.


8.1. RADIAL SYMMETRY 179

If a well penetrates an aquifer and pumping is done at the rate of Q = 150 m 3 /h,
determine u at radial distances T = 0,10,20,40,80,120, and 160 m, assuming
that a constant head Uo = 50 exists at T = 200 m (see Fig . 8.1), and k = 30
m 3/(h.m2 ) . In this problem f = 0 = (3 = ij, since there is no distributed source in
the region. Then from (8.13) the coefficient matrices for seven linear elements are
given by

K(l) =1rk [~1 ~1], K(2) = n]: [3 -3]=


-3 3
K (3)
'

K(4) =
3 [11-11 -11]
1rk
11 ' K(S) = -rrk [17
3 -17
- 17 ]
17 '

K(6) = -rrk [~6 ~6], K(7) = -rrk [17


3 -17
-17]
17 '

and f( e) =
0 for all elements. The boundary conditions are Us = Uo = 50, and
Q~S) = -150. Then solving the system of equations KU = F, we find that
U1 = 46.0864, U2 = 47.6779, U3 = 48.2084, U4 = 48.739, Us = 49.173,
U6 = 49.4539, U7 = 49.7191, and Us = 50. The value of Q~S) = 150.02, which
by definition is given by 2~;T (Us - U 7) = 176.495.•

EXAMPLE 8 .3 . (Vortex-flow temperature separation) Vortex tubes are used


for cooling electronic control cabinets and high speed machine operations. They
provide temperature control in protective helmets and suits. The tube is operated
by compressed air, which accelerating through a nozzle at a near-sonic velocity
enters tangentially at one end of the tube (Fig. 8.2 shows the scheme for a simple
tube). After entering into the tube, the motion of the air resembles that of a free
vortex since there is no external torque . This results in a high velocity vortex that
flows axially down the tube toward the hot air exit. The control valve at this exit
regulates hot and cold air mass flow as well as temperature separation. By a proper
control of this valve the direction of the air is reversed through the center of the
annular flow field.

Air Inlet
N01.Zle

c~ ~
Ellil
Outer Annular Flow , . - Hot Air

= .,
,
(

.. ·
~ Elli l

.,=
~MasS FIOW
Control
Valve

Fig . 8.2. Vortex Core Flow.


180 8. AXISYMMETRIC HEAT TRANSFER

Because of the conservation of the angular momentum, the kinetic energy in


the core flow undergoes separation such that the higher energy particles re-enter the
annular flow, while the lower energy particles remain in the core. This separation
continues throughout the axial length of the tube until the gas field exits the cold
air exhaust end.
The flow is both axial and radical, which during the flow process develops into
a secondary flow field. The circulation exhibits near elliptical stream lines between
the core and the annular region . The secondary flow also undergoes losses due to
momentum, kinetic energy, and high turbulence.
The problem of the vortex-flow temperature separation is simplified by assum-
ing an inviscid fluid flow in the immediate inlet section of the tube. The finite
element method is carried out by considering a small cross-sectional tube element
with an irrotational inviscid compressible flow problem under adiabatic conditions,
which eventually leads to the steady-state conditions. Since the compressed air
inlet conditions produce vortices, it is assumed on the basis of experiments that
the inlet gas pressure (especially the tangential velocity) is a primary factor of the
entire operation.
For an adiabatic isentropic process the relationship between pressure P, density
p, and velocity v is given by

P2 = (P2) I h = V2, (8.14)


PI PI VI

where, denotes the specific heat ratio (= ep/ Cv ). Since the radial pressure gradient
varies considerably depending on the inlet conditions, we will assume empirically,
based on available experimental data, that

p(r) = 3r 3 + 18r 2 + 15.7, (8.15)

where r denotes the radial distance, R I ::;; r ::;; R 2 (see Fig. 8.3).

Fig. 8.3. A Mesh of 7 Elements for 0.2 ::;; r ::;; 0.9 .


8.1. RADIAL SYMMETRY 181

Then from (8.14) and (8.15), we can define the density P and velocity v by

h p(r ) ) u-,
p(r ) ) l
p(r) = PI ( ---p;- , v(r ) = VI ( ---p;- (8.16)

Since we are dealing with a radially-symmetric steady-state flow problem, the


energy equation reduces to

pc Vr dT _ ~~ (kr dT) = 0, (8.17)


p dr r dr dr

where T denotes temperature, k the thermal conductivity, and V r the radial com-
ponent of the velocity field. Using the linear interpolation functions, the finite
element model equation for Eq (8.17) for an element n (e) is

(8.18)

where the matrix M (e) is given by

l
(e)
r2 d¢(e )
M (e) = (e) p(r) v(r) Cp T ¢(e) r dr, (8.19)
r j

and K (e) is defined by (8.8). Thus, the matrices M (e) and K (e ) are given by

(e) M(e)]
M (e) _
-
Cp PIVI
2 p 1.425
[M 11 12
M (e) M( e) ,

[1 -1]
21 22
(8.20)
K( e) = kA (e )
l( e ) -1 1 '

where A(e ) is the area and l (e) the length of the elementn(e). We consider a tube of
radius 2.54 em with an inlet pressure of 206.8 kPa. Note that in the above compu-
tation the values of the component s of the matrix M , namely, Mi~), Mi~) , MJ~),
and MJ~) , are computed by integrating (8.19) by the 4-point Gaussian quadrature
for the known cp and the interpolation functions ¢(e) (see Appendix F). Thus, for
example, Mg) = 29.32494623. The boundary condition is determined from the
dT
fact that dr = 0 at the center of the tube.

Finally, using a mesh of 7 linear elements along the radial axis with nodes at
r = 0.2(0.1)0.9, we find that

T(0 .9) = 31.3, T(0.8) = 14.8, T(0 .7) = 4.0 , T(0.6) = - 2.9,
T(0 .5) = -7.6, T(O.4) = -106, T(0 .3) = -12.6, T(0 .2) = - 21.2.
182 8. AXISYMMETRIC HEAT TRANSFER

These results are compared with the experimental data (Garrity 2000) in Fig. 8.4.
It is obvious from this figure that the finite element results match fairly well with
the experimental data .
The computation of the elements of the matrix M(e) by using the Gaussian
quadrature is left as an exercise (see Exercise 8.2).•

40 r---------------------------,

30
x Experimental Data
• Finite Element Solution

0.8 0.9

-30 ' - - - - - - - - - - - - - - - - - - - - - - - - - - - '


Tube Radius (r em)

Fig. 8.4. Experimental and Finite Element Data for 0.2 < r s:; 0.9.

8.2. Linear Elements

First, we discuss linear triangular and bilinear rectangular elements for heat transfer
in solids. The heat equation based on the Fourier's law of heat conduction in a
three-dimensional solid V is given by

8 2T 82T 8 2T)
- k ( 8x 2 + 8 y2 + 8z 2 = i, in V,
(8.21)
T=T onCl ,
-kVT·n=/3(T-Too ) onC2 ,

where C l , C 2 are two disjoint portions of the boundary of V that make up the
entire boundary, k is the heat conductivity of the material that occupies the volume
V, j is the heat source in V, T the prescribed temperature on C l , Too the ambient
temperature in the exterior of C 2 , /3 the convective heat transfer coefficient, and
n = [nx n y nzV the outer normal on C 2 •

The weak form of the above equation is obtained by multiplying both sides of
8.2. LINEAR ELEMENT S 183

the equation by a test function wand performing integration by parts. This yields

JJJvrr
r (aT aw + aT aw + aT aw) dV = J"
ax ax ay ay az az JJv
f wdV rr
+ /1 (
aT tn: aT)
-a n x + -a n y + -a n z wdS,
C t UC2 X y Z

which can be written as

Ilik'VT ,'VwdV= Ili f wdV + ll.: k'VT·nwdS

= IIi fwdV + lit k'VT · nwdS + I i2 {J (TcXJ - T) wdS.


(8.22)
Suppose that f ,T, k , {J, Too are all functions of (r, z ) only, in which case we say
that the data of the problem is axisymmetric. Suppose also that the solution domain
V can be obtained by revolving a domain n in the (r, z )-plane about the z-axis by
360 0 , in which case we say that the solution domain is axisymmetric. With these
two conditions, we look for a solution T of the boundary value problem (8.21) that
is also a function of (r, z ) only. We write V = [0,21l") x n, 0 1 = [0, 21l") x f l ,
C 2 = [0,21l") x f 2 , and T = T(r,z). Since

we have

aT
because aB = O. Let w = w(r,z). Then

Now, since

aT aw 2 aT aw . 2 aT Biu aT aw aT aw
'VT ·'Vw=--cos B+--sm B + - - = - - + - -
ar Br ar ar az az ar ar az az '
184 8. AXISYMMETRIC HEAT TRANSFER

Eq (8.22) can be rewritten as

12~ fin k ( :' ~~ + ~~ ~:) r dr dz de = 12~ fin f w r dr dz de

+ r2~ r k \IT . nw r dsd e +


Jo Jr 1 Jo Jr2
r
r f3 (Too - T) w r ds dB,

which simplifies to

fin k(~~~~ + ~~~:) rdrdz+ h 2 f3Twrds= finf wrdrdz

+ r k\lT. nwrds + Jr2


Jr
r f3 Too w r ds . (8.23)
1

LINEA R TRIANGULAR ELEMENTS. For simplicity we assume that n is a


polygonal domain. Let us divide the solution domain Y = [0 ,21f) x n into an
union of NE finite element subdomains y(e) = [0 ,21f) x n (e), e = 1,2 , ... , NE,
where n (e) is chosen to be a triangle in the (r, z)-plane with three vertices or nodes
denoted by i, j , and k with 1 :::; i , j, k :::; N as global labels , or 1,2, and 3 as
the corresponding local labels, respectively. Here, N denotes the total number of
nodes in the partition. Letan(e) denote the boundary of the triangle n(e) consisting
of three sides ri;), r;~, and r~:) with lengths i.; I jk, i. : respectively (Fig . 8.5).
Letrj>(e) = [¢~e), ¢~e), ¢~e) r denote the column vector formed by the three linear
shape functions associated with n (e). Let

then
er» = a (¢ (e) )T T (e) ,
__
8T(e)
__ =
a (..I,(e) ) T
'f' T (e).
and
ar ar az az
k

n(e )

z
j
r
Fig. 8.5. Linear Triangular Element.
8.2. LINEAR ELEMENTS 185

Replacing r:l by r:l(e) , T by r ». and w by fj>( e) in (8.23), we have

The residual or error R(e) is a result of replacing the exact solution T by the finite
element approximation T(e ) in the equation . Let

J1 (
8fj> (e) 8(fj>(e))T 8fj>(e) 8(fj>(e))T)
K (e) = k -- +-- r dr dz
n- 8r 8r 8z Bz

+ { (3 fj>(e) (fj>(e))T r ds ,
} 80 (e)nr 2

F(e) = Je ( fj>(e) f r dr dz + { (3T oofj> (e) r ds ,


} O ( e) } 8 0 (e)n r 2

Q(e) = ( k\lT . n fj> (e) r ds .


} 80(e) / r2

Then Eq (8.24) becomes

R (e) = K( e) T(e) _ F (e) _ Q(e). (8 .25)

e e
Let H e), z;e)) , H e) , zj e) ), and (ri )1 zk )) denote the coordinates of the three
nodes on the triangular element n (e), and let In (e)I denote the area of n(e). The
shape functions for the linear triangular element are

-I,(e)
0/ 1,
= a(e) + b(e) r + c(e)
1, tt
Z
'

¢(e) = a(e) + b(e) r + c(e ) Z


J J J J '
-I,(e) _ a(e) + b(e )r + c(e) Z
'+'k - k k k '

where

a~ e) = H e) Zke) - rk zj e))/(21r:l(e)l), aJe) = (rie)z;e) - r;e)zi e)) /(2[r:l(e)I) ,


ake) = (r;e)zj e) _rje) z;e))/(21r:l(e)l), b~ e) = (zje) - zi e))/ (2 1r:l(e)l),
bJe) = (zie) - z;e))/ (21 r:l(e) I), bke) = (z;e) - zY))/ (2 1r:l(e)1)
c~e ) = (rie) - rje)) /(21r:l(e) [) , cJe) = (r;e) - rie))/(21r:l(e)l),
eke) = H e) - r; e))/(21r:l(e)I).
186 8. AXISYMMETRIC HEAT TRA NSFER

e
Let h = ¢~e), l2 = ¢je), and l3 = ¢i ) be the area coordinates. The area integral ,
in view of (5.16b), is given by

Jr r
In (C)
lrl~l~ drdz = m!n!p!
(m+n+p+2)!
2I n(e) l . (8.26)

Suppose that k = k (e ) , f = i'" .


and j3 = j3(e) are constants in n (e) . By using
the area integral equation (8.26) and (8.24), we have

b ee ) bee )
t J

b~e ) bk

(e) (e)
+ Ci Cj

(e) (e)
Ci Ck

where , for simplicity, we have used the notation


(e) + (e ) + (e )
fee) = ri rj rk
3
Let r = ¢~e) r~e) + ¢je)rj e) + ¢ie)r ie). Then we have

and
8.2. LINEAR ELEM ENTS 187

if ante) n f 2 is chosen to be equal to f~;). The other integrals in (8.24) are

[ O~ 3r j(e ) 0+ r k(e)
r j(e) + r (e)
k
rj
+
o r:(e ) ] ,
(e)

r (e) + 3r (e)
j k

and

r k(e) + r.(e) 0 r (e ) + r.(e) ]


[3 o 0
k
o .
r k(e) + r (e)
i
0 r (e)
k
+ 3r;(e)

We also have

f3(e)T~) l~~) { (e ) 0 (e) } f3(e)T~) L(e) {2r ie) + r~e) }


or = J 2r j + r k , or = k1 0 .
6 r)e) + 2rie)6 rie) + 2ri e)
Note that if ante) n f 2 equals to the union of more than one side of n (e), then the
sum of the above type of matrices is possible. This can happen at a corner of I' 2 .
BILINEAR RECTANGULAR ELEM ENTS. Let n (e) be a rectangle in a finite
element partition of the domain n for e = 1, . . . ,NE, where NE denotes the total
number of rectangles in the partition. The coordinates of the rectangle are labeled
as (r (e) z(e)) (r (e) zt e) ) (r (e) z (e)) and (r (e) z(e)) locally and (r(e) z(e))
1 '1' 2 ' 2' 3 ' 3 ' 4 '4 ' t't'

.v:. »: n
(r e), Zj(e) ) ( e) (e )), and ( e):».(e) ) gIobaIIyor
j
f 1 S ~,. J." k L S N . L et

4
T (e)(r, z ) = LTt\p~e) (r, z) for (r,z) E n (e) , e = 1, ... , NE,
i= 1

be the bilinear interpolation function, where

.I.(e)( ) _ (a - r )(b - z) .I.(e) ( ) _ (a + f)( b - z)


'P I r, Z - 4ab ' 'P2 r, Z - 4ab '
.I.(e)( ) _ (a + f)( b + z) .I.(e)( ) _ (a -r)(b+ z)
'P3 r, Z - 4ab ' 'P4 r, Z - 4ab '

in which
_ r 1(e ) + r 2(e) (e) + (e)
_ Z4 ZI
r = r - --"----"-
2
Z = Z - -"------=---=--
2
188 8. AXISYMMETRIC HEAT TRANSFER

The corresponding local matrices are

=~ =~]
2 1 '
1 2
and

where f (e) = ri ; rj for the area integrals. The corresponding boundary integral
contributions are

r (3 ¢(e) (¢( e»)T rds =


f (e) (e) l(e) 21 12 00 0]0 ,
[00 00 00 00
(3 ij
Jr(,; ) 6
~

if 8 n(e) n r 2 is chosen to be equal to r~j), which is the (i, j) side of n (e) with
length li;).
The other integrals in (8.24) are

1
0 0

[~ ~]
r (e) (3(e) l(e)
(3 ¢ (e) (¢ (e») T r ds = j jk
2 1
r(c)
jk
6 1 2 ,
0 0

1 0 0

[~ ~]
A(e)(3(e) l(e) 0 0
(3 ¢(e) (¢ (e»)T rds = r kl
r (c )
kL
6 0 2 ,
0 1

1 0 0

[~ ~]
(e) (3(e) l(e) 0 0
(3 ¢ (e) (¢ (e») T r ds = ri Ii
r~~) 6 0 0
0 0
We also have

1 B O(c) n r 2
(3T oo¢(e) r ds =
f(e ) (3(e) Tj;,) l~j)

2 {g,
8.2. LINEAR ELEMENTS 189

fee)(3(e )Tj:,) li~)


or
2

Note that in the above calculations of the local matrices for the rectangular element,
(e) (e)
(e) (e) (e) (e) ri + rj
we have assumed that r t: = r I ' r J. = r tc > and have replaced r by ----"--
2
for integrals along r;;)and ri~) as well as for the area integrals on
n (e). On
e)
rj~, we have r = rj e) = ri , and on rf:),
we have r = de)
= r fe). With
more elaborate calculations, we can also evaluate these local matrices directly
reel + r eel
with r = if + t 2 J ,and it is left as an exercise (Exercise 8.6).

The above formulas can be compared with formulas (B.12), which are used for
a 4-node rectangular element in the Cartesian coordinates system.
EXAMPLE 8 .4. A can of creamed mushroom soup is heated via hot steam at a
temperature of Too = 250° F for sterili zing purpose and is assumed to have reached
a steady state . The can has a height of 0.16 m and is cylindri cal with a diameter of
0.08 m. The thermal conductivity of the soup is measured as k = 0.256/Wm oC,
and the convection coefficient is (3 = 35.6/WmoC. Approximate the temperature
of the cream soup at the center of the can. Note that for consistency in units, we
convert 250 °F to 121.111 °C in the following calcul ations .
Due to symmetry, we need only solve Eq (8.21) in V = [0, 2;r) x n, where
n= (0,0.04) x (0,0.08). Here I'i = 0, r 2 = [0, 2;r) x [{0.04} x (0, 0 .08) U
(0 ,0.04) x {0.08}] . We divide n into two triangles : n(1) which is the triangle with
. s (r 1 ) 'Zl(1) ) = ( 0, 0, T 2 'Z2(1)) = ( 0.04 , 0.08 ) , (T1
) (1) ) (1)) = ( 0,0.08 ) ,
vertice l 3 ,z3
2 2)
and the triangle n (2) with vertices (ri ) , z i ) = (0 , 0) , (r~2) , z~2)) = (0.04,0.08),
(r~2) , Z~2) ) = (0,0.08) . The corresponding global nodes are: node 1 at (0, 0),
node 2 at (0 .04 ,0), node 3 at (0 .04, 0.08), and node 4 at (0,0 .08), as indicated by
Fig . 8.6. The connectivity matrix is

c= [11 32 4]3 .
The input data for computing the local matrices for n(1) is K (1) = 0.256, f(1 ) = 0,
(3(1 ) = 35.6, rill = 0, r~l) = 0.04, r~l ) = 0, zi l) = 0, Z~l) = 0.08, and
( 1)
Z3 = 0.08. Thus ,

In(1)1 = ~ (0.04)(0.08) = 0.0016 ,


(1)
123 = 0.04,
190 8. AXISYMMETRIC HEAT TRANSFER

(1) (1) (1) 00


f(l ) = T 1 +T 2 +T 3 0+.4+0 0.04
3 3 3 '
= _1_
(l )
[b 1
b(l)
2
b(I )]
3 21 n(l ) 1 [ Z2(1) _ (1)
Z3
(1) _ (1)
Z3 ZI ZI
(1)
-
(1) ]
Z2 '

= 0.0~32 [0.08 - 0.08 0.08 - a 0 - 0.08] = [0 25 -25 ],


( 1) (1) (1) ] _ _1_ [(1) (1) (1) (1) (1) ( 1) ]
[ C1 C2 C3 - 21n(l)1
T3 - T 2 T 1 - T3 T2 - T 1

1
= 0.0032 [0 - 0.04 0 - a 0.04 - 0] = [-12.5 a 12.5],

4 3
(0, 0.08) . - - - - - -__ (0.04,0.08)
3 2
2

3
(0,0) - - - - - - - - (0.04,0)
1 2
Fig. 8.6. Mesh of 2 Triangular Elements.
8.2. LINEAR ELEMENTS 191

+ (36.6i;0.04) [ ~ 3(0.0~) + 0 O.O~ + 0 ]


o 0.04 + 0 0.04 + 3(0)
0.0009 0 - 0.0009 ]
= 0 0.0177 0.0013 ,
[ -0.0009 0.0013 0.0090

p
F ( l ) -_ f ( l ) In(1) I { 2r( 1) )++2r~l) + r~l)
(1 ) + ( 1)
}
+ (3( 1)T,( 1) L(l )
00 23
{ 2r~1) 0+ r~l) }
"i r 2 r 3 6
12 (1 ) (1) (1 ) ( 1) + 2 ( 1)
r1 + r2 + 2r 3 r2 r3

= { ~} + (35.6)(121.111)(0.04) { 2(0.0~) + 0 } = { 2.2~95 } .


o 6 0.04 + 2(0) 1.1497

}
Similarly, for n(2), we get

K (2) ~ (O.08)(O.2~6)(O.0016) ( { ~r [ -25


0
25 0] + { _1 2.5}
12.5

x [0 -12.5 12.5] ) + (36.6)(0.08) [~ 3(0.04) + 0.04


o
0.04 + 0.04
0 ]
12 0 0.04 + 0.04 0.04 + 3(0.04)
0.0068 -0.0068 0]
= -0.0068 0.0085 - 0.0017 ,
[ o -0.0017 0.0017

F (2) = { ~} + (35.6)(121.111)(0.08) { 2(0.04f + 0.04 } ={ 6.~98}


o 6 0.04 + 2(0.04) 6.898

Then the correspo nding global matrices are

0.0009 0 o -0.~009]
K = 0 0 o
0.0177 0.0013
[
-0.~009 ~
0.0013 0.0090
0.0068 -0.0068
+ - 0.0068 0.0085 -0.0017
[ o o
- 0.0017 0.0017 O~O ]
o o o
0.0158 - 0.0068 o -0.~009]
- 0.0068 0.0085 - 0.0017
- 0 -0.0017 0.0194 0.0013 '
[
-0.0009 o 0.0013 0.0090
192 8. AXISYMMETRIC HEAT TRANSFER

F = { O}
o
+ {
2.2995
O}
6.898
6.898 =
{ O} .
6.898
9.1975
1.14970 1.1497

Solving the globa l system K T = F, we get

T = [84.3673 181.1089 162.3107 112.7363 ]T .

It is obvious that this solution is by no means accurate since the exact solution is
[121.111, 121.111, 121.111, 121.111].
However, if we take a mesh of four linear triangular elements (Fig. 8.7), then
the results improve to almost exact values.
008 6
(0.04. 0.08)
2
n(l ) 3

n ll )
2
(0. 0. 4 (0.04. 0.04)
o 0.04 2
3
n O)

n (4)

(0.04. 0)

insulated

Fig. 8.7. Mesh of 4 Linear Triangular Elements.

In fact, in this case we have In(e) I = 8 X 10- 4 , j( e ) = 0 for e = 1,2,3,4, and

K(l) = (0.04)(0.256)(0.0008) ([ ~ 6~5 -~25 ]


3 0 -625 625

+
625 0
0 0
[ - 625 0
- 625])
0
625
+
(35.6)(0.04) [ 0
3
0
0 0.03
0 0.01 0.01
0.~1]
- 3 [ 1.706667 0 -1.706667]
= 10 0 -15.94667 3.04 ,
-1.706667 3.04 8.16

F(I ) =
(3(I )T, ( I ) t(I)
00 23
{ O} = {2.29949
0.08
0 },
6 0.04 1.14975
8.2. LINEAR ELEMENTS 193

K (2) ~ (008)(02~6)(00008) ([ ~~;5 -;'~;5 ~ ]

+ [~ 6~5 -~25] ) + (35.6)(0.04) [~ 0.~4 0.~2]


o -625 625 3 0 0.02 0.04
3.41333 - 3.41333 0 ]
= 10- 3 -3.41333 25.813332 6.08 ,
[ o 6.08 22.4

F (2) = { ~} + fJ( 2)T/;' ) Z~;) { 0.~ 2} = { 3.4~924 } ,


o 6 0.12 3.44924

K (3 ) = (0.04)(0.256)(0.0008) ([ ~ 6~5 -~25 ]


3 0 -625 625

+
625
0
[ -625
0
0
0
- 625] )
0
625
+
(0)(0 .04) [ 0
3
0
0 0.03 0.~1]
0 0.01 0.01

1
1.706667 0 -1. 706667
= 10- 3 0 1.706667 -1.706667
[ -1.706667 -1.706667 3.41333

F (3) ~ {n,
K (4) = (0.08)(0.256)(0.0008) ([ ~O 6~5 -~25]
3 -625 625

625 0 - 625]) (0)(0.04) [ 0 0


+ 0 0 0 + 0 0.03
[ -625 0 625 3 0 0.01
3.41333 0 - 3.41333 ]
= 10- 3 0 22.4 6.08 ,
[ - 3.41333 6.08 25.813332

O} O} = {3.44924
0 }.
{o0 +
fJ(4)T,( 4) Z(4) {
F (4 ) = 00 23 0.12
6 0.12 3.44924
The connectivity matrix is

3346 56]
c = [1 4 3 .
1 24
194 8. AXISYMMETRIC HEAT TRANSFER

After assembly the global matrices are

5.11997 -3.4133 o -1.70667 o o


-3.4133 25.8133 6.08 o o o
o 6.08 49.942 -5.11997 6.08 o
1.70667 o -5.11997 8.5333 o -1.70667 '
o o 6.08 o 38.3467 3.04
o o o -1.70667 3.04 8.16
F= [0 3.44924 6.898483 0 5.74874 1.14974f·

Solving the system KT = F, wegetT1 = 121.107, T 2 = 121.125, T3 = 121.049,


T 4 = 121.071, T 5 = 121.122, and T6 = 121.099, which are very close to the exact
solution . Thus, as the number increases, the finite element solution can be shown
to converge to the exact solution . •

8.3. Linear Elements for Heat Transfer in Fluids

Consider the following steady-state heat equation for heat transfer of fluid in a
circular tube of radius R and length L.

(8.27)

where T = T(r, z) is the unknown temperature of the flow at a location (r, z )


in the domain n = (0, R) x (0, L), and u the velocity of the flow at the same
location, p the density of the fluid, c the heat capacity of the fluid, and k the heat
conductivity of the fluid.
The difference between this equation and Eq (8.21) is the additional term
pc u 8T. The corresponding finite element local matrix is
8z

In the case of a piston flow, u is constant, say u = Urn ' Suppose that p and care
8.3. LINEAR ELEMENTS FOR HEAT TRANSFER IN FLUIDS 195

also constant. Then, for a linear triangular element we have

Similarly, in the case of a viscous laminar power-law flow, u has the value u =

Urn
3n + l ) [1 -
( n +1
(r)n+l/n]
R ' and we have

Explicit matrix forms of the term

can also be derived for the bilinear rectangular element. Typically, a circular tube
has a semi-infinite length that occupies the volume V = [0,21T) x [O ,R) x [0,00).
°
The fluid enters the tube at z = with a prescribed temperature T and heat transfer
through the tube wall atr = Rby convection in the form -k ~~ = {3(T -Too) . It is
frequently assumed that down stream at very large values of z, the fluid temperature
becomes independent of z and an artificial boundary at z = L is taken, on which
we take ~~ = 0, which is equivalent to assuming that there is no heat transfer in
the z direction. Also, some times a wall temperature is directly applied to the fluid,
that is, T = Too at r = R. This boundary condition can be viewed as a limiting
case of the previous one by letting {3 ---t 00. In the implementation of the finite
elements, we simply choose very large values for {3 in the input data.
196 8. AXISYMMETRIC HEAT TRANSFER

8.4. Nonlinear Heat Transfer

The assumptions that the fluid properties are independent of temperature and that
frictional heat generation is negligible are well suited for conventional fluid flows.
However, the viscosity of highly viscous non-Newtonian fluids often changes sig-
nificantly with temperature, and the frictional heat generation is appreciable. The
mathematical model for this heat transfer problem is given by

ZT ZT)
8T
pcu 8 z
= k (8
8r Z
~ 8T
+ r 8r +
8
8zZ +
A -nB(T-T",) Idu
d
I n
-
1
(du)Z
d
e r r '
(8 .28)
where p, c, k, A, B, T rn , and n are positive constants, and u is given by

(8.29)

in which R is the radius of the tube and Urn is the mean flow velocity. Introducing
the dimensionless parameters

vkz I r
Z'
(v + 2)pcu rn R2' r = R'

where v = n + 1 , we have the following nonlinear elliptic partial differential


n
equation:

zT zT
(1-r'V) -
8T= D -
8z
8 -+- 1-8T+ -8- + C e -BnT r tv .
8z ,2 r Br' 8r ' z
(8.30)
' '

For simplicity, we drop the primes and use (r, z ) instead of (r', z"). We use a linear
triangular element to approximate the solution of the heat equation

zT zT
(1 _ r V) 8T _ D8 ~ 8T 8 C -BnT V
8z - 8 zZ + r 8r +8r 2 + e r. (8.31 )

The corresponding local Galerkin finite element system is

which has a nonlinear term

(8.32)
8.4. NONLINEAR HEAT TRANSFER 197

Let

where 6, 6, 6 are the triangular coordinates (§5.4). Then

T (e) = T 1(e)6 + TJe)6 + Ti e)6 ,


and

where J is the Jacobian matrix and its determinant is

zl(e) - Z3(e) I
(e) (e) '
Z2 - Z3

The corresponding global system is

KT - N (T) - F = R.

We now describe a simple Newton's method for the solution of the global system .
First , for a starting initial point we obtain the solution of the linear problem

KT-F=O,

which corresponds to the finite element solution of the linear heat equation without
viscous dissipation:

v et = D~
(l- r )-;:;-
a2T + -1 -;:;-
er +~.a2T (8.33)
uZ uZ r or ur

Then we start the iteration steps by following Newton's method : A simple version
of Newton's iteration method for the nonlinear system R = 0 is given by

(8.34)

which can be written in the following form to avoid evaluation of the inverse of
the Jacobian matrix :

(8.35)

where the Jacobian matrix J (T(k)) is given by


198 8. AXISYMMETRIC HEAT TRANSFER

The computational scheme for this method is given below.

Input: Number N of equations and unknowns; initial solution Xo;


tolerance TaL; maximum number of iteration M.
Output: Approximate solution X or a message that the maximum
number of iteration was exceeded.
Step 1. Set k = 1.
Step 2. While k < M, do steps 3 through 6.
Step 3. Calculate R(X) and J(X), where J(X)i,j = (fJ!i(X)/fJX j)
for 1 ~ i ~ j ~ N .
Step 4. Solve the N x N linear system J(X)Y = J(X)X - F(X) .
Step 5. If IIY - XII < TaL, then output Y; (procedure completed
successfully). Stop .
Step 6. Set k = k + 1, X = Y.
Step 7. Output (Maximum number of iterations exceeded);
(procedure completed unsuccessfully) .
Stop .

The terms N(e) and VN(e) are evaluated by the Gauss -Legendre integration
method in the triangular region. Since the highest order of 6, 6 and 6 in the
integration is 2 +v+1 s.:: 7, we choose 7; 1 = 4-th order Gauss integration
scheme, which uses 7 Gauss points (see Appendix F).

8.4.1. Gauss-Legendre Integration Method.

where W m is the Gaussian weight and i,« denotes the value of I at the moth Gauss
point. We have

where

r; = T 1 (6 )m + T 2 (6 )m + T3 (6 )m,
rm = (6)m r l + (6 )mr 2 + (6)m r 3 ,
8.4. NONLINEAR HEAT TRANSFER 199

8</>'1 8</>'1 8</>'1


8Tl 8T2 8T3
V'Ne(T n) = 8</>2 a</>2 a</>2
= - CB n PI x
8T l 8T2 8T3
8</>~ a</>~ 8</>~
OTl aT2 aT3
( 6)')m (6)m (6)m «,)m(,,)m ]
L
7 [
B n Tm
x Wm e - r~+l (6)m(~I) m ((6 )2)m (6)m(6)m .
m=l (6)m(~I)m (6 )m(6)m ((6)2)m

@
@
@
@
@ @ @
® ® ®
@ @ ® @ @
@ @ ® @ @
@ @ @ @ @
@ ® ® @ ®
@ @ ® ® @
@ @ ® @ ®
@ @ ® @ @
@ ® ® @ ®
@ @ ® @ @ ® @)
® @) @
® (j) @ @ @ @
@ @ @ @
@ @ @ @ @
G) @ @ @) @
@) @ @ @ @
G) @ @ @ @
(}) @ @ @ ©
CD ® @ @ @

Fig . 8.8. Mesh of Linear Triangular Elements.

EXAMPLE 8.5. A typical high-density polyethylene melt in a circular tube


satisfies the relations (8.28) and (8.29), where
n l
Ae-nB(T-Tm) Id
d~ I - = 'r),

U =U m (V
- +2) [l-(R)r v] ,
v-
200 8. AXISYMMETRIC HEAT TRANSFER

and the following velocity and temperature boundary conditions

T(r,O) = To,

- k
aT
or (R, z) = (3 (T(R, z) - Tw ),
aT
oz (r, L) = 0,

where we take U m = 15.0 em/sec, v = (n + l)jn, n = 0.453, R = 0.125 em,


To = 130°C, T w = 160°C, A = 28,200 Pa. s'', B = 0.0240K- 1 , T m = 399.5K,
L = 60 em, and (3 = 10 6 W/m 2K (artificially large) to approximate the constant
wall temperature boundary condition T(R) = 160°C. Numerical experiments
show that a slightly smaller or larger value of (3 does not affect the solution profiles
significantly.
n
The domain is divided into 2000 triangular elements for numerical simula-
tion . Fig . 8.8 shows only 100 of these elements.

Wei and Zhang (2001) found that for large z the temperature can be approxi-
mately given by

+ 2- + 1,
fI 2
T -- Tw I c1 R +
n (8.36)
nB Cl +1
where

enB + (v + 2)2 en B T w ] 2 _ 1 _ enB + (v + 2)2 en B T w


Cl = [ enB cnB
u n +1 (v + 2)n+l
n B T m -'---",---'-:--
C = ~ Ae
k Rn-l

We have presented a comparison between the numerical approximation and the


above analytical solution at z = 5.2 em in Fig. 8.9. It can be seen that the
agreement is good with a maximum deviation of 5.6%. This is to be expected
since the roundoff error cannot be omitted in solving an N x N nonlinear system
with N = 2000. A three-dimensional view of temperature distribution is shown
in Fig . 8.10. A Fortran program for this type of problems is given in §14.4. The
material in this section is taken from Wei and Luo (2003).
8.4. NONLINEAR HEAT TRANSFER 201

500t=- _
Numerical Solution

Analytical Solution

400

200

Fig. 8.9. Comparison of Results.

T
E
M
P
E
R
A
T
U
R
E

Fig . 8.10. 3-D View of Temperature Distribution.


202 8. AXISYMMETRIC HEAT TRANSFER

8.5. Exercises

8.1. Determine the temperature distribution T(r) in a hollow cylinder of height


L with inner and outer radius rl and rz, respectively, where the inner and
outer surfaces are kept at temperatures T 1 and T z, respectively. Assuming that
the material of the cylinder is homogeneous and the cylinder is sufficiently
high so that the end effects can be neglected, or if the ends are insulated, the
steady-state temperature distribution is one-dimensional and is governed by
ddz~
r
+ ~ ddT = 0, and subject to the boundary conditions T (rl) = T1 and
r r
T (rz) = Tz.
· IS
ANS. Exact so Iution . T()
r =
T 1 - I T 1( - /T z) Inr-.
n rz rl rl
8.2. Use Gaussian quadrature to compute the element of the matrix M (e) defined
by Eq (8.19).
8.3. Derive the weak variational formulation and the finite element model equation
for an element n(e) for the following heat transfer problem: Consider an
axisymmetric system in the cylindrical polar coordinates (r, B, z), which is
defined by the equation

a ( rk; aT)
- [ :;:1 8r 8r a ( k aT)] =f(r,z) ,
+ 8z (8.37)
z 8z

where rand z are the radial and the axial coordinates, respectively, and T
denotes the temperature. The temperature gradient is defined by the vector

and the normal derivative of T is given by

(8.38)

where n = n r i + nzj.
ANS. The weak formulation ofEqs (8.37) and (8.38) over an element n(e)
with a test function w yields the bilinear and linear forms

b(w, T) = 21l'
fL (
n ( c)
aw8T
or r
awaT)
k; --;:;- -8 + k z --;:;- --;:;- r dr dz ,
uZ u Z

l(w) =21l' frrIn(c) wfrdrdz+21l'1i- wqnds .


8.5. EXERCISES 203

Then the finite element model equation is given by

where

rdrdz ,

8.4. Use a mesh of 1 bilinear rectangular and 2 linear triangular elements, shown
in Fig. 8.11, to approximate the solution of Example 8.4.
8 .5. Use a mesh of 2 bilinear rectangular elements of equal size (Fig. 8.12) to
approximate the solution of Example 8.4.

6
(0. o. 3 (0.4.0.8) (0. 0. 6 (0.4. 0.8)
4 4 3

n O) n O)

2 I 2
4 4
2 (0.04. 0.04) (0. 0.0 4 3 (0.04. 0.04)
3
n U)
n U)
n (3)

2 2
(0. (0.4. 0) (0. (0.4. 0)

Fig. 8.11. Fig. 8.12.

8.6. Derive an explicit form of the local matrix

Ji
arjJ(e) T
PC U -j::l
_ (rjJ(e» ) rdrdz
0 (0) uZ

for the bilinear rectangular element.


204 8. AXISYMMETRIC HEAT TRANSFER

8.7. A hot air-like gas is being transported through a circular pipe of length
L = 100 ft and radius R = 10 ft. The inlet fluid temperature is controlled at
T= 10000P and the mass flow rate is U m = 1000 ftlhr. The convective heat
transfer coefficient is j3 = 0.03898 BTU/(hr' ft2) and the ambient temperature
is approximately Too = 60°F. The properties of the gas at 60 0P are given by
p = 0.0735Ibrnlft3 , cp = 0.240 BTU/(hr·ft·op), and k = 0.01516 BTU/(hr·
ft·oP) . Assuming that a steady state has been reached, compute the outlet gas
temperature.
8.8. Solve the axisymmetric heat transfer problem on a circular cylinder of radius
6 em and height 24 em, governed by

where the thermal conductivities k; = k z = k = 30 W/(moq, subject to the


boundary conditions shown in Pig. 8.13, and a constant internal heat generation
of 10 = n X 10 7 W/m 3 .

Tz = 0

-- -- - - - - - - - - - -
T=To
5 6 7 8
4 3
CD (;) G)
I 2

insulated 2 3 4
Tz = 0

Pig. 8.13.

HINT. Because of the symmetry, consider the right lower quarter region
in the (r, z )-plane with a mesh of 3 rectangular elements (each a square of side
0.006 m). Use formulas (B.14) and (B.16). Then
8.5. EXERCISES 205

ANS. T 1 = 250.796 = T5 , T2 = 238.23 = T6 , T3 = 187.965 = T7 .


82U 82u 82u)
8.9. Consider the Poisson's equation - ( 8x 2 + 8 2 + 8z 2 = 0 in a cylin-
y
drical domain V = [0, 27r) x n, where n = (0, 1) x (0, 2). The bound-
ary conditions are U = 1 on the top and the bottom of the domain , where
z = 0 or z = 1, and - ~~ = 2(u - 1) on the side surface defined by
{(8,r, z): 0:::; 8 < 27r, T = 1,0 < z < 2}. Use a mesh of 1 axisym-
metric bilinear rectangular element and 2 axisymmetric triangular elements
(Fig. 8.14), and approximate the solution of the boundary value problem . De-
fine connectivity matrix, calculate the local matrices, and form the global sys-
tem by assembling the local matrices and applying the boundary conditions.
Finally, solve the 2 x 2 linear system for the nodal solutions .
8u
HINT. Note that - 8n = 2(u - 1).
ANS . U1 = U2 = U3 = U4 = U5 = U6 = 1.
5
(0, 2)6...4 - - - - - - 3 - - . (I, 2)

1 2
(0, 1)4 . . . - - - - - - - - 1 1 3 (1,1 )
3 2
3

2
(0, 0) lL_ __e (1,0)
1 2

Fig. 8.14.

8.10. An apple of diameter of 4 in and initial temperature of 80°F is to be cooled


to a temperature of 38°F with air at 28°F. The data for the apple are: p = 52 .4
lb/ft", cp = 0.91 Btu/Ib-Pf, k = 0.242 Btu/h-ftPf; and (3 = 7.8 Btulh·ft2 .0F.
Neglecting the heat of respiration, approximate the temperature distribution in
the apple by using a mesh of 9 linear triangular elements shown in Fig. 8.15.
HINT . The curve in this figure represents the central cross section of the
apple. Its polar equaton is T = 2(1 - sin t), 0 :::; t < 27r. The cartesian coor-
dinates of the global nodes are: 1 (0, -4),2 (0, -3),3 (0, 0), 4 (1, -3.8051 ),
5 (1, -2),6 (1, 0),7 (1, 0.493377 ), 8 (2.54404, -2) , and 9 (2, 0) , which are in
inches and must be converted into feet. For an upside-down apple the equation
206 8. AXISYMMETRIC HEAT TRANSFER

of the curve becomes r = 2(1 + sin B), 0 ::; B < 2tr. The axis of symmetry
can also be taken horizontal. In that case the polar equation of the curve is
r = 2(1 ± cos B), 0 ::; B < 2tr. If the axis of symmetry is oblique, it can be
rotated by an appropriate angle and made vertical or horizontal.

y
7

---;<:......-----+<=--+--~:..-~x

'P----f--=------j 8

Fig. 8.15. A Mesh of 9 Linear Triangular Elements.


9
Transient Problems

We discuss the finite element analysis of one- and two-dimensional transient prob-
lems by using a semidiscrete weighted residual method and approximating the solu-
tion u by taking u(x , t) ~ ~7=1 u~e)(t) <p~e)(x) in the one-dimensional case, and
taking u(x , y, t) ~ ~7=1 u~e) (t) <p~e) (x, y) in the two-dimensional case, where
<p~e) are the interpolating shape functions, and u~e) are determined by finite differ-
ence methods .

9.1. Classical Methods

Given an equation of the form

0 < x < L, t > 0 ,

we obtain a quas i-variational weak form by fixing t and then following the proce-
dure of §1.3. We explain this approach by an example.
EXAMPLE 9.1. Consider

0 < x < 1,

subject to the initial and boundary conditions u(x, 0) = 1, and u(O , t) = 0 = ~~.
Using w as a test function , we obtain the quasi-weak form

(9.1)

P. K. Kythe et al., An Introduction to Linear and Nonlinear Finite Element Analysis


© Springer Science+Business Media New York 2004
208 9. TRANSIENT PROBLEMS

If we take a 2-parameter semidiscrete Rayleigh-Ritz approximation u(x, t) ~


2
L Ui(t) (/Ji(x , y), where (PI (x) = x and (P2(x) = x 2, and replace w by ¢j (x ), we
i=l
get from (9.1)

j = 1,2, (9.2a)

or, in matrix notation


Mu j Ku = 0, (9.2b)

where the dot denotes the time derivative, and

i ,j = 1,2. (9.2c)

In this example, we have M l1 = 1/3 , M 12 = 1/4 = M 21 , M 22 = 1/5, K l1 = 1,


K 12 = 1 = K 21 , and K 22 = 4/3 . Then, from (9.2b) we obtain

1/ 3 1/4]{Ul}
[ 1/4 1/5 U2 +
[1 4/31]{Ul}
1 U2
{O} = 0 .
(9.3)

This is the semidiscrete formulation of the given problem. Now, Eq (9.3) can be
solved by a combination of the following three methods: Galerkin, Laplace trans-
form, and the 8-scheme. In these methods the initial condition must be satisfied.
However, the initial condition leads to U1(0) X + U2(0) x 2 = 1, which cannot be
E Ul
satisfied for all x (0,1) . Thus, we determine the initial values of (0) and U2(0)
by the Galerkin and the Laplace transform method, and then use the 8-scheme to
approximate the values of u.
t = 0 is r = u(x , 0) - 1 = U1(O) x +
1 ¢~e) 1 ¢i~e)
GALERKIN METHOD . The residual at
1 1
U2(0) x 2 - 1. Then r dx = 0, and r dx = O. These two equations
simplify to

1/ 3 114]
[ 114 1/5
{Ul(O)} =
U2(0)
2}'
{1 /
1/3
which yields Ul (0) = 4, and U2(0) = -10/3.
LAPLACE TRANSFORM METHOD . Apply the Laplace transform toEq (9.2b) ,
with 8 as the transform variable and Ui denoting the Laplace transform of Ui for
i = 1,2. Then we get
(8 - 1) Mil + K il = 0,
or
8/ 3 + 1 814+1] {Ul(O)} = {1 / 2}
[ 814 + 1 8/5 + 1 U2(0) 1/3 '
9.1. CLASSICAL METHODS 209

which, by using the inversion formulas (see Kythe et al. 2000)

yields

52t 15 52t]
[cosh 3 + 13 sinh 3 '
52t 3
Ul(t) = 4e- /

- e- 52t /3 [ cosh -52t + -9 sinh -52t] .


U2(t) . = -10
3 3 13 3

Thu s, we find the initial values as Ul(O) = 4, and U2(0) = - 10/ 3.


8-SCHEME. We use the 8-scheme (C.3) (see Appe ndix C), and get

1/ 3 + 8b.t 1/ 4 + 8b.t ] { Ul (O)}


[ 1/4+8b.t 1/5 + 8b.t U2(0) n+l
(9.4)
= [1 / 3 - (1 - 8)b.t 1/ 4 - (1 - 8)b.t ] { Ul (O)}
1/ 4 - (1 - 8)b.t 1/5 - (1 - 8)b.t U2(0) n '

We take the time step b.t = 0.05. Then using the above initial values Ul (0) = 4,
and U2(0) = - 10/3, we take tn+l = n b.t and solve Eq (9.4) for 8 = 1/ 2 (Crank-
Nicolson scheme) and 8 = 2/3 (Galerkin scheme). The results are given in Table
9.1. The exact solutio n is

I >->.;,t sin X.,»,


00
(2n +1)7l"
u(x , t) = An = 2
n =O

Table. 9.1.

x 8 = 1/ 2 () = 2/3 Exact

0.2 0.5 0.554 0.561 0.553


1.0 0.785 0.791 0.772
0.4 0.5 0.337 0.344 0.336
1.0 0.477 0.487 0.476
0.6 0.5 0.205 0.211 0.205
1.0 0.290 0.299 0.290
0.8 0.5 0.125 0.130 0.125
1.0 0.176 0.183 0.177
1.0 0.5 0.076 0.080 0.076
1.0 0.107 0.113 0.108 .
210 9. TRANSIENT PROBLEMS

EXAMPLE 9.2 . The unsteady transverse motion of a uniform beam clamped


at both ends is governed by

(9.5)

subject to the (essential and necessary) boundary conditions U = 0 = ~~ at

x = 0,1 for t > 0, and the initial conditions u = sin 7rX - 7rx(l - x), ~~ = 0
at t = 0 for 0 < x < 1. We use the semidiscrete Galerkin (or Rayleigh-Ritz)
n
approximation u ~ :2:Ui(t)¢i(X) with the dof = n, where ¢i = cos2i7rx,
i=l
i = 1,2, .. . , n. Each ¢i satisfies the boundary conditions. The semidiscrete
weak form of Eq (9.5) with a test function w is given by

(9.6a)

j = 1, ... , n,

or, in matrix form


Mir-l Ku = 0, (9.6b)

r r
where
d2¢i d2¢j
M i j = io ¢i ¢j dx, K i j = io dx2 dx 2 dx. (9.6c)

We will determine a one-parameter approximation (n = 1) of Eq (9.6) by the


following three methods.
METHOD 1. From Eq (9.6c) we find that

1
1 3
M ll = (1 - cos 27rX)2 dx = -,
o 2

Then Eq (9.6a) yields

(9.7)

The general solution of this equation is

U1 (t) = k 1 cos At + k2 sin At, (9.8)


9.1. CLASSICAL METHODS 211

where the initial conditions determine k l and k 2 . Thus , since

U = Ul¢l = (k l cos At + k2 sin At) (1 - cos 27!'x) ,

we find after using the initial conditions that

sin7!'x-7!'(x-x2)
Ul(t ) = 1 - cos27!'x
. cos At.

It is obvious from this result that c(O) depends on x. So we use the Galerkin method
to compute Ul(0). In view of the initial conditions the residuals are given by

Tl = residual in u(x, 0) = u(x , 0) - sin 7!'X + 71' (x - x 2)


= Ul(O)¢l(O) - sin 7!'X + 71' (x - x 2) ,

T2 = residual in ~~ (x, 0) = ~~ (x, 0) - 0 = Ul (0)(1 - cos 27!'x) ,

which give, respectively,

The first equation simplifies to give Ul(0) = ~ (~ - 71') :::::: 0.110715, and
then from (9.8) we get Ul( O) = k l . Similarly, from the second equation we get
Ul (0) = 0, which in view of (9.8) yields k2 = O. Thus , Ul(t ) = k l cos At , and the
one-parameter approximation is given by

or
U(X,t) :::::: 0.110715 (1 - cos 27l'X) cos 22.7929t. (9.9)

METHOD 2. To solve Eq (9.6) we use the Newmark scheme (§C.2) with


a = 1/2,(3 = 1/4, where ao = 4/!:!.t2, al = 4/!:!.t a, 2 = 1, a3 = !:!.t/2 = a4;
then M = M l l = 3/2, K = K ll = 871'4 , aoM = 871'4 + 6/!:!.t2, F = 0 , and
{!:!.t}n+l =n!:!.t. ThenEq (C.l2) gives

Initially , tlt n = 0, !:!.tn+l = !:!.t . Using Eq (9. 10) successively, forward in time,
with !:!.t = 0.05, we obtain

871'4 _6_) U (0 05) _ 6Ul (0)


( + 0.0025 1 · - 0.0025 '
212 9. TRANSIENT PROBLEMS

which gives UI (0.05) ~ 0.083578. Then we use (9.7) and (9.8) to compute
successive values OULI and UI . The results are given in Table 9.2.

Table 9.2. (llt = 0.05).

t UI UI UI

0.00 0.110715 0.0 -14.37953


0.05 0.083578 -0.68069 -10.85501
0.10 0.063092 -1.14628 -8.19431
0.15 0.047628 -1.24964 -6.18582
0.20 0.359536 -0.95108 -4.66961-

The above methods , although useful , are limited in their application. They fail
to deal with complicated domains, boundary conditions, and variable coefficients
in the governing equations. We will present the finite element methods which are
more suitable for such general situations.

9.2. One-Dimensional Transient Problems

Consider the unsteady model equation (3.1), which contains both time-dependent
second-order (with C2 = 0 = b) and fourth-order (with CI = 0 = a) equations.
The semidiscrete weak form ofthis equat ion in an element n( e) with a test function
W IS

(9.11)
where
2
Qi e
) = [-aax
au + ~ (b a u)]
ax ax2 x=x ~e) '
Q (e ) =
2
[-b aaX2u]
2

(e )
,
X= X1

aX2u] .
Q(e) _ [a
au- -
a (ba2u)]
-2 Q (e) =
2
[b a
3 - ax ax -ax x =x~e) ' 4 (c)
x=x 2
9.2. ONE-DIMENSIONAL TRANSIENT PROBLEMS 213

Let u be approximated by the semidiscrete approximation


n
u (x, t) ~ Lu~e) (t) ¢~e)(x) , (9.12)
i= l

where
n{ ;: : 2 if b = 0 and Cl or C2 is equal to zero ,
=4 if a = Cl = 0,
and ¢~e) are the interpolation shape functions . Note that the approximation (9.12)
cannot be used in problems involving the wave equation where the solutions are of
the form f (x ± ct ) and, therefore, the time and space variables cannot be separated.
Substituting the approx imation (9.12) for u and ¢)e) for win (9. 11), we gel

l
x~e ) n d (e) d 2 (e) d A,(e) dA,(e)
~ [Cl ¢(e )¢(e) ~ + C2¢(e)¢(e) ~ + a _ 'I-'_i_ _ 'I-'
_J _
(e) Z:: 'J dt 'J dt 2 dx dx
Xl i= l

+
d 2 (e) ¢(e)
b U,(e) ~ _ J_
'
dx 2 dx 2
+ CUt(e) '1-',
A,(e) _ A,(e)
'1-',
f] d
X
_ Q (e) A,(e) (
1 'l-'J Xl
(e) )

_ Q( e) [_ 8 d¢)e) ( (e) ) ] _ Q(e)A,(e) ( (e) ) _Q( e )[_ 8 d¢)e) ( (e) ) ] =0


2 8dx Xl 3 'l-'J X2 4 8 dx X2 ,
(9.13)
or in matrix notat ion,
L (e) u t e) +M (e) u t e) + K (e )u (e) = F (e) , (9.14)
where
K (e) = K l (e) + K 2(e) ,

L (e) =
IJ
l Xl
(e)
x 2

(e)
Cl ¢(e) ¢(e) d x
' J '
M (e)
IJ
=
l
x ~e )

(e)
Xl
. C2 A,(e) A,(e) d x
'1-'1 'l-'J '

d2 A,(e)

l
x~e) d A,(e) dA,(e)

l
x~e ) d 2 A,(e)
K ijl (e ) 'l-'i 'l-'J d 2
K ij(e) b 'l-'i 'l-'J d
=
X( e)
a - - -d-
dX X
X, = (0 )
- Xd 2 - Xd 2 X,
I Xl

l
x ~e )
p te) = Q (e) _ ¢(e) f dx i , j = 1, 2, .. . , n. (9.15)
J J (e) J '
Xl

EXAMPLE 9. 3 . Consider the problem of Example 9.1. For a linear element


we take n = 2. Let l (e) = x~e) - x ~e) . Then, with ¢~e) = I- xl l (e) , ¢~e) = XI l(e),
o::; x < r», we obtain
Lg) = 1 (¢~l)) 2
1« )
dx = [(e) 13,

L~~ = 1 (¢~l)) 2
/ (e)

d x = [ (e) / 3 ,
214 9. TRANSIENT PROBLEMS

which gives

L(1 ) =~
6
[21 21] .
Also ,

K:~1) = 11« ) (
d:~
(e)) 2
= in», K 1( 1) =
12 J
o
r
1« ) (e)
dePl
dx
(e)
deP2
dx
= -su»
_ K 1( 1)
- 21 ,

(e))2
K~~l) = ( d:~ = ui»,

which gives

K( l)=_l_[l -1].
l( e) - 1 1

Again, K( 2) = 0, and Ff2) = Ql, Fi 2) = Q2. Thus, for the element n(e) we find
from (C.l) that .

l (e)
6"'
[2 1]
1 2
{Ul} + lW
U2
1 [1
-1
-1]
1
{Ul}U2 =
{Q1}
Q2 .
(9.16)

Applying the O-scheme (C.3) to Eq (9.16), and writing l for l( e), we get

(9.17)

For a one-element finite element model we have l = 1, and the boundary conditions
on the element n(e) become ul = 0 and Q 2 = 0 for any n, and for t > 0, while
the initial condition is uy
= 1= ug
for any x E (0,1) . We take = 1/2 (Crank- e
Nicolson) and tJ.t = 0.05. Then, using these boundary and initial conditions, Eq
(9.17) reduces to

0.3583 0.1417]
[ 0.1417 0.3583
{O} _{0 .1917U
U2
2+0.05 Q1}
0.3083
n+l - U2 n'

which is solved to give

n+l _ 0.3083 n _ 0 86045 n


(9.18)
u2 - 0.3583 U2 -. U2'
9.2. ONE-DIMENSIONAL TRANSIENT PROBLEMS 215

This equation computes U2 successively, moving forward in time with the time-
step tlt = 0.05. However, the one-element model is defective , because the initial
condition u~ = 1 and the boundary condition u1 = 0 are contrary to each other.
It gives rise to a singularity. Therefore, we must use a nonuniform mesh of finite
elements with smaller elements near the singularity. This will ensure a more
accurate solution . For example, we can use the following nonuniform mesh for
both linear and quadratic elements at the given coordinates X i, i = 1, . .. , 9:

Mesh at
Linear (Quad) Xl X2 X3 X4 Xs X6 X7 Xs Xg

2(1) 0.0 0.2 1.0


4(2) 0.0 0.2 0.5 0.75 1.0
6(3) 0.0 0.1 0.2 0.35 0.5 0.75 1.0
8(4) 0.0 0.1 0.2 0.35 0.5 0.6 0.75 0.9 1.0

The results for u(x , t) for the linear elements 2,4,6 ,8 are compared with the
exact solution in Table 9.3.

Table 9.3. (8 = 1/2)

tlt t 2 4 6 8 Exact
0.2 0.8182 0.7614 0.7738 0.7393 0.7723
0.4 0.4620 0.4648 0.4736 0.4771 0.4745
0.5 0.6 0.2626 0.2822 0.2867 0.3242 0.2897
0.8 0.1493 0.1711 0.1051 0.1860 0.1769
1.0 0.0848 0.1037 0.1051 0.1860 0.1080

0.2 0.8129 0.7694 0.7753 0.7410 0.7723


0.4 0.4627 0.4665 0.4727 0.4775 0.4745
0.025 0.6 0.2632 0.2828 0.2870 0.3244 0.2897
0.8 0.1497 0.1715 0.17421 0.2363 0.1769
1.0 0.0851 0.1040 0.1057 0.1863 0.1080

Notice that better results are obtained for tlt = 0.5. •

EXAMPLE 9 .4 . Consider the problem of Example 9.2,and solve it by the


Newmark scheme (§C.2). For a bilinear rectangular element D(e) we have n = 4,
and we use the test functions ¢~e) (x), i = 1,2,3 ,4 defined by (5.7). Then we get

M (e)_~ M( e ) _ _ 111 _
210 -
2
M (e) M (e ) _ !!i
70 -
_ M 3(e1 ) '
11 - 36' 12 - 21' 13 -
216 9. T RANSIE NT PROBLEMS

2
M(e) _ 131 _ M (e)
14 - 420 - 41 '
M (e) -
22 -
!:.-
105 ' 23 -
2
M (e) _ _ 131 _ M (e)
420 - 32'

M (e) - _~ - M (e) M (e) _ 12l M(e) _ _ 1112 _ M (e)


24 - 140 - 42 ' 33 - 35 ' 34 - 210 - 43 '

(e) l3
M 44 = 105 '

which gives

13/ l - ll l/210 9l/70 2


13l /420]
M (e) = l3/105 - 13z2/ 420 - l3/ 140
(9.19)
[ 12l/35 11l2/210 .
sym l3/105

Also , K1 (e) = 0, and

K 2(e) _ 12 K 2( e) _ _ ~ _ K 2(e) K 2(e) __ 12 _ K 2(e)


11 z3 ' 12 - l2 - 21 , 13 - z3 - 31 ,

K 2(e) __ ~ _ K 2(e) K 2(e) _ ~ K 2 (e) _ ~ _ K 2(e)


14 - l2 - 41 ' 22 - l' 23 - l2 - 32 ,

K 2(e) _ ~ _ K 2(e) K 2(e) _ 12 K 2 (e) _ ~ _ K 2 (e)


24 - l - 42' 33 - z3' 34 - l2 - 43 ,

K 2 (e) = ~
44 l'

which gives

12/ l 3 - 6/l 2 -12/l


3
-6/l
2]
4/l 6/ l 2 2/l
K 2 ( e) = (9.20 )
[ 12/l 3 6/l 2 . ,
sym 4/l
and
F (e) = [Qie ) Q ~e) Q ~e ) Q~e ) ] T . (9.21 )

Substituting (9.19)-(9.21) into (9.14), we obtain

(9.22)

For a one-element model half-beam we have l = 1/2. For this element the
boundary and initial cond itions give U1 = 0 = U2 = U4 for t :::: 0, and U3 =
sin 7r/ 2 - 7r/ 4 = 1 - 7r/ 4; also, (h = O. Then Eq (9.22) leads to

M 33 U3 + K 33 (1 - 7r/4) = 0,
(1) .. ( 1)
9.3. TIME-DEPENDENT HEAT CONDUCTION 217

or
K (l)
U3 = - ~;) (1 - 7': /4) ~ -110.9317 at t = O.
M 33
We use the Newmark scheme to compute the unknown displacement U3 as follow s:
Let Ilt = 0.0025, ex = 1/2 , /3 = 1/4. Then, ao = 4/(llt) 2, a1 = 4/llt, a2 = 1,
a3 = Ilt /2 = a4. Substituting these values and the above boundary conditions on
U1 , U2 , U4 into (C.12), we get

n +1
(1) (
M 33 aoU!3 + a2U
.. )
!3
U3 - (1) (1)'
K 33 +aOM33

This equation is solved for U3 at different times, starting at Ilt = 0 (for n = 0),
which gives U3(0) = 0.2144, and so on. Table 9.4 shows a comparison of the finite
element solution with the Galerkin solution of Example 9.2 for the half-beam .

Table 9.4. (Il t = 0.0025; NE = Number of Elements.)

NE=2 NE=4 NE=6 Galerkin


0.1 0.2098 0.2097 0.2097 0.2157
0.2 0.1950 0.1951 0.1951 0.1988
0.3 0.1695 0.1696 0.1696 0.1716
0.4 0.1345 0.1348 0.1348 0.1356
0.5 0.0930 0.0932 0.0933 0.0925
0.6 0.0480 0.0483 0.0483 0.0447
0.7 0.0014 0.0016 0.0016 - 0.0055
0.8 - 0.0464 -0.0458 - 0.0458 - 0.0553
0.9 -0.0916 -0.0928 -0.0921 -0.1023
1.0 -0.1346 - 0.1341 -0.1 341 -0 .1441.

9.3. Time-Dependent Heat Conduction

Consider the following initiallboundary value problem

pcA et = ax
a ( Akx x aT)
ax + /3P (Too - T) + q, 0 < x < L, t > 0,
7it
T(O, t) = Tb(t),
aT
-k x x ax (L , t) = /3 (T (L ) - Too(t)), t > 0,
T( x ,O) = To(x) , 0 < x < L. (9.23)
218 9. TRANSIENT PROBLEMS

This problem determines the temperature distribution T(x , t) in a cylindrical fin of


length L with an initial (t = 0) temperature distribution To (x) and an internal heat
source q(x ,t ), subject to an applied temperature TB(t) at one end (x = 0) which
is fixed. The fin is submerged in a fluid with temperature Too (t). It is assumed
that the cross-sectional area at x is A(x) , with p(x) as the perimeter around this
cross -sectional area. The heat conductivity kx x , density p, heat capacity c, and
convetive film coefficent f3 are all dependent only on x.

{:i:;},
9.3.1. Derivation of Finite Element Equations. By multiplying

both sides of Eq (9.23) by <p(e) = where ¢~e) and ¢~e) are the linear

1
x~e)
x~ )
e peA tel -a dx=
¢2 t
r:
shape functions, and integrateing over [x ~e) , x~e) ], we get

{ ¢(e) } er
x ~)
e
¢(e) }[ a er
tel -a(Ak x x - )
¢2 X ax
+ f3p(Too - T) +q
]dx .
(9 .24)
By using integration by parts formula, we have

¢(e) } a er et { ¢(e) }
1
x~c) { I x~e)
e tel -a (Ak x x -a ) dx = Ak x x -a tel ,
x~ ) ¢2 X X X ¢2 x~e)

1 a:r
l
x
&e) { } (A k aT) dx
x~c ) a:~) xx ax (9.25)

_ { -Akx x
- aT,
~~ i , } _l x
&e)
Ak x x
{ a:r) } aT
a dx.
Ak -
xxaX xi )
e (e)
a¢2 x
X=X2
(e)
ax
--

Combining (9.24) and (9.25), and rearranging the terms , we get

1
(e )
x2

x~c )
( )

peA { ¢t:) } a;::dx +


¢2 U~
1 (c)
x2

x~e)
Ak x x
{ a¢~e) }
are) ~Tx dx
a¢2
ax

1
x~e) { ¢(e) } (9.26)
+ e tel f3pT dx
x~ ) ¢2

=
-Ak
{ Ak aT
aT,
ax x=x~e)
xx
}
I
+ 1 {¢I
x2
(e)

x~e)
( e) }

¢~e)
(f3pT.
00
+ q)dx .
xx ax x =x&e )

In the terms on the left side of (9.24) we replace T (z, t ) by the finite element interpo-
T (e)(t) } 1x~e) { ¢(e) } er
lation [¢~e)(x) ¢~e)(x)] { 1e) . Thenthetenn e peA tel -a dx
T2 (t) x~ ) ¢2 t
9.3. TIME-DEPENDENT HEAT CONDUCTION 219

is replaced by

r~e) a:~: }~T


l
the term Ak xx{ dx is replaced by
Jxi
C
) a<P2 x
ax

x~c) a<p~e) a<p~e) a<p~e) a<p~e) ]


(Jr xi c )
k
xx
A ax ax
a<p~e) a<p~e)
ax ax
a<p~e ) a<p~e)
ax ax ax ax

and the term li x~e)


c)
{ (e )}
:~e) (3pT dx is replaced by

As a result, we get

where
220 9. TRANSIENT PROBLEMS

Q (e) _ aTI
-Akx x -
ax
(c) }
- { Akxx~
aTI '
X=X 1
(9.27)
_
ox X- X2
(c)

and R( e) is the residual error due to replacement of the exact solution T by the
finite element interpolation. By setting the residual error R (e) = 0, we obtain the
local finite element system

(9.28)

where, if we further assume that p, c, A , k x x ,(J, Tb , and Too are all constants de-
noted by p (e) , c (e), A (e), k (e) , (J(e ) , T~e), and T !:;, ), respectively, in the inter val
[x ~e) , x~e)], then

(e) = p (e) c( e) A (e) l (e) [2 1]


M 6 1 2 '

K
(e) = k( e)A (e )
l( e)
[1 -1] +
-1 1
(J(e) p (e) l (e)
6
[2
1

F (e ) = ( (J(e) p( e)r.(e)
00
+ q (e)) l (e) { 1}
2 1 '
_A (e)k( e )aT ( x (e))} .
Q( e ) _ ax 1
(9.29)
- { A(e)k(e) ~~ (x~e)) .

The derivative boundary condition -k ~~ (L) = (3 (T (L ) - T oo) is applied when


x~e) = L. In such a case, we have

and the corre sponding local finite element system has the following form

p (e)c(e) A (e) l (e) [2 1] {t 1e ) }


6 1 2 tie)
+(
k (e)A (e)
l (e)
[1 -1] +
-1 1
(J(e) p(e) l(e)
6
[2 1]+[0 0
1 2 0 A (e)(J(e)
]){T} e )}
r.(e)
2

( (J(e) p (e )T!:;, ) + q (e)) l( e) { 1} { 0 } { Qi ) }


e
= 2 1 + A (e) (J(e )T 00 + 0 . (9.30)
9.3. TIME-DEPENDENT HEAT CON DUCT I ON 221

9.3.2. Numerical Time Integration. The time-dependent problem


(9.23)-(9.24) is solved numerically by a finite differenc e scheme. We begin by
assuming that the two temperature states T i at time t, and T i+ 1 at time ti+l are
related by

(9.31)

where !:It = ti+l - ti denotes the time step. The relation (9.31) follows from the
trapezoidal rule, where the parameter B is chosen by the user (see Appendix C).
Next, we express Eq (9.29) in the global form as

F=KT+MT.

By using ti and ti+l , we have

F i=KTi+MT i ,
(9.32)
F i+l = K T i+l + M T i+l '

Then multiplying the first equation in (9.32) by (1 - B) and the second by B, we


obtain
(1 - B) (KT i + MT i) = (1- B) F i ,

B (K T i+1 + M T i+l ) = BF i+l,

which, after adding together, gives

Now, using Eq (9.31), we delete the time derivative terms and get

Rewriting this equation, we have

The time integration to solve for T is carried out as follows:


• Given a known initial temperature To at time t = a and a time step !:It;
• Determine T 1 at t = !:It ,which is not known, by using (9.33);
• Use T 1 to determine T z at t = 2 !:It; and so on.
222 9. TRANSIENT PROBLEMS

e
Note that is chosen by the user. Depending on the value of bt may have e,
e
an upper limit for the numerical solution to be stable. Thus , if < 1/2, the largest
bt for stability is bt = ( _ ~) A ' where Am ax is the largest eigenvalue of
1 2 max

(K - AM) T = O.

If e
~ 1/2, the numerical solution is unconditionally stable (see Appendix C). In
the following example we will use the Galerkin scheme (9.33) for which = 2/3. e
EXAMPLE 9 .5 . A circular fin oflength 4 em and radius 0.2 em is made of pure
aluminum with the thermal conductivity k x x = 237W/(m.O) , {3 = 150 W/(m 2. 0C) ,
mass density p = 2702 kg/m", and specific heat c = 903 j/(kg·°C) (lj=l watt -s).
The initial temperature is 25° C. The right tip of the fin is insulated, and the base
of the fin is maintained at a temperature of 85° C. We choose bt = 0 .1 s, and
e = 2/3, and consider a 2-element mesh of equal length of [(1) = [(2) = [ = 2 em
each (Fig. 9.1). We will determine the temperature distribution up to 3 s.

d =0.4 em
~~~X0
L=4em

Fig . 9.1. A Circular Fin .

We use the following three methods.


METHOD 1. CALCULATION BY AN ELECTRONIC CALCULATOR OR
MATHEMATICA .

K(l) =K(2) = Ak[ [1-1 xx -l]+ {3P[


1 6
[21 21]
= 7r(0.002)2(237) [1 -1]
0 .02 - 1 1
150(27r)(0 .002)(0.02) [2 1]
+ 6 1 2 .
9.3. TIME-DEPENDENT HEAT CONDUCTION 223

On assembly the global stiffness matrix is

K~2)]
K (l ) K(l)
11 12
K-
-
K(l)
21
K(l)
22
+ K(2)
11 12
[ K(2) K(2)
O 21 22

-0.1~2628]
0.161478 -0.142628
= -0.142628 0.322956 .
[ o -0.142628 0.161478

The components of the force vector are

F(l) = F(2) = (JT;pl {']

= (150)(25)(27l')(0.002)(0.02) {1} = {0.471239}


2 1 0.471239 '

Thus, from (9.27)

f?) } { 0.471239 }
F = fJ1) + f~2) = 0.942478 .
{ fJ2) 0.471239

Next, each element mass matrix is

M(l) = M(2) = cpAl


6
[21 1]
2
(903)(2702) 7l'(0.002)2(0 .02) 2 1] = [0.204405 0.102203]
6 [1 2 0.102203 0.204405 .

On assembly we get

M (l )
11
M(l)
12 0 ]
M -
-
M(l)
21
M(l)
22
+ M(2)
11
M(2)
12
[ o 21
M(2)
22
M(2)

0.10~203] .
0.204405 0.102203
0.102203 0.40881
[ o 0.102203 0.204405

Now, using the results in (9.33), we have

1 [2.1517 0.92694 0]
AM + BK = 0.92694 4.30341 0.92694 ,
ut 0 0.92694 2.1517

1 [1.99022 1.06957 0]
AM - (1 - B) K = 1.06957 3.98045 1.06957 ,
ut 0 1.06957 1.99022
224 9. T RANSIENT PROBLEMS

where B = 2/3 and !:i.t = 0.1 s.


Thus, the finite element system at time t = 0.1 s is

2.1517 0.92694 0 ] { 85}


0.92694 4.3041 0.92694 T2
[ o 0.92694 2.1517 T3
1.99022 1.06957 0 ] { 85} { 0.471239 }
= 1.06957 3.98045 1.06957 25 + 0.942478 .
[
o 1.06957 1.99022 25 0.47 1239

Since F , = F i + 1, so (1 - B) F i + BFi+l = F, holds for all time. Hence,


T 2 = 27.192° C, and T3 = 24.0557° C. Again, at time t = 0.2 s. we have

2.1517 0.92694 0 ] { 85}


0.92694 4.3041 0.92694 T2
[ o 0.92694 2.1517 T3

1.99022 1.06957 0 ] { 85 } { 0.471239 }


= 1.06957 3.98045 1.06957 27.192 + 0.942478 .
[ o 1.06957 1.99022 224.0557 0.471239

which gives T2 = 29.1168° C, and T 3 = 23.4427° C. The results through a time


period of 3 s are tabulated in Table 9.5 (on page 221) and plotted in Fig. 9.2.

Node 1
85 :
80 :

70 :
U
0
60 .
~
:::l

~Q) 50 :
0-
S
Q) 40 :
E-<
30 :

20 -

IJ 0.5 1.5 2 2.5 3


Time (s)

Fig. 9.2. Temperature Distribution in the Circular Fin.


9.3. TIME-DEPENDENT HEAT CONDUCTION 225

Table 9.5. Temperature Distribution in 0 C.

Time (s) At Node 1 At Node 2 At Node 3

0.0 85 25.0000 25.0000


0.1 85 27.1920 24.0557
0.2 85 29.1168 23.4427
0.3 85 30.8188 23.0992
0.4 85 32.3346 22.9746
0.5 85 33.6943 23.0270
0.6 85 34.9230 23.2221
0.7 85 36.0413 23.5315
0.8 85 37.0664 23.9320
0.9 85 38.0122 24.4045
1.0 85 38.890 7 24.9333
1.1 85 39.7113 25.5055
1.2 85 40.4823 26.1106
1.3 85 41.2103 26.7399
1.4 85 41.9007 27.3864
1.5 85 42.5584 28.0443
1.6 85 43.187 28.7089
1.7 85 43.7899 29.3764
1.8 85 44.3697 30.0437
1.9 85 44.9286 30.7084
2.0 85 45.4687 31.3683
2.1 85 45.9914 32.0220
2.2 85 46.4982 32.6681
2.3 85 46.9903 33.3057
2.4 85 47.4685 33.9340
2.5 85 47.9338 34.5525
2.6 85 48.3869 35.1606
2.7 85 48.8284 35.7581
2.8 85 49.259 36.3448
2.9 85 49.679 36.9205
3.0 85 50.089 37.4852

M ETHOD 2. By ANSYS P ROGRAMMING. We treat this problem as a


two-dimensional problem and use two-dimensional elements under the Ansys
type 'Plane75 Axisymmetric-Harmonic Thermal Solid' . A 16-element rectan-
gular mesh, shown in Fig. 9.3, is considered. The Ansys code is given in §14.2.1.
We choose a set of eight solutions at time t = 1, 2, .. . , 8 s.
The results of the Ansys programming are included in Table 9.7, given below,
for t = 0.1, 1.0, 2.0, 3.0 s, and x = 0, L/2( = 0.02), L(= 0.04) m. Note that in
this output, T 1 occurs at node 1, 2, or 3, of T 2 at node 17, 24, or 8, and of T 3 at
226 9. TRANSIENT PROBLEMS

node 12, 13, or 4 (refer to Fig. 9.3 for the location of these nodes). The output files
are available as Listings 14.2.1 through 14.2.8 in §14.2.1.

13
4

~~
11
~~
10
"5
9
2
8
2,
7

"'~ 6
2J 5

2
1

Fig. 9.3. 16-Element Mesh.

METHOD 3. EXA CT SOLUTION. We solve Eq (9.23), where


f3p
q = A s f3 (T - Too) = -A (T - Too) ,

subject to the boundary conditions

T( x ,O) = 25, T(O, L) = 85, Tx(L, t) = 0, Too = 25, (9.34)


where p is the perimeter (circumference) and A the area of the cross section of the
cylindrical rod. After substituting u(x, t) = T(x , t) - Too, Eq (9.23) reduces to

kxx ~
a2u = pc ~
2
au + -f3p u, (9.35)
uX uX a

and the boundary conditions (9.34) become

u(x ,O) = 0, u(O , t) = 60, ux(L , t) = O. (9.36)

By using the method of separation of variables , the exact solution is given by

u(x , t) = C1 eQ X + C2 e- Q X

_ ~ 60(2n - 1)1T . (2n - l )1Tx - (>.~+ a3) t/a2


~ (2n _ 1)21T2 Sill 2l e , (9.37)
n=1 Q:2 L2 + -'---_--'-_
4
9.3. TIME-DEPENDENT HEAT CONDUC T I ON 227

where a = {lf
Ak'
p
xx
a3 =
{3p
11' a2 =
2
pc, and An = k x x
(2n - 1)21T 2
4L2 ' and 0 1
and O2 are given in Exercise 9.9. Thus, finally we have T(x , t) = u(x, t) + Too =
u(x, t) + 25.
Using the following data: L = 0.04 m, (3 = 150 W/(m 2 . 0C) , k = a1 = 237 xx
W/(m .°C), p = 2702 kg/m" , c = 903 j/(kg 0C), r = 0.002 m, p = 21Tr, A = 1Tr 2 ,

we find that a = ±v f7JP


A'k: = ±25.1577, 0 1 = 7.07307, O 2 = 52.9269,
a3 = 150000, a2 = 2439906, and Ai = 3.65484 x 105 , A~ = 3.28935 X 106 ,
A~ = 9.13709 X 106 , A~ = 1.79087 X 10 7 , and A~ = 2.96042 X 107 . Since
An i 00 as n --4 00 , we will use only these five values of An. Then

A21 + a 3 = 0.21127, A22 + a 3 = 1.04963,


a2 a2
A5+ a3 = 3.80633, A~ + a3 = 7.40139,
a2 a2
A§+ a3 = 12.1948.
a2
The results of computation are presented in Table 9.6.

Table 9.6. Exact Temperature Distribution ( 0C).

Time,s At Node 1 At Node 2 At Node 3


0.1 85 26.3479 25.0000
0.2 85 25.0003 25.0001
1.0 85 32.5438 25.4680
2.0 85 41.8870 29.6437
3.0 85 48.1277 35.3135

Finally, a comparison of the results from the three methods are presented in
Table 9.7. •

Table 9.7. Comp arison of the Results .


Time (s) 1 Node 11 Node 2 Node 3
___I I Ca lculator* Ansys Exact I Calcul at or* Ansys Exact
0.1 I 85 I 25.1920 25.039 26.34791 25.0570 25.001 25.0000
1.0 I 85 I 38.8907 33.786 32.54381 25.9333 26.196 25.4680
2.0 I 85 I 45.4687 42.430 41.88701 31.3683 30.919 29.6437
3.0 I 85 I 50.0890 73.228 48 .127~ 37.4852 68.381 35.3135
* Using an electronic calculator or Mathematica.
228 9. T RANSIENT PROBLEMS

9.4. Two-Dimensional Transient Problems

We use the semidiscrete weak formulation and derive the finite elemen t equation
for the two-d imensional model equation

CI au _ !..- (k l aU ) _ !..-
at ax ax ay
(k 2 aU )
ay
+f = 0 in n, 0 < t ::; to, (9.38)

subje ct to the mixed boundary conditions


au au ,
k l ax n x + k2 ay n y + ,8 (u - uoo ) + q = 0
(9.39a )
(9.39b)
and the initial condition u = Uo in n for t = 0, where CI, k l , k2 , f , ,8, u oo , ij, and
it are prescribed functions of x and y, and possibly t. The sem idiscrete weak form
of Eqs (9.38)-(9.39) on an element nee) with a test function w is

0= JeIrn<c) auat _!..-


[C I (kauax ) _!..-ay (k auay ) + f] wdxdy 2

'r [ wat:au + k awau


l
ax

J
= I n(C) CI
aw aU)
ax ax + k ayay + fw dx dy
l
]
2 (9.40)

+ r
J r <c)nr 2
w [,8 (u - uoo ) + ijn] ds + r
Jr(C)/ r 2
w qn ds,

au au . ()
where qn = - k l ax n x -k y ay n y. We approximate u(x, y,t) on theelement w e

by the semidiscrete appro ximation


n
u(x , t) ~ L u~e)(t)¢~e)(x, y), (9.4 1)
i =1

where ¢~e) are the interpolation shape functions . Substituting the appro ximation
(9.41) for u and ¢]e) for w in (9.40), we get the local finite element equa tions

J1
d . ( a",(e) a¢(e) a",(e) a¢~e» )
",(e) ",(e) --.!2 d d + k _'+'_i J_ + k _ '+'_i J_ d d
{ nee) [CI '+'1 '+'J dt x Y I a
x
ax 2 a
Y aY x y

+r
Jr<C)nr2
,8¢;e)¢]e) dS] u i } +Jer
I n<C)
f ¢Je) dx dy - r
(,8u oo - ij) ¢i ds = 0,
Jr<c)nr 2
for j = 1, 2, . . . ,n, (9.42a)
or, in the matrix notation,
(9.42b )
9.4. TWO-DIMENSIONAL TRANSIENT P ROBLEMS 229

where

Mi~) = Ie ( ¢~e) ¢;e) dx dy,


i-;
1
ci

aA,(e) a¢~e ) aA,(e) a¢(e»)


K~) = kl _ 'f'_i J_ + k2 _ 'f'_i J_ dxdy + (3¢~ e)¢;e) ds,
Iin ee)( ax ax ay ay r <e)nr 2

Fj(e) = { ((3Uoo+ Q)¢j ds + I "{ f ¢;e)dxdy, j = 1,2, . . . ,n.


Jr<e)nr 2 i-
(9.42c)
EXAMPLE 9 .6. Consider the problem of the unsteady flow of a viscous
incompressible fluid in a closed region n E R 2 . We assume that there is no flow
in the z direction , and the velocity field (u , v, 0) along the x and y axes does not
vary with respect to z. Then this flow is governed by the equations
2u
Cl -
au -
a - J.L-
2J.L- a (au
- + -av) + -Bp - fx = 0,
at ax 2 ay ay ax ax
2u
av a (au av ) a ap
Cl at - J.L ax ay + ax - 2J.L ay2 + ay - fy = 0, (9.43 )

au au _ 0
ax + ay - ,
for t > 0, where p denotes the pressure, I-, f y the body forces (if any) along the x -
and y-axis , respectively, and Cl , J.L are material constants. The natural and essential
boundary conditions on r 1 are

a2u . ( au av ) ,
2J.L ax 2 n x + J.L ay + ax n y - pn x = t- ; (9.44a)

au av ) a2v '
J.L ( ay + ax n + 2/1- a y2 n y
x - p ny = t y , (9.44b)

and the initial conditions on r 2 are u = uo, v = Vo for t = 0, where r 1 U r 2 = an.


The weak form ofEqs (9.43)-{9.44) over an element n (e) is given by

0= [C1Wl -au + 2J.L -aWl -au + J.L -aWl (au


- av)
+ax
- --aWl
p - wdx] dxdy
If at ax ax ay ay ax
n <e)

- J wdxds, (9.45 a)
fr ee)
0= I f [C1W2 au + 2J.L aW2 au + J.L aW2 (au + av) _ aW2 p - w2 f ] dxdy
at ax ax ay ay ax ax y
n<c)
(9.45b)

(9 .45c)
230 9. TRANSIENT PROBLEMS

where W I, W2, W3 (variations in u, v , p, respectively ) denote the test function s.


For the element n (e) we approximate u, v, and p by the following semidiscrete
interpolations:
n n m
u;:::: L u}e)(t) r/>}e) (x,y), v;:::: L v~e) (t)r/>}e) (x, y), p;:::: LP} e)(t) 1/J~e)(x , y),
i= 1 i =1 i= 1
(9.46)
where r/>}e) and 1/J(e) are the interpolation polynomials of degree nand m (n ~ m ),
respectively. Substituting (9.46) into (9.45), we obtain the finite element system

where M}j = Mr; is given by M};) in (9.42c), and, in view of (9.42c ) we have
K 11 = 2pS11 + pS22 , K I2 = pS1 2 , K 22 = pS 11 + 2pS 22 , K 33 = 0,
(9.48)
where

11 _
S ij -
11 fJr/>} e) fJr/>je)
~ ~ dxdy ,

11
n {e) ox ox
22 _ fJr/>}e) fJr/>;e)
Sij - ~ ~ dxdy,

Ii:r
n{ e) vy vy
r
Fj = f x r/>je) dxdy , +
i:J txr/>je) ds,

F; = Ii:r
r
f y r/>je ) dxdy, + J tyr/>je) ds ,
fr{e)
11
11
fJr/>(.e) fJ1/J(e) fJr/>(e ) fJ'ljJ(e)
Kj~ = - +----jLdxdy , K;~ = - + ----jLdx dy,
n( e ) ox os: n (e ) vy ox
i, j = 1,2, . .. , n ; k = 1,2 , . .. ,m.
(9.49)
Thus, we write Eq (9.47) as

MU +KU=F , (9.50a)

where

(9 .50b)

Note that Eq (9.50a) is of the type (C.l ) and its temporal solution can be obtained
as in Example 9.3.•
9.4. TWO-DIM ENSIONAL T RANSIENT PROBLEMS 231

EXAMPLE 9.7. Consider the equations of motion of a plane elastic solid in a


n E R2:
region

cPu a (au av) -C33-a (au av)


at ox Cl lox
al---
2 ay
- + C12 -
ay - ay+ox
- - fx =O ,
(9.51)
a2a-v a (au av) - - a (au av ) -f =0 ,
2

ox -ay +-
at 2- C33- ox ay C 12-+C22-
ox ay Y

where aI , a2 are constants that depend on the density of the material , and Cm n ,
(m , n = 1,2 ,3) are the coefficients in the stress-strain relations (see §ILl). The
natural and essential boundary conditions are

(Cll~~ +C12~~) n X+C 33:


x (~~ + :~) n y =tx ,

C33: x (~~ + :~) n x + ( C12 ~~ + C22~~) n y = ty , (9.52)

u = fL, v =v on r~e),

and the initial conditions are

au . -av = Vo. ()e


u = Uo, v = va, at = uo, at on r 2 for t = 0, (9.53)

where r~e) u r~e) = r (e). The weak form ofEqs (9.52) over an element n (e) with
WI, W2 as test functions is given by

We approximate u and v over the element n (e) by

n n
u(x , y) ~ I>i(t)<e) ¢~e) (x, y), v(x, y) ~ I>i (t) (e) ¢~e) (x , y).
i=1 i= l
232 9. T RAN SIENT PROBLEMS

The semidiscrete finite element model equation is obtained by substituting these


approximations into (9.54a,b) and by replacing WI , W2 by ¢~e) :

~
L
J1 [ ",(e) ",(e) d2u~e)
a l '1', 'l'J dt 2
+ 8¢~e)
8
(C8¢~e)
8 11
(e)
U,
+ C8¢~e)
812
(e))
V,

i
i=1 nee) X X Y

8 ",(e) (8 'l'i
",(e) 8",(e) ) (e) ]
+ C 33 - 8'l'- j -8- + - 8-
'l' i
- f X¢ j
.
dx dy -
' (e)
t X¢ j ds = 0,
y y (9.55a)

J1 [a2 ¢~e)¢~e) d dVt~(e) + C


X r (e)

L
n

i=1 nee)
2
33 T
8 ¢ (e) ( a¢ (e)
X
T u~e) +
Y
71-
a¢(e)
vie)
X
)

8 ¢ (e) (
C _a ¢ i_
(e) a",(e) ) ]
u(e) + C zrs.. v(e) - f ",\e) dx dy
+ _ 8 J_
y 12 ax ' 22 ay' Y 'l' J

-i r ( e)
ty¢~e) ds = O. (9.55b)

or
Lii + Kllu + K 12y = F 1 ,
(9.55c)
Mv + K 21 u + K 22 y = F ,
2

or, in the matrix notation ,

o M0] {ii}
[L v + [K
K
ll
K {u}
K v {FF 21
12
22 ] =
1
2 } , (9.55d)

where

L ij = Jr( ¢~e) ¢~e) dx dy , M ij = Jr( ¢~e) ¢~e) dx dy ,


i:
Cl C2
i-:
11 _
K ij -
J1 nee)
( Cll a
a¢~e) a¢~e)
8 + C 33 a
a¢~e) a¢~e))
Y
a
Y
dxdy ,

J1 (
X X

12 _ a¢~e) 8¢~e) a¢~e) a¢je) )


K ij - C 12 8 8 + C 33 a a dx dy ,

J1 (
nee) X Y X Y
21 _ 8¢~e) a¢~e) 8¢~e) 8¢~e))
K ij - C 33 8 8 + C 12 8 a dx dy ,

J1 (
nee) X Y x Y
22 _ 8¢~e) 8¢~e) a¢~e) 8¢~e) )
K ij - C 33 a
x
8
x
+ C 22 8 8 dx dy ,

i
nee) Y Y

pJ = Jr(
I n(e)
fx¢~e)dx dy +
r «:
tx¢~e) ds ,
pJ= J"(
i- ;
fy ¢ je) dx dy + 1 ty ¢ je) ds .
x: (9.56)

Note that Eq (9.57) is of the type (C.6) and can be solved as in Example 9.4.
9.5. EXERCISES 233

9.5. Exercises

9.1. Use the Galerkin method to solve ~~ + w2u = 0, such that u(x , 0) = uo.

ANS . Approximate the solution by taking u = Uo + (Ut - uo) j t:J..t, where


t:J..t is a small time step, Ut the (unknown) value of U at the end of t:J..t, and Uo
the prescribed initial value. Substitute this value of u into the given equation .
This gives the residual

_ Ut - Uo
r- t:J..t +w
2 [
Uo +
(Ut - uo)
t:J.t
t] .
This residual must be orthogonalized with respect to U chosen above. Since
Uo is fixed, the variation of U yields JoLl.t r t dt = 0, or

l o
Ll.t [Ut - Uo
t:J.
t
+ w2 ( Uo + (Ut -t:J.tuo)t)] t dt = 0,

which after integration gives

3 - w2 t:J.t
3 + 2w2t:J..t·
U - U
t - 0

This solution is compared with other solution in Table 9.8 with t:J.t = 0.1. •

Table 9.8. (t:J.t = 0.1)

t Galerkin Trapezoidal Exact


0.0 1.0 1.0 1.0
2.0 0.1392 0.61351 0.1353
4.0 1.9498 x 10- 2 1.8254 X 10- 2 1.8314 X 10- 2
6.0 2.7219 X 10- 3 2.4663 X 10- 3 2.4787 X 10- 3

9.2. Use the Galerkin method to solve the diffusion equation ~~ k ~:~,
subject to the initial condition u(x, 0) = Uo (1 - x 2) for -1 :::; x :::; 1, and the
boundary conditions u(±l, t) = 0 for t ~ O.
. d2u du
9.3. Solve by the Galerkin method the one dof system m dt 2 +C dt + ku = f(t),
subject to the initial conditions u(O) = 0, du(O) jdt = WO o
9.4. Compute the element matrices K and F for Eq (9.40d) for a linear triangular
element by using formulas (5.3)-(5.4).
234 9. TRANSIENT PROBLEMS

ANS . For a linear element ~ (e)

KlJ = cll HlJ + C33H'f} = 4~(e) [cu /h Bj + C33/i/'j],

K ij2 = C12Hij2 + c23H'f] = 4~(e) [C33tJi/i + C12tJi/d,


K'f] = [Kij2]T = C33H~1 + C12Hij2 = 4~(e) [C12tJi/'j + C33tJi/i ],
K~2 = C33H~2 + C12H~2 = 4~(e) [C33tJitJj + Cl2'Yi/'j],
1 A (e) 1 2 A (e) 2
F j = -3- (Ix + qn) , F j = -3- (Iy + qn) ,

where q~ = q~ n x + q; ny, (k = 1,2), i.e.,

9.5 . Solve the plane-wall unsteady heat conduction problem ~~ = ~:~, subject
to the boundary conditions T(O , t) = 1, T(l , t) = 0, and the initial condition
T( x,O) = 0, by using a mesh of 4, and 8 linear elements . Compare the finite
element solution at t = 0.5 with the exact solution which is given by

oo

U( t = 1 - x - -2 '"'"
x,) 1.
~ - Sl Il mrx e
_n2 7l"2t
.
7r n=l n

ANS . The finite element solution at t = 0.5 is compared with the exact
solution in Table 9.9.

Table 9.9. (~t = 0.025)

x 4 Elements 8 Elements Exact


0.125 0.8734 0.8732 0.8733
0.25 0.7467 0.7465 0.7468
0.375 0.6211 0.6206 0.6208
0.5 0.4953 0.4951 0.4954
0.625 0.3712 0.3706 0.3708
0.75 0.2472 0.2466 0.2468
0.875 0.1234 0.1232 0.1233
1.0 0.0000 0.0000 0.0000
9.5. EXERCISES 235

9.6. Solve the unsteady heat conduction problem a:; - \J2T = 1 in the domain
n= {(x,y) : 0< x,y < I}, subject to the boundary conditions T = a on
f = {x
aT
= 1, Y = I} and an = a on f 2 = {x = 0, y = a} for t 2: 0, and
1

the initial condition T( x , 0) = 0, by using a 2 x 2 mesh of 8 linear triangular


elements.
HINT. Take D..t = 0.05 and solve Eq (9.27b) with Cl = 1, k1 = k2 = 1,
f3= 0, and f = 1. The boundary condition implies that T3 = T6 = T 7 =
Ts = Tg = 0, and the initial conditions are Tj = a for all j = 1,2, . . . ,9.
This reduces the finite element to the same type as (9.35a). Usc the t1-scheme,
which gives
LTn + 1 = NTn + G,
where

L=M + t1 D..tn+l K , N = M - (1 - (1) D..tn+l K , G = D..tn+l .


Choose t1 = 1/2 and D..t = 0.05.
ANS . The results for t = 1 are T1 = 0.2993, T 2 = 0.2278, and T 5 =
0.1791. The exact solutions are: T1 = T(O, 0) = 0.2947, T 2 = T(0.5 , 0) =
0.2293, and T 5 = T(0.5, 0.5) = 0.1811.
9.7. A cylindrical can of creamed mushroom soup with an initial temperature of
lOO°F(37.78°C) is heated for sterilizing purpose under hot steam at acontrolled
temperature of

T. (t) = { lOO°F + 15 t, a < t < 10,


00 2500F , 10 < t < 00.
The can has a height of 0.16 m and diameter 0.08 m. The thermal conductivity
of the soup is measured as k = 0.256 W/(m °C) , the convection heat transfer
coefficient is f3 = 35.6 W/(m 2 0C), the specific heat capacity is C = 3220
Ws/(kg°C), and the density is p = 1186.1 kg/m", Simulate the temperature of
the cream soup in the can for a reasonable period of time by using the Galerkin 's
finite difference scheme in time and the linear triangular element in space.
HINT . Refer to §8.2 for calculation of the local matrices ; see also Example
8.4.

9.8. For the differential equation a ~~ + bu = I, where u = u(t), take the linear
2

approximation u(t) ~ L c, cPi(t), where cPl = 1-t/D..t, and cP2 = t ]D..t , and
i= l
derive the associated algebraic equation. Compare the result with that obtained
by the t1-scheme.
ANS. (a + bt1D..t) Un+! = (a -2bt1!:::.t) un + (0.5tt1fn+t1fn+d D..t.
236 9. TRANSIENT PROBLEMS

9.9. Use the method of separation of variables to solve Eq (9.35) subject to the
initiallboundary conditions (9.36).
fp u au .
nT I h .
SOLUTION . vve so ve t e equation al ax 2 = a2 at + a3 u, subject to
the initial and boundary conditions (9.36). Assume that u(x, t) = X(x) T(t),
which after substitution into Eq (9.35) yields

where al = kxx , a2 = pc, and a3 = (3p/A. The only situation where these
two expressions can be equal to each other is for each of them to be constant,
say, each equal to r: Then the above equation is equivalent to the two equations

(9.57)

We notice that for a nonzero solution of this problem the values of /'i, must
be either positive or negative. First, we set r: = fL2, which gives the general
solution of Eq (9.57) as

which, using the boundary conditions (9.36), reduces to

where, with fL2 = a3 , we get

Next, we set r: = - A2 . Then the general solution of Eq (9.57) is

which , in view of the boundary conditions (9.36) , becomes

Hence , u(x, t) = u(/l) (x, t) + u(>')(z , t) can be written as a series of the


form

u(x
,
t) = C e"'x + C e-"'x
1 2
~ Dn
+ L..- sin (2n -l)1l"x
2L
e-(>.~,+a3)t/a2 '
n =l
9.5. EXERCISES 237

(2n - 1)7rylal
where An = 2L . Since u(x , 0) = 0, we find that

~ . (2n - 1)7rx
C1 e
QX
+ C2 e- QX
+ LJ D« sin 2L = O.
n =1

To determine the coefficients D n , since


oo
(2n - 1)7rx
C 1 eQ X + C 2 e- + L QX
D n sin 2L = 0'
n=1

let f( x) = -C1 eQ X - C2 e- Q x • Then f( x) can be represented as a Fourier


sine series :

o; = L J
2 rL
f( x) sin
. (2n - 1)7rx
2L
_
dx = D n ,1 + Dn ,2 ,
o
where

QL . (2n -l)7r (2n-l)7r]


[ oe sin 2 + 2L '

------=---- La e- QL sin. (2n-l)7r


[
2
(2n -l)7r]
+ 2L .

Then
60(2n - 1)7r
Dn = D n ,1 + D n ,2 =
2£2 (2n - l )27r2'
a + 4
which gives the required solution (9.37) .
9.10. Solve U t = U X X ' -7r < X < 7r, subject to the conditions u(x ,O)
f(x), u( - 7r,t) = u(7r, t) , and u x ( -7r , t) = Ut(7r, t), where f(x) is a periodic
function of per iod 27r. This problem describes the heat flow inside a rod of
length 27r which is shaped in the form of a closed circular ring . The exact
solution is (Kythe et al. 2002, p. 153)

L e-
00

u(x , t ) =
n 2t
( an cos nx + bn sin nx) ,
n=O

where a n = -1
7r
L -1r
f( x) cos nx dx, and bn = -1
7r
f1r
- 1r
f (x ) sinnxdx.

9.11. Sol ve Ut = V' 2 u, r < 1,0 < z < 1, such that u(r , z, 0) = 1, u(l , z, t) =
0, and u(r , 0, t ) = °
= u(r, 1, t) . Thi s problem describes the temperature
238 9. TRANSIENT PROBLEMS

distribution inside a homogeneous isotropic solid circular cylinder. The exact


solution is (Kythe et aI. 2002, p. 153)

l:=
00

u(r,z,t) = Cmne-(A~+n211'2)tJO(A.mr) sinmrz,


m ,n=!

where A. m are the zeros of Jo, and

9.12. Solve Ut = Uzz - cu, where c is a constant, given that u(z , 0) = 0, and
u(O, t) = uo, z......
lim u(z, t) = 0 for t > O. This problem corresponds to the flow
oo
of a viscous fluid on an infinite moving plate under the influence of a constant
magnetic field applied perpendicular to the plate. The exact solution is (Kythe
et aI. 2002, p. 182)

u = Uo {ez..;c
2
erfc (_z_
20
+ Jet) + e-z..;c (_z_ -vet)}.
20
erfc

Also , discuss the case when c = O.


9.13. Solve Ut = Uxx, subject to the initial condition u(x,O) = 0 and the
boundary conditions ux(O, t) = 0, and u x (1, t) = 1. The exact solution is
(Kythe et aI. 2002, p. 183)

9.14. Solve u, = Uxx, subjectto the initial condition u(x, 0) = 0 and the boundary
conditions ux(O, t) = 0 and u(l, t) = 1. The exact solution is (Kythe et aI.
2002, p. 183)

u=
~
L) -It [2n
erfc
+Vi
I - x
+ erfc 2n + Ir:+ x] ,or
°
f)°
2 t 2yt
u = I - _I)n 4cos(2n + 1)1l'xj2 e-(2n+l)211'2t/4.
(2n+I)1l'
9.15. Solve the problem: Ut = Uxx, x > 0, t > 0, such that u(O, t) = 0 and
U(x, 0) = f(x) = 1. The exact solution is (Kythe et aI. 2002, p. 290-291)

U(x,t) = erf C~) .


9.5. EXERCISES 239

Fig. 9.4.

9.16. Solve the heat conduction problem

er = -:;1 ora
at ( aT)
r or ' 0::; r < a, t > 0,
0 ifr < a,
T(r,O) = { .
To if r = a,

( a aT
at
+~
a
aT)
or
I
r=a,
= 0
t> o '
such that lim T
r-->O
< 00.

This problem describes the temperature distribution in a cold cylinder of radius


a which at time t = 0 is encased in a thin heated cylindrical sleeve that is
thermally insulated outside, and a denotes its thermal capacitance (Fig . 9.4).
It is assumed that initially the cylinder and the sleeve have temperature zero
and To, respectively, and any temperature drop inside the sleeve is neglected.
The exact solution is (Kythe et al. 2002, p. 302)

T( ) = T. [~ ~ J o hn ria) e-'Y~t/a2 ]
r,t 0 1 + 2a + ~ 1 a ')'; ,
n=1 (1 + 2a + -2-) John)
where "t« are the consecutive positive roots of the equation J 1(x) + a ')' Jo(x) =
O.
9.17. Solve the problem of one-dimensional heat diffusion down a semi-infinite
channel filled with a fluid that moves at a uniform velocity. The entrance
of the channel is kept at a constant temperature and the sides of the channel
are insulated. This provides variations in temperature through the channel.
The channel is 1 m long and 1 em wide . The entrance is kept at temperature
T 1 = 1233° K, and the initial temperature of the channel material is To = 298 0
K. The fluid velocity is uniformly maintained at 0.5 mls. This problem deals
with the advection-diffusion analysis and is governed by
m: or a2T
p Cp at + U ax = k ax 2 ' 0 < x < 1, (9.58)
240 9. TRANSIENT PROBLEMS

where p denotes the density, Cp the specific heat, k the thermal conductivity, x
the axial distance along the channel, and t the time. The boundary and initial

°
conditions are: T (O , t) = T 1 = 1233 for t > 0, and T (x , 0) = To = 298 for
< x < 1. The exact solution of this problem is (Carslaw and Jaeger 1959,
p.388)

T(x, t ) = To + ~ (T1 - To ) [erfC (~~) + eUx/ex erfc ( ~~)] ,


(9.59)
k
where a = - . Use the data: p = 2698 kg/m", cp = 900 j/kgO K, and
pCp
k = 226 W/m°K.
ANS . The solution (Table 9.10) shows a wiggle effect at x = 0.18 m and
t = 0.5 sec, with Cit = 0.01. Despite large oscillations, the solution is very
close to the exact solution , as shown in Fig. 9.5 (see Sahai 1991, for details) .
A Fortran program to compute the exact solution (9.59) is given in §14.4.2.

TempOK

Finite Element Solution


Exact solution at 0.5 sec

0.1 0.2 OJ

Fig. 9.5.
10
Nonlinear One-Dimensional Problems

The mathematical models considered in this chapter involve only a single nonlinear
differential equation with one unknown, which is one-dimensional in the space
variable. These equations are encountered mostly in problems of radiation heat
transfer, stress in plastics bars, non-Newtonian fluid flows between parallel plates,
and turbulent flows in tubes . We introduce the standard Newton's method , the
method of steepest descent, and some nonlinear conjugate gradient methods for
numerical solutions of the corresponding finite element nonlinear problems. Both
Galerkin and Rayleigh-Ritz finite element methods are used to drive the finite-
dimensional finite element equations from nonlinear differential equations and
their respective boundary condit ions in idealized situations. For simplicity, only
linear elements are used in the finite element methods .

10.1. Newton's Method

Let y = f( x) denote a real-valued function from R I to R I. We are interested in


solv ing for x such that f( x) = O. One of the most important methods for finding
such an x numerically is the so-called N ewton's method. We describe a simple
algorithm for the Newton's method as follows:
STEP 1. Take an initial guess Xo and check if f (xo) is zero (most likely not).
If f( xo) 1:- 0 go to Step 2; otherwise stop.
STEP 2 . Calculate !, (x) and check if !,(xo) equals zero . If !'(xo) 1:- 0 go to
Step 3; otherwise go back to Step 1.
STEP 3 . Write down the equation of the tangent line of the graph of y = f( x)
at (xo, f(xo)), which is y - f(xo) = !,(xo)(x - xo).

P. K. Kythe et al., An Introduction to Linear and Nonlinear Finite Element Analysis


© Springer Science+Business Media New York 2004
242 10. NONLINEAR ONE-DIMENSIONAL PROBLEMS

STEP 4. Find the x-intercept of the tangent line by solving - f(xo)


f'(xo)(x - xo) for x, and obtain x = Xo - J,~::)).
STEP 5. Denote the x-intercept obtained in Step 4 by Xl, and check Xl - Xo
or f(xo) to see if either is smaller than a prescribed tolerance. If the tolerance is
satisfied, then take Xl as the numerical approximation of a solution of f(x) = 0
and stop; otherwise go back to Step 1.
The basic idea in the above-described Newton's method is to use the formula
Xl = Xo - J,~:~) repeatedly until Xl - Xo or f(xr) becomes sufficiently small.
Each time, when Xo is given and the next value Xl is calculated, we say that an
iteration is complete, and Xo is referred to as the old value and Xl the new or current
value. The first value of Xo is very important and is referred to as the initial guess,
and the subsequent values of Xo are the values of Xl obtained from the previous
iteration. It is well known that Newton's method works very well if the initial
guess Xo is good and there is no guarantee of convergence if the choice of Xo is
inappropriate.
EXAMPLE 10.1. To solve the cubic equation + 9x - 2 = 0 numerically
x3
by using the Newton's method described above, note that by the intermediate
value theorem, there is a zero of f(x) = x 3 + 9x - 2 in the interval (0,1) since
f(O) = -2 < 0 and f(l) = 8 > O. Let Xo = 0,5 and calculate f'(x) =
3x 2 + 9x, then we have f(0 .5) = 2.625 and f'(0 .5) = 9.75. Thus, Xl = Xo -
J,~::)) = 0.230769. This completes one iteration in the Newton's method. If one
continues the iteration procedure, the sequence {x n : n = 1,2, . . , } so obtained
will converge to 0.221023, which is one of the three solutions of the equation. As
one can see that after only the first iteration, the number 0.2308 obtained is a good
. , . herelati d'ffi . 0.230769 - 0.221023
approximationto Oizz'luzs sincet ere ative 1 erence is 0.221023
0.044095. In fact, the other two solutions are imaginary as they can be explicitly
given by the cubic formula (see, e.g., Uspensky 1948).

Now, let us consider vector-valued functions from R 2 to R 2 . Let x = {~~ },


and f = { ffl ((Xl, X2)) }. The corresponding formula for the Newton 's method is
2 Xl, X2

where Xo is the old value, Xl is the new value, and


10.2. RADIATION HEAT TRANSFER 243

is the gradient matrix of f evaluated at xo. The matrix V'f(x) is also called the
Jacobian of f and is denoted by J(x) or Z~h , h~. For vector-valued functions
Xl,X2
from R N to R N ,

..
ah (xo) ah (xo) ah (xo)
aXl aX2 aXN
x= { ] , and V'f(xo) =
aiN'(xo) aiN (xo) 8fN (xo)
aXl aX2 aXN

EXAMPLE 10.2. Solve the system of equations numerically by using the


above-described Newton's method:

XI - x~ + 6Xl - 4 = 0,
xI + X2 -1 = O.
Note that

f(x) = {h(X)}
h(x)
= {XI -2X~ + 6Xl - 4},
Xl + X2 - 1

and

Vf'(x) = [2X~x: 6 -~X2].

Let Xo = },
{~ } be the initial guess. Then f(xo) = { =~ vr (xo) = [~ ~],
and Xl = {~} - [~ n -1 { =~} = {~:~~~~}. Repeating, we get X2 =
0.6333 } { 0.6513 } { 0.6513 } { 0.6513 }
{ 0.6000 ' X3 = 0.5762 ' X4 = 0.5759 ' and X5 = 0.5759 . One can

check that { ~:~~;~~ } is indeed a solution of the system .•

10.2. Radiation Heat Transfer

For simplicity, we first consider a straight cooling fin of length L with cross sec-
tional area A(x) (Fig. 10.1). It is assumed that one end of the fin located at X = 0
is attached to a base kept at a fixed temperature To; the other end located at X = L
is allowed to undergo gray body radioactive heat transfer to the ambient tempera-
ture T ex" and the lateral surface of the fin is insulated. It is also assumed that the
244 10. NONLINEAR ONE-DIMENSIONAL PROBLEMS

temperature at the cross section located at x is constant throughout that section and
an equilibrium state has been reached in the fin.

Base 1- - -- - - - - - - - - - - - - -
Fig. 10.1. A Cooling Fin.

A mathematical model that determines the temperature distribution in the fin


can be defined as the following two-point boundary value problem:
d2T
-kA dx 2 = q, 0 < x < L,
(10.1)
T(O) = To , and - k ~~ (L) = (J£(T 4(L)
- T~),
where T = T(x) denotes the temperature, q = q(x) the heat source, A = A(x) the
cross-sectional area at the location x , k the conductivity, IJ the Stefan-Boltzmann
constant, and e the emissivity constant. The conductivity k considered here is
usually a constant or piecewise constant, although it could depend on x in certain
applications. For details of derivation of Eq (10 .1), see, e.g., Sparrow and Cess
(1978).
If the lateral surface of the fin is also allowed to undergo gray body radiation
heat transfer, we have the following slightly more general problem:

d 2T
-kA dx 2 + AslJc(T 4 - Too)
4
= q, 0 < x < L,
(10.2)
T(O) = To , and - k ~~ (L) = 1Jc:(T 4
(L) - T~),
where the additional quantity As denotes the total lateral surface area of the fin.
Now, we proceed with the linear interpolation shape functions and the Galerkin
method to approximate the temperature distribution T determined by the problem

(10.1). Letfp(e)(x) = {q)t:;(X) } , where q)~e)(x), i = 1,2, defined by (2.2), are


q)2 (x)
the linear interpolation shape functions. We start with the Galerkin finite element
local method. Multiplying both sides of Eq (10.1) by ¢(e)(x), then integrating
e)
over [xi , x~e)] and using the integration by parts formula, we get

r r
x(e) (e) x(c ) (c)

2 kA d¢ (X) dT dx= 2 ¢(e)(x)qdX+kA¢(e)(x)dT\ X2 • (10.3)


Jxr. ( C)
1
dx dx } x (e)
1
dx x ~c)
10.2. RADIAT ION HEAT TRA NSFER 245

T (e) }
Let T (e) =
{ Tj e) , where Tie) and Ti e) denote the nodal temperatures at nodes
x~e) and x~e), respectively. The linear interpolation equat ion of the line connecting
the two points (x 1(e) , T 1(e») and (x 2(e), y,(e»)
2 is

y,(e) T (e)
T (e)(X ) _T{e)
1
= 2 -
(e)
1 ( _
(e) X
(e» )
Xl ,
x2 - Xl
which can be rearranged in the form

We have

dT( e)(x) = [ <p~e)(X) <p~e) (X) ] {Ti e)} = !:...(<p{e)(x)) TT (e).


dx ~ dx Ti e) dx

Replacing T (x ) in Eq (10.3) by T {e)(x ) on the left side, we get

where the term R (e) = { Rt


R(e)}
e) is the resulting error due to the replacement of the

true solution T (x ) by the linear interpolation T {e)(x ). For simplicity, we assume


that k, A , and q are equal to the constants k{e), A (e), and q(e), respectively, in the
. [ (e) (e)] . d (e )() 1 d (e)( ) 1
element Interval X l , x 2 . SInce dx <PI X = ~ and dx <P2 X = - ~ '
where [(e) = x~e) - x~e), we have

1 ] dx
__
[(e )
= k(e)A(e) [ 1
[{e ) - l
-1]
1 .

(10.5)
Also,

1x,
(e)
x2

( 0)
q<p(e)(x) dx = 1x, (e )
x2

(e)
q{e) {
( )

<PIe (x)
",{e ) (X)
'1'2
} dx = e»:
( ) ( )

2
{I} .
1
(10.6)
246 10. NONLINEAR ONE-DIMENSIONAL PROBLEMS

( 0)

Let Q(e) = Q ~e)


e
( )
} dTlx2 + R(e) , then
denote the term kAfjJ(e)(x)-d
{Q x xl")
2

_ke elA(e) dT (x (e») + R(e) }


Q (e) _ dx 1 1
- { keelA (e)dT (x(e») + R(e) .
dx 2 2

By using these calculations, Eq (lOA) can be rewritten as

k(e)A (e)
lee )
[1 -1]
-1 1
{TI e)} = q(e)l(e)
r. (e ) 2
{I} + {Q~e)}
1 Q(e) , (10.7)
2 2

which is the local finite element system K( e)T(e) = F (e) + Q (e) , or simply the
local syst em. The derivative boundary condition -k ~~ (L) = h(T(L) - Too ) for
the boundary value problem (10.1) is applied when x~e) = L, and TJe) = u(L) is
unknown. For one-dimensional problems, this usually happens when the element
number e = N and x~N) = L at the end point. In such a case , the corresponding
boundary term Q (e) can be written as _K~e) + F~e) + R (e) , where

and the corresponding local finite element system has the following form :

-11] {~i:; }+ [~ A(e)crc~Tie»)4]


= -2-
q(e)l(e) {I} + { 0 }+
1 A(e)crcTj
{Q (e) }
Q~e) ,

with obvious boundary contributions. In the idealized case when the coefficients
k, A, and the source q are constants in the entire solut ion interval [0, L], and
the element intervals are chosen to have an uniform length l = lee) =
1, . .. , N, where N denotes the total number of partitions of [0, Lj , the global
t, e =

system corresponding to the standard labeling is

KT=F+Q ,
where
1 -1 0 o
-1 2 -1
keelA (e)
K= lee) 0
-1 2 -1
0 o -1 1
10.2. RADIATION HEAT TRANSFER 247

1
2
and Q=
2 Q ~N-l ) + Q ~N)
1 AO"c(Tt+1 - Tj)

Once again , in order to obtain a complete algebraic system with equal number of
equations and unknowns, the inter-element quantities Q~ + Qi , . .. , Q~- l + Q~
are set to zero and the result is

Q=

Since the temperature at x = 0 is prescribed as T(O), there are only N unknown


nodal temperatures , and the final complete algebraic system is

1 - 1

k(e)A(e) -1 2
l (e ) o
o

which has N + 1 equations and N + 1 unknowns, namely, Q ~l), T 2, . . . , TN+l .


Note that if the derivative boundary cond ition is k ~~ (0) = O"C (T4 (0) - T~) ,
the Dirichlet condition is T (L) = To, and x~e) = 0, then

(e) {A( e)O"cT~ }


and Fb = (e) .
Q2

This usually happens at the first element e = 1. Then the corresponding final
248 10. NONLINEAR ONE-DIMENSIONAL PROBLEMS

system is

1 -1 0 0 T1
k(e)A(e) -1 2 - 1 T2
l(e) 0
-1 2 -1 TN
0 0 - 1 1 T(L)
1 Aae((T1 ) 4 - T,;,)
q(e)l(e) 2 0

2 +
2 0
1 Q ~N)

EXAMPLE 10.3. Consider a very thin cylindrical cooling surface (fin) made
of pure copper with cross-sectional area A = 7f R 2 , with R = 4.0 m and length
L = .04 m. The following data are given: k = 400 W/(m .oK), T(O) = 100°C
(= 373.15 °K), Too = O°C (= 273.15°K), a = 5.675 x 10- 8 W/m 2 .oK4, and
e = 0.76. The following assumptions are made: (i) the lateral surface is insulated ;
(ii) the temperature variation in the lateral direction is negligible, and hence, a one-
dimensional analysis is sufficient; (iii) no heat is generated or dissipated within
the surface; and (iv) the surface behaves like a gray body (the emissivity and the
absorptivity of the surface are the same) . With these assumptions, the system
(10.1) can be used as the mathematical model with the x-axis set at the center of
the surface as indicated by Fig. 10.2.

cp
Too

T(O)~ :;,
X
I

I
L
Fig. 10.2. A Cylindrical Cooling Surface or Cooling Fin.

We now use two linear elements and Newton's method to approximate temper-
atures located at x = 0.02 m and x = 0.04 m. The corresponding global system
becomes

-1
kA
2
I - 1
10.2. RADIATION HEAT TRAN SF ER 249

Numerically, we have the closed algebraic system

[ ~1o ~1 ~1]{37~~15}+a{~1
-1 1 T3 (T3 )4
}=a{ ~
(273.15)4
}, (10.10)

where the quantity a = 1.6565 x 10- 12 . Uncoupling the first equation from the
system, we have

373.15}
[1 - 1 0)
{
~~ +aQ1 =0, (10.11)

and

[-;;1 :1 ~1] r~n + { ;')4}~ {(273015)'} a (10.12)


Reorganizing Eq (10.12), we get

[~1 ~1 ] {~~ } + 1.6565 X 10-


12
{ (T~)4 } = { 9.22~:3~1~0-3 l
(10.13)
Applying Newton 's method , we get T 2 = 348.7696°K, T3 = 324.3892°K, and
Q1 = - 75086.3W. The exact solution for the temperature is obtained by T(x)
solving the boundary value problem
2T
kr ddx 2 = 2<Jc (T 4 - Too
4) + 2/3 (T - Too ) ,
dT
T (O ) = T B , and dx (L) = 0,
which has the soluti ons (see Sparrow and Cess 1978)

-1M
1{I ( jGT~
jGT~+ M--IM) I (V GT3+ M - -IM ) }
n + M + -1M - VGT3 + M + -1M =X n
'f
1 /3 1= 0,

T = (T; 3/2- 1.5GX) - 2/ 3 if /3 = 0,

where M = 2/3 / (kR ), G = 4pc/ (5kR); T B denotes the base temperature of the
fin, and x the distance along the fin. The values ofT obtained by the finite element
method arc compared with the exact solution at x = 0.02 and 0.04 in Table 10.1.•

Table 10.1.

x FE Solution Exact Solution


0.02 348.7696 373.4
0.04 324.3892 373.05
250 10. NONLINEAR ONE-DIMENSIONAL PROBLEMS

EXAMPLE 10.4. Consider a cooling fin of cylinder shape made of the same
pure copper as in Example 10.2 of cross-sectional area A = 1f R 2 , with R = 0.002
m and length L = 0.02 m (see Fig. 10.2). The material data are as follows:
k = 400 W/m.oK , T(O) = 100°C (= 373.l5°K), Too = O°C (= 273.l5°K), a =
5.675 x 10- 8 W/(m .2oK4 ) , and e = 0.15 . The following additional assumptions
are made: (i) the lateral surface is allowed to undergo radiation and convection
to the surroundings; and (ii) no heat is generated or dissipated within the surface;
therefore, q = O. With these assumptions, the system (10.2) can be used as the
mathematical model.
We again use two linear elements and Newton's method to approximate tem-
peratures at x = 0.01 m and x = 0.02 m. The corresponding local system is

(10.14)

where the local stiffness matrix K(e) (T(e)) is a nonlinear function of T(e) and is
given by

(T[e)) 4

2 1 1 (T[e)) (Tie))
3 2

~ (T[e)) (Tie))
iJ
15 10 15
+ A s ac;l(e) [
1
2 2
1 1 2
30 15 10 15 T[e) (Ti e))3
(Tie)f

Then the boundary term Q(e) is decomposed as the sum K~e) + F~e), where

For two linear elements of equal length l = L/2 and the boundary conditions, the
10.2. RADIATION HEAT TRA NSFER 251

corresponding global system is

o ]{ T(O
)}
~1
- 1
LkA
- 2 -1 T2
2 [ - 1 2 T3
1 2 1 1 1
- - - - - 0 0 0 0
6 15 10 15 30
L 1 1 1 2 1 2 1 1 1
+A SO'€ "2 -
30
-
15
-
10
-
15
-
3
-
15
-
10
-
15
-
30
x
1 1 1 2 1
0 0 0 0 - - - - -
30 15 10 15 6
(10.15)
(T( O) )4
(T(0)) 3(T2)2
(T(0)) 2 (T2)2

X
T (O) (T2)3
(T2)4
(T 2)3T3
-~
-
L
4
r} {
2
1
+ 0
Q, }
AO'€ (Ti - T~) .
(T2)2(T3)2
T2 (T3)3
(T3)4

By using the data: k = 400 W/m.oK, 0' = 5.675 X 10- 8 , e = 0.15, T (O ) =


373.15°K, Too = 273.15°K, q = 0, A = 1TR2 = 1.2566 X 10- 5 m2 , As =
21TRL = 2.5133 x 10- 4 m2 , we get

- 1
5.0265 X 10-
5
[ ~1 2 -1
-1 2
o ]{ 373T 15 }
T3
2

1 2 1 1 1
- - - - - 0 0 0 0
6 15 10 15 30
1 1 1 2 1 2 1 1 1
+ 2.1384 X 10- 14 - - - - - - - - - x
30 15 10 15 3 15 10 15 30
1 1 1 2 1
0 0 0 0 - - - - -
30 15 10 15 6
1.9388 X 1010
5.1958 X 107 T 2
1.3924 X 105 (T2)2
373.15 (T 2 )3
x (T2 )4 a, }.(10.16)
(T 2 ) 3 T 3
=
{ 1.0697 x 10- 13
0
(Ti - 5.5668 x 10 9
)
(T2)2 (T3)2
T2(T3)3
(T3)4
252 10. NONLINEAR ONE-DIMENSIONAL PROBLEMS

~:C:U:I~:::h:::~:~:.ti:1frU~ r1;;~}::.::::'~~:::::
1.9388 x 1010
5.1958 X 107T2
1.3924 X 105 (T2 )2
373.15 (T 2 )3
2 1 1 (T2 )4 (10.17)
15 10 15 (T2 )3 T3
(T 2 ) 2 (T3 ) 2
T 2 (T3 )3
(T3 )4
and
5.0265 x 10-
5
[~1 ~1 ] {~~ }
1 2 1 1

+21384 x 10- [I: I: 12


~ 3
1
15
1
10
1
15
2
30 15 10 15
5.1958 x 107T2
1.3924 X 105 (T2)2
373 .15 (T 2)3
(T2 ) 4
x
(T 2 )3 T 3 = { 1.0697 X 10- 13 (~ - 5.5668 x 109 ) }
(T2)2(T3)2
T 2 (T3 ) 3
(T3 )4
- 1.8756 x 10- 2 { ~1 } -4.1459 XlO- 2 {1/030}. (10.18)
Numerical iterations using Newton's method yield

10.3. Stress Analysis of Plastic Rods

For many metals and alloys, the stress and strain can approximately satisfy the
following generalized Hooke's law
(10.19)
where K and n are constants with values for a given metal or alloy depending on
its thermomechanical history, and e denotes the strain. These types of metals are
10.3. STRESS ANALYSIS OF PLASTIC RODS 253

classified by engineers as plastics, yet they are not the usual plastics we use in
everyday life. This generalized Hooke's law is also called the power-law . Note
that for n = 1, Eq (10.19) reduces to the usual Hooke's law for elastic materials
with Young's modulus E in place of K. Table 10.2 is a partial list of values of K
and n for some commonly used industrial alloys (see Shackelford 2000).

Table 10.2.

Alloy K[MPa(ksi)] n

Low-carbon steel (annealed) 530(77) 0.26


304 Stainless steel (annealed) 1275(185) 0.45
Brass, 70Cu-30Zn (annealed) 895(130) 0.49
2024 Aluminum alloy (heat treated) 690(100) 0.16

Stress analysis of machine parts or structures made of these type of metals or


alloys is very important in industrial applications. For simplicity, consider a rod
made of a plastics material satisfying the power-law (Fig. 10.3) .

I====::::jt:_ f
(x_) --I~X
L

Fig . 10.3. A Plastic Rod .

Suppose that the rod is subject to only longitudinal body force j(x) and its
cross-sectional area is denote by A (x). Suppose that one end of the rod located at
x = 0 is fixed while the other end is stress-free. When the rod is at equilibrium,
by balance of forces, we have
dAO"
+ Af =
A

dx 0,

which with (10.19) gives the ordinary differential equation

_.!!- (KAIdu I n
-
1
dU) = Aj '
dx dx dx

where j(x) represents the axial body force at x. Let A j = f. Then we have the
following two-point boundary value problem:

d ( Idu
- dx KA dx
In - 1 dU)
dx
_
- I, 0 < x < L,
(10 .20)
du
u(O) = 0, and dx (L) = 0,
254 10. NONLINEAR ONE-DIMENSIONAL PROBLEMS

which will determine the longitudinal displacement u(x) as its solution. For linear
finite elements, if A, K, and f have the constant values A(e), tc»,
and fee) ,
respectively, in the element interval [x~e) , x~e)], then the corresponding local system
is
(10.21)

where the local stiffness matrix K(e) (u(e)) is given by

K(e) (u( e)) =


K( e)A(e) lu( e) - u(e)
lee)
2
lee)
1
I
n- 1 [ 1
-1
-1]
1 '

and

The corresponding global matrices K, F, and Q are

= Iu i(e) - I
n- 1 .
where we have used the notation
• _0
lIi ,i+1 (e)
ui+l ,2 = 1,2, .. . , N , and

In order to obtain a complete algebraic system, the inter-element quantities Q~I) +


(2 ) (N-l)
Q 1 , . .. , Q 2 + Q (N)
1 are set to zero and the result
.
IS

..
Q~1)
0
Q=
0
Q~N)

EXAMPLE 10.5. Consider a prismatic bar made of 304 annealed stainless


steel rigidly fixed at the top point and subject to a vertical load of 1000 lb/in.
10.3. STRESS ANALYSIS OF P LAST IC RODS 255

Determine the deflections, reactions, and stresses for the properties data: L = 420
in, A = 10 in 2 , K = 77,000 lb/in '', n = 0.26, and p = 0.2 8907Ib/in 3 . We set the
coordinate origin at the top point and let the positive direction of the x-axis point
to the bottom of the bar. The body force f equals the sum of the applied con stant
load oflOOO Ib/in and the gravitational body force of pA(L - x) = 2.8907(L - x)
lb/in.
For simplicity, we will use only two linear finite elements of equ al length to
approximate the solution. Thus, by Eq (10 .21), the finite element system is

[ (e)
I
K( e)A (e) u~e) - u~e)
[(e)
In- 1 [ 1
-1
-1] {~te)
1
(e) }
=
{ / e)}
f~e) +
{ Q(e) }
Qt e) ,
(10.22)
where the local load vector

I} {J:t-~) (L - x)¢~e)(X)dX}
{ + 1
2.8907
J?c)(L -
( e)
e
( )
X)¢2 (x) dx
.

Xl

After some calculation, we get

5L2 }
{ f Jl)
f~l) } = lOOOL { 1 } + 2.890 7 24 = 105 { 2.1123}
4 1 { L2 1.8999 '
6
and

{
1~2)
f J2 )
} = lOOOL
4
{I}
1
+ 2.89 07 { ~~L2 10
} = 5 {1.4749} .
1.2623
24

Since K (e) = K, A(e) = A , [(e) = L /2 , I (e) = I, u ~1) = U1 , u~1) = U2 = U~2),


and u~2) = U3, we have the global system

KA2 n [ IU2 - U1ln-l - IU2 - U1l n-l


-- n- -IU2 - U1ln-l IU2 - U1ln-l + 1U3- 2! n- l u

«,} ~l
L 0 -IU3 - U21 n-1

x { ~~
U3
} = {
I J2)
+{
Q3
} .

Applying the boundary conditions U1 = u(O) = 0, Q3 = ~~ (L ) = 0, substituting


numerical values for the known quantities, uncoupling the first equation from the
256 10. NONLINEAR ONE-DIMENSIONAL PROBLEMS

rest, and simplifying, we get

1.9174 X 105 [-IUz l- O.74 0] {~:} = 2.1123 X 105 + a. ,

and

These nonlinear algebraic equations can be solved explicitly by adding and sub-
tracting the two equations, which yields Uz = 29.8613 in, and U3 = 30.0616
m. •

lOA. Power-Law Pressure Driven Flow between Two


Parallel Infinite Plates

Similar to solids, a class ofvery important non-Newtonian fluids is called power-


law flu ids . Their rheological behavior can be described by the following simple
relationship between stress and strain in the fluids:
n- l
Tx y =K I,xy I , xy ,

where T is the shear stress, ,xy the shear strain, K the consistency coefficient, and
n the fluid behavior index. For n = 1, it reduces to the usual Newtonian model.
This model is appropriate for many polymers as well as food fluids (see, e.g.,
Steffe 1992). The fluids are said to be shear-thinning for a < n < 1, and shear-
thickening for 1 < n < 00 . Tables 10.3 and lOA list some values of K and n
for some commonly used industrial polymer melts and liquid foods. In a simple
situation, we consider a power-law fluid that is sandwiched between two parallel
infinite plates one of which is moving with a constant speed uo in the positive x
direction and the other is kept stationary (see Fig. lOA).

Fig. lOA. Couette Flow between Two Parallel Infinite Plates.


10.4. POWER-LAW P RESSURE DRIVEN FLOW BETWEEN PLATES 257

This is referred to as the plane Couette flow, which is frequently used to model
flows in thin channels. We assume that the flow is steady -state and gravity forces
are neglected . The distance between the plates is denoted by L . By balance of
force s, we have
fhx y
ap _ 0 0 < y < L,
ay ax - ,
_

from which we obtain the differential equation

_K~(ldul n-l dU) __ dp 0 < y < L.


dy dy dy - dx '

Since ~~ = 0, we find that u is only a function of y and ~~ equals to a constant.


Thus, after replacing aay by ~
dy
and aa by dd ,respectively, in the ab ove equation,
x x
we have the following two-point boundary value problem:
n- 1
_ K~ ( ldUl dU) = _ dp, 0 < y < L,
dy dy dy dx (10 .23)
u(O) = 0, u(L) = uo.
Table 10.3.

Polymer T (Kelvin) n

Nylon 493 2.62 X 10 3 0.63


. Polystyrene 463 4.47 x 104 0.22
High-Density Polyethylene 453 6.19 x 10 4 0.56
Polycarbonate 553 8.39 X 103 0.64

Table lOA.

Food Products T (0C) K (Pa· s" ) n


Mayonnaise 25 6.4 0. 55
Mustard 25 27.0 0.34
Minced Fish Paste 3- 6 8.55 0.91
Carrot Puree 25 24.16 0.228

In this model , the fluid velocity u is determined along the y-axis for a given
constant pressure gradient ~~. The above differential equation can be derived
dire ctly from the general Navier-Stokes equations in three dimensions as well. By
using linear elements the corresponding local finite system is

(10 .24)
258 10. NONLINEAR ONE-DIMENSIONAL PROBLEMS

where the local stiffness matrix K(e) (u(e)) is given by

K( e) (u(e)) =
l< e)
(e)
...!!:...-I U2 - U1
l(e)
(e) n-l [
I 1
-1
-1]
1 '
and
F(e) = _ dxdp ~2 {I}
1
.
If f is constant in the entire interval [0, L], then the corresponding global system
is
K(U) U = F+Q,
where K (U), F, and Q are analogous to those given in §1O.3.
EXAMPLE 10 .6 . A sample human blood is assumed to satisfy the power-
law model, in which the flow consistency index K is experimentally measured as
16.66 x 103 pa -s", and the non-Newtonian behavier index n as 0.708 (see Hussain,
et aI., 1999). Suppose that the sample is being transport through a narrow channel
between two stationary parallel plates of 0.01 m apart driven by a constant pressure
grad ient of - 2.59 x 103 N/m 2 . Approximate the vilocity profile ofthe blood sample
in the chann el by using two linear finite elements.
Let z -axis be set on the mid-line between the two plates . Due to symmetry,
we assume that d~~O) = 0 at the center line of the channel. We also assume that

u (L ) = u(0 .005) = 0 due to viscosoty and the no-slip condition at the plate wall.
The calculations are simlilar those in Example 10.5. By using the same elements
of equ al length, we have the global system

du
Applying the boundary conditions Ql = K dx (0) = 0, U3 = u(0.005) = 0, and
substituting K = 0.980, n = 0.708, L = 0.005, and - ~: = 2.59 x 103 into the
above equation, we get
10.5. MIXING-LENGTH EQUATION FOR TURBULENT FLOW 259

and
-1.1586 X 103IU2\-o.292U2 = 3.2375 + Q3.
The first nonlinear algebraic equations can be solved explicitly by adding and
subtracting the two equations; thus, Ui = 1.1 X 10- 3 mis, U2 = 8.5679 X 10- 4
mls. The last equationgives Q3 = -l1.0375N/m2 , which gives the shear stress
on the plate wall. •

10.5. Mixing-Length Equation for Turbulent Flow in


Pipes

Fluid flows can be classified as laminar or turbulent. The flows in a pipe or


along a flat plate are classified according to the so-called Reynolds number R e =
(P:D), where p is the density of the fluid, p, its viscosity, Vthe bulk velocity,
and D the dimension of the geometry involved. A laminar flow exists in a pipe
when the Reynolds number is less than about 2100, while a turbulent flow may
occur when it is above 2100. The following is a very simple mathematical model
that is used for modeling turbulent flows near a wall:
du
Tx du -
= -p,- p(KY) 21 -I-,
du
y dy dy dy
where T x y is the total average shear stress; u is the mean velocity, which is the
instantaneous velocity averaged over a finite time interval determined by the fluc-
tuation pattern of the flow; K is the von Karman constant, which has a numerical
value of approximately 0.4; and Y is the distance from the wall. The following
two-point boundary value problem can be used to determine the average velocity
for turbulent flow in a a circular pipe of radius R:

_~
dy
(p, dy
du + P(Ky)21 du I dU)
dy dy
p
= _ d ,
dx
0 < Y < R,
(10.25)
Bu
By (R) = 0, u(O) = O.

The condition u(O) = 0 corresponds to the no-slip wall condition and :~ (R) = 0
is due to the symmetry of u . The corresponding local finite element system for
linear elements is
K(e)( u( e)) u (e) = F(e) + Q( e), (10.26)
where the local stiffness matrix K( e) (u(e)) is given by

K( e) (u(e)) =...!!.-
lee)
[1
-1
-1]
1
+ P (Ky)2 u~e) - u~e) I
lee) 1 lee) [ 1 -1]
-1 1 '
260 10. NONLINEAR ONE-DIMENSIONAL PROBLEMS

(e) + (e)
.h - YI Y2 d
WIt Y= l(e) , an

Q(e) = {~l:~ }, F(e) = (- ~:) (e) l~) {l}


EXAMPLE 10.7. Water at 20°C flows through a circular pipe of 0.2 m in
diameter, subject to a pressure gradient of 2.59 kPaim and a flow rate of 4 x
10- 2 m 3 / s. The density of water is 988 kg/m", and /-L = 1.004 X 10- 6 m 2 / s.
Approximate the velocity profile by using two linear finite elements.
The bulk velocity for this flow is

Q 4 x 1O- 2 m3 / s
V = A = 7[(0.1 m)2 = 1.2732 mis,

and the Reynolds number is

VD (1.2732m1s)(0.2m) 5
Re = - II = 1.0 04 x 10- 6 m 2/ s = 2.536 x 10 .

This flow can be classified as a turbulent flow, and by using (10.26), we get

K(I)(U(I)) = 2.008 X 10- 4 [ ~1 ~1 ] + 39.52 Iu~l) _ u~l) I [1 -1]


-1 1 '

K(2)(u(2)) = 2.008 X 10- 4 [ ~1 ~1 ] + 39.52Iu~2) _ u~2) I [1 -1]


-1 1 '

and

F(l) = F(2) = 6.4750 { ~}.

The corresponding global system is


10.6. RAYLEIGH-RITZ AND NONLINEAR GRADIENT METHODS 261

Applying the boundary conditions U1 = 0, Q3 = 0, and uncoupling, we get

Adding the last two equations and simplifying, we get IU2 ! U2 + 5.08 X 10- 6 U2 =
0.4915. The final finite element solution is U2 = 0.701 mis, U3 = 3.2465 mis,
and Q1 = -19.4203 m 2/s 2 . As a result, the shear stress on the pipe wall is
approximately T = -PQ1 = -988 kg/m'' x (-19.4203 m 2/s 2 ) = 1.9187 x 104
N/m2 . •

10.6. Rayleigh-Ritz and Nonlinear Gradient Methods

Let

F(x ,u,u') KA
= - -
n+1
Iu' jn+1 - fu, I(u) = 1 L
F(x,u(x),u'(x))dx ,

1 1
0

oo}.
L L
n
V = {uIU(O) = 0, \u(x)\n+1 dx < 00, and lu'(x)l +1 dx <

Consider the following minimization problem:

Minimize I( u),
{ u E V. (10.27)

The Euler-Lagrange equation for problem (10.27) is

of d (OF) -_0
ou - dx ou' in (0, L), (10.28)

subject to the natural boundary condition

of
ou' = 0 at x = L. (10.29)

The boundary value problem corresponding to (10.27), (10.28), and (10.29) is


. of
(10.20), SInce ou - dxd (OF) d (KA Iu'In-1 u.
ou' = f - dx ')
262 10. NONLINEAR ONE-DIMENSIONAL PROBLEMS

Let <Pi(X), i = 1, .. . , N + 1 denote the linear global shape functions corre -


sponding to the partition 0 = Xl < X2 < .. . < XN < XN+l ' Let

be the set of piecewise linear functions joining the points ( X l , Ud , (X2, U2) , . . . ,
(XN+l, UN+d . The restriction of u E Vi in the element interval [x~e) , x~e) ] is
the equation of straight line u(e)(x ) = ¢~e)(x) u~e) + ¢~e) (x) u~e), and I (u ) =
N+l
L I (e)(u ), where
e= l

Assuming that A and j arc the constants A (e) and j (e), respectively, on the element
interval [x ~e) , x~e)], we have

I (e)(u ) = K (e)A (e) lu 2(e) -


n +1 l(e)
u (e)
I
n +l
l(e) I j (e)l(e)
_
2
( (e )
ul
+ (e» )
u2 .

(e) _ (e) n + l
Note that in matrix form, u I 2 l(e)u I I
can be written as

I
u~e) - u~e)
l(e)
r' u~e)
I - u~e) 12 =
l(e) (l(e»)2 l(e)
_1_ 1
u~e) - u~e) In - l(
[1 - 1 J{ :t })2
(e)
e)

1
= (l(e») 2
Iu(e)l(e)
2
- u(e) I
I
n
-
l
(
{ u~e) _u~e)}
{ 1 } [u (e)] )
- 1 [ 1 - 1] ute)

_ 1
- (l(e» )2
lu 2(e) - u (e)
l(e)
I l
n- 1
[ul
(e ) (e)
U2]
[1 - 1
_ l ] {U(e)
1
I }
u~e) '
and
j (e)l(e) ( e) (e») _ j (e)l(e) [ (e) (e) { 1 }
2 u l + u2 - 2 ul u 2] -1 .

Let

K (e)(u(e») = K (e)A (e) Iu (e) - u (e)


l(e)
2
l(e)
I
r[ 1
- 1
-1]
1 '
and
(e) _ j (e)l(e) { 1 }
F - ,
2 -1
10.6. RAYLEIGH-RITZ AND NONLINEAR GRADIENT METHODS 263

then

Instead of solving (10.27), we now solve the following finite-dimensional problem:

Minimize I(u) ,
{ u E Vi, (10.30)

that is, find the minimum of the energy lover a restricted set of admissible func-
tions. If Vi is a good approximation of V, then one expects the solution Ul of
(10.30) be a good approximation of the solution u of (10.27) as well. For a fixed
N+I
u(x) = 2: UiifJi(X),(UI,U 2 , .. . , UN , UN+ d E R N+ I , U I = 0, the value of
1
I(u) depends only on the N -tuple (U2 , . .. , UN + I ) . Therefore, one can think of
I(u) as a function from R N to R. Let us denote this function by F(x), where
x denotes (U2 ,' . . ,UN+d . One can see now that the problem (10.30) is really a
unconstrained nonlinear program

MinimizeF(x) ,
{ xER N ,

where F : R N 1---+ R I is the nonlinear function defined by I(u) for u E Vi ,


To begin, let us use the method of steepest descent for a numerical approxi -
mation of the nonlinear program . The method can be described by the following
simple formula:

Xi+I = Xi + a i P i,

where Pi = -,;; f:;; I is called the search direction, and a i is the solution to
the one-dimensional nonlinear program

Minimizeg(a) = F(Xi + o p.),


{ a E RI .

EXAMPLE 10.8. Consider a very simple situation of Eq (10.30) , in which


L = 1, A = 1, K = 1, n = 0.5, f = 2, leI) = l(2 ) = 0.5, UI = I
) = 0, ui
264 10. NONLINEAR ONE-DIMENSIONAL PROBLEMS

I(u) = 2- t' IU'I1.5 dx - (2udx


1.5 io io
= 1\ 1°.5
1
(</>~l)(X)U~l) + </>~1)(x)u~1))'11.5 dx
_ 1°·5 2 (</>~1) (x )U~l) + ¢~1) (x )u~1)) dx

15
+ 1.5
1 r11( </>~2)(X)U~2)
iO.5 + </>~2\x)uf) )'1 . dx

-1 1
5 2 (¢~2)(X)U~2) + ¢~2)(X)U~2)) dx

=
3
(1)
~I U2 - U1
0.5
(1) 15
I·_0.5(U(l) + U(l))+~ IU2 - U1
1 2 3
(2)

0.5
(2) 5
11. _ 0.5(U(2) + u(2))
1 2

"31 IU 0.5 1 IU3


2 - U1 11.5 U211.5
0.5 (U1 + U2) + "3 - 0.5 (U2 + U3 )
-
= - 0.5 .

Let U1 = 0, U2 = Xl, U3 = X2, and F = I(u). Then

and

Table 10.5 shows the results of three iterations using the method of steepest descent.

Table 10.5.

of of
(}:i xl xi2
OX1 OX2
0 0.75 1.0 -0.48236 0.207107
1 0.8672117 1.015357937 -0.227352 0.044327
2 0.277777 0.930365163 1.00304474 -0.01717 -0.11874
3 0.619048 0.940996726 1.07655048

The exact solution to problem (10.20) is

u(x) = _n_ (L)l/n [L(n+l)/n _ ~ (L _ x)(n+1)/n], 0 < x < L.


n+1 AK n
10.6. RAYLEIGH-RITZ AND NONLINEAR GRADIENT METHODS 265

For the given set of special data, we have

u(x) = ~ [1 - (1 - X) 3] , 0 < x < 1.

Therefore, at the nodes X2 ::= 0.5 and X3 = 1.0, the exact solution is

u(0.5) = 1.1667, u(l) = 1.3333 .

It is obvious from the above table that the approximate finite element solution using
the method of steepest descent after three iterations is

U2 = 0.9409967263 , U3 = 1.076550482.•
Now, we introduce some nonlinear versions of the conjugate gradient methods .
Like its linear versions, these methods generally converge faster than the method of
steepest descent. Conjugate gradient methods, like the method of steepest descent,
are also schemes that search for the minimizers of F(x), and they usually have the
following common form:

ifi = 1,
if i ~ 1,

where 0 i is the step size, Pi the search direction, and (3i the parameter. The
difference is that they usually choose better search directions. We introduce
three popular choices of the parameter (3i as follows : The first is the so-called
'Hestenes-Stiefel' formula, that is, (3fs ::= yT-l VF(X i)/yT-1Pi-l, with Yi-l =
V F(Xi) - V' F(Xi-l) ; the second one is the 'Dai-Yuan' formula, that is, (3f Y =
VF(Xi)TV'F(Xi)/yT_1Pi-t. with Yi-l = V'F(xJ - VF(Xi-d; and the third
is a combination of these two choices such that (3i = max{ 0, min {(3f s, (3fY} }.
Any of three choices can be implemented similarly. For example, the algorithm
using the third choice is as follows:
Set P-l = 0, (30 = 0, Xo = 0, and
set the convergence tolerance 8 > 0
For i = 0,1 , ...
if IIV!(xi) 11 < 8 stop
if i > 0 set
Yi-l = V F(Xi) - V' F(Xi-d
(3f s = yT-l V!(Xi)/yT-l Pi-l
(3pY = V F(Xif V' F(Xi)/yT-l Pi-l
(3i = max{O,min{(3t,f3;}}
end if
266 10. NONLINEAR ONE-DIMENSIONAL PROBLEMS

Set Pi = - 'VF(Xi) + f3i Pi-l


Use a line search to determine (li
and set Xi+! = Xi + (li Pi
end if
The implementation of this scheme is similar to that of the method of steepest
descent and is left as an exercise.

10.7. Exercises

10.1. In Example 10.3, if the cooing fin is surrounded by air of Tf = 25°C with
a convection coefficient f3 = 150 W/m 2°C, solve for the corresponding finite
element solution by taking into account of heat loss due to convection.

10.2. In problem (10.30), let L = 1, A = I, K = I, n = 1.5, f = I,


l(l) = l(2) = .5, Ul = u~l) = I, U2 = U~l) = u~2), U3 = u~2). Solve for U2
and U3 .
10.3. Consider a prismatic bar made of 304 annealed stainless steel rigidly fixed
to a bar made of 2024 heat-treated aluminum alloy and subject to a vertical
point load of P = 10000 Ib at the bottom. The density of steel is 0.28907Ib/in3
and of aluminum 10000 lb/in". The structure is supported at the top point and
is also subject to a gravity (body) force (see Fig. 10.5, next page) . Determine
the deflections, reactions, and stresses for the properties given in Table 10.6.

Table 10.6.

Element lee)

1 420 in 77000Ib/in 2 0.26 0.28907 lb/in'


2 240 in 100000 Ib/in2 0.16 0.10000Ib/in 3

10.4. A tomato paste is tested to satisfy the power-law: Tx y = 49.3 !lxyl-O.743 'Yxy .

Suppose that the pressure gradient is ~: = 3.69 kPalm, and the distance be-
tween the plates L = .09 m. Solve the problem (10.23) for the velocity u at
y = 0.03 m and y = 0.06 m.
10.5. In problem (10.30), let L = I, A = I, K = I, n = 1.5, f = 1,
[(1) = [(2) = 0.5, Ui = U~l) = I, U2 = u~l) = u~2), and U3 = U~2). Solve
for U2 and U3 by using the method of steepest descent.
10.6. In problem (10.30), let L = 1, A = 1, K = 1, n = 1.5, f = I,
10.7. EXERCISES 267

z(1) = Z(2) = 0.5, U1 = u~l) = 1, U2 = u~l) = U~2), and U3 = u~2). Solve


for U2 and U3 by using the nonlinear conjugate gradient method described
above.
10.7. Consider two immiscible power-law fluids between two flat plates shown
in Fig. 10.6. A pressure gradient is imposed from the inlet to outlet to cause
flow. The lower half of the region is filled with fluid I and the upper half with
fluid II. Approximate the velocity profile by using four linear elements.
10.8. The following model is also used for non-Newtonian flows between parallel
plates

d (1+ IdUln-1
- J.L - - -dU) _ -dp
- 0< y < L.
dy dy dy - dx'

Derive the local linear finite element system .

f CD
~

! Power-Law Fluid I

I:
I
I

~ Power-Law Fluid "

P = 10.000 Ib

Fig. 10.5. A Prismatic Bar. Fig. 10.6. Two Immiscible Power-Law Fluids .

10.9. A wall of an industrial oven is made of three different materials, as shown


in Fig. 10.7. The first layer is composed of 5 em of insulating cement with
a clay binder which has a thermal conductivity of 0.08 W/m ·oK; the second
layer is made of 15 em of 6-ply asbestos board with a thermal conductivity of
0.074 W/m ·oK; and the exterior is made of 10 cm silica bricks with a thermal
conductivity of O. 72 W1m· °K. The inside wall temperature of the oven is 200°C,
and the outside air is 30°C with a convection coefficient (3 = 40 W/m 2 . 0 K.
268 10. NONLINEAR ONE-DIMENSIONAL PROBLEMS

Determine the temperature distribution along the composite wall.

k =0.08 W/m~K k =0.074 W/m~K

ti_
I I
15 em 10em
x
-.._ - - - -=- - - - _• - - -=-- - _ •
2 3 4

Fig. 10.7. A Composite Wall ofan Industrial Oven.

HINT . Since the lateral surfaces are insulated, the net change in heat is zero
and the steady-state is reached, we can use a one-dimensional mesh of three
elements. The equation to be solved is 40.358308 T4 + 4.767 x 10- 8 (T4 )4 =
12698 .1312 .
ANS . T 4 = 304.483.
10.10. Consider the following nonlinear two-point boundary value problem:

-U" - eU = 0, 0 < x < 1,


{
u(O) = 1, u'(I ) = o.
Use a mesh of two linear finite elements and Newton 's method to approximate
the solution.
ANS. U1 = u(O ) = 0, U2 = u(0.5) = 1.302564, U3 = u( l) =
1.7193321.
11
Plane Elasticity

Steady -state problems ofplane elasticity are studied in this chapter. These problems
involve two dependent variables u and v, which are each functions of x and y. The
finite element method is therefore accordingly modified, since each node has two
degrees of freedom . The stiffness matrix and the load vector for a linear constant-
strain triangular element and a bilinear rectangular element are derived, and some
steady -state plane elasticity boundary value problems are solved.

11.1. Stress-Strain Relations

In the case of plane stress problems for very thin bodies, the stresses in the z-
direction are negligibly small, i.e., /jz = Tyz = Txz = 0, whereas for plane strain
problems (body is thick) the strains in the z-direction are zero, i.e., Cz = r yz =
I XZ = 0. The following equations govern steady-state plane elasticity problems.
(i) Equilibrium equations in terms of stress 17:

aTxy
a/jx
ax + ay + f x -_ 0, }
aTxy a
/jy f -
inD, (11.1)
ax + ay + y - 0,

where fx , fy denote the body forces along the x- and y-direction, respectively.
(ii) Strain-displacement relations:

au au au av
Cx = ax' Cy = ay' IXY = ax + ay ' (11.2)

P. K. Kythe et al., An Introduction to Linear and Nonlinear Finite Element Analysis


© Springer Science+Business Media New York 2004
270 11. PLANE ELAST ICIT Y

(iii) Stress-strain relatio ns:

O"x = C ll t7 x + C 12t7 y, O"y = C 12t7 x + C22 t7y , t xy = C33,xy, (11.3)


where Ci j (= C j i ) are the material (elastic) constants . For an isotropic elast ic
body they are defined in terms of the modul us of elasticity E and the Poisson's
ratio v by

Plane stress:
E vE E
Cl l = C 22 = - -2 ,
1- v C 12 = C21 = -1 - v2 ' C33 = 2(1 + v)
C 13 = C 23 = C31 = C32 = 0, (11.4)
Plane strai n:
E(1 - v) C - C _ vE
Cll = C 22 = (1 + v)(1 - 2v) ' 12 - 21 - (1 + v)(1 - 2v) ,
E
C33 = 2(1 + v) , C 13 = C23 = C31 = C32 = O. (11.5)

(iv) Boundary conditions: Natural: { O"xnx + 7xyny = ~x, on f 1 , (11.6)


7xyn x + O"yn y = t y,
Essential: u=ii., v =v on I'j , (11.7)
where f l U f 2 = an.
After substituting Eq (11.2) into (11.3) , and Eq (11.3 )
into (11.1) and (11.6) we express the above equations in terms of the displacement
vector u = (u , v) :

_~ ( Cll au + C12 aV) _ C33~ (aU + aV ) = i x, }


ax ax ay ay a y ax
_C33~(au+aV) _~(C12au+ C22aV)=f inn, (11.8)
ax ay ax ay ax ay y

t x == ( Cll : : + C12 :~) n x + C33 (~~ + :~) n y = i: }


au aV) (aU aV) • on r. ,
t y == C33 ( ay + ax nx + C12 ax + C22 ay ny = ty (11.9)

where Cij = h C i j , h being the thickness of the plate. In view of (11.4) and (11.5),
the elastic coefficient matrix C = [Ci j ] can be written as

[~ l~"
V
E
1 for plane stress ,
1- v 2
a ]
C=

~ 1~2"
V
E
I-v for plane strain .
(1 + v)(1 - 2v ) [' v
a ]
(11.10)
11.2. CONSTANT-STRAIN TRIANGULAR ELEMENT 271

11.2. Constant-Strain Triangular Element

We assume that the elastic body is linearly orthotropic. For steady-state problems,
since the displacements u and v are the primary degrees of freedom at each node,
and since only the first derivatives of u and v with respect to x and y appear in Eqs
(11.8) and (11.9), the displacements u and v are approximated in an element n (e)
by the Lagrange family of shape functions ¢~e)(x, y). Thus, we assume that

L u~e) ¢~e) , L v~e)¢;e) ,


N N
u(x, y) ~ v(x, y) ~ (11.11)
i=1 i =1

respectively. Note that for an element, whether triangular or rectangular, there are
two degrees of freedom (u;e) , v~e)) at each node , Thus , for a linear triangular
element (N = 3) there are a total of six nodal displacements for each element,
whereas for a bilinear rectangular element (N = 4) there are a total of eight nodal
displacements for each element (Fig. 1I. 1(a), (b)). Since the first derivatives of ¢~e)
for the triangular element are constant for each element, all strains (E x , E y, T x y)
computed for such an element are, therefore, constant. Thus, in problems of plane
elasticity, the linear triangular element is known as the constant-strain triangular
(CST) element. The situation for a rectangular element is different, because the
8¢(e)
first derivatives of ¢;e) are not constant; in fact, in view of (5.7), 8~ is linear in
8J,(e)
. x, whereas
yan d constant 10 ----au
'f'i '
IS I'mear 10
in zx and . y.
an constant 10

(a) (b)

Fig. 11.1. Linear Triangular and Bilinear Rectangular Elements.


272 11. PLANE ELASTICITY

11.3. Virtual Displacement Finite Element Model

We rewrite Eq (11.2) in the matrix form as

a
ax ao
o ay (11.12a)
{ :: } =
'Yxy a a
ay ax
or
E: =[)u, (l1.12b)

where u = { ~ }, and n denotes the matrix of differential operators. Now Eqs


(11.1) and (11.3) can be written as

[~
o
a (l1.13a)
ay
or
[)T (T +f = 0, (11.13b)

and
(T = CE:. (11.14)

Note that [) and [)T are matrices of partial differential operators, such that

a
ax ao
o ay
a a
ay ax
The displacements and strains for an elastic element n(e) are given by
11.4. WEAK FORM FINIT E ELEMENT MODEL 273

e (e) (e) (e) (e) (e) ]T


X [u i ) U2 ... UN Vl V2 VN
¢~e)

« ¢~) ]
0 0 ... ¢W
= [¢t ¢~e)
0 0
X [ u ~e) U2
(e)
UN
(e)
Vl
(e )
V2
(e)
.. . VN
J
(e ) T

= ¢ (e) u (e) , (11.15)

where

¢~e) ¢~) . ..
¢~) ] ,
[ ¢(e) 0 0
4> (e ) = ~
0 0 ¢ie ) ¢~e) . ..
u (e) = [u i e ) U2(e) UN
(e) (e )
Vl V2
(e)
... VN(e) JT '
c( e) = B (e)u(e), u (e) = c (e)u (e) , B (e) =, 84> (e ). (11.16)

B (e) is known as the strain matrix for an element n(e) . Using the dynamic principle
of virtual displacement applied to an element n(e) ofthickness h (e) of a plane elastic
body, we obtain the system of algebraic equations

(11.17)

where
K (e ) = h eel J"{ (B (e))T c (e)B (e) dxdy ,
i-;
r (e) = h eel J"{
I n<<)
(4) (e ))T { fx} dxdy,
fy
(11.18)

Q (e ) = heel r
Jr«)
(¢ (e))T {tt y x } ds ,

and f x , f y denote the components of the body forces and t x t y the components of
the boundary stress vectors , respectiv ely, and r eel = ou».

11.4. Weak Form Finite Element Model

The weak variational formu lation for Eqs (11.8) on an element nee) is obtain ed by
taking W l as the test function for the first equation and W2 as the test function for
the second . Then
274 11. PLANE ELASTICITY

We use the finite element interpolation of the form


n n
U ~ ' " u (e)",(e) V ~ ' " v( e)",(e)
LJ ) '1') , LJ ) '1') ,
j=1 j=1

and replace W1 ,2 by ¢~e) in Eqs (l1.19a,b). This leads to the matrix system

Kllu + K 12y = F 1 ,
(11.20)
K 21 u + K 22 y = F 2 ,
where

Note that
(a) The matrix K 12 corresponds to the coefficients of v in the first equation, i.e.,
the first superscript corresponds to the equation number and the second to the
variable number.
(b) For a bilinear rectangular mesh of sides a and b, the coefficient matrices (11.21)
are computed by using the shape functions (5.7).
(c) For the boundary conditions the primary degrees of freedom (i.e., the displace-
ments) or the secondary degrees of freedom (i.e., the forces) can be specified
at any node on the boundary in one of the following four distinct cases:
Case 1. u and v are prescribed (then t x and t y are unknown).
Case 2. u and t y are prescribed (then t x and v are unknown).
Case 3. t x and v are prescribed (then u and t y are unknown).
Case 4. t x and t y are prescribed (then u and v are unknown).
U.5 . ST IFFNESS MATRIX AND LOAD VECTOR 275

The weak form is the principle of virtual work which states that if a solid
in equilibrium is subjected to a system of small virtual displacements within a
compatible state of deformations, then the virtual workof externalactions is equal
to the virtual strain energy of internalstresses.

11.5. Stiffness Matrix and Load Vector

Tocomputethe stiffness matrixK (e) and the load vector F (e) for an element n (e),
we willdiscussthecasesof a linearCST elementanda bilinearrectangularelement
separately, and derive formulasfor K (e) and F (e) in each case.
11.5.1. Linear CST Element. First, we use formula (5.4) to compute
the valuesof band c, which dependon the coordinates of the global nodes of the
triangular element. Let (X l, Yl ), ( X 2 , Y2 ), and ( X 3 , Y3) denote the coordinates of
such an element (see Fig. 5.1). Then, as in Example 5.1,

b (e=
) [ b(e)
1
b(e)
2
b(e) ]
3
[Y2 - Y3 Y3 - Yl u, - Y 2]
2jn(e) I 2In(e) I
(e) (e) (e) ] [ ]
(11.22)
c (e) = [ Cl C2 C3 = X3 - X2 X l - X3 X2 - Xl
2In(e) I 2In (e)I
The strain matrix B (e) for a linear triangularelement n( e) is defined by
B (e) = vel>
e) a¢~e) a¢~e )
a¢i
0 0 0
ax ax ax
e)
a¢ i a¢~e) a¢~e)
0 0 0
e)
'7fYe) '7fY ay
(11.23)
a¢i a¢i a¢~e) a¢~e) a¢~e) a¢~e)

l ay ax ay ax ay ax

[br
b(e) b(e)
0 2 0 3
Cl
(e)
0
(e)
C2 0 o(e) ]
C3 .
(e) b(e) (e) b(e) (e) b~e)
Cl 1 C2 2 c3

Then the stiffness matrix K (e) is givenby

K (e) = h(e)ln(e)j (B (e ))T CB(e) = h(e)ln (e)1 [k(e)


11
k (e)
12
]
k(e) , (11.24)
sym 22

where h ( e) is the thickness of the element n (e), and


(e) ) 2
Cll (b 1 + C33 (Cel )) 2 (
C12 +
C33 1 C l
) b(e) (e)
Cll 1 2
b (e)b (e)
+ C33 C(e)
l C2
(e) ]

k (e) ( (e) )2 (b (e) ) 2 b(e) (e) b(e) (e)


11 = C22 C l +
C33 1 C12 2 C1 + C33 1 C2 '
[
sym (e) ) 2
Cll (b 2 + C33 ( C2(e))2
276 11. PLANE ELASTI CITY

(e»)2
C22 ( C2 + C33 (b 2(e») 2 C12b 3(e) C2(e)+ C33 b(e)
2 C3
(e) (e ) (e)
C22 C2 C3 + b(e)b (e )]
C33 2 2
(e)
k 22 = Cn
(b (e») 2
3 + C33 ( C3e»)2 (
C12 + ) b(e) (e)
C33 3 C3 .
[
sym (e») 2
C22 (b 3 + ( (e») 2
C33 C3
(11.25)

Note that the element matrix K (e) is a symmetric 6 x 6 matrix. The load vector
F (e ) is given by

2tni + t nj }
F (e) = M e)z (e) 2t St· + t SJ.
(11.26)
6 { t -« + 2t nj ,
t si + 2t sj

where tni and tsj are the normal and surface tract ions, respecti vely, on the side
i - j of the triangular element (Fig. 11.2(a)).

hag hag
I tnj 4 3
I
I
/
I lsj hbg 4 hbg 3
I
I
4 3
I
I
I
I
de )
I -----+ n
I
I de)
I

2
hbg\ hbg
2
lni
/ // / / / / r hag] hag 2
tsi
(a) (b)

Fig . 11.2. (a) Tractions on a Linear Triangular Element;


(b) Constant Nodal Forces .

11.5.2. Assembly of CST Elements. We consider the case of a mesh


of two linear triangular elements with two degrees of freedom at each node , as
shown in Fig. 11.3, where the global nodes are marked within the square brackets
11.5. ST IFF NESS MATRIX AND LOAD VECTOR 277

and the local nodes for each element are marked within parentheses.

[7,8] [5,6] [7, 8] [5,6]


4 3 4 3
(5, 6) (3, 4) (3, 4) (1, 2)

(5, 6) (5,6)

(1,2)
CD (5, 6)
CD
(1,2) (3, 4) (1,2) (3, 4)

1 2 1 2
[1,2] [3,4] [1,2] [3,4)
(a) (b)

Fig. 11.3. Mesh of 2 CST Elements.

The relat ionship between the global and local nodes becomes clear if we note
that

Global (dof) Local (dof) Globa l....... Local

1 [1 , 2] 1 of element n (1) (1,2) K 11 ....... K (I )


11
+
K (2)
11
1 of element n (2) (1,2) K 22 -> K (I )
22
+
K (2)
22
K 12 ....... K (l ) +K (2)
12 12

2 [3,4] 2 of element n(1 ) (3, 4) K 33 ....... K ( l)


33
K 44 ....... K (I )
44
K 34 ....... K (I )
34

3 [5, 6] 3 of element n (l ) (5,6) K 55 ....... K (I )


55
+ K 33(2)
2 of element n (2) (3,4) K 66 ....... K (I ) + K (2)
66 66
K 56 ....... K (I )
56
+ K 34(2)
K (2)
4 [7,8] 3 of element n (2) (5,6) K n ....... 55
K 88 ....... K (2)
66
K 78 ....... K (2)
56
278 11. PLANE ELASTICITY

After assembly the stiffness matrix K and the load vector F are given by
K(l)
11
+K(2)
11
K(l)
12
+ K(2)
12
K(l)
13
K(l)
14
K(l)
15
+
K(2)
13
K(l)
21
+K(2)
21
K(l)
22
+ K(2)
22
K(l)
23
K(l)
24
K(l)
25
+
K(2)
23
K(l) K(l) K(l) K(l) K(l)
31 32 33 34 35
K(l) K(l) K(1) K(l) K(l)
K = 41 42 43 44 45
K(l)
51
+K(2)
31
K(l)
52
+ K(2)
32
K(l)
53
K(l)
54
K(l)
55
+
K(2)
33
K(l)
61
+K(2)
41
K(l)
62
+ K(2)
42
K(l)
63
K(l)
64
K(l)
65
+
K(2)
43
K(2) K(2) 0 0 K(2)
51 52 53
K(2) K(2) 0 0 K(l)
61 62 63
K(l)
16
+ K(2)
14
K(2)
15
K(2)
16
K(l)
26
+ K(2)
24
K(2)
25
K(2)
26
K~~) 0 0
K~~) 0 0
K(l) + K(2) K(2) K(2)
56 34 35 36 (11.27)
K(1)
66
+ K(2)
44
K(2)
45
K(2)
46
K(2) K(2) K(2)
54 55 56
K(2) K(2) K(2)
64 65 66
p(l) + p(2)
1 1
F.(l) + F.(2)
2 2
p(1)
3
FJ1)
F= (11.28)
F.(1)
5
+ p(2)
3
F.(l)
6
+ p(2)
4
p,(2)
5
p(2)
6

The above stiffness matrix K, which is an 8 x 8 symmetric matrix, represents the


assembly of the two element matrices K(1) and K(2), each of which is a 6 x 6
matrix.
Another method of obtaining the assembled stiffness matrix K and the load
vector F consists in the use of formula (11.24), which determines the element
matrix K(e), e = 1,2. This method is equivalent to the above method and is
also easy to manipulate with Mathematica or an electronic calculator. Note that
each element matrix K(l) and K(2) can be decomposed into 2 x 2 blocks, which
represent the combination of (u, v)-pair of values at each global node; thus, we
write

k(2)
13 ]
(2)
k 23 ' (11.29)
k(2)
33
11.5. ST IFFNESS MATRIX AND LOAD VECTO R 279

where, for e = 1, 2,

[K [K
(e) (e)
k (e) _ 11 K 12
(e)] k (e) _ 13 K 14
(e) ]
11 - K (e) K (e) , 12 - K (e) K (e) ,
21 22 23 24

[K [K
(e) (e)
k (e) _ 15 K 16
(e) ] k (e) _ 33 K(34e) ]
13 - K (e) K (e) , 22 - K (e) K (e) , (11.30)
25 26 43 44

[K [K
(e) (e)
k (e) _ 35 K 36
(e)] k (e) _ 55 K 56
(e) ]
23 - K (e) K (e) , 33 - K (e) (e) .
45 46 65 K 66

Then the global stiffness matrix K is obtained by properly adding the above two
matrices, which is

[k(IJ
11
k (l )
12 o k(l}] [k(21 0
13 11
k (2)
13
k(2}]
12
k (l ) k (l ) k (l ) 0 0 0 0
K = 21 22 o 23 + (2)
0 0 o 0 k 31 0 k (2)
33
k (2)
32
k (l )
31
k (l )
32
o k (l )
33
k (2)
21 0 k (2)
23
k (2)
22
(11.31)
[k(l}+ k(2}
11 11
k (l )
12
k (2)
13
k(l}+ k(2}]
13 12
k (l ) k (l ) k (l )
21 22 0 23
k (l ) k (2) (2) .
31 0 33 k 32
k (l )
31
+ k (2)
21
k (l )
23
k (2)
23
k (l )
33
+
k (2)
22

Note that an extra column and row of four 2 x 2 null matrix 0 are inserted in the
third column of the above decomposed matrices K (1) and K (2), and the third and
fourth rows of K (2) are interchanged in the final form of the global matrix K.
Similarly, the load vector F , defined by (1 1.28), can be computed from

(11.32)

where for e = 1,2

(11.33)

EXAMPLE 11.1 . Consider a thin uniform square steel plate of side 5 in and
thicknes s h = 0.2 in. This plate is subjected to a uniform load of 0'0 = 10000
lb/in? (psi). The plate is composed of an isotropic material with Young' s modulus
E = 3 X 10 7 psi and Poisson's ratio 1/ = 0.3. Use plane stresses to compute the
280 11. PLANE ELASTICITY

displacements u and v in the x and Y directions. The geometry of the plate and a
mesh of two CST elements are presented in Fig.11.4.

-
3 2
f--
3

.5 -
f --

-
- 0:
V)
~o

--+h-
5 in

x
-
f --

x
2

(a)

Fig . 11.4. Thin Uniform Squar e Plate .

The elastic coefficient matrix C for both elements n (l ) and n (2) (Fig. 11.3(b))
is given by (11.1 0) as

E [1 0.3 o ]
C = - -2 0.3 1 . (11.34)
1- v 0 0 0.35

Since the values of band c depend on the coordinates of the global nodes of
each element, we have for element n (1 ): (Xl , YI) = (0 ,0), (X2' Y2) = (5,0), and
(X4, Y4) = (5,5 ), which give 2/n (l ) I = 25, and

b el) = [-5 5 0 ] /25, C(l ) = [0 -5 5] /25.

Similarly, for the element n (2), we have (Xl , YI) = (0,0), (X3' Y3) = (0 , 5) , and
(X4' Y4) = (5 ,5), which give 2In(2)I = 25, and

b (2) = [0 5 - 5] /25, c (2) = [-5 0 5] / 25.

The strain matrix B (e) for each element is computed from (11.23); thus ,

B (l ) = ~ [ -5
0
0
0
5
0
0
-5
o
o 0]
5 ,
25 0 - 5 - 5 5 5 0

B(2) ~ 2-25 P-5


0
-5
0
5
0
0
0
0
5
-5
0
5
~
-5
]
11.5. STIFFNESS MATRIX AND LOAD VECTOR 281

Then, the stiffness matrix for each element is given by

K(I ) = hln(1)1(B(1){ C B(I)


25 0 - 25 7.5 o -7.5
o 8.75 8.75 -8.75 * -8.75 o
-25 8.75 33.75 -16.25 - 8.75 7.5
=g
7.5 - 8.75 - 16.25 33.75 8.75 - 25
o - 8.75 -8.75 8.75 8.75 o
-7.5 0 7.5 - 25 o 25
(11.35)
K (2) = hln(2)1 ( B(2))T CB (2)
8.75 0 0 -8.75 -8.75 8.75
o 25 -7.5 o 7.5 - 25
o - 7.5 25 o - 25 7.5 (11.36)
=g - 8.75 0 0 8.75 8.75 -8.75
-8.75 7.5 -25 8.75 33.75 -16.25
8.75 - 25 7.5 -8 .75 -1 6.25 33.75

hE
where g = ( 2)' and h = h (e) for e = 1,2. The assembly of the two
50 1 - v
stiffness matrices is carried out by using formula (11.31), which gives

K =gx
43.75 0 - 25 7.5 - 8.75 8.75 0 -16.25
0 33.75 8.75 -8.75 7.5 - 25 - 1.25 0
-25 8.75 33.75 - 16.25 0 0 - 8.75 7.5
7.5 -8.75 - 16.25 33.75 0 0 8.75 - 25
- 8.75 7.5 0 0 33.75 - 16.25 -25 8.75
8.75 -25 0 0 -16 .25 33.75 7.5 -8 .75
0 - 1.25 -8.75 8.75 -25 7.5 33.75 0
-16 .25 0 7.5 -25 8.75 -8.75 0 33.75

The boundary conditions imply that UI = VI = V2 = U3 = O. Since there are


no body forces or thermal inputs, we have f (1) = f (2) = O. The surface tractions
give

which, in view of the data z(e) = 5, h = 0.2, ani = anj = 10000, and
asi = asj = 0, gives F = [0 0 5000 0 0 0 5000 0 ]. Applying the
boundary conditions, we solve the constrained system KU = F for the unknown
282 11. PLANE ELASTICITY

33.75
o
0
33.75
-8.75
7.5 7.5 ]
-8.75 {U2}
V 3 1 {5000
0 }
[ -8.75 7.5 33.75 o U4 =9 5000 .
7.5 -8.75 o 33.75 V4 0

The solut ion is U2 = 0.00166636, V3 = -0.000497691, U4 = 0.00166607, and


V4 = -0.0004999329. This solution matches with the exact solution, which is
given by
(To
u(x,y) =E '
X
v(x, Y) = -e-'
V(ToY

This gives the exact solution as U2 = U4 = 1/600 = 0.00166667, and V3 =


-0.0005 = \14. •
EXAMPLE 11 .2 . Consider a thin elastic rectangular plate of dimensions a x b
units and thickness h units, which is subject to a uniformly distributed edge load of
magnitude (To (Fig. 11.5, where the global nodes are marked within square brackets
and local nodes within the parentheses).

------
y [7,8] [5,6]
4 3
3 (5,6) (3,4) 2
3

---
5,6
0

---
b
- °0
CD
-
0,2
1
1 (1, 2) (3,4) 2
- x
-h- a 1 2
[1, 2] [3,4]

Fig. 11.5. Thin Rectangular Elastic Plate.

This example can be solved by using formulas (11.27)-(11.28) or formulas


(11.31)- (11.33).

We discretize the plate to a mesh of2 triangular elements . For the element n(l)
we have X l = Yl = Y2 = 0, X 2 = X3 = a, Y3 = b. For the element n (2) we have
X l = Yl = X 3 = 0, X2 = a, Y2 = Y3 = b. Then we compute the values of K (l )
and K (2) directly from formula (11.29). In either case the solution is obtained by
11.5. STI FFNESS MATRIX AND LOAD VECTOR 283

solving the system

K (l )
33
K (l)
34 K 35
(1) K 36
(1) 1 UV uob/2

K~~) + K~;) K~~) + K~~)


2
(l ) K (l ) K (l) K (l) 0
K 43 44 45 46 2 _

[
K (l )
53
K(l)
54 + + { U3} - { UOb/2} .
K(l) K (l ) K ( l) K( 2) K(l) K( 2) V3 0
63 64 65 43 66 44
(11.37)
We use the following data: a = 100 in, b = 150 in, h = 0.03 in, IJ = 0.25,
E = 27 X 106 lb/irr', and Uo = 8 Ib/in. Then the above matrices K (l ), K (2) , and
Eq (11.37) become

225 0 -225 37.5 0 -37.5


0 84.375 56.25 - 84.375 - 56.25 0
= 102 -225 56.25 262.5 - 93.75 - 37.5 37.5
K (l)
37.5 - 84.375 -93.75 184.375 56.25 -100
,
0 - 56.25 -37.5 56.25 37.5 0
-37.5 0 37.5 -100 0 0
37.5 0 0 -56.25 - 37.5 56.25
0 100 -37.5 0 37.5 -100
102 0 -37.5 225 0 - 225 37.5
K(2) =
- 56.25
,
0 0 84.375 56.25 - 84.375
-37.5 37.5 -225 -56.25 262.5 -56.25
56.25 -100 37.5 -84.375 -56.25 184.375

and

262.5 -93.75
{U2}
-37.5
27 x 102 -93 .75 184.375 56.25 37.5 ]
-100 V {600}
0
2 _

[ -37.5 56.25 262.5 o U3 - 600 '


37.5 -100 o 84.375 V3 0

respectively. Thus, we find that

[U 2 V2 U3 V3 ]T
= [0 .00104129 -0.000086338 0.00101382 -0.000565121]T .

Alternatively, if the local nodes are taken as marked in Fig. 11.3(b), then,
although K (1 ) = K (2), the assembled matrix K becomes different; we have the
284 11. PLANE ELASTICITY

following relat ionship between the global and the local nodes .

Global (dof) Local (do f) Global--+ Local

1 [1 ,2] 1 of element n (l ) (1,2) K 11 --+ K (I)


11
+
K (2)
55
3 of element n (2) (1,2) K22 --+ K (I)
22
+
K (2)
66
K 12 --+ K (I) +K (2)
12 56

2 of element (3,4) K (I )
2 [3, 4] n(l ) K 33 --+ 33
K 44 --+ K(I )
44
K (I)
K 34 --+ 34

3 [5,6] 3 of element n(1 ) (5,6) K 55 --+ K (I)


55
+ K 11
(2)

1 of eleme nt n(2 ) (1,2) K 66 --+ K(I)


66
+ K 22
(2)

K 56 --+ K(I)
56
+ K 12
(2)

K (2)
4 [7,8] 2 of element n(2) (5, 6) K 77 --+ 33
K (2)
K 88 --+ 44
K (2)
K 78 --+ 34

This is left as an exercise (Exercise 11.1). Recall that the matr ix K and the vector
F can be constructed by either of the two equivalent methods described in §11.5. •

11.5.3. Bilinear Rectangular Element . First , we use formula (5.12)


to derive the strain matr ix B (e) . Since there are four global nodes with two degre es
of freedom at each node, we have

B(e) = 8fjJ(e)
fJ¢ie ) 0
a¢~e)
0
a¢~e)
0
a¢~e)
0
ax ax ax ax
a¢i ) e
0
a¢~e)
0
a¢~e )
0
a¢~e )
0
ay ay ay ay
a¢i ) a¢i )
e e a¢~e ) a¢~e) a¢~e ) a¢~e) a¢~e) a¢~e)
ay ax ay ax ay ax ay ax
(11.38)
Note that since ~ = x - Xm = as and 17 = Y- ym = bt, we find that
a 1 a a 1 a
- = - - and - = - -.
ax a as ay a at
U .5. STIFFNESS MATRIX AND LOAD VECTO R 285

Then, from (11.37) we get

1-t
o 1-t
o 0 - 0]
[-~
a l+t l+t

B(e) = 1- s
--b- o 1+ s
- -b-
O
a 1+s
-b-
Oa 1-s
b '

l- s _ 1-t 1+ s 1- t l+ s 1+t 1- s _l+t


- -b- a --b- a -b- a - -b- a
(11.39)
In view of the relation (11.15), the stresses on a rectangular plate of length a, width
b, and thickne ss h are expres sed by

(llAO)

where, in view of the shape functions ¢~e) , i = 1,2 ,3 ,4, defined by (5.12), we get

-bCu(1- t) -aC12(1 - s ) bCu(1 - t ) - aC 12(1 + s)


CB (e) = -bC 12(1 - t) - aC 22(1 - s) bC12(1 - t ) -aC22(1 + s)
[ - aC (1 - s) -bC
33 33(1 - t) aC33(1 + s ) -bC33(1 - t)
bCu(1 + t) aC 12(1 + s) -bCu (1 + t) aC 12(1 - s) ]
bC12 (l + t) aC22(1 + s) - bC 12(1 + t) aC22(1 - s) .
aC33 (1 + s) bC33(1 + t) aC33(1 - s) - bC33(1 + t )
(ll Al)
Thus , the stiffness matrix K (e) for a rectangular element, which is an 8 x 8 matrix,
is given by

l
(e)
au a (e) (e) ]
a 13
12
K (e ) = ~ a le) a le) (llA2)
12 22 23 '
sym a 33
le)

where , with r = Ii[ a which is known as the aspect ratio of the rectangle , and
recall ing that Cij = h Ci j ,

[4 (cll r + c3dr) 3 (C12 + C33 ) 2(C33/r - 2C llr ) ]


alue) -- 4 (C22/r + C33r ) 3 (C33 - C12) ,
sym 4 (cu r + C33/r)
[ 3(c ,, -c,, ) -2 (C33/r + cur) - 3 (c" + cd ]
al e) _ 2 (C22/ r - 2c33r) -3 (C12 + C33) -2 (c22/r + C33r) ,
12 -
-3 (C12 + C33 ) 2 (cu r - 2c33/ r) 3 (C12 - C33)

a le) _
[2(cll r - 2c33/r) 3( c,, -c,, ) ]
13 - 3 (C1 2 - C33) 2( C33r- 2c22/ r ) , (1l .43)
-2 (cur + C33/r ) 3 (C12 + C33 )
[4(C22/r + C33r) 3 (C33 - C12 ) 2(c"r - 2C22 /r)]
a le) -
22 - 4 (cu r + C33/r ) 3 (C12 + C33 ) ,
sym 4 (C22/r + C33r)
286 11. PLANE ELASTICITY

3 (C12 + C33) -2 (C22/r + C33r)]


a~~ = 2 (C33/r - 2cllr) 3 (C12 - C33) ,
[ 3 (C33 - C12) 2 (c22/r - 2c33r)
a (e) _
33 -
[4 (Cllr + C33/r) -3(C22/r
sym 4
(C12 + C33) ]
+ C33r ) .

The load vector F(e) is defined by

(11.44)

where g(e) denotes the constant nodal forces on the element n(e) (Fig.11.2(b)).
EXAMPLE 11.3 . Consider the plane stress problem of a thin rectangular plate
of length 4 in, width 2 in, and thickness 0.1 in, which is subjected to a moment
as shown in Fig.11.6(a) . As the simplest case we will consider a single element
model (Fig.11.6(b)). Recall that even such a model involves lengthy computations
by an electronic calculator (or Mathematica). The stiffness matrix and the load
vector are computed as

4.8 1.95 -0.6 -0.15 -2.4 -1.95 -1.8 0.15


1.95 8.7 0.15 3.3 -1.95 - 4.35 - 0.15 -7.65
-0.6 0.15 4.8 - 1.95 -1.8 -0.15 -2.4 1.95
-0.15 3.3 -1.95 8.7 0.15 -7.65 1.95 -4.35
K=d
-2.4 -1.95 -1.8 0.15 4.8 1.95 0.6 -0.15
,
-1.95 -4.35 -0.15 -7.65 1.95 8.7 0.15 3.3
-1.8 -0.15 -2.4 1.95 0.6 0.15 4.8 -1.95
0.15 -7.65 1.95 -4.35 -0.15 3.3 -1.95 8.7
F = [0 a -2500 0 0 0 2500 O]T,

where d = hE/ (12 (1- v 2 ) ) . Since UI =0= VI = U3 = V 3, we solve the


system

4.8 -1.95 -2.4 2


hE -1.95 8.7 1.95 1.95 ] { U
-4.35 V2 } { -2500
0 }
[ -2.4 1.95 4.8 -1.95 U4 = 2500 '
1.95 -4.35 -1.95 8.7 V4 0

which gives the solution as U2 = -0.004524, V2 = -0.001352, U4 = 0.004524,


and V4 = 0.001352.
Next we consider a mesh of 2 x 2 rectangular elements shown in Fig. 11.6(c).
11.5. STIFFNESS MATRIX AND LOAD VECTOR 287

The stiffness matrix K( e) and the load vector F(e) for e = 1,2 are computed as

5.4 1.95 -3.3 -0.15 -2.7 -1.95 0.6 0.15


1.95 5.4 0.15 0.6 -1.95 -2.7 -0.15 -3.3
-3.3 0.15 5.4 -1.95 0.6 -0.15 - 2.7 1.95
-0.15 0.6 -1.95 5.4 0.15 -3.3 1.95 -2.7
K=d
-2.7 -1.95 0.6 0.15 5.4 1.95 -3.3 -0.15
,
-1.95 -2.7 -0.15 -3.3 1.95 5.4 0.15 0.6
0.6 -0.15 - 2.7 1.95 -3.3 0.15 5.4 - 1.95
0.15 -3.3 1.95 -2.7 -0.15 0.6 -1.95 5.4
F = [0 a - 2500 a a a 2500 OjT. (11.45)

2
2 2500

CD CD
( 1,2) (3,4) (1, 2) (3,4) _ 2500
123
[1, 2] [3. 4] [5.6]
(c)

Fig. 11.6.

After assembly, we solve the system

K 33 K 34 K 35 K 36 K 39 K 3,1O K 3,1l K 3,12


K43 K 44 K 45 K 46 K 49 K 4,1O K 4,1l K 4,12
K 53 K 54 K 55 K 56 K 59 K 5, IO K 5,1l K 5,1 2
K 63 K 64 K 65 K 66 K 69 K 6,IO K 6 ,1l K 6,12
K 93 K 94 K 95 K 96 K 99 K 9,IO K 9 ,1l K 9 ,12
K I O,3 K IO,4 K lO,5 K I O,6 K IO,9 KlO ,lO KlO ,ll K lO,12
K ll,3 K ll,4 K ll ,5 K ll ,6 K ll,9 KU ,IO KU ,ll K ll ,12
K 12,3 K 12,4 K 12,5 K 12,6 K 12,9 K 12,1O K 12,1l K 12,12

X [U2 V2 U3 V3 U5 V5 U6 v6 f =
[13 i4 i5 i6 fg !to i11 !t2]T . (11.46)
288 11. PLANE ELASTICITY

Since the stiffness matrix is symmetric, the correspondence between the global
and local nodes for the upper diagonal elements is given below.

Thus , from using the values from (11.45) , we obtain from Eq (11.46) the system

10.8 O. -3.3 -0.15 -5.4 O. 0.6 0.15


O. 10.8 0.15 0.6 O. - 5.4 -0.15 - 3.3
- 3.3 0.15 5.4 -1.95 0.6 -0.15 - 2.7 1.95
- 0.15 0.6 - 1.95 5.4 0.15 - 3.3 1.95 - 2.7
d x
-5 .4 O. 0.6 0.15 10.8 O. - 3.3 - 0.15
O. - 5.4 -0.15 - 3.3 O. 10.8 0.15 0.6
0.6 -0.15 -2.7 1.95 -3.3 0.15 5.4 -1.95
0.15 -3.3 1.95 -2.7 -0.15 0.6 -1.95 5.4
[U2 V2 U3 V3 Us Vs U6 V6 ]T =
[ -2500 0 -2500 0 2500 0 0 O]T .

This gives the solution U2 = -0.0078, V2 = 0.000668, U3 = - 0.0143226, V3 =


-0.00245104, Us = - 0.00231111, Vs = -0.000668056, U6 = -0.00589965,
and V6 = 0.00245104.•

11.5.4. Two or More Bilinear Rectangular Elements. The ele-


ment matrix K (e ), defined by formula (11.42), is computed depending on the
number of elements. Then their assembly is carried out on the same lines as in the
case of CST elements, with the only difference that now K (e) is an 8 x 8 matrix ,
and F (e) an 8 x 1 vector. We rewrite this stiffness matrix K (e) by decomposing it
11.5. STIFFNESS MATRIX AND LOAD VECTOR 289

into four blocks of 2 x 2 submatrices as follows :


k (e) k (e) k (e)
11 12 13 14 ]
k (e)
(e) (e) (e) k( e)
K (e) _ k 21 k 22 k 23 24
- k (e) k( e) k (e) k (e) , (11.47)
[ 31 32 33 34
k( e) k (e) k (e) k( e)
41 42 43 44

where kW,...,kW are defined in (11.30) for e = 1,2 , and


k(e ) _ [ K 17
(e) K 18
(e)] k (e) _ [ K 37
(e) K 38
(e) ]
14 - K (e) K (e) , 24 - K (e) K( e) ,
27 28 47 48
(11.48)
k (e) _ [ K 57
(e) K( e) ] k (e ) _ [ K 77
(e) K(78e )]
58
34 - K (e) K (e) , 44 - K( e) (e) .
67 68 87 K 88

The details for a mesh of two and four bilinear rectangular elements are describ ed
below.
First, we consider the case of a mesh of two bilinear rectangular elements
(Fig . 11.7(a)), where the eight global elements are marked within square brackets
and the local elements within parentheses. We start with the matrix K (e), defined
by (11.47), for e = 1,2. Since there are six global nodes with two degrees of
freedom each , each element matrix K (e) is expanded, to yield the following two
matr ices K 1 , and K 2 :

1 2 3 4 5 2 3 4 5 6
k (l )
11
k (l )
k ( l)
12
k (l )
0 k (l )
14
k (l )
k(ll]
13
k (l )
k (2)
11
k (2)
k (2)
12
k (2)
0 k(2)
14
k (2)
k (2)
13
k (2)
21 22 0 24 23 21 22 0 24 23
K 1 = 0 0 0 0 o ,K 2 = 0 0 0 0 0
k (l ) k(l ) k (l ) k (l ) k (2) k (2) k (2) k (2)
31 32 0 34 33 31 32 0 34 33
k (l ) k (l ) k (l ) k (l ) k (2) k (2) k (2) k (2)
41 42 0 44 43 41 42 0 44 43

where the third and fourth rows of each elemental matrix K (e) are interchanged in
each expanded matrix K e , and the numbers on the top line represent the columns
where the entry is made in the assembled global stiffness matrix K, which is a
square matrix (12 x 12) given by

2 3 4 5 6
k (l ) k(l ) k (l ) k (l )
11 12 0 14 13 0
k (2)
12
k (l )
22
+ k (2)
11
k (2)
12
k (l )
24
k (l )
23
+ k(2)
14
k (2)
13
k (2) k (2) k (2) k(2) (11.49)
0 12 22 0 24 23
K=
k (l ) k (l ) k (l ) k (l )
14 24 0 44 43 0
k (l )
13
k(l )
23
+ k (2)
14
k(2)
24
k (l )
43
k (l )
33
+ k (2)
44
k (2)
43
k (2) k (2) k (2) k (2)
0 13 23 0 43 33
290 11. PLANE ELASTICITY

[7,8] [9, 10] [11, 12]


4 5 6
(7,8) (5,6) (7,8) (5,6)

(1,2) (3,4) (1,2) (3,4)


1 2 3
[1,2] [3,4] [5,6]

(a)

7 [13, 14] 8 [15, 16] 9 [17, 18]


(7, 8) (5,6) (7,8) (5,6)

(1,2)
CD (3, 4)[~ 10] (1,2)
8)
(3,4)
4 6
[7,8] (7,8) (5,6) (7,8) (5,6) [11, 12]
CD G)
(1,2) (3,4) (1,2) (3,4)
1 [1,2] 2 [3, 4] 3 [5,6]

(b)

Fig. 11.7.

Next we con sider the case of a mesh of 4 rectangular elements (Fig. 11.7(b)) .
Again we start with the stiffness matrix K (e), defined by (11.42), for e = 1 ,2,3 ,4.
Since there are nine global nodes with two degrees of freedom each, each elemental
matrix K (e) is expanded to yield the following four matrices K 1 , K 2 , K 3, and
K 4 , where K 1 and K 2 are defined above, and

4 5 6 7 8 5 6 7 8 9
k (3) k (3) k (3) k (3) k (4) k(4) k (4) k (4)
11 12 0 14 13 11 12 0 14 13
k (3) k (3) k (3) k (3) k (4) k (4) k (4) k(4)
21 22 0 24 23 21 22 0 24 23
K 3= 0 0 0 0 0 , K 4= 0 0 0 0 0
k (3) k (3) k (3) k (3) k (4) k (4) k (4) k (4)
31 32 0 34 33 31 32 0 34 33
k(3) k (3) k (3) k (3) k (4) k(4) k (4) k (4)
41 42 0 44 43 41 42 0 44 43

where, as before , the third and fourth rows of each elemental matrix K (e) are
interchanged in each expanded matrix K e, and the numbers on the top line represent
11.6. EXERCISES 291

the columns where the entry is made in the assembled global stiffness matrix K ,
which is a square matrix (9 x 9) given by

k (l ) k (l ) k (l ) k (l )
11 12 0 14 13
k (2)
12
k (l )
22
+ k (2)
11
k (2)
12
k (l )
24
k (l )
23
+ k (2)
14
k (2) k (2) k (2)
0 12 22 0 24
k (l )
14
k (l )
24 0 k (l )
44
+ k (3)
11
k (l )
43
+ k (3)
12
K = k (l )
13
k (l )
23
+ k (2)
14
k (2)
24
k (l )
43
+ k (3)
12
k (l )
33
+ k (2)
44
+ k (3)
22
+ k 11
(4)

0 k (2)
13
k (2)
23 0 k (2)
43
+ k (4)
12
k (3) k (3)
0 0 0 14 24
0 0 0 k (3)
13
k (3)
23
+ k (4)
14
k (4)
0 0 0 0 13
0 0 0 0
k (2)
13 0 0 0
k (2)
23 0 0 0
k (3) (3)
0 14 k 13 0
k (2)
43
+ k (4)
12
k (3)
24
k (3)
23
+k (4)
14
k (4)
13 (11.50)
k (2)
33
+ k 22(4) 0 k (4)
24
k (4)
23
k (3) k (4)
0 44 43 0
k (4)
24
k (4)
43
k (3)
33
+k (4)
44
k (4)
43
k(4) k (4) k (3)
23 0 43 33

The load vector F can be determined in a similar manner. This is left as an exercise.

11.6. Exercises

11.1. Determine the assembled stiffness matrix K for the mesh of two CST
elements shown in Fig. ll.3b, and solve Exampl e 11.2 with the same data.
ANS. [ U2 V2 U3 V3 ]T =
[0.0010535 -0.00324829 0.00592593 - 0.00364335 r.
11.2. Write the correspondence between the global and local nodes for the mesh
292 11. PLANE ELASTICITY

shown in Fig. 11.8, where each node has two degrees of freedom (u, v).

~t- =-[9:....,1-.:.0] 5
3
(5,6)

1
[1 ,2] ---...c--
(3,4)2 1(1,2)
---=-,
2
[3,4]

Fig. Il.8.

(1) (1) (1) (1) (2)


ANS. K ll -t K ll ' K 22
K 22 ' K 12 - t K 12 ' K 33 - t K 33
-t K ll ' +
(1) (2) (1) (2) (1) (2)
K 44 - t K 44 +
K 22 ' K 34 - t K 34 +
K 12 ' K 55 - t K 55 K 77 ' K 66 - t +
(1) (2) (1) (2) (2) (2) (2)
K 66 +KSS ' K 56 - t K 56 +K7S ,K77 - t K 33 ,Kss - t K 44 ,K7S - t K 34 ,
(2) (2) (2)
K gg - t K 55 ' KlO ,lO - t K 66 ' K9,1O - t K 56 .

11.3. Write the correspondence between the global and local nodes for the mesh
shown in Fig.II.9, where each node has two degrees of freedom (u, v).
(1) (1) (1) (1)
ANS. K ll -t K ll ' K 22 -t K 22 ' K 12 - t K 12 ' K 33 - t K 33 ' K 44 - t
(1) (1) (1) (2) (1) (2)
K 44 ' K 34 - t K 34 ' K 55 - t K 77 +
K 55 ' K 66 - t K ss K 66 ' K 56 - t +
K (l )
78
+
K(2) K
56'
K(l)
77 -> 55
+ K(2)
11
+
K(3) K
11'
K(l)
88 -> 66
K(2)
22
K(3)
22 '
+ +
(1) (2) (3) (2) (3) (4)
K 78 - t K 56 + K 12 +
K 12 ' K gg - t K 33 K 55+ +
K 55 ' K lO,10 - t
(2) (3) (4) (2) (3) (4) (3) (4)
K 44 + K 66 + K 66 ' Kg , 10 - t K 34 K 56 + +
K 56 ' KU,ll - t K 33 K ll ' +
(3) (4) (3) (4) (4)
K 12,12 - t K 44 + K 22 ' K 11,12 - t K 34 +
K 12 ' K 13,13 - t K 33 ' K 14,14 - t
(4) (4)
K 44 ' K 13 ,14 - t K 34 .

[11,12] 6

Fig. 11.9.
11.6. EXERCISES 293

11.4. A steady-state axisymmetric problem in a two-dimensional domain n is


governed by the equations of motion

O(1r ((1r - (10) OTrz


~ + +~=O ,
or r oz
OTrz Trz O(1z
- + - + - = 0.
or r OZ
The boundary an
consists of two parts: f l and f 2 , such that the displacements
are u and v prescribed on I' I, while the tractions or stresses are prescribed on
f 2 as l c; + mTrz = tr(s), and lTrz + m a; = tz(s). Determine the weak
form for this problem in the form (2.10).
ANS. On an element nee) we have
-Jj{
O- WI (O(1r
-
or
+ ((1r-(10) + -
r
OTrz
- ) + W2 (OTrz
OZ
- - + -Tr z + -O(1z)} r dr dZ
or r OZ
O ( e)

= - JJ {(1r 0;:/ + (10 ~I + Tr z ( 0l:zI + 0;2) + (1 z 0l:z2 } r dr dz


o ( e)

+ r
Jao(e)
{WI (l(1r+mTrz) +W2 (lTrz+m(1 z)}rds ,

by using divergence theorem,

= _ Jj { OorWl (Cll au
or
+ C12~ + C 13 OWl )
r OZ
+ WI
r
(C12 AU +
or
C22~
r
O (e )

+ C23 :~) - :~ ( C 3:~ + C23~ + C33:~) } r dr dz


1

+
aO«)
J {WIi r(S)+ W2 tA s)}rds,

= -b (WI , W2;u,v) + l (WI ,W2 ) .

11.5. Verify the expression for K( e) given by Eq (11.24) .


11.6. Verify the expression for K given by Eq (11.31).
11.7. Verify the expression for K( e) given by Eq (11.42) .
11.8 . Solve the problem of Example 11.3 for a rectangular plate of length 3 in,
width 4 in, and thickness 0.25 in; use a single element.
ANS . U2 = -0.00145766 = U4, V2 = -0.0007787 = V4.
11.9. Determine the load vector F for a mesh of two and four bilinear rectangular
elements.
294 11. PLANE ELASTICITY

11.10. Use a mesh of two linear CST elements to compute the displacements of
the global nodes 3 and 4 of the structure shown in Fig. 11.10, with the following
data for each element n( e) , e = 1,2: E = 15 X 109 N/m 2 , h = 5 X 10- 3 m,
and u = 0.25.

4
1 10000 N

E
N

3
~
2 ---'l -'-+ X

3
2m

Fig . 11.10. A Mesh of 2 Linear CST Elements.

HINT . Solve

11

5 x 10
6
~3
[
-3

ANS. U3 = 2.52101 X 10- 5 , V3 = -6.72269 X 10- 5 , U4 = 24.6499 X


10- 5 , V4 = -15.4062 X 10- 5 .

11.11. Consider a cantilever beam of length 6 x 2 x 1 em", with E = 3 X 10 7


N/cm 2 and u == 0.3. This beam is subjected to a bending moment of 600 N ern
at the free end (Fig. 11.11). Find the correspondence between the global and
local elements of the stiffness matrix.
ANS. The stiffnes s matrix correspondence is given below.
11.6. EXERCISES 295

Nod e dof Global-> Local

K(l )
1 [1,2] K l1 -> 11
K (l )
K 22 -> 22
K (l )
K 12 -> 12
2 [3,4] K 33 -> K (I )
33
+
K (2)
11
K 44 -> K (I ) +K (2)
44 22
K 34 -> K (I ) +K (2)
34 12
K (2)
3 [5,6] K 55 -> 33
K (2 )
K 66 -> 44
K (2)
K 56 -> 34
4 K 77 -> K (I ) +K (3)
[7,8] 77 11
K 88 -> K ( I ) +K(3 )
88 22
K 78 -> K ( I ) +K (3 )
78 12
5 [9,10] K 99 -> K (I)
55
+
K (2)
77
+ K 33(3) + K 11(4)
KlO ,lO ->
K (I)
66
+
K (2)
88
+ K 44(3) + K 22(4)
K 9 ,10 ->
K (I )
56
+
K( 2)
78
+ K 34(3) + K 12(4)
6 [11,12] Kll ,ll -> K (2)
55
+
K (4 )
33
K (2) +K (4 )
K 12 ,12 -> 66 44
K ll ,12 ->
K (2 )
56
+
K (4)
34
K (3)
7 [13,14] K 13 ,13 -> 77
K (3)
K14 ,14 -> 88
K (3)
K 13 ,14 -> 78
8 K (3) + K(4)
[15,16J K 15 ,15 -> 55 77
K (3) + K(4)
K 16 ,16 --00 66 88
K 15,16 ->
K (3)
56
+
K (4 )
78
K (4)
9 [17,18] K 17 ,17 --00 55
K (4)
K 18 ,18 --00 66
K (4 )
K 17 ,18 --00 56

8 [15. 16) 9 [ 17.1 8)


(5. 6)3 4(7,8) (5. 6)3
CD (3. 4) 2[9. 1011 (1,2)
@
(3.4)2
E 6
u
(5,6)3 4(7. 8) (5. 6)3 [11, 12)
'"
CD (3, 4)2 1(1.2)
@
(3. 4)2

II 2 [3. 4) 3 [5.6)
lem I
3em 3 em

Fig. 11.11. Cantilever Beam with a Mesh of 2 x 2 Rectangular Elements .


296 11. PLANE ELASTICITY

11.12. Use a mesh of twoCST elementsshown in Fig. 11.12to determinethe dis-


placements at the global nodes (ignore the body forces) for a two-dimensional
load plate of thickness h = 0.5 in, E = 30 X 107 psi, and v = 0.25.

4
3 2
3

2
2

Fig. 11.12.

HI NT. We find that

0.983 - 0.5 -0.45 0.2 -0.533


1.4 0.3 - 1.2 0.2 0.3 ]
-0.2
K (l) = K (2 ) = 107 0.45 o o -~ .3 .
[ 1.2 -0.2
sym 0.2

Then solve

ANS. U1 = 1.90877 X 10- 5 in, U2 0.873799 X 10" in, U4


- 7.41554 x 10" in.
12
Stokes Equations and Penalty Method

In this chapter, we will introduce the penalty finite element formulation for both
Newtonian and power-law non-Newtonian Stokes flows. The penalty finite element
method has been recognized as one of the most effective numerical methods in
solving fluid flow problems by many in academics as well as in industry (see, e.g.,
Fastook 1993 and Fidap 1999). We will derive local finite element matrices by using
the linear triangular and the bilinear rectangular elements. We will use the method
of steepest descent, the conjugate gradient methods for the linear Stokes problem ,
and nonlinear conjugate gradient methods for the nonlinear Stokes problem. The
cavity problem will be used as an example of numerical implementation.

12.1. Equality-Constrained Programs and Lagrange


Multipliers

Consider the equality-constrained problem

Minimize f(x), x E R N,
{ subject to g(x) = 0, (12.1)

where f : R N f---4 Rand 9 : R N f---4 R are real-valued functions with continuous


first-order derivatives . The Lagrange function in this case is

L:(x, >') = f(x) - >'g(x),

where>' E R. A solution of the optimization problem can be determined by


solving the system

'VL:(x , >') = 0

P. K. Kythe et al., An Introduction to Linear and Nonlinear Finite Element Analysis


© Springer Science+Business Media New York 2004
298 12. STOKES EQUATIONS AND PENALTY METHOD

for a stationary point (x*,),*). This gives

\7x.c (x*, ),*) = \7f (x*) - )'*\7g(x*) = 0,


{ \7,\.c(x*,),*) = -g(x*) = O. (12.2)

EXAMPLE 12.1 . Solve the constrained program

Minimize f(XI , X2) = xi + x~ - 4XI,


(12.3)
{ subject to Xl - X2 = 0,

by using the Lagrange function.


Here we have

L (Xl, X2,),) = xi + x~ - 4XI - ), (Xl - X2) ,

x
x
\7 .c ( * *) = { 2xi - 4} _ ),* { 1 }
x , 2x; - 1 '

and
\7)..c (x*,'\*) = -(xi - x;) .
The system is

[o21 -102 -1]


1
0
{X!} _{54},
0
X2
),*
-

which gives the stationary point (xi,x; ,'\*) = (1,1,-2) . The minimizer is
(XI,X2) = (1,1) .•
An alternative way of solving Eq (12.1) is the penalty method. A simple
version can be described as follows. Let 0 < e :::; 1, and let I" (x) = f(x) +
1
2€ [g(x)] 2 . For each fixed e, solve the unconstrained program

Minimize f ,,(x),
{ xE RN , (12.4)

for a minimizer x. ; The key to the success of this method is that for very general
conditions on f and g it can be shown that x, --> x as e --> 0 if x denotes the
solution of Eq (12.1) . For each e, the solution x, of the unconstrained program
(12.4) can be found by solving
1
\7f,,(x) = \7f(x) + -e gc(x) \7g" (x) = O. (12.5)

By taking the limit, the solution x ofthe corresponding constrained program (12.1)
is given by
x = lim x.;
,,->0
12.2. PENALTY FORMULATION FOR LINEAR STOKES EQUATION 299

EXAMPLE 12.2. Solve the constrained program

Minimize !(Xl, X2) = xi + x~ - 4XI,


{ subject to Xl - X2 = 0, (12.6)

by using the penalty method.


We have

and

The system is

2c + 1 -1] {XI}={4c},
[ -1 2c + 1 X2 0

.
which has the solution
. (XI ,X2)€ =
(2c + 1 1)
- - , - - . It
. easy to check that
IS
c+1 c+1
lim (Xl, X2)€ = (1,1) , which is the solution to Eq (12.6).•
€-->O

12.2. Penalty Formulation for Linear Stokes Equation

Consider the boundary value problem ofthe Stokes equation for viscous Newtonian
fluid flows:

-v~u + "il . p = g in n,
{ u (12.7)
=u on an,

where v is the viscosity of the fluid, u the velocity vector, p the pressure, g the
body force, n a bounded open subset of R 2 , and an the boundary ofn. Eq (12.7)
is associated with the following linear program with constraint

Minimize I(u), u E V,
{ subject to "il . u = 0, (12.8)
300 12. STOKES EQUATIONS AND PENALTY METHOD

where

I(u) =!:. { ID(u)1 2 dxdy - { g . udxdy,


2 In In
u = { ~ }, g = { :: } ,

au 1 (aU av)]
D(u) = ax 2 ay + ax
[~(av+aU) av '
2 ax ay ay
ID(u)21 = (au)2 + ~ (au + av)2 + (av)2,
ax 2 ay ax ay
V'·u--+-
au av
- ax ay '
and V is the set of admissible functions of I, which satisfy the boundary condition
u = on u an. u
It can be verified that for a large class of functions g and and the
domain n, both the boundary value problem and the optimization problem have
exactly one solution. These problems are equivalent in the sense that if (u,p) is
the solution for Eq (12.7), then u is the solution of Eq (12.8); conversely, if u is
the solution of Eq (12.8) then one can find a function p so that (u, p) is the solution
ofEq (12.7).
The corresponding penalty formulation of Eq (12.8) is given by

Minimize I,,(u), (12.9)


{ UEV,

where

The Euler-Lagrange equation corresponding to Eq (12.9) is

-vL'l~ - ~ V'(V' . u) = g inn,


(12.10)
{
u = u , on an.

Let u" denote the solution of Eq (12.9) or (12.10). Then the pressure p of the
solution ofEq (12.7) can be approximated by p" = -~ V'. U ". In fact, it has been
c
verified that (u, p) = lim (u, , p,,).
,,-0
12.3. PENALTY LINEAR T RIANGULAR STOKES ELEMENT 301

12.3. Penalty Linear Trtangular Stokes Element

Let nee) be a triangle in a finite element partition of the solution domain n for
e = 1, . . . , NE, where NE denotes the total number of triangles in the partition.
Let N denote the total number of nodes. The coordinates of the triangle are
labeled as (x~e) , y~e) ), (x~e) , y~e)), (x~e), y~e)) locally, and (x~e), y~e)), ( x ; e ), YJ e )) ,
(x ie ) , yke ) ) globally for 1 ::; i, j , k ::; N. Let the piecewise linear interpolation
function be defined for (x,y) E nee), e = 1, . . . , NE, by

where

1 x (e) (e)
Y 1 Xl YI
(e) (e)
1 X2 Y2 1 X Y
(e) (e) (e) (e)
1 X3 Y3 1 X3 Y3
(e) (e) , (e) (e) ,
1 Xl YI 1 Xl YI
(e) ( e) (e) (e)
1 X2 Y2 1 X2 Y2
(e) (e) (e) (e)
1 X3 Y3 1 X3 Y3
(e ) (e )
1 Xl YI
(e) (e)
1 X2 Y2
1 X Y
(e) (e)
1 Xl YI
(e) (e )
1 X2 Y2
(e) (e)
1 X3 Y3

are the three associated linear shape functions. It can be calculated that for (x, y) E
nee),

where

x~e) y~e) I _111 y~e) I 1 x~e) I

I (e)
x3
(e)
Y3
(e)
Y3 11 x 3(e)
(e) (e) (e) (e) (e) (e)'
1 Xl YI 1 Xl YI 1 Xl YI
(e) (e) (e) (e) (e) (e)
1 x2 Y2 1 x2 Y2 1 x2 Y2
(e) (e) (e) (e) (e) (e)
1 x3 Y3 1 x3 Y3 1 x3 Y3
302 12. STOKES EQ UATI ONS AND PENALTY METHOD

I
a(e) -
2 -
-

1
I X (e)
X3
l
(e)

(e)
YI(e)
Y3
(e)
(e) ,
b(e) _
2 -
1
I~
YI(e)
Y3
(e)
(e)
I
(e) ,
c(e) -
2 -
-I ~
1
(e)
Xl(e)
(e)
X3
(e)
1

,
Xl YI Xl YI Xl YI
(e) (e) (e) (e) (e) (e)
1 X2 Y2 1 X2 Y2 1 X2 Y2
(e) (e) (e) (e) (e) (e)
1 X3 Y3 1 X3 Y3 1 X3 Y3

a(e) -
3 -
1
I
X (e)
X2
l
(e)

(e)
YI(e)
(e)
Y2
I
(e) ,
b(e) -
3 -
1
-I~ (e)
YI(e)
(e)
Y2
(e)
I
,
c (e) _
3 -
1
I~
X (e)
X2
(e)
l
(e)
I
(e)
Xl YI Xl YI Xl YI
(e) (e) (e) (e) (e) (e)
1 X2 Y2 1 X2 Y2 1 X2 Y2
(e) (e) (e) (e) (e) (e)
1 X3 Y3 1 X3 Y3 1 X3 Y3

Therefore, for (x,y) E nee),

Thus,
NE
I e (u ) = LI~e) (u) ,
e= l
12.3. PENALTY LINEAR TRIANGULAR STOKES ELEMENT 303

where

I~e) (u) = ~ r
2 In<e)
2
ID(u)1 dx dy - r
In(e)
g.u dx dy + ~ r
2€ In<e)
IV.uI 2

~ ~ In(elI[ (t, ul'lblel)' + ~ (t, ule)c~el + t, vlelblel)'


+(t, vlelcle))']- In(el I[9;H')h Ie)+ ule») + g; Hel+ vle) + vie»)]
+~ In(e)1 (t, ule)b~el t, vlelcle»),
+

For i = 1, 2, 3, we get

In matrix notation,
304 12. STOKES EQ UATIONS AND PENALT Y METHOD

{ l
where
b(e )b(e ) b(e )b(e )

M (e} = In(e}1
1,1
(v+-) 1
IE:
I
b(e}b(e}
2
I
I
I
b(e}b (e}
2
2
2
b(e}b(e } b(e}b(e}

l
3 I 3 2

C(e)
I CI
(e) C(e) C(e) C(e) C(e) ] }
4 4 c~e)
I 2 I 3
+~ e }ci e } c~e) c~e} e} ,
2

{l
(e) (e) (e) (e) (e) (e)
C3 CI C3 C2 C3 C3

b (e} (e) b(e} (e) b(e } (e) ]


= In (e)I ~
I CI 1 C2 I C3
M (e)
1,2
b(e} (e)
2 CI
b( e} (e)
2 C2 .
b(e} (e)
2 C3
IE: b (e} (e) b(e} (e ) b(e ) (e)
3 CI 3 C2 3 C3

l
C(e}
I
b(e)
I CI(e )b(e)
2 C(e}
I
b(3e) ] }

+ ~ (e}b( e)
C2 1
(e} b( el
C2 2
(e} b( e)
C2 3 '

{ l
2 (e} b( e) (e}b (e ) (e} b(e )
C3 1 C3 2 C3 3
(e) (e) (e ) (e )
1 C1 C1 CI C2
M~~{ = In (el I (v+ -) c~e} cie) 4el c~e)
IE: (e) (e) (e) (e)
C3 c1 c 3 C2

l
b(e}b(e} b(e}b(e} b(e}b(e } ] }
V 1 1 1 21 3
+_ b(e )b(e} b (e}b(e} b(e}b(e}
2 21 2 2 23 '
b~e} bi e} b~e} b~e } b~e } b~e}
(e} b(e) (e} b(e) (e) b(e) ]
CI I CI 2 CI 3
M (e} = In(e}1 ~ (e} b(e) (e} b(e) (e} b(e)
2,2 C2 I C2 2 C2 3
{ IE: [ (e} b(e) (e} b(e) (e} b( e)
C3 1 C3 2 C3 3

b (e}
I C(e)
I
b(e}
I C(e) 2
b(e}
I C(e) ] }3
+~ b(e} (e)
2 CI
b(e ) (e)
2 C2
b(e ) (e)
2 C3 .
2 [ b (e) (e) b(e ) (e) b(e ) (e)
3 C1 3 C2 3 c3

Therefore, the local element gradient is given by

r
V'[ e(el = K e(e)u (e} _ F (e},
where

"1(' ) ~ [ mJ')
a[~ e} a[~e} a[~ e} a[~ e } 01)')
e a (e) a (e) a (e) a (e) a (e) a (e)
UI vI U2 V2 U3 V3

u te) = [u i e ) (e)
VI
(e)
U2
(e)
V2
(e)
U3 -r
V3 '

In(e}1
F (e) = - - [9x 9y 9x 9y 9x 9Y f ,
3
12.3. PENA LTY LINEAR TRIANGULAR STOKES ELEMENT 305

and
(e) p( e) (e ) p(e) (e) p(e)
0:1 ,1 1,1 0:1,2 1,2 0: 1 ,3 1,3
(e) (e) (e) (e) (e) (e)
1]1,1 1'1,1 1]1,2 1'1 ,2 1]1,3 1'1,3
(e) pre) (e) pre) (e) pr e)
K (e )
e
= 0: 2,1
(e )
2,1
(e)
0:2 ,2
(e)
2,2
(e)
0:2 ,3
(e)
2,3
(e) ,
1]2,1 1'2,1 1]2,2 1'2,2 1]2,3 1'2,3
(e) pre) (e) p( e) (e) pr e)
0: 3 ,1 3,1 0: 3 ,2 3,2 0: 3 ,3 2,3
(e) (e) (e) (e) (e) (e)
1]3, 1 1'3 ,1 1]3 ,2 1'3,2 1]3,3 1'3 ,3
in which
o:(e) = In(e) I[ ( v + ~) b(e)b~e) + ~ e re ) e (e) ] ,
' ,J E • J 2 ' J

p( e) = In(e)I (~b(e) e(e) + ~ de) b (e ))


' ,J E' J 2' J '

1'( e) =
.,J
In(e) I [ ( v +~)
E
e (e) e (e )
• J
+~ b (e ) b (e )] ,
2 ' J

1](e) = In(e)! (~ e(e) b(e) + ~ b (e )e (e ))


.,J E' J 2 ' J '

for 1 :::; i, j :::; 3.

Notice that each term in the expression of \D(u)1 2


and IV' . ul 2
is a quadratic
function in ( u~e) , vie), u~e) , v~e) , u~e), v~e) ) and can be rewritte n in terms of matrix
multiplications. For example,

(e) (e) ]T
u3 v3 '
306 12. STOKES EQUATIONS AND PENALTY METHOD

and

(e) ( e) ( e) (e) (e) ]


VI U2 V2 U3 V3
(e ) ( e) ( e) b( e) ( e) ( e) ( e) b (e) (e ) (e) ( e)b( e) (e)
CI CI CI I CI C2 CI 2 CI C3 CI 3 UI
b( e) (e) b( e) b( e) b( e) (e) b(e) b( e) b(e) (e) b(e) b ( e) (e )
I CI I I I C2 I 2 I C3 I 3 VI
(e ) (e) ( e) b( e) (e) ( e) ( e) b( e) (e) (e ) (e)b ( e) (e )
x
C2 CI C2 I C2 C2 C2 2 C2 C3 C2 3 U2
b (e) (e) b( e) b(e) b (e) (e) b( e)b( e) b( e) (e) b( e) b( e) (e)
2 C1 2 1 2 C2 2 2 2 C3 2 3 V2
(e ) (e ) ( e) b(e) (e) (e) (e) b( e) (e) ( e) (e) b( e) (e )
C3 C1 C3 I C3 C2 C3 2 C3 C3 C3 3 U3
b( e) (e ) b (e)b(e) b( e) (e ) b(e)b( e) b(e) (e) b( e)b( e) (e)
3 C1 3 I 3 C2 3 2 3 C3 3 3 V3

Therefore, 1£e) (u) can be written in the following matrix form :


I£el(u) = ~ (u(e))T K~e)u(e) _ (F (e))T u(e).

Suppose that , corresponding to the local nodes 1,2, and 3, the global nodal numbers
of the triangle n (e) are i, j, k , where i < j < k. Let N denote the total number of
nodes in the finite element mesh. We have :
u~e) = u; u~e) = u; u~e) = u.. vi e) = Vi , v~e) = Vj, v~e) = V k ,
Fi = Fie) + , Fj = F~e) + ... , Fk = FJe) + ...,
K ii = Ki~) + , K ij = Ki~) + , K ji = K~~) + , K ik = Kg) + ,
K ki = K~~) + , K jk = KW + , K kj = K~~) + , Kkk = K~;) + ,
wher e ... denotes contributions from other elements that share the same node. Let

iT = [UI VI u, v, UN vNf,
F(e) = [0 0 gx gy ... 0 of ,
12.3. PENALTY LINEAR TRIANGULAR STOKES ELEMENT 307

and

0 0 0 0 0 0 0 0

(e) /3(e) (e) /3(e ) (e) /3(e)


0 0: 1 ,1 1,1 0: 1 ,2 1,2 0: 1 ,3 1,3 0
(e) (e) (e ) (e ) (e) (e )
0 171 ,1 1 1,1 1]1,2 1 1,2 "11 ,3 1 1,3 0

(e) /3(e ) (e) /3(e) (e) /3(e)


0 0: 2 ,1 0: 2 ,2 0: 2 ,3 0
K( e) = (e )
2,1
(e ) (e)
2,2
(e) (e)
2,3
(e)
0 1]2,1 12,1 1]2,2 12,2 1]2,3 1 2,3 0

(e) /3(e) (e) /3(e) (e) /3(e )


0 0: 3 ,1 3, 1 0:3, 2 3, 2 0: 3 ,3 2,3 0
(e) (e) (e) (e) (e) (e)
0 1]3,1 1 3,1 1]3 ,2 1 3,2 1]3 ,3 13,3 0

0 0 0 0 0 0 0 0

Then
NE NE
Ie(u) = ~ (u( (2:K( e») U - (2: (F (e»)T) U.
e= l e= l

EXAMPLE 12 .3 . Consider the cavity Stokes problem, which is a special case


ofEq (12.7). In the cavity the flow turns around , with small separation zones along
the boundary walls. The solution domain n is the unit square (0, 1) x (0,1), and
the flow is described by the velocity vector u, the first component of which denotes
the horizontal velocity and the second the vertical velocity, such that u = { ~}
(see Fig. 12.1). Letu = OonthewallOA UABUDB,anduo = {~o} = {O}
onOD.
II = UO' V= O
D

II =0 II =0
V =0 V =0 II

A B

II =O . V =O

Fig . 12.1. Cavity Structure.


308 12. STOKES EQ UATIONS AND PENALTY METHOD

We use eight linear triangular penalty elements to approximate the solution of


this cavity problem. The connectivity matrix is given above. The global nodal
coordinates are marked in Fig. 12.2. The local matrices are comp uted as

1/ 0 0 -1/ -1/ - 1/
2 2
0 2 (1/ + ~) E:
0
E:
2 (1/ + ~)
2 2
K (e)=
0
E:
2 (1/ + ~) 0 -2 (I/+~) E:
e -1/ 0 0 1/ 1/ -1/
2 2 2
1/
E:
-2 ( 1/ + ~) 1/ 31/ + -
E:
- (- + 1/)
E:
2 2 2
-1/ -2 (I/ + ~) E:
-1/ -( -+1/)
E:
31/ +-
E:

9x
9y
F (e) =~ 9x fore = 1, .. . ,8.
24 9y
9x
9y

Since on the boundary nodes we have (U2, V2) = (Uo,O) and (UI, Vd = (U3, V3)
= (U4 , V4 ) = (U6 , V6 ) = (U7 , V7 ) = (Us, Vs) = (0,0), the only unknowns are
Us and Vs . By using the connectivity matrix , we get (uf) ,V~2») = (uP),v~3»)
= (u~4),v~4») = (u~S) , v~s») = (ur ), vf») = (Us , Vs ). The globa l system is
assembled and simplified to

{_(::~) } Uo +
[
_ (~:~) -(2"++~~) ] {U,}~ ~ {9, }.
8
121/
E:
Vs 4 9y

(1/2, I)
1 2 3 (I, I)
(0, I) Connectivity Matrix
e k
1 4 2 1
2 2 4 5
3 5 3 2
(0, 1/2)4 6(1, 1/2) 4 3 5 6
5 7 5 4
6 5 7 8
7 8 6 5
8 6 8 9
(1,0)
8 9
(1/2,0)

Fig. 12.2. Unit Square Cavity Stokes Problem.


12.4. PENALTY BILINEAR RECTANGULAR STOKES ELEMENT 309

Suppose that in the above example the cavity is a square of 1 em, u = 1.004 X
10- 2 cm 2/s , Uo = 10 cm/s, and there is no body force. Let e = 0.5. Then the
system is
16.12048 -8.02008] {Us} {0.2008}
[ -8.02008 16.12048 Vs - 8.01004 .
The solution is (Us, Vs) = (0.3451,0.6686).
REMARK . It is well known that in the penalty method for the Stokes problem,
the penalty finite element solution converges to the exact solution of the Stokes
problem if the size of the elements is proportional to the penalty parameter e and
the rate of convergence is only in the first order of e. Therefore, the numerical
solution obtain ed in the above calculation is by no means accurate. Finer partitions
of n and proportionally smaller e are required to obtain more accurate numerical
solutions. _

12.4. Penalty Bilinear Rectangular Stokes Element

Let n( e) be a rectangle in a finite element partition of the solution domain n for


e = 1, . .. ,NE , where NE denotes the total number of rectangle s in the partition.
Let N denote the total number of nodes. The coordinates of the rectan gle are
labeled as ( x (e) y(e) ) ( x (e) y (e)) (x (e) y (e)) (x (e) y(e) ) locally and (x(e) y (e) )
1 '1 '2 ' 2 ' 3 ' 3 '4 ' 4 ' ""
. . k , l :S N . For (x ,y) E
I baIIy forl :S t,],
( X (e) 'Yj(e)) , ( X ( e) ' Yk(e) ) , ( XI( e) ' YI(e)) go
j k
where e = 1, . .. , NE, let a piecewise bilinear interpolation function be
n( e) ,
defined by

where
d,(e) ( ) _ (x - aWj - b) d,(e) ( )_ (x + a)(y -b)
'f'1 X, Y - 4ab ' 'f'2 X,Y - 4ab '
d,(e) ( ) _ (x + a)(y + b) d,(e) ( ) _ (x - a)(y + b)
'f' 3 X, Y - 4ab " 'f' 4 X, Y - 4ab '
in which
_
(e)
Xl
+ X (e)
X = X - ~---=-
2
2
(e) + (e) .
{ y -_Y - Y4 2 Y1 .
(Continued on page 311)
w
......
o

u b 1 u b 1 -u b -1 -u b -1 u 1 u -1 -u -1 -u 1
3+6 6+6 ......
A
8 3 12 8 6 12 8 8 3 4 3 4 6 4 6 4 !V
1 u b -1 u b -1 -u b 1 -u b 1 b 1 - 1 -1
-b -b b tr:
8 6+3 8 6 6 8 12- 6 8 12+3" 4 3 4 6 4 6 4 3 '"'3
u b -1 u b -1 -u b 1 b 1 u 1 u -1 -u -1 -u 1 o
:;.:
--- -
- u- -
3 12 ""8 3+6 8 6 12 8 6 12 8 3 4 3 4 6 4 6 4 gJ
1 u b -1 u b -1 -u b 1 -u b -1 -b -1 b 1 b 1 -b
1 trJ
K (e )
e
= V 8 6 6 8 6+3" 8 12+6 8 12 6 4 6 4 3 4 3 4 6 D
-u b -1 -u b -1 u b 1 u b 1
+-E: -u -1 -a 1 a 1 a -1 C
--- --- 3"+6 6 4 6 4 3 4 3 4
6 12 8 6 12 8 8 3 12 8 ~
-1 -u b 1 -u b 1 u b -1 u b -1 -b -1 b 1 b 1 -b oZ
8 A 12 6 8 12+3" 8 6+3 8 A 6 6 4 6 4 3 4 3 4 6 rn
-u -1 -a 1 a 1 u -1
-u b 1 -u b 1 u b -1 u b -1 >
6+6 --- 3+6 6 4 6 4 3 4 3 4 Z
8 6 12 8 3 12 8 8
-1 1 b 1 -b -1 -b -1 b tl
-u b 1 -u b 1 u b - 1 u b
4 3 4 6 4 6 4 3 '"0
8 12+3 8 12 6 8 6 6 8 6+ 3" trJ
ut e) Z
1 >
v (e) gx
1 gy S3
ute ) -<
2 -gx
v (e) -gy a:::
u te) = u~e) I( ' F(e) = In(e) I
gx ~
v (e) 4 gy ::r:
3 o
u te) -gx tl
4 -gy
v( e)
4
12.5. PENALTY TRIANGULAR POWER-LAW STOKES ELEMENT 311

Similar calculations as in §12.3 yield

where K~e), u(e), and F(e) are defined on page 310. In these matrices we have
used the notation a = ~, b= ~; and ID (e) I as the area of rectangle D(e). We use
four such bilinear rectangular penalty Stokes elements to approximate the solution
of the cavity problem.

12.5. Penalty Linear Triangular Power-Law Stokes


Element

Consider the stationary power-law Stokes problem :

-KV· (ID(u) In-1D(u)) + Vp = g in D,


V · u = 0 in D, (12.11)
u = u on aD.

Note that if K = II and n = 1, the power-law Stokes equation (12.11) reduces to


the linear Stokes equation for Newtonian flows.
The corresponding penalty formulation is given by

Minimize Ie(u),
{ uEV, (12.12)

where in this case

Ie(u) =
n + 1 in
r
~ !D(u)l n+1 dxdy - g. udxdy
in
r
+ 1 r
IV .u!n+1dxdy.
(n+1)c in

The Euler-Lagrange equation corresponding to Eq (12.12) is

n 1
-K~ . (ID(u) r - D(u)) - ~ V (IV. u!n-1 V . u) = g, inD ,
{
u = u, on aD.
(12.13)
Let (u, p) denote the solution of Eq (12.11) and u, the solution of Eq (12.9) or
(12.10). Then the pressure p of the solution of Eq (12.7) can be approximated by
312 12. STOKES EQUATIONS AND PEN ALTY METHOD

PE = - ~€ 1\7 .UEln - 1 \7 -u". In fact, it has been verified that (u, p) = lim (u, , PE). E ~O
For the linear triangle shape functions, we have

I~e)(u) = ~
n +1
r
Jo {e)
n
ID(u)l + dxdy -
1
r
J o{e)
g. udx dy

+ (n +1 1)e 1I
O {e )
I
n 1
\7 . u + dx dy ,

where

I
3 3

In . n+ 1 =
v U
(aU aV) n+l =
aX + a ( ' " (e)b .
L.,; u, ' " (e )
, + L.,; v, .) n+l
C, .
Y i= l i= l

Thus,
12.6. SOL UTIONS BY CONJ UGAT E GRADIENT METHODS 313

12.6. Solutions by Conjugate Gradient Methods

Suppose that {n ee) : e = 1,2, . . . , NE } is a finite element partition of the solution


domain n. Let C(NE , NL) be the corresponding NE x NL connectivity matrix,
where NE is the number of elements in the partition and NL the number of local
nodes on each element domain n (e). Let {<l'i(X, y)l(x , y) E nh=I,N denote the
set of global shape functions corresponding to the finite element partition, where

<l'i(X,y) = { ¢je)(x , y) . if i = C(e,j) and (x ,y) E n ee),


o otherwise.

Let

If. Ui<l'i(X, y) }
=
S~(n) = {v( x,y ) : v
{L~ Vi<l'i( X, y) '
i
(UI, VI, U2,V2..., UN, VN) E R 2N , (x , y) E n} ,
be the set of all possible global interpolation functions determined by the local shape
functions associated with the partition and the connectivity matrix. Without loss of
generality, we assume that the last M nodes are the boundary node s (1 ::; M < N)
and use U2(N - M) +l, V2(N-M)+I"" , UN, VN to denote the boundary values of
U on these boundary nodes. For x = (Xl , X2"" , X2(N -M )- I,X2(N-M )) E
R 2( N -M ) , let

NtM X2i- l <Pi(X, y) +


i= 1 i=N- M+l
f U2i- 1 <Pi(X, y) }
U = N- M N '
{ L x 2i <l'i(X, y) + L V2i<l' i(X, y)
i= 1 i=N- M+l
and con sider the unconstrained nonlinear program:

min f e(x) , (12.14)


xE R 2 ( N - M )

where f e(x) = Ie(u ), and

Ie(u)=~ {!D(u)l n+Idxdy_ (g ,UdXd Y+ ( 1) (1\7,ul n+ldxdy .


n + I1n in n + 1 e in
If n = 1, then we have the Newtonian linear Stokes problem and f e(x) is a
quadratic function in x. The minimization problem can be written in the stand ard
form as
314 12. STOKES EQUATIONS AND PENALTY METHOD

where matrix A " is obtained by writing each I~e) in the matrix multiplication form
and summing over the number of elements .
EXAMPLE 12.4. Consider the same cavity linear Stokes problem as described
in Example 12.3. Use four bilinear rectangular elements to approximate the penalty
solution. We use the connectivity matrix

C=
1 4
3 8 4 1
9 5]
[2 1 5 6 '
7 3 1 2

where global nodal coordinates are marked in Fig. 12.3.

(112, 1)
9
(0, 1) 8 (1,1)

CD 0
(1/2, 112 )
(0, 112'1 I 3(1, 112)

Q) 8)

(0,0) 6 (1,0)
2 7
(1/2, 0)

Fig. 12.3. Four Bilinear Rectangular Elements.

Here the global nodes are numbered differently from those in Fig . 12.2. In this
a
simple partition, = b= 1, and we have

I~l) = ~ [U1 VI U4 V4 Ug Vg Us Vs] x


1 1 1 1 -1 -1 - 1
- - - - - - 0 -
2 8 4 8 4 8 8
1 1 -1 -1 -1 1 1
- - - 0 - - - -
8 2 8 8 4 8 4 U1
1 -1 1 -1 -1 1 -1 1
- - - - - - VI
4 8 2 8 4 8 4 8
1 -1 1 -1 1 1 -1 U4
- 0 - - - - V4
x 8 8 2 8 12 8 4
-1 -1 - 1 -1 1 1 1 1 Ug
- - - - - - - -
4 8 4 8 2 8 4 8 Vg
-1 -1 1 1 1 1 -1
- - - - - - 0 Us
8 4 8 4 8 2 8 Vs
1 -1 1 1 - 1 1 -1
0 - - - - - - -
8 4 8 4 8 2 8
-1 1 1 -1 1 -1 1
- - - - 0 - -
8 4 8 4 8 8 2
316 12. STOKES EQUATIONS AND PENALTY METHOD

1 1 1 -1 -1 -1 -1 1
- - - - - - - -
3 4 3 4 6 4 6 4
1 1 1 1 -1 -1 -1 1
- - - - - - - -
4 3 4 6 4 6 4 3 UI
1 1 1 -1 -1 -1 -1 1
- - - - - - - - VI
3 4 3 4 6 4 6 4
-1 1 1 Uo
-
4
--1
6
-1
-
4
-31 -
4
-
3
1
-
4
-1
-
6 0
x -1 -1 -1 1 1 1 1 -1 0
- - - - - - - -
6 4 6 4 3 4 3 4 0
-1 -1 -1 1 1 1 1 -1
- - - - - - - - 0
4 6 4 3 4 3 4 6 0
-1 -1 -1 1 1 1 1 -1
- - - - - - - -
6 4 6 4 3 4 3 4
1 1 1 -1 -1 -1 -1 1
- - - - - - - -
4 3 4 6 4 6 4 3
gx
gy
-gx
1 -gy
- - [U VI Uo 0 0 0 0 0]
16 I gx
gy
- gx
- gy

il m}
1
-

Uol [i
8
v 1
= -2 [U I VI -
2
-1
-
8
1 1
-

[i r '} ,
-
4 3

Uol UJ
1 1 1
+ -E: [UI VI
Uol -
3 4
1 1
VI
U
o
- - [U I
16
VI

li ~HU' } ~ It tJ{ ~: }
- -
4 3

v
= -2 [ UI + E: [U,

n,}
VII ~ VI VII
2
1
- -
16
[U I Vd { : : } + V~o [U 1 VII

U
+ -E:O [UI VI]
C} -
~
V 2
+ :1 (Uo) + 3E: (Uo) + 16 gx ii;
1 2 1
12.6. SOLUTIONS BY CONJUGATE GRADIENT METHODS 317

Similarly,

111

1 (2) = ~ [Uo UI Vtl 2


2 14
_4
121
~~1 { u,~o~ }

i ~:t I{~~}- ~
8 8 2

~
+ [ti;
e
U, VI I ~
!1 !1 VI 16
[U o UI

1 1
- -
4 4 3

~ ~ [U, Vd [~I ] {~: } + ~ [U, Vd[ ~I j{ ~: }


1 1

-I~[U' Vd{=::}+U~O [U, Vd{n+~O[U' Vd{ ~1}

I] {~: }~
u 2 1 2 1
+ "4 (Uo) + 3€ (Uo) + 16 gyUO'
1 -1]
1 (3) ~ ~ [U, Vd [~I + [U, Vd [~I t {~}
~

[! !j{ ~: } ~
- [UI vtl { -gx } ,
16 -gy

1 (4) ~ ~ [U, Vd + [U,


16
- 1 [UI Vtl { : : } .

Thus,

I, ~ t, ~ ~ [U,I ;' ) Vd [! i] {~: }~ + [U, Vd [~ ~]{~:}


+ v~o [UI VI] { ~ } + ~o [UI VI] { 4~3 } + ~ (UO)2
2
+ 3€ (Uo)2+116 (gx + gy)Uo
318 12. STOKES EQUATIONS AND PENALTY METHOD

vUo 4UO}
VI] -2-+&
{
°
Since v = 1.004 X 10- 2 cm 2/s, Ui, = 10 cm/s, gx = gy = 0, and e = 0.5, we get
the minimizer (Ul, Vd = (1.9107,0) .•
A standard conjugate-gradient method for the above quadratic unconst rained
program can be described by the following algorithm .
Set xo=O, ro=b=b-Axo,P-l=0 ,(30=0,
set the convergence tolerance 8 > °
For i = 0,1, ....
If [r.] < 8, stop .

An alternative way of solving the matrix equation Ax = b, as is done in


the example in §12.3, is to solve the quadratic program by a direct minimizing
method such as the conjugate gradient method described above when the matrix
A is symmetric and positive definite .
If n =j:. 1, we have the power-law Stokes problem and Eq (12.14) is a nonlinear
nonquadratic program. Nonlinear versions of conjugate gradient methods are
available for solving for its minimizer. In general, a nonlinear conjugate gradient
method has the following form:
Set P-l = 0, (30 = 0, and set convergence tolerance 8 >0
For i = 0, 1, . ..
If IIV'f(Xi)II< 8, stop .
If i > 0, set
(3i = V'(J(Xi))~V'f(Xi)
V'(J(xi-d) V'f(X i-d
Set Pi = -V' f(X i) + (3i Pi-l
Use a line search to determine xi+l = Xi + O:"i Pi

The following are three similar versions of the nonlinear conjugate-gradient


methods that differ in the formula for (3i. When applied to a quadratic function
these version s are equivalent, but they behave differently on general nonlin ear
12.6. SOLUTIONS BY CONJUGATE GRADIENT METHODS 319

functions. The three best-known formulas for f3i are:


'V(J(Xi) {'V!(Xi)
1. f3i = T '
'V(J(Xi-l)) 'V!(Xi-l)

2. f3i = (Yi-l) T;, j(Xi) ,


'V(J(xi-d) 'V!(xi-d

. _ (Yi_l)T'V!(Xi)
3. 13
t - T
(Yi-l) Pi-l
The following hybrid nonlinear conjugate gradient method seems to be a better
choice for finding the minimizer of the above nonlinear program compared with
the above three formulas ,
Set P-l = 0, 130 = 0, Xo = 0, and set the convergence tolerance
8> 0.
For i = 0,1, ....
If 11'V!(Xi) II <8, stop .
Ifi>O,set
Yi-l = 'V!(Xi) - 'Vf(Xi-d
. T
f3HS = (Yi-I) 'Vf(Xi)
t (Yi-l{Pi-1
f3DY = 'V(J(Xi)) T 'V!(Xi)
t (Yi_1)T Pi-l
f3i = max {O, min{f3[is, f3py} }
end if
Set Pi = -'Vf(Xi) +f3iPi-l
Use a line search to determine (Yi and set Xi+l = Xi + (Yi Pi

In this algorithm, a back tracking line search scheme is used and the step length
(Yk is the first element of the sequence: 1,2- 1,2- 2,2- 3 , . . . , 2- i , .. . that satisfies
a sufficient decrease condition (see, e.g., Nash and Sofer 1996, for details). This
hybrid nonlinear conjugate gradient method was first proposed by Touati-Ahmed
and Storey (1990) . It has been tested to perform well for many difficult numerical
examples (Dai et al. 1998) in comparison with the Polak-Ribiere-Polyak scheme
(see Polak and Reviere 1969, and Polyak 1969), and it does not require the line
search scheme to satisfy the strong Wolfe condition. A listing of the Matlab
program that implements the above scheme to calculate finite element solutions
for the cavity problem is given in §14.3.1.
EXAMPLE 12.5. We will give a numerical solution by using this Matlab
program. For simplicity, we only test the method by using the linear triangular
elements on a cavity Stokes flow problem. The numerical example is problem
(12.11) with n
= (0,1) x (0,1), K = 1, g = 0, and the boundary conditions
320 12. STOKES EQUATIONS AND PENALTY METHOD

Cavity Power-law Stokes flow at r=2.0, h=0.05, eps=O.05


1.2 ~--~----,----~--~----,----~---,

• • • • • • • • • • • • , • • • • • I
- - • 1 \

, \

0.8

0.6

0.4

0.2

-0.2
-0.2 0 0.2 0.4 0.6 0.8 1.2

Fig . 12.4. Newtonian Stokes Flow in a Square Cavity.

Cavity Power-law Stokesflow at r=3.0.h=0.05. eps=0.05

- •••••• I ••••• _,

0.8

0.6

0.4

0.2

_0.2 l - -- - - ' ' - - - - - - - - ' - - - - - - - ' - -- - - ' - - - - - ' - - - - - L -_ _--.J


- 0.2 o 0.2 0.4 0.6 0.8 1.2

Fig. 12.5. Power-Law Stokes Flow in a Square Cavity.


12.7. EXERCISES 321

lie = (1,0) on (0,1) x {I} and lie = (0,0) otherwise. Figure 12.4 (page 320)
is the graphic solution of the problem for the Newtonian Stokes flow with r = 2,
i.e., n = 1. Figure 12.5 (page 320) is the graphic solution of the problem for the
corresponding power-law non-Newtonian Stokes flow with n = 2. In both of these
numerical experiments we take e = 0.05 and h = 0.05. Note that r = n + 1 in
the Matlab code . These numerical experiments support our analysis. Oscillations
in the gradient of the objective function are observed. One apparent differenc e in
the two cases is the center of rotation. For the shear thinning flow, the center is
lower, which coincides with physical intuition.•

12.7. Exercises

12.1. Water is flowing above an open square cavity of 1 em with a horizontal


velocity of 10 cm/s. The viscosity of water is measured as 1.004 x 10- 2
cm 2/s . Use four equal-size bilinear rectangular Stokes penalty elements with
e = 0.5 to approximate the velocity at the center of the cavity.
12.2. Solve the cavity problem of Exercise 12.1 by dividing one of the four rect-
angles into two triangles as shown in Fig. 12.6. Combine the three remaining
three bilinear rectangular elements with the two resulting linear triangular ele-
ments and solve the penalty finite element equation for velocities at the center
of cavity.

(0.5, 1)
9 4 8 (1, 1)
(0, 1)

(0,0.5) 5
CD

0
1
X (3)

CD
3 (1,0.5)

(0,0) 6 0 ,0)
2 7
(0.5, 0)

Fig. 12.6.
12.3. Blood is flowing above an open square cavity of 1 em with a horizontal
velocity of 10 cm/s. The consistency coefficient and the fluid index of the
322 12. STOKES EQUATIONS AND PENALTY METHOD

blood are measured as 1.029 Pa s" and 0.703, respectively. Use four equal-
size bilinear rectangular Stokes penalty elements with e = 0.5 to approximate
the velocity at the center of the cavity.
12.4. Develop a computer program for Exercise 12.2 by using Matlab, Mathemat-
ica, or another programming language, such as Fortran or C.
12.5. In Exercise 12.1, solve for the finite element solution by minimizing the
energy I , using the conjugate gradient method as in Example 12.4.
12.6. In Exercise 12.3, solve for the finite element solution by minimizing the
energy Ie using the three nonlinear conjugate gradient methods (three different
(3/ s given on page 319). Compare the results.
12.7. In Exercise 12.3, solve for the finite element solution by minimizing the
energy I , using the hybrid nonlinear conjugate gradient method given on page
319.
12.8. Solve the linear Stokes problem of Example 12.4 for the cavity shown in
Fig. 12.7, using the same data with Uo = 10 cm/s.

v =0. u = IOcmls
(0. 0) (3. 0)

O. - I)

0,2.5) (3. 2.5)

Fig. 12.7.
13
Vibration Analysis

In this chapter we set up the finite element matrix systems for the vibration of elastic
rods, Euler beams, and flat elastic structures and derive finite element solutions
of some examples. We also solve the eigenvalue problems that arise from the
transient paraboli c and hyperbolic equations and discuss their relationship with
the Helmholtz equation.

13.1. Hamilton's Principle

The equations governing the vibration of elastic rods, the Euler beams , and flat
elastic structures are

(13.1)

(13.2)

(13.3)

where

u = {u((X, y)) }, g = {9X((X, y)) } , (x, y) E fl,


V x,y 9y x ,y

<JXX
<J y y } = [Cll
C21
C
C 22
12
0]
a {CE:
Xx}
yy ,
{
Tx y a a C33 TX y

P. K. Kythe et al., An Introduction to Linear and Nonlinear Finite Element Analysis


© Springer Science+Business Media New York 2004
324 13. VIBRATION ANALYSIS

and E: xx , E: yy, and IX Y are defined in (10.2). For simplicity, we assume that n is a
bounded polygonal domain in the two-dimensional space R 2 •
The Hamil ton's principle involves an energy functional I( u) which can be
written as the kinetic energy Ek minus the elastic potential energy E p plus the
work W done by an external force ; that is, if

then

I(u) = ~
2Jv
r p(u fudV - ~2Jrv uTedV + JvrgTud+ i:r eudS.
For an clastic body occupying a volume V, we have

u = [u , V, (ufU = it? + v2 + 'Ii? , gT = [gx, gy, gz]'


wjT ,
gT u = gxu + gyV + gzw, t T = [tx, ty, tzJ, gT u = gxu + gyV + gzw,
t T U = txu + tyv + t zw, e = [e: xx e:yy e: zz IXy IXZ I yz ]T,
U =[CT xx CT yy CT zz Txy Txz Tyz]T .

This general form of the total energy can be reduced to simpler forms for the above
three cases as follows.
CASE 1. For an axial elastic rod with cross-sectional area A, density p,
Young's modulus E, and subject to an axial force f , we have m = pA, a =
[CT xx , 0, 0, 0, 0, OjT ,e: = [e xx , 0, 0, 0, 0, ojT, u = [u(x), a, ojT,g = [I, a, alT,

exx = ~:' and CT xx = Ee xx . Thus, the total energy is

r 2dx - "2 Jr EA ( au)


I (u) ="2 J pAit,
1
L
ax
2 r L
dx + Jo fud x.
1
L

o o
CASE 2. For the Euler beam, it is assumed that u(x , y) = -y ~~, v = v(x) ,
and g = [a, r(x), alT , Then
au a 2v
xx
e = ax = - Yax 2 '
1 ( au av )
e: xy ="2 ay + ax = a,
eyy = E: xz = e yz = e: z z = a.
13.1. HAMILTON 'S PRINCIPLE 325

Thus, the elastic potential energy is given by

e, = ~
2 iv
rO"XXCxx dV = ~2 i ro i;rEc~x dA dx,
L

21 l«r i r E (a (r )
2V)2 2v)2
1 t'
L
(a
= A y aX2 dA dx 2 i E i y2 dA
= aX2 dx A
o

21 l«t' EI
2v)2
(a
= ax 2 dx ,

where I = fA y2dA is the second moment of area of the cross section about the
z-axis, and the kinetic energy is

Ek = - IlL
2 0
pAv 2 dx.

Thus, the total energy is

21 i t' pAv2dx - 21 i t' EI t' rvdx .


2V)2
(a
I(v) = ax2 dx+ i
o o o
CASE 3. For a flat elastic body of thickness h occupying an area of n in
the xy plane, the displacement vector is given by u = {~~:: ~~ }, and e =

au av (aU aV) ] T.
+ ax The stress-strain
. reIanon
" = h e IS
C c, were '
[ ax ay ay IS (J"

defined by (10.10) . The total energy is

I(u) =~ rph(iJ?+v
2in
2
) dA-~
2in
rheTCedA
+ r (gxu + gyv)dV + iC2r (txu + tyv) ds,
in
where t = { ~: } is the applied boundary forces per unit arc length on O2 , which
is that part of the boundary an where t is applied.
Recall that in steady-state cases when displacement is independent of time
we seek a displacement u that minimizes the total energy I among all possible
displacements within certain constraints as required by the principle of virtual
work. This implies a necessary condition

d~ [I(u + TV)] 17=0= 0


for all possible V and any real number T so that u + TV satisfies the constraints.
Here I represents the total potential energy, I = E p - W . The principle of virtual
326 13. VIBRATION ANALYSIS

work is a special case of Hamilton 's principle which deals with time dependent
situations. Specifically, Hamilton's principle requires that we seek a displacement
u such that U(tl ) = U(t2) and u(td = U(t2) for any time interval (h,t2) ;
and that for all displacement of the form u + TV, with V (t d = V (t2) = 0 and
v v
(h) = (t2) = 0, where T is any real number, the function u satisfies

I
t2
d
-d [I( u(t) + Tv(t))l! dt = 0 for all such V and for any interval (t l , t2),
tl T T= O

where I = Ek - E p + W . It can be shown that if the displacement u satisfies this


equation of Hamilton's principle, then it must also satisfy the differential equations
(13.1), (13.2) and (13.3) , respectively, in each case, provided that all the quantities
appearing in the equations have continuous derivatives . For the elastic rod in case
I above , the above necessary condition gives

Jtlr J{L
o
(
pAuv - EA ax ax
au av + I v ) dxdt = O.

Using integration by parts and taking v (td = v (t2), we get

Supp ose that the quantities p, E , A, I , ~:~ , and ~:~ are all continuous and the
set of admissible functions satisfies the boundary condition u(O, t) = u(t) , then
u(O, t) + TV(O , t) = u(t), and we have v(O, t) = 0 for all t, and v(L, t) is arbitrary.
This gives

l1t2 L
(PAil - EA ~:~ - I) v dx dt = 0

for arbitrary v satisfying v(O, t) = v(L, t) = O. This last equation implies that

pAil - EA ~:~ - 1=0 in (O ,L) x (0, 00) ,

which is the wave equation for the elastic rod. The natural boundary condition
au
EA ax (L , t ) = 0 comes from

au v I dt = 0,
j
2 L
t
EA-
tl aX 0

since v(O, t) = 0 and the interval (t l , t2) and v(L, t) are arbitrary. The other two
wave equ ations and boundary conditions can be derived similarly.
13.2. FREE AXIAL VIBRATIO NS OF AN ELAST IC ROD 327

13.2. Free Axial Vibrations of an Elastic Rod

For simplicity, first we use linear shape functions to approx imate the axial dis-
placement u. Let

u(e )(x, t) = ¢i (x)uie)(t) + ¢~e) (x) u~e) (t)


e
)

= [¢ie)(x) ¢~e) (x) ] {~~:;~:~} = (¢(e) f u (e)(t )

be the linear interpolation function in x. Here the nodal displacement vector


u (e) (t) =
u2
{u t:;(t)(t) }
is assumed to be a function of time t. Then, using the

method of separati on of variables, we have

The kinetic energy for an element is given by

E (e) =
k
~
2
l
Xl
<e)
x2

<e)
p(e)A(e) (u(e))2dx

[2 1]
where
M (e)= P(e)A(e)l(e)
_
3 1 2 .
Similarly, the local elastic potential energy is given by

E (e) = ~ (u (e))TK(e)u(e)
p 2 '
where

l
d¢ie) d¢ie) d¢~e) d¢~e) ]
dx dx dx dx dx
d¢~e) d¢ie) d¢~e) d¢~e)
----
dx dx dx dx

[1 -1]
E (e)A(e)
l(e) -1 1 '
328 13. VIBRATION ANALYSIS

and the work for the element is given by

{I}
where
j(e) z(e)
F( e) = - 2 - 1 .

Therefore, the local energy is

and the total energy is given by

NE
I(U) = LI(e)(u(e)) = ~ (Uf(t) MU(t) - ~ UT(t)KU(t) + UTF,
e= l

where U is the corresponding global nodal displacement vector, and M, K, F are


the global matrices obtained after assembly of the local matrices. By Hamilton's
principle, we need to solve for U from the global system

MU(t) + KU(t) = F,

which satisfies the appropriate initial and boundary conditions. A problem of


interest in vibration is to determine solutions of the form

Uri) ~ X ,lnw! ~ {}~ } sin ex,

where X is independent of time in the corresponding homogeneous problem

MU(t) + KU(t) = 0,

which is equivalent to solving for wand X from

where X also satisfies appropriate conditions translated from that of U . A solution


to the last equation is a pair of eigenvalue and eigenvector (w 2 , X), where X is
known as the free vibration mode and the associated w the corresponding frequency .
13.2. FREE AXIAL VIBRATIONS OF AN ELASTIC ROD 329

EXAMPLE 13.1. Compute the axial modes of vibration of an elastic rod with
Young's modulus E = 29 X 106 lb/irr', density p = 0.29Ib/in 3 , cross-sectional
area A = 1 in 2 , and length L = 100 in, by using two linear finite elements
(Fig. 13.1). Assume that one end of the rod at x = 0 is fixed and the other end is
free of stress, i.e., E~~ (L) = O.

Fig . 13.1. A Mesh of 2 Linear Elements

We have
M(e) = 0.29·50 [2
3 1
1] = 145 [2
2 30 1 ~] ,
6
K (e) = 29 x5010 [1
-1
-1]
1 = 5.8 x 10
5 [ 1
-1
-1]
1 '
e = 1,2.

The global system is

-1
2 0] - w2 [21
-1 1 4 1 O]){XI}
X {O}
0 .
2 =
-1 1 0 1 2 X3 0

Applying the boundary condition UI = u(O) = 0 gives Xl = 0 for the fixed left
end of the rod. Then

The solutions are

WI = 398.0578, X~1) } = { 0.5774 }


{ xj -0.8165 '

and

= 113.9298 , X~2) } = { 0.5774}


W2
{ xl 0.8165'·

In general, if F ;f 0, the global system

MU(t) + KU(t) = F
330 13. VIBRATION ANALYSIS

is said to be nonhomogeneous, and it can be solved by numerical schemes based


on finite difference and/or finite element methods . The choice of the numerical
schemes may depend on the form ofF.

13.3. Free Vibrations of a Euler Elastic Beam

Let v(e)(x, t) = (</>(e)fu(e) be the Hermite interpolation of v over a element


interval [x~e), x~e)], where

are the Hermite shape functions and the local nodal variables, respectively. Similar
to the steady-state case, we have

The local energy is

Using matrix products, we write

U. 4(e)] X
13.3. FREE VIBRATIONS OF A EULER ELASTIC BEAM 331

cP~e)cP~e) cP~e) cP~e)] {u~e)


-+.(e) -+.(e) -+.(e) -+.(e) . (e)
}
'1'2 '1'3 '1'2 '1'4 U2
-+.(e) -+.(e) -+.(e) -+.(e) . (e) .
'1'3 '1'3 '1'3 '1'4 U3
-+.(e) -+.(e) -+.(e) -+.(e) . (e)
'1'4 '1'3 '1'4 '1'4 U4

Thus, we get

[(e) = ~ (u(e)fM(e)u(e) _ ~ (u(e)fK(e)u(e) + (u(e)fF(e),


where

(e)
Let ~ = X ~(e~l be the local coordinate . Then x = x(~) is a function of ~ for
x~e) ::; x ::; x~e), 0 ::; ~ ::; 1, and ~; = l(e). The four local shape functions can
be written as

cP~e)(x(O) = 1 - 3e + 2e, cP~e)(x(~)) = l(e)~(l _ ~)2,

cP~e)(x(~)) = 3e - 2e, cP~e)(x(~)) = l(e)~(e - ~).

We get
dcP~e)(x(~)) d~ 1 dcP~e)(x(~))
d~ dx LW d~
and
d2cP}e) (x(~)) 1 d2cP~e)(x(~)) ~
dx 2 LW de dx
332 13. VIBRATION ANALYSIS

Therefore,

and

F;e) = (l(~)) 1r¢)e)(x(~))d~.


1

Let b = EI. Now we evaluate the entries in the local matrices under the assumption
that both band r have the constant value b(e) and r(e), respectively, in [x~e) , x~e)l.
For demonstration purposes, we only show calculations of some of them as follows:

A complete calculation gives the following local finite element matrices for the
Euler beam:
54
131(e)
156
-22z( e)

12 61(e) -12 61(e) ]


b(e)
K(e)- _ _
61(e) 4(1(e))2 -61( e) 2(l(e))2
-61(e) , (13.5)
- (1( e))3 -12 -61( e) 12
[
61(e) 2(1(e))2 -61(e) 4(1(e))2

(e) _ r(e)l(e) { l~) }


(13 .6)
F - 12 6 '
_l(e)

EXAMPLE 13 .2 . Consider an elastic Euler beam of L = 100 in long , with


cross-sectional area of A = 12 in2 , moment of inertia I = 4 in" , density p = 0.29
13.3. FREE VIBRATIONS OF A EULER ELASTIC BEAM 333

lb/in'' , Young' s modulus E = 30 X 106 psi. Suppose that one end of the beam
is fixed and clamped, and the other end is free of constraint or any force (see
Fig. 13.1). Let us use two Hermite elements of equal length to approximate the
first two vibration modes. Here l (e) = 50 in, b(e) = 4 x 30 X 106 = 1.2 X 108 ,
r( e) = 0.0 for e = 1, 2. The local matrices are

455.7 22.4 -269.3 ]


[ 646
M (e) = 455.7 4143.0 0.2693 -310.73
22.4 269.3 64.6 - 455.7 '
- 269.3 -3107.3 - 455.7 4143.0
288000 -11520 288000 ]
[ 11520
K (e ) = 280000 9600000 - 288000 4800000
- 11520 -288000 11520 - 288000 '
4800000 - 288000 9600000

g},
288000

F (' ) = for e = 1,2.

The corresponding global matrices are

64.6 455.7 22.4 -269.3 0 0


455.7 4143.0 0.2693 - 310.73 0 0
22.4 269.3 129.2 0 22.4 - 269.3
M=
-269.3 -310.73 0 8286.0 0.2693 - 310.73
,
0 0 22.4 269.3 64.6 -455 .7
0 0 - 269.3 -3107.3 -455.7 4143.0

11520 288000 -11520 288000 o o


280000 9600000 -288000 4800000 o o
K =
- 11520 -288000 23040 o - 11520 288000
288000 4800000 o 19200000 -288000 4800000 '
o o -11 520 - 288000 11520 -288000
o o 288000 4800000 -288000 9600000

The boundary condition at x = 0 is v(O , t) = 0 and v'(O , t) = 0, which translates


334 13. VIBRATION ANALYSIS

into Xl = 0 and X 2 = O. Thus, we obtain the global system

23040 0 -11520 288000]


o 19200000 - 288000 4800000
( [ -11520 -288000 11520 -288000
288000 4800000 - 288000 9600000
129.2 0 22.4 -269.3 ] )
2 0 8286.0 269.3 -310.73
- w
[ 22.4 269.3 64.6 -455.7
-269.3 -3107.3 -455.7 4143.0

The correspondingsolutions are

xj
XJ } _ {0.3554}
-0.0366
WI = 43.3197, and
{ XJ - 0.9201 '
XJ 0.1604

Xi} _ {0.6052}
XJ -0.0021
W2 = 11.7558, and
{Xl - -0.7947
Xg
- 0.0454
'

Xl} _ {-0.0238
xi -0.4088 }
W3 = 57.6717, and
{Xg - -0.8897
X~ -0.2020
'

Xi} {-0.3161
xi -0.0109
}
W4 = 2.00000, and
{xt
xt
-0.9486 . •
=
-0.0135

13.4. Free In-Plane Vibrations of an Elastic Plate

For a flat elastic body of thickness h occupyinga region n in the xy plane, let

u(e)(x y t) = {u(e)(x, y, t) }
, , v(e)(x, y, t)
= [4>~e)(X,y) 0 4>~e)(x,y)
o 4>~e)(x, y) 0
13.4. FREE IN-PLANE VIBRATIONS OF AN ELASTIC PLATE 335

u~e) (t)
u~e) (t )
u~e) (t)
X for (x ,y) E n (e),
u~e) (t)
u~e) (t)
u~e) (t)

whereO(e) is a triangle in the finite element partition oro, and ¢~e) (x , y), ¢~e) (x , y),
and ¢~e) (x, y) are the three corresponding linear shape functions, i.e.,

u~e)(t) u(x~e) , y~e) ,t)


u~e) (t) v(x~e), y~e) , t)
u~e) (t) u(x~e), y~e) , t)
u~e) (t) v(x~e), y~e), t)
u~e)(t) u(x~e), y~e),t)
u~e) (t) v(x~e), y~e) , t)
is the local nodal displacement vector. Suppressing the variables x, y, and t , the
strain vector is given by
au(e)
ax
au (e )
ay
au(e) av(e)
- - +--
ay ax
a¢~e ) a¢~e)
ax
o o ax
o
a¢~e ) a¢~e ) a¢~e)
a 0 a
ay ay ay
a¢ ~e ) a¢~e) a¢~e) a¢~e ) a¢~e) a¢~e )
ay ax ay ax ay ax

Let u( e) denote the displacement vector, and let

G (e) =
", (e)
'1'1
a ",( e)
'1'2
a ",(e )
'1'3
0]
[ a ¢~e) 0 ¢~e) 0 ¢~e)'

and
a¢~e) a¢~e) a¢~e)
0 0 a
ax ax ax
a¢~e) a¢ ~e) a¢~e)
n (e) = 0 0 0
ay ay ay
a¢~e ) a¢~e) a¢~e) a¢~e) a¢~e) a¢~e)
ay ax ay ax ay ax
336 13. VIBRATION ANALYSIS

Then

(u (e))2 + (V(e))2 = [U(e) V(e) ] {~~:~} = (U(e)(G(e))T) (G (e)U(e))

= (u( e)f(G(e)fG(e)u( e),

(u (e)fg(e) = (g(e)fe(e)g(e) = ((U(e))T(D( e))T) ere) (D(e)u(e))


= (u( e)f ((o (e))T e(e)O(e ))u(e),

(t~e)u(e) + t~e)v(e)) = [u(e) v(e ) ]{ ~: } = (u(e)f (G(e)f) { ~: }


= (u( e)f(G(e ))T {~: } .
The total local energy is

which can be rewritten in the quadratic form

where, for the linear triangular element, we have

2 a 1 a 1 a
a a
1 a
2 1 1
= phln(e)1 1 a 2 a a 1
a 1 a 2 a 1 ,
M (e) ph (G( e)fG(e)dA =
O (e) 12
1 a 1 a 2 a
a 1 0 1 0 2
K(e) = 1 O (e)
h (D(e )fe (e)D(e) d~ = h In( e)1 (D (e))T e (e)o(e),

where
b(e)
1
0 b(e)
2
a b(e)
3
D(e) = 0 c~e) 0 c (e) 0
[ (e) b(e) (e) b(e) (e )
C1 1 C2 2 C3

d11e) d12e)
ere) = de ) C( e) ao ] ,
[ o2 1 o22 C (e)
33
13.4. FREE IN-PLANE VIBRATIONS OF AN ELASTIC PLATE 337

such that
9x
9y
r (G( e)f {9X} dA = In(e)1 9x
io« ) 9y 3 9y
9x
9y

tx
ty °

tx
ty
J (G(e )f {~: } ds = l~ tx
, or
l (e)
23 x
l (e)
, or .B...
°
C 2n 80«)
ty

°
2 ty
tx
2
°
tx

if O 2 n 8n(e) = r~~ , or r~1, or r~~),


°
respectively.
ty ty

EXAMPLE 13 .3. Consider a square elastic plate of unit thickness with Young's
modulus E = 3.0 X 10 6 psi, Poisson's ratio v = 0.33, and density p = 0.29Ib/in3 •
The dimension of the plate is 10 in by 10 in. I t is assumed that one side of the
plate is clamped and fixed and the plate can only undergo in-plane displacements.
Calculate the finite element frequencies and the corresponding mod es by using two
linear triangular elements (Fig . 13.2).

y
4 . - - - - - - - -....3

Fig . 13.2. Mesh of two Linear Triangular Elements.

We divide n into two triangles: n(1) which is the triangle with vertices
(x~1) , yP)) = (0, 0), (X~l) , y~1)) = (10 , 0) , (x~1) , y11 ) ) = (10 ,10) ; and n (2 )

which is the triangle with vertices (x~2) , y~2)) = (0 , 0) , (x~2) , y~2)) = (10 ,10),
(X~2) , y12 ) ) = (0 ,10). The corresponding global nodes are node numb er 1 at
338 13. VIBRATION ANALYSIS

(0,0) , node 2 at (10,0), node 3 at (10, 10), and node 4 for (0,10) as indicated in
Fig. 13.2. The connectivity matrix is

[1 2 3]
c= 1 3 4 .

The element input data for calculating the local matrices in D(I) are

h = 1, E(I) = 3.0 x 106 , p(1) = 0.29, v(l) = 0.33,


XCI) - 0 X CI) - 10 XCI) - 10 '
1 -' 2-' 3-
(I ) _ 0 y(I) - 0 y(I) - 10
Y1 - ' 2-' 3- '

ID(1) I = ~ (10)(10) = 50.


2

Then

(b(1»)T = [bP) b~I) b~I) ]

_ _1_ [(1) (1) (1) (1) (1) (1) ]


- 2ID(1) I Y2 - Y3 Y3 - YI YI - Y2

1
= 100 [10 - 10 10 - 0 0 - 10] = [0 0.01 -0.01] ,

(c(l)f = [e~l) c~I) e~l)]


_ _1_ [(1) (1) (1) (1) ( 1) (1) ]
- 2ID(2)1 X3 -X2 Xl -X 3 X 2 -Xl

1
= 100 [0 - 10 0- 0 10 - 0] = [-0.01 0 0.01] ,
2 0 1 0 1 0
020 1 0 1
M(I) = phID(e)1 1 0 2 0 1 0
12 0 1 0 2 0 1
1 0 1 0 2 0
o 1 0 1 0 2
2.4167 o 1.2083 0 1.2083 o
o 2.4167 0 1.2083 o 1.2083
1.2083 o 2.4167 0 1.2083 o
o 12083 0 2.4167 o 1.2083 '
1.2083 o 1.2083 0 2.4167 o
o 1.2083 0 1.2083 o 2.4167

n(1) =
b( I )

~
o ~l)
C~I) 0 e(l)
0 br) 0]0 C~I)
[ (1) (I ) (1) b(I ) (1) b(I)
ci b1 e2 2 C3 3
13.4. FREE IN-PLANE VIBRATIONS OF AN ELASTIC PLATE 339

O~l ] ,
0 0 0.01 0 -0.01
= 0 0 0 -0.01 0
[o 0 -0.01 0.01 0.01 -0.01
E (1) E(l ) 1/(1)
0
(1 - 1/(1)2) (1 - 1/(1)2)
E(1 ) 1/(1) E (l)
C (l ) = 0
(1 - 1/(1 )2) (1 - 1/(1)2)

o o 2(1 + 1/(1 »)

= 10
6
[~:~~~~ ~:~~~~ ~] ,
o 0 1.278
K (l) = h In(1)1(D(1)fc(1)n(1 )
o 0 0 0 o o
o 0 0 0 o o
o 0 23223 -11945 -23223 11945
o 0 -11945 23223 11945 -23223
o 0 -23223 11945 23223 - 11945
o 0 11945 - 23223 -11945 23223

Similarly, for n (2), we have

h = 1, E (2) = 3.0 x 10 6 , p (2) = 0.29, 1/(2) = 0.33,


x (2) - 0 x (2) - 10 x (2) - 0
1 - ' 2- ' 3 - '
(2) _ 0 y (2) - 10 y (2) - 10
Y1 - ' 2- ' 3 -'

In (2) I = ~2 (10)(10) = 50 .

Then

(b(2 »)T = [b f ) b~2) b~2) ]

_ 1 [ (2) (2) (2) (2) (2) (2) ] _ (b( l l )T


- 2In(2)1 Y2 - Y3 Y3 - Y1 Y1 - Y2 - ,

(c (2) f = [ c~2 ) C~2) c~2) ]

_ _1_ [ (2) (2) (2 ) (2) (2) (2) ] -_ ( c( l» ) T


- 2In(2)1 X3 - x2 Xl - X3 X2 - Xl ,

201 0 1 0
020 1 0 1
M (2) = ph In(e)1 1 0 2 0 1 0
12 0 1 0 2 0 1
=M(l ) ,
101 0 2 0
010 1 0 2
340 13. VIBRATION ANALYSIS

b(2) (2)
[ b( 2j 0 2 0 3
D(2) = ~ C1
(1)
0 C(l) 0 C3 o]
(1)

(1) b(l ) (1) b(l) (1) b~l )


C1 1 C2 2 C3

~ [~ O~l ] ,
0 0.01 0 -0.01
0 0 -0.01 0
0 -0.01 0.01 0.01 -0.01
E(2) E(2 )V(2)
0
(1 - v(2)2) (1 - V(2)2)
E(2)v(2 ) E(2)
C(2) = 0 = C(1) ,
(1 - v( 2) 2) (1 - v (2) 2)
E( 2)
0 0
2(1 + v(2»)
K(2) = hIO(2)1 (D(2)fc(2)D(2)

0 0 0 0 0 0
0 0 0 0 0 0
0 0 23223 -11945 -23223 11945
0 0 -11945 23223 11945 -23223
0 0 -23223 11945 23223 -11945
0 0 11945 -23223 -11945 23223

The corresponding global matrices are

2.4167 0 1.2083 0 1.2083 0 0 0


0 2.4167 0 1.2083 0 1.2083 0 0
1.2083 0 2.4167 0 1.2083 0 0 0
0 1.2083 0 2.4167 0 1.2083 0 0
M= 1.2083 0 1.2083 0 2.4167 0 0 0
0 1.2083 0 1.2083 0 2.4167 0 0
0 0 0 0 0 0 0 0
0 0 0 0 0 0 0 0

2.4167 0 0 0 1.2083 0 1.2083 0


0 2.4167 0 0 0 1.2083 0 1.2083
0 0 0 0 0 0 0 0
0 0 0 0 0 0 0 0
+ 1.2083 0 0 0 2.4167 0 1.2083 0
0 1.2083 0 0 0 2.4167 0 1.2083
1.2083 0 0 0 1.2083 0 2.4167 0
0 1.2083 0 0 0 1.2083 0 2.4167
13.4. FREE IN-PLANE VIBRATIONS OF AN ELASTIC PLATE 341

4.8334 o 1.2083 o 2.4166 o 1.2083 o


o 4.8334 o 1.2083 o 2.4166 o 1.2083
1.2083 o 2.4167 o 1.2083 o o o
=
o 1.2083 o 2.4167 o 1.2083 o o
2.4166 o 1.2083 o 4.8334 o 1.2083 o
o 2.4166 o 1.2083 o 4.8334 o 1.2083
1.2083 o o o 1.2083 o 2.4167 o
o 1.2083 o o o 1.2083 o 2.4167

o 0 o o o o 0 0
o 0 o o o o 0 0
o 0 23223 -11945 -23223 11945 0 0
K=
o 0 -11945 23223 11945 -23223 0 0
o 0 -23223 11945 23223 -11945 0 0
o 0 11945 -23223 -11945 23223 0 0
o 0 o o o o 0 0
o 0 o o o o 0 0

6390 o o 0 0 - 6390 -6390 6390


o o o 0 0 o o o
o o o 0 0 o o o
o 16833 o 0 -5555 o 5555 -16833
+ o -5555 o 0 16833 o -16833 5555
-6390 o o 0 0 6390 6390 -6390
-6390 5555 o 0 -16833 6390 23223 -11946
6390 -16833 o 0 5555 -6390 -11946 23223

6390 0 o 0 o -6390 -6390 6390


o 16833 o 0 -5555 0 5555 -16833
o 0 23223 -11945 -23223 11945 o 0
=
o 0
-11945 23223 11945 - 23223 o 0
o -5555 -23223 11945 17668 -11945 -16883 5555
-6390 0 11945 -23223 -11945 29613 6390 -6390
-6390 5555 o 0 -16833 6390 23223 -11946
6390 -16833 o 0 5555 -6390 -11946 23223
Applying the boundary condition (UI, VI) = (U2 , V2 ) = (0,0) , we obtain the
system

4.8334 0
1.2083 0]
o 4.8334 o 1.2083
( [ 1.2083 0 2.4167 0
o 1.2083 o 2.4167
17668 -11945 -16883
2 -11945 29613 6390 -6390 X6
5555 ]) {XS} { 0}
- w [ -16833 6390 23223 -11946 X7 - 0 .
5555 -6390 -11946 23223 Xs 0
342 13. VIBRATION ANALYSIS

The corresponding solutions are:

C} C
367O
XJ _ -0.3129
WI = 160.4567, and
xi -
XJ
-0.6739
0.5596
}'

C} r
7345
Xg _ -0.2229
W2 = 16.90069, and
xi - - 0.6066 '
}

C}
Xg -0.2071

X~ _ {0.3142}
0.3702
W3 = 90.0671, and
xs - -0.5592 '
xg -0.6719

C} r
2278
= 67.0517, and
xt 0.7319
W4
xi = -0.1982 . •
}
xl 0.6109

13.5. Axial Vibrations of a Plastic Rod

For a plastic rod such as the one made of annealed steel, the stress and strain
relationship can sometimes be written in the form CY xx = K lexxln-Iexx in the

L
axial direction. In this case the potential energy is

Ep
_
- --
n+l
1
1 0
ACYxxexxdx - - -
n+1
_ 1 1L 0
Alexxl
n+l
dx.

The total energy is

I(u)=~ rLpAiJ?dx _ _
1_ rLKAlaaUln+ldX+ rLjudx.
2 Jo n+ 1 Jo x Jo
For n = 1 the above energy reduces to that for the elastic rod, for which K = E .
The only difference is the potential energy term. The corresponding local potential

l
energy is

E(e) = __
1 x (e)
K A
a
j_U_l (e) n +l 1
dx = - - (u( e)fK(e)u(e),
p n +1 x (e )
1
ax n +1
13.5. AXIAL VIBRATIONS OF A PLA STI C ROD 343

[1 -1]
where
K (e) = K (e )A(e) (e) _ I (e)ln-l
(l (e)r u2 ul - 1 1 .

Now, since the matrix K (e) depends on u (e), we also write K (e) (u (e») in place of
K (e). The local energy is

where

M
(e) = p (e )A (e)l (e)
6
[21 21] '
are the same as those for the clastic rod.
EXAMPLE 13.4. Consider a prismatic rod of 304 annealed stainless steel
rigidly fixed at one end. The density of the steel is 0.28907 lb/in" , the cross-
sectional area is A = 1 in 2 , the length L = 100 in, the power-law constants are
K = 77000 Ib/in2 , and n = 0.26. Suppose that there is no body force or any other
external force acting on the rod. Derive the corresponding free vibration system
for a two linear finite element approximation. Here, [ (1) = [ (2) = 50, A(1) =
A (2 ) = 1, K (1) = K (2) = 77000, n(l) = n (2 ) = 0.26, pel) = p( 2) = 0.28907.
Then we have

M
( 1) _
-M
(2) _
-
p (e )A(e)l(e)
6
[21 1]2 '

= 2 7846 x 104 Iu (1) _ u (l ) In- l [1 - 1]


. 2 1 - 1 1 '

K( 2) _ K (2)A(2)
- ([ (2»)n
I (2) _
U2 ul
(2) ln - l [1 -1]
-1 1

= 2.7846 X 104 Iu~2) - u~2) ln-l [!1 ~1].


Let U 1 = u (1 ) , U~1) = U 2 = u~2), U(2) = Us be the connection between the local
and the global nodes. Then the assembled system is

2 1 0] {U1}
[o1 41 12 tj2 + 1.156 Us
X 104 x
344 13. VIBRATIO N ANALYSIS

IU2 -
U
- IU2 - 1 1- .
O 74

U11- O.74 + 1U3 - U2 j- O.74


0
0
]
- jU3 - U2 \ - O.74 IU3 - U2 1- 0 .74
X [U 1 U2 U3 ]T = [0 0 O]T .

Applying the boundary condition U1 = 0, we get

For ii(t) = 0 this system reduces to that of Example 10.5. Thus , we can use the
initial conditions U2(0) = 29.8613 , U3 = 30.0616, [[2(0) = 0 = [[3(0), and
solve this system numerically. This system can be solved by using an ordinary
differential equation numerical technique, such as a Runge- Kutta or a Euler scheme.
See Exerci se 13.13, and also the authors' websites for more details.•

13.6. Eigenvalue Problems

We presented a one-dimensional eigenvalue problem in Exercise 3.13. Now we


discuss two-dimensional eigenvalue problems and present their finite element so-
lution s.
13 .6.1. Hyperbolic Equations. The transverse vibrations of a membrane
n, with material densit y p, are governed by the wave equation
2u
p -a - -a ( all -au ) - -a ( a22-
au ) + cou = f .
mn, (13.7)
at 2 ax ax ay ay

where all and a22 denote the tensions along the x and y axis, respectively, u
denotes the transverse deflection , CO the modulus of elastic foundation on which
the membrane is stretched, and f the transversely distributed force. The boundary
conditions are such that u = Uo or qn = tin on an
for t ~ 0, and the initial
conditions are u(x, y , 0) = uo(x, y ) and ~~ (x, y , 0) = vo(x , y). The weak form
of Eq (13.7) over an element n (e) with a test function w is
13.6. EIGENVALUE PROBLEMS 345

The semidiscrete finite element model for Eq (13.7) over the element ne e) is

0= 'NE
" ( d2 (e)
M(~) ~ + K (e) u( e)) _ f ee) _ Q (e)
L..J tJ dt2 tJ t J J '
(13.8a)
i= l

or, in matrix notation,

(13.8b)

where Mi~)' Ki~)' f j e) , and Q;e) are the same as defined in §9.3. As above, the
eigenvalue problem is defined as finding a solution of the form U j (t ) = U, e- iwt at
the global nodes such that Eq (13.7) holds for homogeneous boundary conditions
and the initial conditions and f = O. Substitution for u ;e) in Eq (13.8a) gives

(13.9)

which after assembly for all elements n( e) , e = 1,2 , . . . , NE, leads to the system

(K - w2 M) u (e) = Q. (13.10)
N
The eigenvalues w2 and the eigenfunctions LUi ¢i(X, y) are computed from Eq
i= l
(13.10). Note that in a membrane problem w denotes the radian frequen cy of its
vibration.
EXAMPLE 13 .5. Consider the problem of a square vibrating membrane of
side a with all four edges fixed, which is governed by the wave equation \72u =
2
C 8 u h d enotes the uutia
. . . I tension
. In. the mem brane , su b'ject to t he
To 8t2 ' were rt:
.L 0

boundary condition u = a on all four edges . Solve the related eigenvalue probl em
where ..\ = pw2 ITo, by taking a mesh of four linear triangular element s shown in
Fig . 13.3.

4 5

n(4)

3
n(2) n (3)

n (j)

1 2

Fig. 13.3. A Square Membrane.


346 13. VIBRATION ANALYSIS

Note that

o -2
2 -2 14 22 10 OJ1
8 8 2 2 .
2 4 1
4

Thus, we solve K 33 - AM 33 = 0, and obtain A = 24/ a 2 • However, this finite


element solution, when compared with the exact value 2n 2/a2 , shows an error of
22%. Better results are obtained by taking refined finite element meshes .•
13.6.2. Parabolic Equations. We consider the general parabolic equa-
tion (9.23) in a domain n, which is subjected to the boundary conditions (9.24)
with 13 = 0, and the initial condition u(x, y, 0) = uo(x, y) in n. The semidiscrete
finite element formulation, as described in §9.3, leads to the system (9.27b), that is,
M( e)u(e) + K(e)u(e) = F(e), where F(e) = g(e) + Q(e). The eigenvalue problem
is defined as finding a solution of the form uJe) (t) = UY)
e->.t at the global nodes,
and computing the nodal eigenvalues A and the corresponding eigenfunctions, sub-
ject to the homogeneous boundary conditions and the initial condition, and f = O.
This substitution for uJe) in Eq (9.27b) gives

(13.11)

which after assembly for all elements n(e), e = 1,2, .. . ,NE, leads to the system

(K - AM) = 0, (13.12)

where the right side of Eq (13.11) becomes a zero vector as a result of the homo-
geneous boundary conditions. A nontrivial solution of Eq (13.12) exists iff the
determinant IK - AMI = 0, which, after expansion, yields a polynomial equa -
tion in A of degree NE, and the solution of this equation gives the consecutive
eigenvalues Aj, j = 1,2, . . . ,NE.
EXAMPLE 13.6 . Consider the heat transfer problem discussed in Example
9.7 with 13 = 0 (no convection). Using a mesh of two linear elements as in Fig . 9.1,
and using the same data, we have

0.0699
0.0~99] .
0.50984 -0.49952 0] [2.1398
K= 1.01788 -0.49951 , M= 0.2796
[ sym 0.50894 sym 0.51398

We solve IK - AMI = 0, which gives the polynomial equation

0.0096746 - 0.75564146 A + 0.7830936 A2 - 0.0725025 A3 = O.


13.7. EXERCISES 347

The solution ofthis equation gives three consecutive eigenvalues >'1 = 0.0129775,
A2 = 1.05662, and A3 = 0.73132 . •
13.6.3. Helmholtz Equation. This equation arises when we separate the
spatial and time variables in the heat and wave equations, defined by Eq (9.23) and
(13 .7), respectively. Let us consider the following simplified equations:

2 2
8v = k (8 V 8 V)
8t 8x 2 + 8 y2 '
2v 2 2 (13 .13)
8 _ 2 (8 8V V)
8t2 - C 8x 2 + 8 y2 '

where the first is the two-dimensional heat equation and the second the wave
equation. In either equation, if we assume the solution as v(x , y, t) = u(x, y) T(t),
then, after separating the variables, we obtain the Helmholtz equation

\7 2U(x , y) + Au(x , y) = 0, (13 .14)

where A denotes the eigenvalue. The related eigenvalue problem on a rectangle


{(x, y) : 0 ::; x ::; a, 0 ::; y ::; b} is defined if Eq (13.14) is subjected to the
boundary conditions u(O,y) = 0 = u(a ,y) = u(x,O) = u(x , b) . Then each
eigenfunction Umn corresponds to the eigenvalue A = Amn = (~7r) 2 + (~7r) 2,
whereu m n = ~ sin m7rX sin n1 l'Y
form,n = 1,2, ... (see Kytheetal. 2002,
vab a b
pp. 280-281). Similarly for both equations in the cylindrical polar coordinates
(r, e), the Helmholtz equation on a disk of radius a is defined by

V'2 u(r,B) + Au(r,B) = 0, 0 < r < 2, 0 ::; B < 27r, (13.15)

where A = w 2 / c2 . This equation, subject to the boundary conditions u(a, B) = 0,


o::; B < 27r, leads to the eigenvalue problem on the disk where each eigenfunction
.
umn corresponds to the eigenvalue", = "'mn = ~
,,2 (amn ) 2
, for m = 0,1 , . . . and
n = 1,2, . .. , where a mn is the n-th positive zero of the Bessel function J m (ar).
The corresponding eigenfunction umn(r, B) is given by cos mB Jm (Amnr) and
sin me Jm (Amnr). Note that for m = 1,2, . . . , we obtain two distinct eigenfunc-
tions for a given eigenvalue (see Kythe et al. 2002, pp. 292-293).

13.7. Exercises

13.1. Derive the wave equation (13.2) from Hamilton's principle.


348 13. VIBRATION ANALYSIS

13.2. Derivethe finiteelementsystem MU (t) +KU( t) = F from the discrete en-


1 . .
=L
NE
ergy function I(U), defined by I(U) I(e) (u(e)) = 2" UT(t)MU(t) +
e=l

~ U T (t)KU(t) - UTF, by using a discrete version of Hamilton's principle.


HINT. Calculate -d dII
T r=O
=0.
13.3. Use three linear elements to estimate three modes and frequencies for the
elastic rod discussed in Example 13.1.
13.4. Derive the local finite element system for estimation of the free vibration
modes of the elastic rod discussed in Example 13.1 by using the quadratic
element. Find the four modes and four frequencies by using two such quadratic
elements.
13.5. Calculate the free vibration modes of the elastic rod discussed in Example
13.1 by using one quadratic element and one linear element.
13.6. A Euler elastic beam of 10 meters long is claimed on one end. The material
properties are E = 65 X 109 N/m 2 , v = 0.33, and p = 2600 kg/m", Calculate
the modes and frequencies by using two Hermite elements.
13.7. Derive the local finite element system for free in-plane vibration of the
elastic plate by using the bilinear rectangular element. Find the finite element
modes and frequencies for the problem discussed in Example 13.3 by using
two such elements.
13.8 . Find the finite element modes and frequencies for the problem discussed in
Example 13.3by using two linear triangularelements and one bilinearquadratic
element.
13.9. Compute the eigenvalues and the eigenfunctions for the heat conduction
problem governed by Ut - V'2 u = f in the unit square n = {(x, y) : 0 :=:;
x, y :=:; I} , subject to the boundary conditions ~~ (0, y , t) = 0 = ~~ (x, 0, t) ,
u(x, 1, t) = 0 = u(l , y,t), and the initial condition u(x, y, 0) = 0, by taking
(a) a mesh of two triangularelementsand (b) a mesh of one rectangularelement.
ANS . (a) and (b) A = 6, (h(x,y) = (1 - x)(l- y).
13.10. Compute the naturalfrequenciesfor the vibrationproblem of a rectangular
membrane n of length 4 feet and width 2 feet with all edges fixed (i.e., u - 0
on an). In view of the symmetry about both the x and y axes, consider the
first quadrant by taking a mesh of (a) 2 x 2 rectangular elements and (b) 4 x 4
rectangular elements (Fig. 13.4), and using the data: all = a22 = 12.5 and
p = T = 2.5.
ANS . (a) Wll = 4.3303; (b) Wll = 4.0285, W21 = 5.0285, W31 = 7.2522,
W12 = 7.9527, W22 = 8.6603, W41 = 9.9805, W32 = 12.7157, W51 = 13.17,
13.7. EXERCISES 349

W42 = 14.0734.
. .
NOTE. The exact solutIOn . b
IS given y wm ,n = 1r V(TPV~
--;,2 + b2' which IC

gives: Wu = 3.5124, W21 = 4.4428, W31 = 5.6636, W12 = 6.4765, W22 =


7.0248, W41 = 7.0248, W32 = 7.8539, WSI = 8.459, W4 2 = 8.8857. Notice
that the finite element solution is not close to the exact values. Also, a complete
evaluation of all global natural frequencies can be obtained by considering the
entire rectangle with a more refined mesh of finite elements.
u =0
7 8 9

au
(a) (h=o
4 5

.>

~~\
3

16t--1>----+--+---+20
4ft (b) 11 15
6 to
2 3 4 5

Fig. 13.4.

13.11. The axial motion of an elastic bar is defined by

8 2u 8 2u
EA 8x 2 = P8t 2 ' 0 < x < l;

subject to the boundaryconditions u(O, t) = 0, EA ~~ (i, t) = 1, and the initial


condition u(x , 0) = O. Take l = 200 mm, A = area of the cross section = 1
mm", E = modulus of elasticity = 2.5 x 104 Nzmm", p = density = 0.005
N· p.s 2/ mm4 . Use a mesh of 4 linearelements and ~t = 0.0025p.s and compute
the axial displacement u.
13.12. For the matrix differential equation (9.14), use the central difference
scheme defined by

"} _ {u}n-t - 2 {u}n + {U}n+1 [u] _ {u}n+l- {U}n-I


{U n - (~t)2 , U n - 2~t '

and derive the algebraic equation for the solution of {u }n+l in the form

M{u}n+l = {f}n - K{u}n - C{U}n-l.


Compare with formulas (C.2) and (C.12) .
350 13. VIBRATION ANALYSIS

13.13. Solve numerically the last system of two ordinary differential equations
in Example 13.4 for U2 and u;
HINT . Use the notation : a = 1.156 x 104 , p = IU2 j- O.74 , and q =
IUa - U2 1- o.74 • Then the given system becomes

{Q
u,2} = _~7 [2 -1]
-1 4
[p+q
-q
-q]
q
{U2}
u, .
By taking Yl = U2, Y2 = o; Ya = Ua, and Y4 = ii; the above system reduces
to a system of four first-order equations

o
~~ } = [_ a(2~: 3q) ~ a
-
7
{ Ya 0 0 o
. a(-p-5q) 4aq
Y4 0
7 7

The following Matlab code solves this system numerically.

y1=29 .8613;
y2=0;
y3=30 .0616;
y4=0;
a=1.156*10 4 ; n=1000; dt=l/n;
OutU2=[] ; OutU3=[] ;
for i= 1: n*l
p=1/(abs(y1)r 0.74;
q=1/(abs(y3-y1))' 0.74 ;
tmpYl=yl+dt*y2;
tmpY2=y2-dt*a*( (2*p+3*q)*yl- 3*q*y30/7.0;
tmpY3=y3+dt*y4;
tmpY4=y4-dt*a*( (-p-5*q)*yl +5*q*y3)/7.0;
yl=tmpYl;
y2=tmpY2;
y3=tmpY3;
y4=tmpY4;
OutU2=[OutU2;i*dt,y1];
OutU3=[OutU3;i*dt,y3];
i=i+1;
end
14
Computer Codes

Different computer codes used in the book are presented. The cross-references to
the sections, examples, and exercises are provided.

14.1. Mathematica Codes

Mathematica is used to compute solutions of the matrix equation KU = F and in


some cases to draw the graphs. In the cases of solving the above matrix equation,
the Mathematica command is LinearSolve [m, b] which gives the vector x as the
solution of the matrix equation m x = b . The commands Plot, ListPlot, and
ImplicitPlot are generally used to generate graphs of functions or data. In this
section we provide Mathematica codes for some representative examples solved
in previous chapters. Most of the examples and exercises can be similarly solved.

EXAMPLE 3.3.
In~]: m={ {892200,-356800,44600},{-356800,712600,-356800},
{44600,-356800,312200} };
In[2]: b = 8333.3 {1, 4, 1};
In[3]: LinearSolve[m, b]
Out[3]: 0.0865988, 0 .227477, 0.274294
In[4]: Ql = -5.8*10 5 * 0.0865988
Out[4] : -50227.3

P. K. Kythe et al., An Introduction to Linear and Nonlinear Finite Element Analysis


© Springer Science+Business Media New York 2004
352 14. COMPUTER CODES

EXERCISE 3.2. FOURNoDECUBIC

(* 4-node cubic element


Determine the stiffness matrix and the force vector.
Write x_e and x_{e + 1} as a and b, respectively.
Note b - a = L- (e), where we write lower case el as L.
Then we have xbar = (L/2)(1 + xi),
or xi = 2 xbar/L - 1. We will write xbar as x *)

Inrl} :
phi1[xJ ;= -(9/16)* (1 - (2 x/L - 1))*(1/9 - (2 x/L - 1)-2);
phi2 [xJ (27/16) *(1 - (2 x/L - 1) - 2) *(1/3 - (2 x/L - 1));
phi3[xJ ;= ( 27/16)*(1 - (2 x/L - 1)-2)*(1/3 + (2 x/L - 1));
phi4[xJ ;= -(9/16)* (1 + (2 x/L - 1))*(1/9 - (2 x/L - 1)-2);
d1 = D[phi1[x] , x];
d2 = D[phi2[x] , x] ;
d3 = D[phi3 [x] , x];
d4 = D[phi4[x] , x];
kl1 = Integrate[a*(d1)-2, x, 0, L]
+ Integrate[c*phi1[x]-2, x, 0, L]
k12 = Integrate[a*(d1) (d2) , x, 0, L]
+ Integrate [c*phi1 [x] *phi2 [x] , x, 0, L]
k13 = Integrate[a*(d1) (d3) , x, 0, L]
+ Integrate [c*phi1 [x]*phi3[x] , x, 0, L]
k14 = Integrate [a*(d1)(d4), x, 0, L]
+ Integrate [c*phi1 [x]*phi4[x] , x, 0, L]
k22 = Integrate[a*(d2)-2, x, 0, L]
+ Integrate[c*phi2[x]-2, x, 0, L]
k23 = Integrate [a*(d2)(d3), x, 0, L]
+ Integrate[c*phi2[x]*phi3[x] , x, 0, L]
k24 = Integrate [a*(d2)(d4), x, 0, L]
+ Integrate [c*phi2[x] *phi4[x] , x, 0, L]
k33 = Integrate[a*(d3)-2, x, 0, L]
+ Integrate[c*phi3[x]-2, x, 0, L]
k34 = Integrate [a*(d3)(d4), x, 0, L]
+ Integrate[c*phi3[x]*phi4[x] , x, 0, L]
k44 = Integrate[a*(d4)-2, x , 0, L]
+ Integrate[c*phi4[x]-2, x, 0, L]
f1 = Integrate [f*phi1 [x] , x, 0, LJ
f2 = Integrate [f*phi2[xJ , x, 0, LJ
f3 = Integrate [f*phi3 [x] , x, 0, LJ
f4 = Integrate [f*phi4[xJ , x, 0, LJ
14.1. MATHEMATICA CODES 353

Out{l}:
The Mathematica output is suppressed; but it yields the answer given on page 66.

E X A MPL E 4 .3.

(* 3-Element Mesh, keeping EI with U_i *)

In~}: k1 = (2/512)*{ {6, -24, -6, -24}, {- 24 , 128, 24, 64} ,


{- 6, 24 , 6, 24} , {-24, 64 , 24, 128} };
k2 = (2/64 ) *{ {6, -12, -6, - 12} , {-1 2, 32 , 12, 16} ,
{- 6 , 12, 6, 12}, {- 12, 16, 12, 32} };
k3 = k2 ;
K=
{ {k1[[1, in. k1[[1, 2JJ, k1[[1, 3JJ , k1[[1, 4JJ, 0, 0, O,O},
{k1[ [2, 1]], k1[ [2, 2JJ , k1[ [2 , 3JJ , k1[ [2, 4J J , 0 , 0, 0, O},
{k1[ [3 , 1JJ, k1[[3, 2JJ, k1[[3, 3JJ + k2[[1, ui. kl[[3, 4JJ +
k2[[1 , 2JJ ,k2[[1, 3JJ, k2 [ [1, 4JJ , 0, O},
{kl[ [4 , 1JJ, kl[[4 , 2JJ, kl[[4, 3JJ + k2[[2, 1JJ, kl[[4 , 4JJ +
k2 [ [2, 2JJ, k2 [ [2, 3JJ, k2 [ [2, 4JJ, 0, O},
{O, 0, k2[[3, 1JJ, k2[[3, 2JJ, k2[[3, 3JJ + k3[[1, 1JJ, k2[[3,
4JJ + k3[[1 , 2JJ , k3[[1 , 3JJ, k3[[1 , 4JJ} ,
{O, 0 , k2[[4, 1JJ, k2[[4, 2JJ , k2[[4, 3JJ + k3[ [2, 1J J ,
k2[ [4, 4JJ + k3 [[2, 2JJ , k3 [ [2 , 3JJ , k3[[2, 4JJ },
{o, 0, 0, 0 , k3 [ [3 , 1J ] , k3 [ [3 , 2JJ , k3 [[3, 3J] , k3 [ [3 , 4JJ },
{O, 0, 0, 0, k3 [[4, 1JJ, k3[[4, 2JJ, k3[ [4 , 3JJ, k3[[4, 4JJ} }
Out{l} : K=
3333 3 1 3 1
{ { 128 '-32' - 128 ' -32 ' O,O,O,O}, {-32 '2 ' 32 '4 'O' O,O,O},
3 3 27 9 3 3 3 1 9 3 3 1
{-128 ' 32 ' 128 ' -32 '-16 ' -8 'O,O}, {-32 ' 4 '-32 '2 '8 '2 '0,0} ,
33333 313 1
{O, O, - 16 ' 8 ' 8 ,0 , -16 ' - 8 }, {O,O, - 8 ' 2 ,0 , 2, 8 ' 2} '
3333 3 13
{O ,O,O,O, - 16 ' 8 ' 16 ' 8} ' {0,0 ,0 ,0 , -8' 2' 8 ' I }}
In{2] : f = -(600*8/12)* {6, -8 , 6, 8}
Out[2] : {- 2400, 3200, - 2400, -3200 }
(* We solve for EI Ui, i = 2, 3, 4, 5, 6, 8*)
In{3}: m=
{ {1/2, 3/32, 1/4, 0, 0, O}, {3/ 32 , 27/128, -9/32 , -3/16, - 3/ 8 ,
O}, {1/4, -9/32 , 3/2, 3/8, 1/2, O} ,
{O, -3/16, 3/8, 3/8 , 0 , - 3/ 8} , {o , - 3/8 , 1/2 , 0, 2 , 1/2},
{O, 0 , 0 , -3/8, 1/2 , 1} };
354 14. COMPUTER CODES

b = {3200, -2400, -3200, -2500, 0, 9000}/185000000j


(* b = F/EI, with EI = 18.5 x107 *)
LinearSolve[m, b] // N
Out[3] :
{0.000316757, -0 .0013982, -0.00004, -0.000867027, -0.000209189,
-0 .000171892}
In[4]:Qll = (-3/32)*(0.000316757) + (-3/128)*(-0.0013982) + (-
3/32)*(-0 .00004) + 2400/185000000
Out[4]: 0.0000197973
(* Now we solve for the 2 - element mesh.
The matrix K is EI*kl, and fl is the same as f above.
Compute f2 for the element 2: *)
In[5] :
phil [x.] : = 1 - 3*x~ 2/64 + 2*x~ 3/512
phi2 [xJ : = -x« (1 - x/B) ~ 2
phi3[x_] := 3*x-2/64 - 2*x-3/512
phi4[x_] : = -x*(x~2/64 - x/8)
£12 = phi1(4] j
f22 = phi2[4] ;
f23 = phi3[4] ;
f24 = phi4[4] j
f2 = -2500*{f12, f22, f23, f24}
Out[5] : {-1250, 2500, -1250, -2500}
In[6] : fvector = {-2400, 3200, -2400, -3200, 0, O}
+ {O, 0, -1250, 2500, -1250, -2500 + 9000}
Out[6]: {-2400, 3200, -3650, -700, -1250, 6500}
In[n : m = (2/512)*{ {128, 24, 64, O}, {24, 12, 0, -24} ,
{64, 0, 256, 64}, {O, -24, 64, 128} };
b = {3200, -3650, -700, 6500}/185000000;
LinearSolve[m, b] // N
Out[n: {0 .000316757, -0.0013982, -0.00004, -0 .000171892}
In[S]: Q11two = (2/512)*( -24*(0 .000316757) + (-6)*(-0.0013982)
+ (-24)*(-0.00004)) + 2400/185000000
Out[S] : 0.0000197973
In[9] :Q32two = (2/512)*( (-6)*(-0 .0013982) + (24)*(-0.00004) +
24*(-0.000171892)) + 1250/185000000
14.1. MATHEMATICA CODES 355

Out[9} : 0.0000196622

EXERCISE 6.20.

(* use (6.15), with a = 5, b = 6, and K-e = Wl1 + W22;


also take fo = 2. Since all 4 elements are similar,
we compute K ' 1, denoted by matk1, and f1 *)
In~}: matk1 = (1/5)*{ {2, -2, -1, 1}, {-2, 2, 1, -1},
{-1, 1, 2, -2}, {1, -1, -2, 2}} +
(5/16)*{ {2, 1, -1, -2}, {1, 2, -2, -1},
{-1, -2, 2, 1}, {-2, -1, 1, 2}}
Outp} : {{41/40, -7/80, -41/80, -17/40},
{-7/80 , 41/40, -17/40, -41/80},
{-41/80, -17/40, 41/40, -7/80},
{-17/40, -41/80, - 7/ 80 , 41/40}}
In[2}: f1 = (2*5*6)/4*{1 , 1, 1, 1}
Out[2} : {15, 15, 15, 15}
(* Denote the assembled stiffness matrix related to
U 5 and U 6 by m, and the force vector by b *)
In[3}: m = {{matk1[[3, 3]] + matk1[[4, 4]] + matk1[[2, 2]] +
matk1[[1, 1]], matk1[[4, 3]] + matk1[[1, 2]], matk1[[3, 4]]
matkl + matkt ] [2, 1]], [[3, 3]] + matk1[ [2, 2] J} }
Out[3}: {{41/10, -7/40}, {-7/40, 41/20} }
In[4} : b = {f1[[3]] + f1[[4]] + f1[[2]] + f1[[1]] -
( matk1[[2, 4]] + 46*(matk1[[2, 3]] + matk1[[1 , 4]]) +
191*matkl[[1, 3JJ), f1[[3JJ + f1[[2JJ -
(46*matk1[[2, 4]] +191*matk1[[2, 3]])}
Out[4}: {395/2, 539/4}
In[5} : LinearSolve[m, b] // N
Out[5} : {51. 1628, 70 .0993}

EXAMPLE 7.1.
(* Heat transfer without convection; Rectangle size 4a x 2a;
16 linear triangular elements *)
Inp}: k1={ {2,-1 ,0,0,-1,0,0,0},{-1,4,-1,0,0,-2,0,0},
{0,-1,4, -1,0,0,-2,0},{0 ,0,-1,4,0,0,0,-2}
{-1,0,0,0,4,-2,0,0},{0,-2,0,0,-2,8,-2,0}
{0,0,-2,0,0,-2,8,-2},{0,0,0,-2,0,0,-2,8}};
356 14. COMPUTER CODES

£1= {O, 0, 0, 0, T, 2*T*Cos [71'/S], 2*T*Cos[7!' /4]' 2*T*Cos[31l'/S]} ;


LinearSolve[k1,f1JIIN
OutV} : {0.759838 T, 0.701999 T, 0.537286 T, 0 .290777 T,
0.817677 T, 0.755435 T, 0.578185 T, 0.312911 T}
(* Heat At node 5 is given by Fs = Qs = -(k/2)T4 *)
In[2} : F5 = -(k/2)*(0.290777 T)
Out[2} : -0. 145389 k T
(* Same problem with 8 rectangular elements *)

In[3} : k2 = { {4, -1, 0, 0, -1, -2, 0, O},


{-1, 8, -1, 0, - 2 , -2, -2, O},
{O, - 1 , 8, -1, 0, -2, - 2 , -2},
{O, 0, -1 , 8, 0, 0, -2, -2},
{-1, -2, 0, 0, 8, -2, 0, O},
{-2, -2, -2, 0, -2, 16, -2 , O},
{O , -2, -2, -2, 0, -2, 16 , -2},
{O, 0, -2 , -2 , 0, 0 , -2, 16} } ;
f2 = {O, 0, 0, 0, T + 2 T* Cos[7!'/S]'2T+2T*Cos[7!'/ 8]
+ 2T * COS [71'/ 4]' 2T * COS[71' /81 + 2T * COS[71'/ 4] + 2T * Cos [371' /8],
2T * Cos[[Pi] /4 ] + 2T * Cos[3 [PillS] };
LinearSolve[k2, f2J II N
Ou t[3J: {0 .749839 T, 0 .692761 T, 0 .530216 T, 0 .286951 T,
0.809968 T, 0 .748313 T, 0 .572734 T, 0 .309961 T}
(* Heat at node5 i s given by Qs*)
In[4]: Q5 = -(kT/6) * (0.2S6951 + 2 * 0.309961)
Out[4}: -0 .151146 k T
(* Exact Solution *)
T[x,Yj := T * (Cosh[(7!'y )/(8a)] Cos[(7!'x)/(Sa)])/Cosh[7!' /4]/ /
T able[T[x , y], {x , 0, 4a,a}, {y, 0, 2a, a} ]
Out[4}:
{ {0.75494 T, 0 .813902 T, T},
{0.697473 T, 0.751948 T, 0.92388 T},
{0 .533823 T, 0 .575516 T, 0 .707107 T},
{0.288903 T, 0.311467 T, 0.382683 T},
{O.,O.,O.} }

EXAMPLE 7.2 .
(* Heat transfer with convection; Too = 0 *)
14.1. MATHEMATICA CODES 357

In!l]: m = { {40.8, -9 .8, 0, 0, -4.9, -10, 0, O},


{-4.9, 81 .6, -4.9, 0, -10, -9.8, -10, O},
{O, -9.8, 81.6, -9.8, 0, -10, -9 .8, -10},
{O, 0, -9 .8, 81.6, 0, 0, -10, -9.8},
{-4.9, -10, 0, 0, 20.4, -4.9, 0 , O},
{-10, -9.8, -10, 0, -4.9, 40.8, -4.9, O},
0, -10, -9.8, -10, 0, -4 .9, 40 .8, -4 .9},
{O, 0, -10, -9.8, 0, 0, -4.9, 40.8}
b = {1600, 1000, 1000, 3020, 1260, 600, 600, 1990};
LinearSolve[m, b] II N
Out~]: {87 .138, 66 .6033, 67 .096, 63 .6882, 130.62,
86 .0481, 76.2619, 87.2249}
(* Now with Too = 20 each element of the right side vector c is *)
In[2]: c1 = (107 * 0.012)/2 + (0.01 * 2 * 105)/2 + 2 * 60 * 0.01 * 20;
c2 = 107 * 0.012 + 4 * 60 * 20 * 0.01;
c3 = 107 * 0.012 + 4 * 60 * 20 * 0.01;
c4 = 107 * 0.012 + 4 * 60 * 20 * 0.01 + 20.4 * 100;
c5 = (107 * 0.012)/4 + (0.01 * 2 * 105 )/ 2 + 60 * 20 * 0.01;
c6 = (107 * 0.012)/2 + 2 * 60 * 20 * 0.01;
c7 = (107 * 0.012)/2 + 2 * 60 * 20 * 0.01;
c8 = (107 * 0.012) /2 + 2 * 60 * 20 * 0.01 + (15 - 10 * 0.01) * 100;
c = c1 ,c2,c3,c4 ,c5,c6,c7,c8
Out[2]: {1624., 1048., 1048.,3088.,1262.,624.,624., 2014.}
In[3] : LinearSolve[m, c] II N
Out[3}: {89 .0276, 68.2868, 58 .8475, 65.2117, 133.077, 88.5034 ,
78.5511, 88 .8836}

EXAMPLE 7 .3 .
(* Torsion of a unit square bar *)
In[l}: m = 0.5*1, -1, 0, -1, 4,2,0, -2,4;
b = 29 .08882087*1, 3, 3 ;
LinearSolve[m, b]
Out!l]: 94.5387, 36.361, 61. 8137
In[2}: g = 8*106 ; () = 0.01 * 1f /180; A = 1/32 ;
2 * g * () * A/3
Out[2} : 29 .0888
(* Computation of shear stresses *)
(* For elements 1, 2, 3 : * )
358 14. COMPUTER CODES

In[3}: gradientl = 16*{{-1/4, 1/4, O}, {O, -1/4, 1/4}}.{94 .5387,


36.361, 61.8137}
Out[3}: {-232 .711, 101.811}
In[4}: gradient2 = 16*{{1/4, 0, -1/4}, {-1/4, 1/4, 0}}.{94.5387,
36 .361, 61 .8137}
Out[4} : {130.9, -232.711}
In[5} : gradient3 = 16*{{0, -1/4, 1/4}, {1/4, 0, -1/4}}.{94 .5387,
36.361, 61.8137}
Out[5} : 101. 811, 130.9
(*gradient4 = gradientl *)

EXERCISE 7.5 .

Inp} : m = 5*{ {4, -1, -1, -2, 0, O}, {-1, 8, -2, -2, 0, O},
{-1, -2, 8, -2, -1, -2}, {-2, -2, -2, 16, -2, -2},
{O, 0, -1, -2, 4, -1}, {O, 0, -2, -2, -1, 8} +
(2/15) *{ {4 , 1, 1, 0, 0, O}, 1, 8, 0, 2, 0, °},
{1, 0, 8, 1, 1, O}, {O, 2, 1, 16, 0, 2},
{O, 0, 1, 0, 4, 1, 0,0, 0, 2, 1, 8}};
b = 1000*{1,2, 2, 4, 1, 1} +16*{1, 2, 2, 4, 1, 1} +
(1/200)*{300000, 0, 300000, 0, 300000, O} +
{O, 5*400, 0, 5*500, 0, 5*300};
LinearSolve[m, b] II N
Out[l}: {450.357, 326.768, 416.402, 311.457, 441.355, 288.786}

EXERCISE 7.8 .

(* Element 4 *)
Inp}: xl = 1; yl = 0.885618083; x2 = 0; y2 = 1; x3 = 0;
y3 = 0 ;
0:[1] = x2 * y3 - x3 * y2;
0:[2] = x3 * y1 - xl * y3;
0:[3] = xl * y2 - x2 * y1;
List [0:[1]' 0:[2],0:[3]]
Out[l}: 0, 0, 1
In[2} : A = 0:[1] + 0:[2] + 0:[3]
Out[2} : 1
14.1. MATHEMATICA CODES 359

In[3}: ,6[1] = y2 - y3;


(j[2] = y3 - yl;
(j[3] = yl - y2;
'1'[1] = x3 - x2;
'1'[2] = xl - x3;
'1'[3] = x2 - xl;
List[(j[l], (j[2], ,6[3]]
List['Y[l] 1'1'[2], '1'[3]]
Out[3): {1, -0.885618, -0.114382};
;{O, 1, -1}
In[4}: kl1 = «(j[1]2 + 'I'[1]2)/(4A);
k12 = ((j[l] * (j[2] + '1'[1] * 'I'[2])/(4A);
k13 = ((j[l] * (j[3] + '1'[1] * 'I'[3])/(4A);
k22 = (,6[2]2 + 'I'[2]2)/(4A);
k23 = ((j[2] * 1][3] + '1'[2] * 'I'[3])/(4A);
k33 = (,6[3]2 + 'I'[3]2)/(4A);
k21 = k12;
k31 = k13;
k32 = k23;
matk4 = {{kll, k12, k13}, {k21, k22, k23}, {k31, k32, k33} } / / N
Out[4}: {{0.25, -0.221405, -0 .0285955},
{-0.221405, 0.44608, -0.224675},
{-0 .0285955, -0.224675, 0.253271}}
In[5} : f4 = (2/3)*{1, 1, 1}

Outf5}: {2/3, 2/3, 2/3}


(* Remaining Elements 1, 2, 3, 5, 6, 7. 8
Notation: m denotes the stiffness matrix K for the element
e;fe denotes the force vector for the element e *)
In[6} : matk1 = 0 .5*{{2, -1, -1}, {-1, 1, O}, {-1, 0, 1}};
f1 = 0.53269207*{1, 1, 1};
matk2 = matk1 j
f2 = f1 j
matk3 = 0.564577451*{{1.784319388, -0.784319388, -1},
{-0.784319388, 0 .784319388, o}, {-1, 0, 1}};
f3 = 0 .295206027*{1, 1, 1};
matk5 = 0.312876192*{{3.553847573, -1, -2 .553847573},
{-1, 1, o}, {-2.553847573, 0, 2. 553847573} }j
f5 = 0.53269207*{1, 1, 1};
matk6 = matk5;
f6 = f5;
360 14. COMPUTER CODES

matk7 = 0.353285686*{{3.338166961, -0.784319388, -2. 553847573},


{-o. 784319388 , 0.784319388, O}, {-2. 553847573 , 0, 2 . 553847573} };
f7 = 0.471761729*{1, 1, 1};
matk8 = 1.244016935*{{1 .161542732, -1, -0. 161542732} ,
{-1, 1, O}, {-0.161542732, 0, 0. 161542732} };
f8 = 0.133974596*{1, 1, 1};
(* Assembly: The assembled stiffness matrix is denoted
by m; the assembled force vector is denoted by b :*)
m = {{matk1[[1, 1]] + matk2[[3, 3]], matkl[[l, 2]], 0,
matk2[[3, 2]], matkl[[l, 3]] + matk2[[3, 1]], matkl[[l, 2]],
matkl[[2, 2]] + matk5[[1, 1]] + matk6[[3, 3]], matk5[[1, 2]], 0,

matkl[[2, 3]] + matk6[[3, 2]], 0, matk5[[1, 2]],


matk5[[2, 2]] + matk8[[3, 2]], 0, 0, matk2[[3, 2]], 0, 0,
matk2[[2, 2]] + matk3[[1, 1]] + matk4[[3, 3]],
matk2[[2, 1]] + matk3[[1, 2]], matk1[[l, 3]] + matk2[[3, 1]],
matk1[[2, 3]] + matk6[[3, 2]], 0, matk2[[2, 1]] + matk3[[1, 2]],

matkl[[3, 3]] + matk2[[1, 1]] + matk3[[2, 2]] + matk7[[2, 2]]


Out[6}: {{I.5, -0.5, 0, 0, -1.}, {-0.5, 2.41095, -0.312876, 0, O},
{O, -0.312876, 0.312876, 0, O}, {O, 0, 0,1.76066, -0.942809},
{-I., 0, 0, -0.942809, 2.2199} }
In[7}: b = {fI[[I]] + £'2[[3]] , fI[[2]] + £5[[1]] + £6[[3]],
£5[[2]] + £8[[2]], £2[[2]] + £3[[1]] + £4[[3]],
fI[[3]] + £'2[[1]] + £3[[2]] + £6[[2]] + £7[[2]]}
Out[7} : {1.06538, 1.59808, 0.666667, 1.49456, 2.36504 }
In[8} : LinearSolve[m, b]
Out[8] : 4.32553,2.11027,4.24104,3.18775,4.36778
(* Exact Solution *)
In[9}: ulx., y.] := (72/13)*(1 - x2 /9- y2 14}1IN
List[u[O, 0]' u[l, 0]' ufO, 1]]
Out[9] : {5.53846, 4.92308, 4.15385}

EXERCISE 8 .7.

In[l]: k = { {0.045, -0 .015, 0, 0, -0 .03, O},


{-0.015, 0.12, -0 .015, -0.03, -0 .03, -0 .03},
{O, -0 .015, 0.12, 0, -0.03, -0.03},
{O, -0 ,03, 0, 0 .045, -0 .015, O},
14.1. MATHE MATICA CODES 361

{-0.03, -0.03, -0.03, -0.015, 0.12, -0.015},


{O, -0 .03, -0.03, 0, -0 .015, 0 .12}} ;
f = {O .18*11", 0.54 * 11" ,0 .54 * 1r + 100(0.015 + 0.03),0.18 * 11" ,
0.54 * n , 0.54 * 11" + 100(0.03 + 0.015) };
LinearSolve[k, f]
Out~}: {250.796, 238.23, 187.965, 250.796, 238 .23, 187.965}

EXAMPLE 10.1.
In[l}: FindRoot[x-3 + 9x - 2 == 0, {x , O}]
Out[l}: {x - ) 0.221023}
In[2} : FindRoot[x-3 + 9x - 2 == 0, {x, 0 .5}]
Out[2} : {x -) 0 .221023}
In[3}: FindRoot[x-3 + 9x - 2 == 0, {x, 1}]
Out[3} : {x -) O. 221023}
In[4} : FindRoot[x-3 + 9x - 2 == 0, {x, 2}]
Out[4} : {x -) 0.221023}

EXAMPLE 10.2.
In[l}: FindRoot[{x-2 - y-2 + 6 x - 4 == 0, x-2 + Y - 1 == O},
{x, 0.2}, {y, O.1}]
Out[l]: {x - ) 0 .65125, Y -) 0 .575873}
(* For Example 5 .1 , we compute Xl by t aking xo = 0.5 *)
In[2} : f[x-J : = x" 3 + 9 x - 2;
f[0 .5] ;
In[2]: f[x-J : = x" 3 + 9 x - 2;
D [f[x] , x]

Out[2} : 2.625
In[2} : f'{x.] : = x" 3 + 9 x - 2;
9 + 3x- 2

In[3} : fprime[0.5] = 9 + 3 (0.5)2 ;


In[2) : f[x -J : = x" 3 + 9 x - 2;
xi = 0.5 - f[0 .5]/fprime[0 .5]
Out[3} : 0 .230769
In[4] : relativeDiff = (xi - 0 .221023)/0 .22 1023
362 14. COMPUTER CODES

Out[4} : 0 . 044095

EXAMPLE 10.4.

(* First, simplify the two equations *)


In[i} :
eqnl = Simplify[2*5 .0265*10 - (-5)*x - 5.0265*10 - (-5)*y +
2 .1384*10- (-14)*«5.1958*10- (7)/15)*x + (1.3924*10(5)/10)* x"2
+(2*373 .15/15)*x-3 + x-4/3 + 2 x-3 y/15 + x-2 y-2/10 + x
Y- 3/15 + Y- 4/30)
== 1.8756*10 - (-2) - (4.1459*10- (-2))/30]

Out[i) : . 128*10- 15 x 4 + x 3(1.0 6393*10- 12 + 2.8512*10- 15y-


0.000050265 y + 7.128 * 10- 16 y4+ x 2 (2.97750816 * 10 -10 +
2 .1384* 10- 15 y2+X (0.000100604) + 1.4256 * 10-15 y3== 0.017374
In[2) : eqn2 = Simplify [-5 .0265*10 - (-5) *x + 2 *5.0265*10 - (-5) *y
+ 2.1384*10 - (-14) *(x-4/30 + x-3 y/15 + x-2 y-2/10 +
2*x Y- 3/15 + Y- 4/6) == 1. 0697*10 - (-13) * (y - 4 - 5 .5668*10 - 9)]
Out[2} : 0 .000595481 - 0 .000050265x + 7.128*10- 16x4 +
0.00010053y + 1.4256*10- 15x3y + 2.1384*10-15Ix2y2
+ 2 .8512*10- 15 x y 3 + 3.564*10- 15]y 4 == 1.0697*10-13 y4
In[3} : FindRoot[{eqnl , eqn2}, x, Random[]}, {y, Random[]}]
Out[3} : {x -> 225 .637, Y -> 106 .974}
In[4} : FindRoot[{eqnl , eqn2} , {x, 0 .5, 50}}, {y , {1, 100}}]
Out[4} : {x -> 225.633, Y - > 106.973}
14.2. ANSYS CODES 363

14.2. Ansys Code

14.2.1. EXAMPLE 9 .5. For the 16-element mesh, shown in Fig. 9.3, the
Ansys code is as follows:
PLANE75 AXISYMMETRIC-HARMONIC THERMAL SOLID
/filname fin
/units,si
/prep7
antyp, trans
et,1,75
mp,kxx,1,237
mp,dens,1,2702
mp,c,1,903
mp,hf,1,150
mp,kyy,1,237
rectng,0,0.002 ,0,0.04
lesize,1",2
lesize,2" ,8
eshape,2
amesh, l
eplot
/solu
outres, , all
outpr, , last
sfa ,1,2,conv,150,25
sfa,1,3,conv,0,25
tunif ,25
d,1 ,temp ,85
d ,2 ,temp,85
d,3,temp,85
autots,on
time,O.O!
t ime,8&! 8 s
nsubst,80&! process 80 steps
solve
finish
/post26
nsol,2,1,temp"tl
nsol,3,17 ,temp "t2
nsol,4,12,temp"t3
/axlab,y,tempet
plvar,2,3 ,4
finish
364 14. COMPUTER CODES

Output Listings 14 .2.1 and 14 .2.2

LOAD STEP= 0 SUBSTEP= 0 LOAD STEP=O SUBSTEP= 0


TIME= 1 . 0000 LOAD CASE= 0 TIME= 2.0000 LOAD CASE= 0

Node TEMP
1 85 .000
2 85 .000
Node TEMP
3 85 .000
1 85.000
4 26 .196 2 85.000
5 66.610 3 85 .000
6 51.448 4 30 .9 19
7 40 .664 5 71 .836
8 33.786 6 59.905
9 29 .751 7 50 .013
10 27 .559 8 42 .430
11 26 .505 9 37.032
26 .196 10 33 .510
12
11 31. 547
13 26.196
12 30 .919
14 26 .505 13 30 .919
15 27 .559 14 31. 547
16 29 .751 15 33 .510
17 33 .786 16 37 .032
18 40 .664 17 42 .430
19 51.448 18 50 .013
20 66.610 19 59.905
21 66 .610 20 71.836
21 71.836
22 51.448
22 59 .905
23 40.664 23 50.013
24 33 .786 24 42 .430
25 29 .751 25 37 .032
26 27 .559 26 33 .510
27 26 .505 27 31. 547

MAXIMUM ABSOLUTE VALUES MAXIMUM ABSOLUTE VALUES


NODE 1 NODE 1
VALUE 85.000 VALUE 85.000
14.2. ANSYS CODES 365

Output Listings 14.2.3 and 14.2.4

LOAD STEP= 0 SUBSTEP= 0 LOAD STEP= 0 SUBSTEP= 0


TIME= 3.0000 LOAD CASE= 0 TIME= 4. 0000 LOAD CASE= 0
Node TEMP
1 85 .000
2 85.000
Node TEMP
3 85.000
1 85 .000
4 30.919
2 85 .000
5 71.836 3 85 .000
6 59 .905 4 42.332
7 50 .013 5 76 .208
8 42 .430 6 67 .864
9 37.032 7 60 .357
10 33.510 8 53 .977
11 31.547 9 48 .912
12 30.919 10 45 .261
11 43.064
13 30 .919
12 42 .332
14 31. 547 13 42.332
15 33.510 14 43.064
16 37.032 15 45 .261
17 42.430 16 48 .912
18 50.013 17 53 .977
19 59.905 18 60.357
20 71 .836 19 67 .864
21 71. 836 20 76 .208
21 76.208
22 59 .905
22 67.864
23 50.013
23 60 .357
24 42.430 24 53 .97 7
25 37.032 25 48.912
26 33.510 26 45 .261
27 31.547 27 43 .064

MAXIMUM ABSOLUTE VALUES MAXIMUM ABSOLUTE VALUES


NODE 1 NODE 1
VALUE 85.000 VALUE 85.000
366 14. COMPUTER CODES

Output Listings 14.2.5 and 14.2.6

LOAD STEP= 0 SUBSTEP= 0 LOAD STEP= 0 SUBSTEP= 0


TIME= 5.0000 LOAD CASE= 0 TIME= 6.0000 LOAD CASE= 0

Node TEMP
1 85 .000
2 85 .000
Node TEMP
3 85 .000
1 85.000
4 47 .613 2 85 .000
5 77 .505 3 85.000
6 70 .345 4 52 .356
7 63 .822 5 78.545
8 58 .192 6 72.357
9 53.646 7 66.688
10 50.319 8 61.755
11 48.293 9 57.740
10 54.779
12 47.613
11 52.966
13 47 .613
12 52 .356
14 48 .293 13 52.356
15 50 .319 14 52 .966
16 53 .646 15 54 .779
17 58.192 16 57 .740
18 63.822 17 61 .755
19 70 .345 18 66 .688
20 77 .505 19 72 .357
21 77 .505 20 78 .545
21 78 .545
22 70 .345
22 72 .357
23 63 .822 23 66 .688
24 58 .192 24 61.755
25 53.646 25 57 .740
26 50 .319 26 54 .779
27 48 .293 27 52 .966

MAXIMUM ABSOLUTE VALUES MAXIMUM ABSOLUTE VALUES


NODE 1 NODE 1
VALUE 85.000 VALUE 85.000
14.2. ANSYS CODES 367

Output Listings 14.2.7 and 14.2.8

LOAD STEP= 0 SUBSTEP= 0 I LOAD STEP= 0 SUBSTEP= 0


TIME= 7.0000 LOAD CASE= 0 I TIME= 8 . 0000 LOAD CASE= 0
Node TEMP I
1 85.000 I
2 85 .000 I Node TEMP
3 85 .000 I 1 85 .000
4 56 .602 I 2 85 .000
5 79 .424 I 3 85 .000
6 74 .071 I 4 56 .602
7 69 .150 I 5 79 .424
8 64 .853 I 6 74 .0 71
9 61 .339 I 7 69.150
10 58 . 738 I 8 64. 853
11 57 .140 I 9
10
61. 339
58 . 738
12 56 .602 I 11 57 .140
13 56 .60 2 I 12 56 .602
14 57 .140 I 13 56 .602
15 58 .738 I 14 57 .140
16 61.339 I 15 58 .738
17 64 .853 I 16 61.339
18 69 .150 I 17 64 .853
19 74. 071 I 18 69. 150
20 79.424 I 19
20
74.071
79 .4 24
21 79 .424 I 21 79. 424
22 74.071 I 22 74 . 071
23 69 . 150 I 23 69 . 150
24 64 .853 I 24 64.853
25 61.339 I 25 61 .339
26 58 .738 I 26 58.738
27 57 . 140 I 27 57 .140

MAXIMUM ABSOLUTE VALUES MAXIMUM ABSOLUTE VALUES


NODE 1 NODE 1
VALUE 85.000 VALUE 85.000
368 14. COMPUTER CODES

14.3. Matlab Codes

14 .3.1. EXAMPLE 12.4. The listing of the Matlab code for the unit cavity
problem follows.

Main function: Cavity_flow(N,r,nu, epsilon)


%N: number division on each side of the unit square
%n: power-law index
%nu: viscosity of the fluid
%epsilon : penalty parameter
%r=n+1=1 .5, 2.0, 3.0
function Cavity_flow(N,r,epsilon)
% initialization
k=1;
fx=fy=O .O;
U_V=zeros((N+1)-2,2)j
for k=2 :N
U_V((N+1)*N+k,1)=1 ;
U_V((N+1)*(N-1)+k,1)=1/2;
end
U_V
conv_tolerance=10- (-2);
p_Lt=zeros (2* (N+1) - 2,1) j
p_i=zeros(2*(N+1)-2,1);
beta_i=O;
gradient_i_1=zeros(2*(N+1)-2,1);
gradient_i=zeros(2*(N+1)-2,1);
for i=O: 10- 4
i f i=10- 4
error( 'run out of loop and convergence tolerance is not satis-
fied');
end
p_i=p _L1 ;
gradient_i_1=gradient_i;
gradient_i=get_dlduv(U_V,k,g,r,epsilon,N);
error=gradient_i'*gradient_i;
if error < cony_tolerance
error
U_V
error('success');
break;
end
14.3. MATLAB CODES 369

error
if i>O
y_i_l=gradient _i-gradient_i_l;
betal_i=(gradient_i'*y_i_l)/(p_i_l'*y_i_l);
beta2_i=(gradient_i'*gradient_i)/(p_i_l'*y_i_l);
beta_i=max(O,min(betal_i,beta2_i));
end
p_i=-gradient_i+beta_i*p_i_l;
%---- the line-search scheme
for k=O:50
i f k==50
break;
end
predict_U_V=U_V;
for pp=l: (N+l) ~ 2
predict _U_V(pp,l)=predict_U_V(pp,l)+2~ (-k) * p_i(2*pp-l);
predict_U_V(pp,2)=predict_U_V(pp,2)+2~ ( -k)* p_i(2*pp) ;
end
predict-f=get_Ie(predict_U_V,k ,g,r,epsilon,N);
current-f=get_Ie(U_V,k,g,r,epsilon,N);
if predict-f < current.f
break ;
end
end
for pp=l : (N+1) ~ 2
U_V(pp,1)=U_V(pp,l)+2- (-k)* p_i(2*pp-l);
U _V(pp,2)=U_V(pp,2)+2~(-k)* p_i(2*pp) ;
end
if(predict-f < 5)
break ;
end
p_Ll=p_i;
end
X=O:l/N:l;
Y=O:l/N:l;
U=[ I:
V=[ J:
for i=N+l :-1:1
U=[U_V«i-l)*(N+l)+l:i*(N+l),l)' ;U] ;
V=[U_V«i-l)*(N+l)+1:i~(N+l),2)' ;V] ;
end
U
V
quiver(X,Y,U,V) ;
hold off;
370 14. COMPUTER CODES

load first_part first_part;


load penalty penalty;
first _part
penalty
save U_V U-Y;

The following are the subfunctions used in the main function.

function result=get_dlduv(uv,k,g,r,epsilon,N)
[A,connect]=get_connect-matrix(N);
[Al,Bl,Cl,A2,B2,C2,A3,B3,C3]=get_abc(N);
area=l/N~ 2;
for i=1:N*N*2
number=connect(i, :);
ul=uv(number(l),l);
vl=uv(number(1),2);
u2=uv(number(2),1);
v2=uv(number(2),2);
u3=uv(number(3),l);
v3=uv(number(3),2);
al=A1(i) ;
bl=Bl(i);
cl=Cl(i) ;
a2=A2(i) ;
b2=B2(i);
c2=C2(i);
a3=A3(i);
b3=B3(i) ;
c3=C3(i) ;
if number(l)<=N+l I mod(number(l),N+l)==l
I mod(number(1),N+1)==0 I number(1»(N+1)*N
Ie_du(i ,1)=0;
else
sl=(u1*b1+u2*b2+u3*b3)~2;
s1=sl+1/2*(u1*c1+u2*c2+u3*c3+v1*bl+v2*b2+v3*b3)~2;
sl=s1+(v1*c1+v2*c2+v3*c3)~2;
s1=s1 ~ (r/2-1) ;
s2=2* (u1*b1+u2*b2+u3*b3)*b1
+(u1*c1+u2*c2+u3*c3+v1*b1+v2*b2+v3*b3)*c1;
if 0<=«u1*bl+u2*b2+u3*b3)+(v1*c1+v2*c2+v3*c3))
s3=«ul*b1+u2*b2+u3*b3)+(v1*c1+v2*c2+v3*c3))- (r-1)*bl ;
else
s3=-(abs«ul*bl+u2*b2+u3*b3)+(v1*cl+v2*c2+v3*c3)))~ (r-l)*b1;
14.3. MATLAB CODES 371

end
Ie_du(i,1)=k/2*area*s1*s2+1/epsilon*area*s3;
s2=2*(v1*c1+v2*c2+v3*c3)*c1
+(u1*c1+u2*c2+u3*c3+v1*b1+v2*b2+v3*b3)*b1;
if O<=«u1*b1+u2*b2+u3*b3)+(v1*c1+v2*c2+v3*c3))
s3=«u1*b1+u2*b2+u3*b3)+(v1*c1+v2*c2+v3*c3))- (r-1)*c1;
else
s3=-(abs«u1*b1+u2*b2+u3*b3)+(vl*cl+v2*c2+v3*c3)))- (r-l)*cl;
end
Ie_dv(i,1)=k/2*area*s1*s2+1/epsilon*area*s3+g*area*1/3;
end
if number(2)<=N+1 I mod(number(2),N+1)==1
I mod(number(2),N+1)==O I number(2»(N+1)*N
Ie_du(i,2)=O;
else
s1=(ul*bl+u2*b2+u3*b3)-2;
sl=s1+1/2*(ul*cl+u2*c2+u3*c3+vl*bl+v2*b2+v3*b3)-2;
s1=sl+(vl*cl+v2*c2+v3*c3)-2 ;
s1=s1- (r/2-1);
s2=2* (u1*b1+u2*b2+u3*b3)*b2
+(u1*c1+u2*c2+u3*c3+v1*b1+v2*b2+v3*b3)*c2;
if O<=«u1*b1+u2*b2+u3*b3)+(v1*c1+v2*c2+v3*c3))
s3=«u1*b1+u2*b2+u3*b3)+(v1*c1+v2*c2+v3*c3))- (r-1)*b2;
else
s3=-(abs«u1*b1+u2*b2+u3*b3)+(v1*c1+v2*c2+v3*c3)))- (r-1)*b2 ;
end
Ie_du(i,2)=k/2*area*s1*s2+1/epsilon*area*s3;
s2=2* (v1*c1+v2*c2+v3*c3) *c2
+(ul*cl+u2*c2+u3*c3+vl*bl+v2*b2+v3*b3)*b2;
if O<=«ul*bl+u2*b2+u3*b3)+(v1*cl+v2*c2+v3*c3))
s3=«ul*bl+u2*b2+u3*b3)+(vl*cl+v2*c2+v3*c3))- (r-l)*c2;
else
s3=-(abs«u1*b1+u2*b2+u3*b3)+(vl*cl+v2*c2+v3*c3)))- (r-l)*c2;
end
Ie_dv(i,2)=k/2*area*s1*s2+1/epsilon*area*s3+g*area*1/3;
end
if number(3)<=N+1 I mod(number(3),N+1)==1
I mod(number(3),N+l)==O I number(3»(N+1)*N
Ie_du(i,3)=O;
else
s1=(ul*b1+u2*b2+u3*b3) -2;
sl=sl+1/2*(ul*c1+u2*c2+u3*c3+vl*bl+v2*b2+v3*b3) -2;
s1=s1+(v1*c1+v2*c2+v3*c3)-2;
s1=s1- (r/2-1) ;
s2=2*(u1*b1+u2*b2+u3*b3)*b3
372 14. COMPUTER CODES

+(u1*c1+u2*c2+u3*c3+v1*b1+v2*b2+v3*b3)*c3;
if O<=«u1*b1+u2*b2+u3*b3)+(v1*c1+v2*c2+v3*c3))
s3=«ul*bl+u2*b2+u3*b3)+(vl*cl+v2*c2+v3*c3))- (r-l)*b3;
else
s3=-(abs«ul*bl+u2*b2+u3*b3)+(vl*cl+v2*c2+v3*c3)))- (r-l)*b3;
end
Ie_du(i,3)=k/2*area*sl*s2+1/epsilon*area*s3;
s2=2* (vl*cl+v2*c2+v3*c3) *c3
+(ul*cl+u2*c2+u3*c3+vl*bl+v2*b2+v3*b3)*b3;
if O<=«ul*bl+u2*b2+u3*b3)+(vl*cl+v2*c2+v3*c3));
s3=«ul*bl+u2*b2+u3*b3)+(v1*cl+v2*c2+v3*c3))- (r-l)*c3;
else
s3=-(abs«u1*bl+u2*b2+u3*b3)+(vl*cl+v2*c2+v3*c3)))- (r-l)*c3;
end
Ie_dv(i,3)=k/2*area*sl*s2+1/epsilon*area*s3+g*area*1/3;
end
end
test=1;
calLnumber=l ;
for i=1:N+l
for j=l:N+l
result (call-number)=sum(sum«connect==test) .*I e_du) ) ;
call-number=call-number+l;
result(call-number)=sum(sum«connect==test).*Ie_dv));
call-number=call-number+l;
test=test+1;
end
end
result=result';
%---------Generate abc --------------------
function [a1,bl,cl,a2,b2,c2,a3,b3,c3]=get_abc(N)
area=1/N- 2;
[A,connect]=get_connect~atrix(N);
S=size(connect);
for i=l: S (1)
number=connect(i,:);
al(i)=det([A(number(2),:);A(number(3),:)))/area;
bl(i)=-det([l A(number(2),2);1 A(number(3),2)])/area;
c1(i)=det([l A(number(2),1);1 A(number(3),1)])/area;

a2(i)=-det([A(number(1), :);A(number(3),:)])/area;
b2(i)=det([l A(number(1),2);1 A(number(3),2)])/area;
c2(i)=-det([l A(number(l),l);l A(number(3),1)])/area;

a3(i)=det([A(number(1),:);A(number(2),:)])/area ;
b3(i)=-det([1 A(number(1),2);1 A(number(2),2)])/area;
14.3. MATLAB CODES 373

c3(i)=det([1 A(number(l),l);l A(number(2),1)])/area;


end
function result=get_Ie(uv,k,g,r,epsilon,N)
[A,connect]=get_connect~atrix(N);
[Al,Bl,Cl,A2,B2,C2,A3,B3,C3]=get_abc(N);
area=1/N~ 2;
first_part=O;
penalty=O;
for i=1:N*N*2
number=connect(i,:);
ul=uv(number(l),l);
vl=uv(number(1),2);
u2=uv(number(2),1);
v2=uv(number(2),2);
u3=uv(number(3),1);
v3=uv(number(3),2);
al=Al(i);
bl=Bl(i);
cl=Cl(i);
a2=A2(i) ;
b2=B2(i);
c2=C2(i);
a3=A3(i) ;
b3=B3(i);
c3=C3(i);
sl=(ul*bl+u2*b2+u3*b3)~2;
sl=sl+1/2*(ul*cl+u2*c2+u3*c3+vl*bl+v2*b2+v3*b3)~2;
sl=sl+(vl*cl+v2*c2+v3*c3)~2;
sl=sr (r/2);
s2=1/3*(vl+v2+v3);
( r);
s3=(abs«ul*bl+u2*b2+u3*b3)+(vl*cl+v2*c2+v3*c3)))~

Ie(i)=k/r*area*sl+g*area*s2+1/(r*epsilon)*area*s3;
first_part=first_part+k/r*area*sl+g*area*s2;
penalty=penalty+l/(r*epsilon)*area*s3;
end
result=sum(Ie);
save first_part first _part;
save penalty penalty;
374 14. COMPUTER CODES

14.4. Fortran Codes

14.4.1. FORTRAN CODE FOR EXAMPLE 8.2.

C This program uses nonlin in dimensionless form


C PROGRAM MASS FLOW GAUSS INTEGRATION TRIANGLE
COMMON/EL/ESM(4,4),EKM(4,4),EKE(4,4),EFS(4)
COMMON/RZ/R(3),Z(3)\hb
COMMON/ELE/CK,CO,CP,CH,KL,TBULK
COMMON/HCV/IDBC(3000,2), NDBC
COMMON/TLE/TITLE(20)
COMMON/GL/GPHIB(3000),GSM2(3000,3000),GF2(3000)
COMMON/CK/EKU(3,3),SL(3,3),GF(3000)
COMMON/GNON/GNOY(3000),GFT(3000),GSM3(3000,3000)
COMMON/GT/GKT(3000) ,GNO(3000)
COMMON/GSM/GSM(3000,3000),NP
COMMON/LNON/ENO(3),ENP(3,3),T(3)
COMMON/NON/AN,BN,RN,aa,UAVG,TM,V,PI
DIMENSION NEL(3000,4), RC(3000), ZC(3000),IB(3000),BV(3000)
DIMENSION NS(4)
DIMENSION ICK(3000)
INTEGER JBJ
DATA GRAND/90000/,TOL/1 .0E-2/
DATA IN/60/,IO/61/
C
DATA NO/40/
C

BN=O.024
aa=O .125
RN=O .453
AN=2 .82
UAVG=15.0
TM=399.5
V=(RN+1)/RN
PI=3.1415926
C
C INPUT OF THE TITLE CARD AND CONTROL PARAMETERS
C
OPEN(IN, FILE='MFGIT.in' ,STATUS='UNKNOWN')
OPEN(IO, FILE='MFGIT.out' ,STATUS='UNKNOWN')
C
OPEN(NO, FILE='ERRORCHECK.TXT',STATUS= 'UNKNOWN')
14.4. FORTRAN CODES 375

C
READ(IN,3) TITLE
3 FORMAT (20A4)
READ(IN,*) NP, NE, NDBC, ITYP, IPLVL
READ(IN,*) CK,CO,CP
READ(IN,*) (RC(I),I=l,NP)
READ(IN,*) (ZC(I),I=l,NP)
C
C OUTPUT OF THE TITLE AND THE NODAL
C
WRlTE(IO,4) TITLE,NP,NE,ITYP,IPLVL
4 FORMAT(//10X,20A4//10X,5HNP = ,I5/10X,5HNE = ,15/
+ 10X,8HITYP = ,I2/10X,8HIPLVL = ,12)
WRlTE(IO,6) CK,CO,CP
6 FORMAT(/10X,4HK = ,Fl0 .8/10X,4Hp = ,F12.10/
+ 10X,5HCp = ,F7.2)
C
C OUTPUT OF THE NODAL COORDINATES
C
WRITE(IO,ll)
11 FORMAT(//10X,17HNODAL COORDINATES/lOX ,
+ 4HNODE,10X,1HR,14X,1HZ)
WRITE(IO,12) (I,RC(I),ZC(I) ,I=l,NP)
12 FORMAT(10X,I4,2F15 .3)
C
C INPUT AND ECHO PRINT OF THE ELEMENT NODAL DATA
C
WRITE(IO,8) TITLE
8 FORMAT(//10X,20A4//10X,12HELEMENT DATA/
+ 15X,3HNEL,4X,12HNODE NUMBERS)
NID=O
DO 9 KK=l,NE
READ(IN,*) N,(NEL(N,I),I=1,4)
IF«N-l) .NE .NID) WRITE(IO,17) N
17 FORMAT(10X,7HELEMENT,I4 ,16H NOT IN SEQUENCE)
NID=N
IF(NEL(N,4) .EQ.O) WRITE(IO,7) N,(NEL(N,I),I=1,3)
9 IF(NEL(N,4) .NE.O) WRITE(IO,7) N,(NEL(N,I),I=1,4)
7 FORMAT(15X,I3,2X,414)
C
C INPUT AND ECHO PRINT OF THE DERIVATIVE
C BOUNDARY CONDITION DATA
C
IF(NDBC.EQ .O) GOT072
WRlTE(IO,49)
376 14. COMPUTER CODES

49 FORMAT(//10X,34HDERIVATIVE BOUNDARY CONDITION DATA/


+ 15X,7HELEMENT,4X,4HSIDE)
DO 10 I=1,NDBC
READ(IN,*) IDBC(I,1),IDBC(I,2)
10 WRITE(IO,71) IDBC(I,1),IDBC(I,2)
71 FORMAT(15X,I4,9X,I1)
READ(IN,*) CH, TBULK
WRITE(IO,70) CH, TBULK
70 FORMAT(/10X,4HH = ,F10.2/10X,8HTBULK = ,F10.2)
C*********
C ANALYSIS OF THE NODE NUMBERS
C*********
C
C INITIALIZATION OF A CHECK VECTOR
C
72 DO 500 I=l,NP
500 ICK(I)=O
C
C GENERATION OF THE SYSTEM OF EQUATIONS
C
DO 302 I=l,NP
GF(I)=O
DO 301 J=1,NP
GSM(I, J)=O
301 CONTINUE
302 CONTINUE
C DELETE LABEL 30 HERE
DO 32 KK=1,NE
KL=3
C
C RETRIEVAL OF NODAL COORDINATES AND NODE NUMBERS
C
DO 31 I=1,KL
NS(I)=NEL(KK, I)
J=NS(I)
R(I)=RC(J)
31 Z(I)=ZC(J)
C
C CALCULATION OF THE ELEMENT STIFFNESS MATRIX
C AND ELEMENT FORCE VECTOR
C

CALL ELSTMF(KK,IPLVL)

C
14.4. FORTRAN CODES 377

C DIRECT STIFFNESS PROCEDURE


C
DO 33 I=1,KL
II=NEL (KK , I )
GF(II)=GF(II)+EFS(I)
GF2(II)=GF(II)
DO 34 J=1,KL
JJ=NEL(KK,J)
GSM(II,JJ)=GSM(II,JJ)+ESM(I,J)
GSM2(II,JJ)=GSM(II,JJ)
34 CONTINUE
33 CONTINUE
32 CONTINUE
C USE THIS TO INTRODUCE SPECIFIED TEMPERATURE
C IB NUMBER OF THE NODE WHERE THE TEMPERATURE IS DEFINED
C BV SPECIFIED TEMPERATURE
C NP NUMBER OF THE TOTAL NODE
C GSM GLOBAL STIFF. MATRIX
J=1
209 CONTINUE
READ(IN,*)IB(J ),BV(J)

IF(IB(J) .LE.O)GOTO 15
C
C INTRODUCTION OF THE BOUNDARY CONDITION USING THE
C PENALTY METHODS WITH A BIG DIAGONAL TERM
C
XMAX=O .
DO 131 I =l,NP
AK=ABS(GSM2(I ,I))
131 IF(AK .GT. XMAX) XMAX=AK
AGRAND=GRAND*XMAX
IF(IB(J) .GT. O)THEN
ABOUND=BV(J)
GSM2(IB(J),IB(J))=AGRAND
GF2(IB(J))=AGRAND*ABOUND
IF (GSM2(IB(J),IB(J)) .GT.10E+8) GSM2(IB(J) ,IB(J))=10E+8
IF (GF2(IB(J)) .GT .10E+8) GF2(IB(J))=10E+8
ENDIF
J=J+l
GOTO 209
15 CONTINUE
IBJ=J-1
C
C SOLUTION OF THE SYSTEM OF EQUATIONS
378 14. COMPUTER CODES

C
CALL GAUSS
CALL BACK
C DO 55 I=1,NP
C 55 WRITE (10,*) 'GPHIB(' ,1,')=' ,GPHIB(I)
C******
C INITIAL VALUE-GPHIB(I),EVALUATE EACH TERM IN THE
C NEWTON ITERATION
C EVALUATE THE NONLINEAR TERM USING GAUSS INTEGRATION
C******
ITER=O
YTOL=1.0
DO WHILE (ITER .LT.40.AND. YTOL.GT.TOL)
.AND.YTOL.GT.TOL
C
C INITIALIZE GNO(I) AND GKT(I)
C
YTOL=O.O
DO 38 I=1,NP
GKT(I)=O.O
GNO(I)=O.O
DO 39 J=1,NP
GSM3(I,J)=GSM(I,J)
39 CONTINUE
38 CONTINUE
C
C EVALUATE [K] .{T}
C
DO 40 I=1,NP
DO 400 J=1,NP
GKT(I)=GKT(I)+GSM3(I,J)*GPHIB(J)
WRITE (10,*) 'GKT(',I,')=',GKT(I)
400 CONTINUE
40 CONTINUE
WRITE (10,*) , ,
DO 42 KK=1,NE
KL=3
DO 41 I=1,KL
NS (I) =NEL (KK, I)
J=NS(I)
R(I)=RC(J)
Z(I)=ZC(J)
41 T(I) =GPHIB (J)
C
C EVALUATE THE NONLINEAR TERMS N(T) AND N'(T)
14.4. FORTRAN CODES 379

C
C WRITE (10,*) 'in element' ,kk
CALL NONLIN
C
C ADD LOCAL MATRIX OF N(T) TO GLOBAL MATRIX GNO(T)
C ADD LOCAL MATRIX OF N'(T) TO GLOBAL MATRIX GSM(II,JJ)
C
DO 43 I=1,KL
II=NEL(KK,I)
GNO(II)=GNO(II)+ENO(I)

DO 62 IK=1,NP
DO 62 JK=1,NP
WRITE (10,*) ' BEFORE ADDITION'
62 WRlTE(IO,*) 'GSM3(' ,IK,',' ,JK,')=' ,GSM3(IK,JK)
WRITE (IO, *) , ,
DO 44 J=1,KL
JJ=NEL(KK,J)
GSM3(II,JJ)=GSM3(II,JJ)-ENP(I,J)
44 CONTINUE
43 CONTINUE
42 CONTINUE
DO 45 I=1,NP
GFT(I)=-GKT(I)+GNO(I)+GF(I)
45 CONTINUE
C
C EVALUATE J(T) .{T}-F(T)
c
WRITE (10,*) 'EVALUATE J(T) .{T}-F(T),
DO 46 I=1,NP
SUM=O.O
DO 47 J=1,NP
47 SUM=SUM+GSM3(I,J)*GPHIB(J)
GFT(I)=GFT(I)+SUM
C WRITE (10,*) 'GFT2(' ,1,')=' ,GFT(I)
46 CONTINUE
C
C INTRODUCTION OF THE BOUNDARY CONDITION USING
C THE PENALTY METHODS WITH A BIG DIAGONAL TERM
C
DO 133 J=1, IBJ
XMAX=O .
DO 132 I=1,NP
AXMA=ABS(GSM3(I,I))
132 IF(AK . GT. XMAX) XMAX=AXMA
380 14. COMPUTER CODES

AGRAND=GRAND*XMAX
IF(IB(J) .GT. O)THEN
ABOUND=BV (J)
GSM3(IB(J),IB(J))=AGRAND
GFT(IB(J))=AGRAND*ABOUND
IF (GSM3(IB(J),IB(J)).GT.10E+8) GSM3(IB(J),IB(J))=10E+8
IF (GFT(IB(J)) .GT .10E+8) GFT(IB(J))=10E+8
ENDIF
133 CONTINUE
C
C USING GAUSS ELIMINATION METHOD TO EVALUATE J(T) .Y=-F(T)
C
CALL GAUSS1
CALL BACK1
DO 661 I=l,NP
661 YTOL=YTOL+(GPHIB(I)-GNOY(I)) **2

YTOL=SQRT(YTOL)
C WRITE (10,*) 'YTOL=' ,YTOL
DO 134 I=l,NP
134 GPHIB(I)=GNOY(I)
WRITE(IO,*) 'ITER=',ITER
WRITE(IO,*) 'NODAL TEMPERATURE'
WRITE(IO,601) 'NODE' ,'TEMPERATURE'
601 FORMAT(lX, A4, A15)
WRITE(IO,662) (I,GPHIB(I),I=l,NP)
662 FORMAT(lX,I4,10X,E10 .4)
ITER=ITER+1
END DO
C
C OUTPUT OF THE CALCULATED VALUES
C
WRITE(IO,*) 'NODAL TEMPERATURE'
WRITE(IO,600) 'NODE' ,'TEMPERATURE'
600 FORMAT(lX, A4, A15)
WRITE(IO,660) (I,GPHIB(I),!=l,NP)
660 FORMAT(lX,I4,10X,El0 .4)
CLOSE(60)
CLOSE(61)
C
CLOSE(NO)
C
STOP
END
14.4. FORTRAN CODES 381

SUBROUTINE ELSTMF(KK,IPLVL)
COMMON/EL/ESM(4,4),EKM(4,4),EKE(4,4),EFS(4)
COMMON/RZ/R(3),Z(3)
COMMON/IPTS/VR(7),VZ(7),WC(7)
COMMON/PDXY/VN(3),PNR(3),PNZ(3),RD,ZD,DET
COMMON/ELE/CK,CO,CP,CH,KL,TBULK
COMMON/HCV/IDBC(3000,2),NDBC
COMMON/CK/EKU(3,3),SL(3,3),GF(3000)
COMMON/NON/AN,BN,RN,aa,UAVG,TM,V,PI
REAL D
DIMENSION B(3),C(3)
DATA IO/61/
C
NO=40
C
C
C LINEAR TRIANGULAR ELEMENT WITHOUT THE DERIVATIVE
C BOUNDARY CONDITION
C
DO 1332 1=1,3
EFS(I)=O.O
DO 1332 J=1,3
EKM(I,J)=O.O
EKU(I, J)=O.0
1332 CONTINUE
PI=3.1416

B(1)=Z(2)-Z(3)
8(2)=Z(3)-Z(1)
B(3)=Z(1)-Z(2)
C(1)=R(3)-R(2)
C(2)=R(1)-R(3)
C(3)=R(2)-R(1)
AR2=R(2)*Z(3)+R(3)*Z(1)+R(l)*Z(2)-R(2)*Z(1)
+ -R(3)*Z(2)-R(1)*Z(3)
IF(ABS(AR2) .LT .0.00001)GOTO 50
RB=(R(1)+R(2)+R(3))/3.0
D=(V*CK/(V+2)/CO/CP/UAVG/aa) **2
DO 10 1=1,3
DO 9 J=1,3
EKE(I,J)=PI/AR2*RB*B(I)*B(J)
EKE(I,J)=EKE(I,J)+PI/AR2*RB*D*C(I)*C(J)
9 CONTINUE
10 CONTINUE
C*****
382 14. COMPUTER CODES

C EVALUATION OF THE MATRIX EKU USING NUMERICAL


C INTEGRATION TECHNIQUE IN TRIANGULAR REGION
C GAUSS POINTS N=6
C****
CALL INGPTS
DO 333 II=l, 7
CALL PDERV(VR(II),VZ(II))

C
DO 222 1=1,3
DO 222 J=1,3
222 EKU(I,J)=EKU(I,J)+CONC1*RD*VN(I)*PNZ(J)*WC(II)
333 CONTINUE
SL(1,2)=SQRT«R(2)-R(1))**2+(Z(2)-Z(1))**2)
SL(2,3)=SQRT«R(3)-R(2))**2+(Z(3)-Z(2))**2)
SL(3,1)=SQRT«R(3)-R(1))**2+(Z(3)-Z(1))**2)
SL(1 ,3)=SL(3, 1)
IF(NDBC .EQ .O) GOTO 70
C
C DERIVATIVE BOUNDARY CONDITION
C
CM=CH
DO 110 I=l,NDBC
IF(IDBC(I,l) .NE.KK) GOTO 110
J=IDBC(I,2)
K=J+l
IF(J .EQ .KL) K=l
EFS(J)=PI*CM*TBULK*SL(J,K)*(2*R(J)+R(K))/3
EFS(K)=PI*CM*TBULK*SL(J,K)*(R(J)+2*R(K»/3
EKM(J,J)=PI*CM*SL(J,K)*(3*R(J)+R(K»/6
EKM(K,K)=PI*CM*SL(J,K)*(R(J)+3*R(K»/6
EKM(J,K)=PI*CM*SL(J,K)*(R(J)+R(K»/6
EKM(K,J)=EKM(J,K)
110 CONTINUE
C
C OUTPUT OF THE ELEMENT MATRICES
C
70 IF(IPLVL.EQ .O) RETURN
DO 500 J=1,3
DO 400 K=1,3
ESM(J,K)=EKE(J,K)+EKM(J,K)+EKU(J,K)
400 CONTINUE
500 CONTINUE
WRITE(IO,80) KK
14.4. FORTRAN CODES 383

80 FORMAT(/10X,7HELEMENT,I4/10X,12HFORCE VECTOR, lOX,


+ 16HSTIFFNESS MATRIX)
DO 90 I=l,KL
90 WRITE(IO,100) EFS(I),(ESM(I,J),J=l,KL)
100 FORMAT(5X,E13.5,10X,4E14.7)
RETURN
C
C DIAGNOSTIC OUTPUT
C
50 WRITE(IO,69) KK
69 FORMAT(//10X,19HTHE AREA OF ELEMENT,I4,
+ 20H IS LESS THAN 0.0001/
+ 10X,39HTHE NODE NUMBERS ARE IN THE WRONG ORDER/
+ 10X,33HOR THE NODES FORM A STRAIGHT LINE/
+ 10X,20HEXECUTION TERMINATED)
STOP
END

SUBROUTINE NONLIN
C*****
C THIS PROGRAM EVALUATES THE INTEGRATION OF THE NONLINEAR TERM
IN
C MASS FLOW EQUATION USING GAUSS INTEGRATION 7 POINTS (ORDER
6)
C IN TRIANGLE REGION, THE INITIAL VALUE OF Ti IS GIVEN
C*****
COMMON/LNON/ENO(3),ENP(3,3),T(3)
COMMON/RZ/R(3),Z(3)
COMMON/PDXY/VN(3) ,PNR(3),PNZ(3) ,RD ,ZD,DET
COMMON/IPTS/VR(7) ,VZ(7),WC(7)
COMMON/NON/AN,BN,RN,aa,UAVG,TM,V,PI
REAL EXPS,SUMVN,RE
DATA 10/61/
DATA U/0.15/,CP/251.0/,CO/794.0/ /
DO 1 1=1,3
ENO(I) =0 .0
DO 1 J=1,3
1 ENP(I,J)=O.O
C*****
C EVALUATION OF THE MATRIX ENO USING NUMERICAL INTEGRA-
TION
C TECHNIQUE IN TRIANGULAR REGION
C****
CALL INGPTS
384 14. COMPUTER CODES

DO 333 II=1, 7
CALL PDERV(VR(II),VZ(II))

CONC=PI*DET*AN*EXP(BN*RN*TM)*(UAVG*(V+2))**(RN+1)/
+ aa**(RN-1)
CONC=CONC/0 .00255
SUMVN=O .O
DO 223 K=1,3
223 SUMVN=SUMVN+VN(K)*T(K)
EXPS=EXP(-BN*RN*SUMVN)
RE=RD**(V+1)
DO 222 1=1,3
ENO(I)=ENO(I)+CONC*WC(II)*VN(I)*RE*EXPS
DO 222 J=1,3
ENP(I,J)=ENP(I,J)-CONC*BN*RN*WC(II)*VN(I)*VN(J)*RE*EXPS
222 CONTINUE
333 CONTINUE
RETURN
END

SUBROUTINE GAUSS
COMMON/GL/GPHIB(3000) ,GSM2(3000,3000) ,GF2(3000)
COMMON/GSM/GSM(3000,3000),NP
DO 40 I=1,NP
PIVOT=GSM2(I,I)
ABSP=ABS(PIVOT)
IPROW=I
DO 10 J=I+1,NP
IF(ABS(GSM2(J ,I)).LE .ABSP) GOTO 10
PIVOT=GSM2(J,I)
ABSP=ABS(PIVOT)
IPROW=J
10 CONTINUE
IF (IPROW .LE.I) GOTO 22
DO 20 J=I,NP
TEMP=GSM2(I,J)
GSM2(I,J)=GSM2(IPROW,J)
GSM2(IPROW,J)=TEMP
20 CONTINUE
TEMP = GF2(I)
GF2(I) = GF2(IPROW)
GF2(IPROW) = TEMP
22 DO 30 J = 1+1, NP
TIMES = GSM2(J,I)/PIVOT
GSM2 (J ,I) = 0.0
14.4. FORTRAN CODES 385

DO 25 K = I+1,NP
GSM2(J,K) = GSM2(J,K)-TIMES*GSM2(I,K)
25 CONTINUE
GF2(J) = GF2(J)-TIMES*GF2(I)
30 CONTINUE
40 CONTINUE
RETURN
END

SUBROUTINE BACK
COMMON/GL/GPHIB(3000),GSM2(3000,3000),GF2(3000)
COMMON/GSM/GSM(3000,3000),NP
10=61
GPHIB(NP)=GF2(NP)/GSM2(NP ,NP)
DO 20 I=NP-1,1,-1
QUOT=GF2 (1)
DO 10 J=1+1,NP
QUOT=QUOT-GSM2(I,J)*GPHIB(J)
10 CONTINUE
GPHIB(I)=QUOT/GSM2(I,I)
20 CONTINUE
RETURN
END

SUBROUTINE GAUSS1
COMMON/GNON/GNOY(3000),GFT(3000),GSM3(3000,3000)
COMMON/GSM/GSM(3000,3000),NP
IO=61
WRITE (10,*) , ,
DO 40 I=1,NP
PIVOT=GSM3 (I , 1)
ABSP=ABS(PIVOT)
IPROW=I
DO 10 J=1+1,NP
IF(ABS(GSM3(J,I».LE .ABSP) GOTO 10
PIVOT=GSM3(J,I)
ABSP=ABS(PIVOT)
IPROW=J
10 CONTINUE
IF (IPROW.LE .I) GOTO 22
DO 20 J=I,NP
TEMP=GSM3(I,J)
GSM3(I,J)=GSM3(IPROW,J)
GSM3(IPROW,J)=TEMP
20 CONTINUE
386 14. COMPUTER CODES

TEMP = GFT(I)
GFT(I) = GFT(IPROW)
GFT(IPROW) = TEMP
22 DO 30 J = 1+1, NP
TIMES = GSM3(J,I)/PIVOT
GSM3(J,I) = 0.0
DO 25 K = I+1,NP
GSM3(J,K) = GSM3(J,K)-TIMES*GSM3(I,K)
25 CONTINUE
GFT(J) = GFT(J)-TIMES*GFT(I)
30 CONTINUE
40 CONTINUE
RETURN
END

SUBROUTINE BACK1
COMMON/GNON/GNOY(3000),GFT(3000),GSM3(3000,3000)
COMMON/GT/GKT(3000),GNO(3000)
COMMON/GSM/GSM(3000,3000),NP
IO=61
GNOY(NP) =GFT(NP)/GSM3(NP ,NP)
DO 20 I=NP-1,1,-1
QUOT=GFT(I)
DO 10 J=I+l,NP
QUOT=QUOT-GSM3(I,J)*GNOY(J)
10 CONTINUE
GNOY(I)=QUOT/GSM3(I,I)
20 CONTINUE
RETURN
END

SUBROUTINE INGPTS
COMMON/IPTS/VR(7),VZ(7),WC(7)
DATA AC/0.05971587/,BC/0.47014206/
DATA C/0.10128651/,D/O .79742699/
C*******
C GENERATION OF THE NINE INTEGRATION POINTS FOR THE
C LINEAR TRIANGULAR ELEMENT GAUSS POINTS N=6
C*******
VR(1)=0.3333333
VZ(1)=O.3333333
WC(1)=O.225
VR(2)=BC
VZ(2)=AC
WC(2)=O.13239415
14.4. FORTRAN CODES 387

VR(3)=BC
VZ(3)=BC
WC(3)=O.13239415
VR(4)=AC
VZ(4)=BC
WC(4)=O.13239415
VR(5)=C
VZ(5)=D
WC(5)=O .12593918
VR(6)=C
VZ(6)=C
WC(6)=O.12593918
VR(7)=D
VZ(7)=C
WC(7)=O.12593918
RETURN
END

SUBROUTINE PDERV(X1,X2)
C******
C THIS SUBROUTINE EVALUATES THE JACOBIAN TRANSFORMATION
MATRIX
C AND USES THE INVERSE JACOBIAN MATRIX TO OBTAIN THE
C DERIVATIVES OF THE SHAPE FUNCTIONS WITH RESPECT TO R
AND Z
C******
COMMON/PDXY/VN(3),PNR(3),PNZ(3),RD,ZD,DET
COMMON/RZ/R(3),Z(3)
COMMON/DERV/VNN(3),PNS(3),PNE(3)
REAL JOCB(2,2)
DATA 10/61/
C*******
C INITIALIZATION OF THE JACOBIAN MATRIX
C*******
DO 1 1=1,2
DO 1,J=1,2
1 JOCB(I,J)=O .O
RD=O .000001
ZD=O.000001
C*******
C EVALUATION OF THE SHAPE FUNCTIONS AND THEIR
C DERIVATIVES
C*******
CALL QDSHFN(Xl,X2)
C*******
388 14. COMPUTER CODES

C CALCULATION OF THE COORDINATES OF THE INTEGRATION


C POINT (RD,ZD) AND THE JACOBIAN MATRIX
C*******
DO 2 1=1,3
RD=RD+VNN(I)*R(I)
ZD=ZD+VNN(I)*Z(I)
JOCB(1,1)=JOCB(1,1)+PNS(I)*R(I)
JOCB(1,2)=JOCB(1,2)+PNS(I)*Z(I)
JOCB(2,1)=JOCB(2,1)+PNE(I)*R(I)
2 JOCB(2,2)=JOCB(2,2)+PNE(I)*Z(I)
C*******
C CALCULATION OF THE INVERSE OF THE JACOBIAN MATRIX
C*******
AD=JOCB(1,1)*JOCB(2,2)-JOCB(2,1)*JOCB(1,2)
BD=JOCB(1,1)
JOCB(1,1)=JOCB(2,2)/AD
JOCB(1,2)=-JOCB(1 ,2)/AD
JOCB(2,1)=-JOCB(2,1)/AD
JOCB(2 ,2)=BD/AD
DET=ABS(AD)
C*******
C CALCULATION OF THE PARTIAL DERIVATIVES WITH RESPECT TO
RAND Z
C*******
DO 3 1=1,3
VN(I) =VNN(I)
PNR(I)=JOCB(1,1)*PNS(I)+JOCB(1,2)*PNE(I)
3 PNZ(I) =JOCB(2,1)*PNS(I)+JOCB(2,2) *PNE(I)
RETURN
END

SUBROUTINE QDSHFN(SI,ETA)
C*******
C THIS SUBROUTINE CALCULATES THE VALUE OF THE SHAPE
C FUNCTIONS AND THEIR DERIVATIVES FOR THE QUADRATIC
C QUADRILATERAL ELEMENT GIVEN SPECIFIC VALUES OF KSI
C AND ETA
C VN - VALUE OF THE KSI AND ETA
C PNS - PARTIAL DERIVATIVE WITH RESPECT TO SKI
C PNE - PARTIAL DERIVATIVE WITH RESPECT TO ETA
C*******
COMMON/DERV/VN(3),PNS(3),PNE(3)
VN(1)=SI
PNS (1) =1
PNE(1)=O
14.4. FORTRAN CODES 389

VN(2)=ETA
PNS(2)=O
PNE(2)=1
VN(3)=1-SI-ETA
PNS(3)=-1
PNE(3)=-1
RETURN
END

SUBROUTINE NONLIN2
C*****
C THIS PROGRAM EVALUATE THE INTEGRATION OF THE NONLINEAR TERM
C IN MASS FLOW EQUATION USING GAUSS INTEGRATION 7 POINTS
C (ORDER 6) IN TRIANGLE REGION, THE INITIAL VALUE OF Ti
C IS GIVEN
C*****
COMMON/LNON/ENO(3) ,ENP(3,3) ,T(3)
COMMON/RZ/R(3),Z(3)
COMMON/PDXY/VN(3) ,PNR(3) ,PNZ(3) ,RD,ZD,DET
COMMON/IPTS/VR(7) ,VZ(7) ,WC(7)
COMMON/NON/AN,BN,RN,aa,UAVG,TM,V,PI
REAL EXPS,SUMVN,RE,AREA
DATA 10/61/
DATA U/0.15/,CP/251 .0/,CO/794.0/ /
DO 1 1=1,3
ENO(I) =0 .0
DO 1 J=1,3
1 ENP(I,J)=O .O
RAVG=(R(1)+R(2)+R(3))/3
TAVG=(T(1)+T(2)+T(3))/3
c WRITE (10,*) 'RAVG=' ,RAVG
C=(UAVG*(V+2)*RAVG**(V-l)/aa)**(RN+l)*aa**2/0.00255
AREA=R(2)*Z(3)+R(3)*Z(1)+R(1)*Z(2)-R(2)*Z(1)
+ -R(3)*Z(2)-R(1)*Z(3)
CONC=2*PI*An*EXP(BN*RN*TM)*EXP(-BN*RN*TAVG)*RAVG*aa*C*AREA
DO 2 1=1,3
DO 2 J=1,3
2 ENP(I ,J)=-BN*RN*CONC/12. 0
DO 3 1=1,3
3 ENP(I,I)=2*ENP(I,I)
DO 5 1=1,3
5 ENO(I)=CONC/3.0
RETURN
END
390 14. COMPUTER CODES

The input and output files nonlin. in and nonlin. out for Example 8.2 are
available at the authors' websites.

14.4.2. The following Fortran program computes the exact solution (9.59) in
Exercise 9.17 .

c File: exactsolution.f (Sahai, 1991)


c This program computes the exact solution from x=O to 1 and
time t as desired.
double precision v(101),x(101),vi,vf,u,con,rho,cp,alpha
double precision time,dem,a,b,c,d,e,f,g,an,ans
open(unit=8, file='exact .out', status='new')
vi=298.
vf=1233 .
u=0 .5
con=226.
eho=2698.
cp=900 .
alpha=con/(rhO*cp)
print *,'input time desired'
read(*,l)time
1 format(dl0.0)
x(1)=O.
do 15 i=1,101
x(i+1)=x(i)+0.01
print *,'xi ', xCi)
15 continue
dem=dsqrt(alpha*time)
do 16 i=1,101
a=(x(i)-(u*time»/(2*dem)
b=(x(i)+(u*time)/(2*dem)
print *,'a, b, dem', a, b, dem
e=u*x(i)
f=e/alpha
g=dexp(f)
v(i)=vi+(0.5*(vf-vi)*(erfcc(a)+(g*erfcc(b) ) )
16 continue
do 17 i=l, 101
print *,x(i), v(i)
write(8,2) xCi), v(i)
2 format(d5 .4, 5x,d6 .6)
17 continue
stop
end
14.4. FORTRAN CODES 391

c
real function erfcc(y)
c23456 returns the complementary error function erfc(x)
c
double precision z, y, ans, t
z=abs(y)
t=1./(1.+0.5*z)
erfcc=t*dexp(-z*z -1 .26441223+t*(1.00002368)+t*(O.37409196+

* t*(O.09678418+t*(-O .18628806+t*(O.27886807+t*(-1.13520398+

* t*(1.48851587+t*(-O .82215223+t*O.17087277))))))))
if(y .lt .O) erfcc=2 .-erfcc
return
end
A
Integration Formulas

Some useful formulas and results on integration are given below. Proofs can be
found in any standard calculus textbook.
Let R" denote the n-dimensional Euclidean space and Z+ the set of nonneg-
ative integers. A useful integration formula is

(A.l)

where r(n) is the gamma function, defined by f(n) = 1 00

t n - 1 e- t dt, n > 0,
such that
n r( n) if n is real,
I' (n+ 1) ={
n! ifn E Z+.

A special case of formula (A. I) for m , n E Z + is

1
1 , I
m n m.n.
x (1 - x) dx = ( )1 ' (A.2)
o m+ n+l .

INTEGRATION BY PARTS. Let f( x) and g(x) be piecewise continuous func-


tions in an interval (a, b) with their first derivatives continuous in (a, b). Then
using integration by parts we get

(A.3)

Further, if f( x) and g(x) are piecewise continuous functions in an interval (a, b)


with their first and second derivatives piecewise continuous in (a, b), and h(x) is
394 A. INTEGRATION FORMULAS

a continuous function in (a, b), then using integration by parts we get

The integration by parts in R", n 2: 2, is defined as a consequence of the gradient


and divergence theorems . Let a finite domain n c R 2 be bounded by a smooth
closed curve I', and let wand F be scalar functions and G a vector function
continuous on n. Then

Gradient Theorem: fk "ilF dx dy = i n F ds,

Divergence Theorem: fk "il . G dx dy = in. G ds,

where n is the outward normal to the curve I', ds denotes the line element that
is defined by ds = J dx 2 + dy2 . These two theorems lead to the following two
2
useful identities in R :

fln("ilF)Wdxd y=- fin ("ilw)Fdxdy + inwFds , (A.6)

- fin ("il 2F)wdxdy = fin v«. "ilFdxdy - i ~: wds, (A.7)

h
were
O
on = n· M a a
v = n x ox + ny oy
. the norma ld envanve
IS
. .
operator.
U ·
smg
the gradient theorem, the component forms of (A.6), with appropriate variables,
are as follows:

fin w ~: dx dy = - fin ~: F dx dy + i nxw F ds,


(A .8)
fin w~: dxdy= - fin ~; Fdxdy+ i nywFds.

The gradient and the divergence theorems are valid in R 3 if the surface integral on
the left side is replaced by a volume integral and the line integral on the right is
395

replacedby a surfaceintegral. Formula(A.8)canbeextended to higherdimensions;


for example, in R3

11k w~: dxdydz = - 11k ~: Fdxdydz+ l~n nxwFdB,


11k w~: dxdydz = - 11k ~; Fdxdydz+ l~n nywFdB, (A.9)

JJLw ~~ dx dy dz JJL~: F dx dy dz + Jfan n w F dB,


= - z

where an denotes the boundary surfaceof n, and dB denotesthe surfaceelement.


B
Special Cases

Some special triangular and rectangular elements lead to different stiffness matrice s
and force vectors . The following three cases are valid for the Laplacian - \72 on a
right-angled triangular element with sides a and b, a 2 b, and the location of the
local nodes 1, 2, and 3, such that the local node 1 is at the origin.
1. For a right-angled linear triangular element n( e) with base a and altitude b, if
the local nodes 1,2, and 3 are at (0,0) , (a, 0), and (a, b), respectively, and the
local node 2 is at the right angle (see Fig. B.I (a» , then

(B.1)

and

(B.2)

2 . For a right-angled linear triangular element n (e ) with base a and altitude b,


if the local node 1 is at the right angle and the nodes 2 and 3 are numbered
counterclockwise (see Fig. B.l(b», then

(B.3)

and the force vector f (e) is the same as in (B.2).


398 B. SPECIAL CASES

3. For a right-angled linear triangular element nee) with base a and altitude b,
if the local node 3 is at the right angle and the nodes 1 and 2 are numbered
counterclockwise (see Fig. B.l(c», then

K(e ) = _1_ (B.4)


2ab

and the force vector r(e) is the same as in (B.2).

(a,b) (O,b) (O,b) (a,b)


3 2
3 3

2 2
(0,0) (a,O) (0,0) (a, 0) (0,0)
(a) (b) (c)

Fig. B.1. Three Cases of a Right Triangle.

4 . For a 4 node bilinear square element nee) of side a (Fig. 5.2 with a = b), the
stiffness matrix and the force vector for the Laplacian - '\72 are given by

!1 ~1 =i
[-2-1 -1-2 -14
-1]
-2
-1 '
r(e) j (e)_
= _ a
4
2
{i}
r t ' (B.5)
4 1

5. For heat transfer problems with convective conductance (3, the following ele-
ments are mostly used, with the respective stiffness matrices .
399

(5a) For a 3-nod e line ar triangular element (Fig. 5.1):

K (e) -
b -
(3(e)Z(e)
-!L!1...
6
[ 2~ ~1 o~]

(3(e) l (e)
+ ~
[0 0 0]
021 +~
(3(e) l (e) [2~ 0
0 (B.6)
6 0 1 2 6 0

f (e) _ (312
b - 2
12
(e)T 00 Z(e)
12 {OIl}

(B.7)

(5b) For a 4-node bilinear rectangular element (Fig. 5.2):

(B.8)

(B.9)

(5c) For a 6-node quadratic triangular element (Fig. 5.7) :

4 2 -1 0 0 0
1 16 2 0 0 0
(3(e) l (e)
K (e)=~ -1 2 4 0 0 0
b 30 0 0 0 0 0 0
0 0 0 0 0 0
0 0 0 0 0 0
400 B. SPECIAL CASES

0 0 0 0 0 0
0 0 0 0 0 0
/3(e) l(e) 0 0 2 -1 0
4
+~
30 0 0 2
16 2 0
0 0 2- 1 4 0
0 0 0 0 0 0
4 0 0 0 -1 2
0 0 0 0 0 0
/3(e) l(e) 0 0 0 0 0 0
+~
30 0 0 0 0 0 0
(B.10)
-1 0 0 0 4 2
2 0 0 0 2 16
1 0
4 0
/3(e) T I 3 l(e) 1 + /3(e)T 35 l(e)
1
f (e) _ 13 00 13 35 00 35
b - 6 0 6 4
0 1
0 0
1
0
/3(e) T 51 l(e)
+ 5 1 00 5 1 0 (B. ll)
6 0
1
4

6. Consider Eq (3.1) with a1 2 = a21 = c = 0, that is,

- -a
ax
( a ll -au )
ax
- -a
ay
( a22 -au )
ay
+ c u - f = 0, (B.12)

whereall ,a22 ,and care presc ribedfunctionsof xandy.lfall = k 1 anda22 =


ka. then Eq (B. 12) defines heat transfer problems, where k 1 and k 2 denote
thermal conductivities. We consider the case when all and a22 are bilinear
functions of the global variables x and y, i.e., all = a~~) + ag) x + ag) y, and
a 22 = a 22
(0) (1)
+ a 22 (2)
x + a 22 y. Then

(6 a) In the case of a linear triangular element n (e) ,

(e) _
K ij -
/1 ( all a
a4>~e) a4>je)
x
a
x
a4>~e) a4>je»)
+ a 22 a y aY dx dy

fl
O le)

= 1 2 ll + a (1)
/3i/3j ( a (0) (2» Y)
u x + all
4 (A (e») O (e )

+ 'Yi'Yj (0)
( a2 2 + a 22
(1) (2» )
x + a 22 Y dx dy
401

= 1 2 [{3i{3j (a ll0)1 00 + all


(1)
lID + all
(2)
120 )
4 (A(e))

+ I n j ( a 22
(0) (2) )]
100 + a 22 lID+ a 22 120 ,
(1)

(B.13)

where 100 , 110 • and 120 are defined in (A.25).


(6b) In the case of a bilinear rectangular element n (e),

(B .14)

where H ll and H 22 are defined in (6.15), and

/1
8A.(e) 8¢ (e)
H ll x = X _ '+'_i J_ dx dy = H ll 1 + xie) n " ,
Ol e ) 8x 8x

/1
8A.(e) 8¢(e)
Hlly = y _ '+'_i J_ dx dy = H 1l2 + yi e) H 11 ,
O l e) 8x 8x
(B.I5)
8A.(e) 8¢\e)
H 22x =
/JO (c)
x _ '+'_i
8y
8A.(e)
J_
8y
8¢(e)
dxdy = H 221 + xie) H 22 ,

H 22y =
/J
O (e )
y _ '+'_i
8y
J_
8y
dx dy :::: H 222 + y~e ) H 2 2 ,

where

H '" ~ s. [..',
12 -1 1
-2
2
-1
1
2 -2 '
!,]H'"
12a
~ ~ [!, - 1
- 1 -1
1
1
1 -1
3
,]
~3 '

-']
1 -1 -2 2 1 - 1 -3
1 -1 1 -1

H
22 1
~ -~f 3
12b - 1 -3
-1 - 1
-3 -
3
1
- 1' ] H 222 = ~ [
l '
1
12
12 2
-1 - 2 2
- 2 -1 1
- 2 - 1
1 .
2
(B.16)

7. Bound ary value problems with axial symmetry are governed by

--1 -8
r 8r
C
a ll r -8U) - -8 a22-8U)
8r 8z 8z
C + aDO U - f
A
r, z = 0,
( )
(B.17)
402 B. SPECIAL CASES

where aoo, a
all, and 22 are functions of rand z. The stiffness matrix and the
force vector are given by
(7a) In the case of a linear triangular element n(e):

where A(e) is the area of the element.


(7b) In the case of a bilinear rectangular element n(e),

K~) = all (r~e) H ll + H 1l1 ) + a22 (r~e) H 22 + H 221 ) ,


(B.20)

f]
(e) = a
2
b j( e )
12
{~}
2' (B.21)
1

where H ll and H 22 are defined in (6.15), and H 1ll and H 221 are defined
above in (B.16).
8. In heat transfer problems when qn = /3 (u - u oo ), we have
FOR A LINEAR TRIANGULAR ELEMENT:

K~e) = f3lr;) [~ ; ~], or = /3lj~) [~ ~ 0] ,


12
6 000 6 01

or = -.!£..
/3l(e) [2a aa a1] ; (B.22)
6 1 a 2

f b(e ) _
-
/3u oo l
2
r;){ 0
11 } _ /3uoolj~ {01 }
' or - 2
_ /3uoolk~)
l ' or - 2
{a
1
l'
} (B.23)
403

FOR A BILINEAR RECTANGULAR ELEMENT:

K(e)
b
= f3l~;)6 [i0 0~ 0~~]0 ,or = (3l;~)6 [~0 ~1 ~2 ~]0 '
000 0000

or -
_ (3li%
6
[~0 ~0 ~~]
2 1 ' or -
_ (3l~~6 [~0 0~ ~0 0~] '
(B.24)
0012 1002

f(e) =
b
(3u(x,l~;)
2
{~}
0' or
= (3u oolW
2
{~}
l' or
= (3u ooli%
2
{~}
l '
o 0 1

or ~ ~u~l~l {~ }. (B25)
c
Temporal Approximations

Finite difference schemes to compute the first- and second-order time derivatives
are considered.

C.l. First-Order Derivative

To compute the vector x E R" in the matrix equation

M x + K x = F, 0::; t ::; to, (C.1)

where x = dx]dt, and M and K are known square matrices and F a known
vector in H"; we use the B-scheme, which approximates the mean value of x at
two consecutive time steps t n and t n + l by the weighted average of x at these two
time steps. This scheme is defined by

Xn+!
A
- Xn = BX. n+l + (1 - B) x' n , O::;B::;l , (C.2)
utn + 1

where D.t n+! = t n+! - tn, and the suffix n represents the value of the quantity at
time tn. The weight B refers to some well-known schemes, which are

0, Forward difference scheme,


1, Backward difference scheme,
B= (C.3)
1/2, Crank -Nicolson scheme,
{
2/3, Galerkin scheme .
406 C. TEMPORAL APPROXIMATIONS

Of these schemes, the forward and backward difference schemes are conditionally
stable, while the other two are unconditionally stable.
Substituting the B-approximation (C.2) for times t n and t n+! into Eq (C.1), we
get

which simplifies to
(C.4)
where
M: = M + B~tn+! K,
K = M - (1 - B) ~tn+! K, (C.5)
F = ~tn+! (BF n+! + (1- B) Fn) .
Formula (CA) determines the unknown solution of Eq (C.1) at time t = t n +! in
terms ofthe known solution at t = tn . Since the solution is known at t = 0, we take
t n +! = n ~t, where ~t is an equally spaced time step . Then we start at t n =
and compute the solution for t n +! = ~t . This process is continued successively,
°
forward in time, moving with the time step ~t each time. This is known as the
forward time-marching process. Also, both F nand F n+! are known since the
vector F is known at all times. For better results we use smaller step size ~t.
The stability analysis shows that if A is the minimum eigenvalue of

det(K - >. M) = 0,

then the solution is


(i) stable without oscillations if°<< >.A<<
(ii) stable with oscillations if -1
1,
0, and
(iii) unstable if A < -1.

C.2. Second-Order Derivative

We compute the vector x in the matrix equation

Mx+Kx=F, (C.6)

where x = d2 x/ de, M and B are known square matrices, and F is a known


vector.
C.2. SECOND-ORDER DERIVATIVE 407

Using the Newmark direct integration scheme, we get

..
0: X n+l +(1
- 0:) X.,n =
xn+l - x n
A , (C.7)
utn +1

(3 X..n +1 + (~2 - (3) " -


x., -
Xn+ l -
A2
Xn _ ~
At' (C.S)
U n+ l U n+l

where the parameters 0: and (3 control the stability and accuracy of the solution.
Thus, for example, 0: = 1/2, (3 = 1/4 give an unconditionally stable solution in
linear problems, while 0: = 1/2, (3 = 1/6 give the linear acceleration scheme.
From (C.8) we get

(C. g)

and (C.?) gives


(C.10)

where

(C.lI)

Substituting (C.?) and (e.8) into (C.6), we obtain for t = tn+l

or

(C .12)

The procedure is as follows. First, we use formula (C.12) to compute X n + l ; then


we use formulas (e.9) and (e.IO), in that order, to compute the first and second
derivatives (velocity and acceleration) at time t = tn+l' For preassigned values of
xo, *0, and Xo at t = 0, we solve (e.9) and (C.10) successively, forward in time,
for a prescribed time step tlt, where t n +1 = n tlt, and compute x, X, and for x
time t > O. The increment tlt can be optimized by the condition that the smallest
eigenvalue of det( (1 - >') ao M - K) = 0 is less than 1. The step size tlt can
also be obtained from formula tlt = Tminlrr, where T min is the minimum period
of natural vibrations associated with the problem.
D
Isoparametric Elements

Isoparametric elements are used for regions with curved boundaries using elements
for curved sides .

n.i. Introduction

The isoparametric concept is based on the principle of mapping a 'reference'


e,
element in the 1J-planeonto the curvilinear element in the x y-plane, the sides of
which pass through the selected nodes. Care must be taken to ensure that no gaps
occur between the adjacent distorted elements, which means that the interpolation
functions must satisfy the required continuity conditions.
D.l.l. Local e-System. The transformation from the global coordinate
system x to a local system ~, when the origin is at the center of an element and
scaled such that ~ = -1 is at the left-end node and ~ = 1 at the right-end node, is
given by (see Fig . D.I)

2x - (x~e) + x~e» )
e= l( e) (D .1)

e
The -coordinates are called normal (or natural) coordinates since these coordinates
are normalized (nondimensionalized) with values between - 1 and 1. The formula
(D. I ) establishes the transformation between points x (x~e) ~ X ~ x~e » ) and the
points e(-1 ~ ~ ~ 1).
The (Lagrange) interpolation functions for a linear element in normal coordi -
nate system are
410 D. ISOPARAMETRIC ELEMENTS

•I bs •2 (D.2a)
2 - node element
(linear)

•I cs
2

3
(D.2b)
3 - node element
(quadratic)

9
¢l = --(1 - e)(1/9 -
16
e)
2

¢2 = ~~ (1 - eH1/3 - e)
(D.2c)
¢3 = ~~ (1 - eH1/3 + e)
9 2
¢4 = -16 (1 + eH1/9 - e ).

Fig. D.l. Linear Elements in the Normal Coordinate System.

Note that the interpolation functions ¢i (ej) are chosen such that

I , ifi = j
¢i(ej) = Dij = { a ·f ·...t·
, I t T J,

whereej denotes the e-coordinate ofthej-th node of the element, ¢i (i = 1, ... , n)


are polynomials of degree n - 1 (n being the number of nodes in the element), and
Dij is the Kronecker delta.
Another method of constructing the functions ¢i (ej) is as follows. Form the
product of n - 1 linear functions e - ej 0 = 1, . . . , i-I, i + 1, .. . , n; j :f- i),
i.e.,

Here ¢i = a at all nodes except the node i. Now, determine Ci such that ¢i = 1 at
e= ei, i.e.,

This gives the required interpolation functions ¢i associated with the node i as
D.l. INTRODUCTION 411

Formula (D.3) can be used to derive the interpolation functions (D.2a), (D.2b), and
(D.2c). For example, to derive (D.2c), note that 6 = -1,6 = -1/3,6 = 1/3,
and e4 = 1. Then, substituting these values, we obtain

D .1.2. Triangular Elements. The following cases are mostly used.

1 2
(a) (c)

Fig. D.2. Three 2-D Triangular Elements.

Fig. D.2(a) :
At the corner nodes : <Pi = ei, i=I,2,3.
Fig . D.2(b):
At the corner nodes: <Pi = (2ei - 1)ei, i = 1,2,3.
j = 4,5,6
At the mid-nodes: <Pi = 4ekel, k = 1, 2, 3 .
{
1= 2,3,1
Fig. D.2(c):
1
At the corner nodes: 2"(3ei - 1)(3ei - 2)ei, i = 1,2,3.
On sides i = 4, 5, 6, 7, 8, 9:

k ~ 1, 2, 2, 3, 3, 1 .
{ I - 2, 1, 3, 2, 1, 3

At the center node i = 10 and <P1O = 27666.


412 D. ISOPARAMETRIC ELEMENTS

D.1.3. Rectangular Elements. We present only those cases that are


most frequently used.

FIRST-ORDER CONTINUITY FUNCTIONS. Refer to Fig. D.3. Here e, TJ are


dimensionless coordinates, defined by

e= -(x
1
a
- xc), (D.4)

with limits ±1. Then

a= 1a a= 1a (D.5)
ax ~a( ay b aTJ'
and

JJr
r
R
f(x , y) dA = 4ab rr -1 -1
f(e, TJ) de dTJ. (D.6)

U2 u.
(- 1,1) .-"------+----_ (1,1)

oL.------+--+

Y (~:-1)-----------e(~,~1)
o
j x

Fig. D.3. Rectangular Element.

LINEAR FUNCTIONS . The simplest model for a rectangle has only corner
nodal unknowns (thus only 4 parameters), which means that the function varies
linearly on the boundaries. Appropriate 2-D interpolation functions for u can be
generated by evaluating

(D.7)

at the nodes and solving for Cl, C2 , C3, C4 , which gives

Ul = Cl + C2 + C3 + C4 at (1, I) ,
U2 = Cl - C2 + C3 - C4 at (-1, I),
U3 = Cl - C2 - C3 + C4 at (-1, -I),
U4 = Cl + C2 - C3 - C4 at (1, -I),
D.l. INTRODUCTION 413

which, on solving for CI, C2, C3, C4, gives

UI + U2 + U3 + U4 UI - U2 - U3 + U4
CI = 4 C2 =-
4
UI + U2 - U3 - U4 UI - U2 + U3 - U4
C3 =-
4
C4 = 4
thus yielding

U = UI + U2 + U3 + U4 _ UI - U2 - U3 + U4 ~
4 4
UI + U2 - U3 - U4 UI - U2 + U3 - U4 c
- 4 TJ + 4 <"TJ
= <PIUI + <P2 U2 + <P3 U3 + <P4 U4,
where
1
<PI = 4(1 - ~)(1 - TJ),
1
<P2 = 4(1 + ~)(1 - TJ),
(D.8)
1
<P3 = 4(1 + ~)(1 + TJ),
1
<P4 = 4(1 - ~)(1 + TJ)·

QUADRATIC FUNCTIONS . A refined rectangular elementis obtained by tak-


ing a quadraticvariation of the function u. This model improves upon the simplest
(linear)rectangleof the previous section. This requiresan additional interiornode
on each side, thus having a total of 8 nodes (see Fig. D.4).

4 7 3

8 6

5 2

Fig. D.4. An 8-ElementRectangle.

We take
414 D. ISOPARAMETRIC ELEMENTS

which , when solved for the Ci , i = 1, ... , 9, at the 8 nodes, gives

At node I: Ul = Cl - C2 - C3 + C4 + Cs + C6 - C7 - CS,
At node 2: U2 = Cl + C2 - C3 + C4 - Cs + C6 - C7 + cs,

At node 3: U3 = Cl + C2 + C3 + C4 + Cs + C6 + C7 + CS ,
At node 4: U4 = Cl - C2 + C3 + C4 - Cs + C6 + C7 - CS,

At node 5: Us = Cl - C3 + C6,
At node 6: U6 = Cl + C2 + C4,

At node 7: U7 = ci + C3 + C6,
At node 8: Us = Cl - C2 + C4·

On solving for Ci, we get


s
U = L¢iUi'
i= l

where
1 1 2
¢ l = -4 (1 - €) (1 - 7])(1 + € + 7]), ¢s = '2(1 - € )(1 - 7]),
1 1
¢2 = -4(1 + 0(1 - 7])(1 + € + 7]) , ¢6 = '2(1 + € )(1 - 7]2),
(D.10)
1 1
¢7 = '2(1 - € )(1 + 7]) ,
2
¢3 = - 4 (1 + €)(1 + 7])(1 - € - 7]) ,
1
¢4 = -4(1 - €)(1 + 7])(1 + € + 7]) , ¢s = ~(1 - €)(1 - 7]2).
2

D.2. Curvilinear Coordinates

To find the transformation from straight to curved sides, we express x , y in terms


of curvilinear coordinates as

The choice of € , 7] depends on the element geometry, i.e., linear, quadratic, cubic,
or a higher dimension (see Fig. D.5).
Generate the interpolation functions in terms of € , 7] and evaluate curvilinear
derivatives as follows:
D.2. CURVILINEAR COORDINATES 415

The general transformation for a function ¢ is given by

8¢ 8¢ 8~ 8¢ 8T!
-=--+--,
8x 8~ 8x 8T! 8x
(D.ll)
8¢ 8¢ 8~ 8¢ 8T!
-=--+--.
8y 8~ 8y 8T! 8y

3
4 3 4

11
11 ~aster Element
2

o ~
4 7 3

8 6

(x2' Y2 )
11 1 5 2 Y ( )
xI'YI (Xs'Ys)
~aster Element ~urved-sideElement
o ~ o x

Fig. D.5. An 8-Element Curvilinear Rectangle .

However, explicit expressions for~, T] in terms of x , yare not easily available . So


we consider
¢ = ¢(x,y) .
Then, using formulas for 8¢/8~ and 8¢/8T], similar to those in (D.lI), we get

{f!}=J{U}'
where J is the Jacobian matrix given by
8x
J = 8(x ,y) = 8{
8(~ ,T!) [ 8x
8t]

On inversion we get

~} --
{ f!.!E.
8y
J-
1
{*} f!.!E.
8t]
'
416 D. ISOPARAMETRIC ELEMENTS

which gives

(D .12)

where IJI denotes the determinant of J. Note that PI must be nonzero and finite
for the transformation to be unique. Also,

(D.13)

If PI #- 0, we have
By
BfJ (D.14)
[ By
B~

where ~ = ~(x, y) and fJ = fJ(x, y). It is obvious from (D.14) that the functions
~(x, y) and fJ(x, y) are continuous and differentiable. It is also required that the
transformation x = x(~, fJ). y = y(~ , fJ) must be algebraically simple. This would
allow us to evaluate the matrix J easily.

D.2.1. One-Dimensional Integrals. For one-dimensional integrals of


the type

l
Xl
XN
f(x) dx = 1 g(~)
1

-1
d (x(~))
d~
d~, (D.15)

where d(x(~)) =J let


d~ ,

(D.16)

where Xl, . . . , X N are the global nodal coordinates. For example, for a 2-node
element n(e), let

where the shape functions <PI and <Pz are given in (D.2a) . Then

(D .17)
D.2. CURVILINEAR COORDINATES 417

Then the new integral is

11
l
X2
l(e)
f(x) dx = - g(~) d~ . (D.18)
Xl 2 -1

D.2.2. Two-Dimensional Integrals. Consider the double integral

(D.19)

where the new variables (~ , TJ) are the natural coordinates, and the integral

(D.20)

where the new variables are the triangular coordinates 6,6 (see §5.3). For each
coordinate system the Jacobian of the transformation is J, i.e.,

Y]
8x 8y ] [ 8x 8
8~ 8~ = J = 86 86 . (D.21)
[ -8x 8y
-
8x
--
8y
8TJ 8TJ 86 86
Now, in the case of a linear triangular element the transformation equations are

x (6,6) = <I>d6 , 6) Xl + <I>2 (6,6) X2 + <I>3(6 ,6) X3,


(D.22)
Y(~1 ,6) = <I>l (~1,6) Yl + <I>2 (~1,6) Y2 + <I>3(6,6) Y3,

where (Xl, Yl), (X2, Y2), (X3, Y3) are the global nodal coordinates. If we replace
the shape functions by their triangular coordinate equivalents, we get

x (6 ,6 ) = 6 X l +6 X 2 +6 X 3 ,
(D.23)
Y (6,6) = 6 Yl + 6 Y2 + (1 - 6 - 6) Y3,

because 6 +6 + 6 = 1. Then the Jacobian J is

(D.24)

This method can be extended to bilinear quadrilateral and other elements.


EXAMPLE D .1. For a bilinear quadrilateral element n( e) shown in Fig . D.6,
the transformation is given by

X = <I>l(~ , TJ) Xl + <I>2(~, TJ) X2 + <I>3(~, TJ) X3 + <I>4(~, TJ) X4 ,


(D.25)
Y= <I>l(~ , TJ) Yl + <I>2(~ , TJ) Y2 + <I>3(~ , TJ) Y3 + <I>4(~ , TJ) Y4,
418 D. ISOPARAMETRIC ELEMENTS

where <Pi, i = 1,2,3,4, are given by (D.8). Taking ~ = 1/2 = TJ, the Jacobian is

= ~ [-(1 - TJ) 1 - TJ 1 + TJ -(1 + TJ)]


Xl Yl1
X2 Y2
4 -(1 - ~) -(1 +~) 1+~ 1- ~ [ X3 Y3
X4 Y4

and thus, IJI = 9/2.•

3
(10,7)

-+----+-----l--4----+--l----1---l--l----1--+-_x
o
Fig. D.6. A Linear Quadrilateral Element.

D.3. Pascal Triangle

After a suitable subdivision of a region into finite elements is made, the next choice
is to represent the element approximation in terms of variational parameters. In
view of the Weierstrass approximation theorem, polynomial approximation in each
element is the choice of such an approximation. Moreover, polynomials are easy
to work with, both algebraically and computationally. The following guidelines
must be followed in the choice of a suitable polynomial.
D.3. PASCAL TRIANGLE 419

1. The number of terms in the polynomial must be equal to the total number
of degrees of freedom (dof) associated with the element. For example, a triangle
with 3 nodes, one dof at each node, requires u = Cl + CZX + C3Y. Again, an
element with 4 nodes and 2 dof at each node will need an 8-parameter polynomial
U = Cl + CZX + C3Y + C4XZ + C5XY + C6Y z + C7XZy + cSxyz.
2. The approximation must have geometric invariance, i.e., there should be no
preference for either x or y direction .
3. The choice of the approximation must enable the element to reproduce
rigid body motions and stress-strain relation precisely, i.e., the convergence of the
method must be required.
4. Higher-order terms must not be retained at the expense of the lower order
terms . Thus choose complete polynomials, i.e., polynomials in which all terms up
to any given degree are present.
The terms of successive degrees of polynomials are conveniently represented
by the Pascal Triangle (Fig. D.?). It can be used to write complete polynomials
or to choose number of terms equal to the dof of any element. Its form up to a
lO-th degree polynomial is given below:

1
x Y
xZ xy yZ
x3 xZy xyz y3
x4 x 3y xZyz xy3 y4
x5 x 4y x 3yz x Zy3 xy4 y5
x6 x 5y x 4yz x 3y3 x Zy4 xy5 y6

Fig. D.? Pascal Triangle.


E
Green's Identities

Let thefunctions M(x , y, z), N(x, y, z), and P(x, y, z) be the components of the
vector G in R 3 , where (x, y, z) is a point in O. Then, by the divergence theorem

Jrrr
JJo.
(BM + BN + BP) dn
Bx By Bz
(E.1)
= J~o.[Mcos(n,x)+Ncos(n,y)+pcos(n,z)] ss,

where dB denotes the surface element, Bn denotes the boundary ofn, and cos (n, x),
cos(n, y), and cos(n, z) the direction cosines of n. If we take M = u ~~,
Bv Bv .
N = u By' and P = u Bz' then (E.1) yields

JJrrl« (BUBx Bxav + auBy ByBv + auBz BzBV)


r dn = JrJanr u BnBv as _ JfrJo.ruV2vdn ,
(E .2)
which is known as Green's first identity. Moreover, if we interchange u and v in
(E.!), we get

dO fr r v Bu es _ ffr r vV 2udn .
ffJo.r (BUBx BxBv + BuBy BvBy + BuBz BV)
r
Bz
=
Jan Bn Jo.
(E .3)
If we subtract (E.2) from (E.3), we obtain Green's second identity:
422 E. GREEN 'S IDENTITIES

whichis alsoknown as Green's reciprocity theorem. Thisresultalsoholds in R2.


Note that Green's identities are valid even if the domain n is boundedby finitely
manyclosedsurfaces; however, in that case the surfaceintegrals must be evaluated
over all surfacesthat makethe boundary of n, and in R 2 the line integrals must be
evaluated over all paths that make the boundary of n.
F
Gaussian Quadrature

Gaus sian quadrature is a very powerful method for numerical integration, which
uses unequally spaced intervals. It is defined as follows :

1a
b
f( x) dx = L
i=1
N
Wi f (Xi)' (F.l)

where
a + b + (b - a)~i
Xi = 2 (F .2)

Here ~i are known as the Gauss points on the interval (-1,1), which are the zeros
of the Legendre polynomial Pn (x) of degree n in the interval ( -1 , 1). Although
tables for the Gauss points and the weights W i are readily available (see, e.g.,
Abramowitz and Stegun 1968), they can be easily generated by Mathematica func-
tion LegendreP [n , x} , For example , the first four Legendre polynomials are:
1 1
Po(x) = 1, P1(x) = X, P2(x) = 2" (3x2 - 1), and P3( x) = 2" (5x 3 - 3x) .
These polynomials of degree n can also be obtained from Rodrigues' formula
n
Pn(x) = -n- I
1 d (2
-d x - I )n , (F .3)
2 n. xn
or from the recurrence relation

(n + I)Pn+ 1 (x) - (2n + 1) X Pn(x) + nPn- 1 (x) = O. (FA)

The orthogonality and normalization relations with the unit weight function are

=J m ,
1
1 { 0 ifn
Pn(x)Pm(x) dx = 2 . (F.5)
-1 -- if n ee rn.
2n+ 1
424 F. GAUSSIAN QUADRATURE

The zeros of each polynomial Pn(x) are real and distinct, and they are located in
the interval (-1,1). The weights Wi and the corresponding Gauss points ~i for
N = 1,2,3,4 are given in Table F.l.
Table F.I . Data for Gaussian Quadrature.

N
1 o 2.0000000000
2 ±0.5773502692 = ±1/J3 1.0000000000
3 o 0.8888888889
±0.7745966692 = ±/375 0.5555555556
4 ±0.3399810436 0.6521451549
±0.8611363116 0.3478548451

If f is a polynomial of degree n, then formula (F.l) is exact if N is chosen to


be 2n + 1.
EXAMPLE F .1. Use the Gaussian quadrature with N = 1,2,3, to compute

1 =
J 1
3 dx
--4
3x +
= -1 In (13)
3
- = 0.206346 (exact value).
7

Set x = 2 + u. Then
1 -11
- -1 3u
du
+ 10 '

Now,forN=l : 1=2(11
0)=0.2;
for N = 2: 1 = [10 _ 3(0.5~73502692) + 10 + 3(0.5~73502692)] = 0.206186;
for N = 3: 1 =9
1[ 5 5]
10 _ 3(0 .7745966692) + 10 + 3(0.7745966692)
= 0.206342.•

EXAMPLE F .2 . Use the Gaussian quadrature with N = 4 to compute

l
1r / 2 2
1 = x 2 cosxdx = ~ - 2 = 0.4674011 (exact value).
o 4

1l"/2+0 1l"/2-0 1l"


Here, Xi = 2 + 2 = "4 (1 + ~i) ' Thus,
1l" 1l"
Xl =- + 6) = - (1 -
(1 0.8611363116) = 0.109063,
4 4
"4 (1 + 6) = "41l" (1 -
1l"
X2 = 0.3399810436) = 0.518378,
425

11" 11"
X3 = 4" (1 + 6) = 4" (1 + 0.3399810436) = 1.05242,
11" 11"
X4 = 4" (1 + ~4) = 4" (1 + 0.8611363116) = 1.46173.
Then
f (xd = xi COSXl = 0.0118241,
f (xz) = xi cos Xz = 0.233413,
f (X3) = xi cos z , = 0.548777,
f (X4) = xi COSX4 = 0.23257,
and

I = ~ [WI f (xd + Wz f (xz) + W3 f (X3) + W4 f (X4) ]


= ~ [(0.3478548451)(0.0118241) + (0.6521451549)(0.233413)
+ (0.6521451549)(0.548777) + (0.3478548451)(0.23257)]
= 0.467402. •

Now, we consider the double integrals on quadrilateral and triangular regions .


QUADRILATERAL REGIONS. The double integral

(F.6)

over a rectangular region {(x , y) : -1 ::; x, y ::; I} is numerically evaluated first


by computing the inner integral, and then computer the outer integral , in the same
manner as in §F.l. Thus, evaluating the inner integral we get

1111 lIN
f(x, y) dx = ~ Wi f (~i) 11
1
g(y) dy, (F.7)

where Wi and ~i are the weight and the Gauss points, respectively. The outer
integral yields

(F .8)

where W j and Tlj denote the weight and the Gauss points, respectively . Hence,
combining (F.7) and (F.8) we obtain

where the values of M and N are obtained by equating (2m - 1) to the highest
power of x and (2n-1) to the highest power of y in the integrand f(x, y) . Formula
426 F . GAUSSIAN QUADRATURE

(F.9) can be viewed as a single sum over M x N = K Gauss points with wiwrtype
products as the corresponding weights .
TRIANGULAR REGIONS. The integral I6 f(x, y) dxdy over a triangular
region t:::, is

Lf(x,y)dXdy = l l ll
- {2 g (6 ,6 ) 1IJl l d6 dx 2
N (F.1O)
= L W i 9 ((6 )i, (6 )i) .
i=l

The Guass points and the weights in Formula (F.lO) are defined in Table F.2 (see
Hammer et al., 1956, for details) .

Table F.2. Gauss Points and Weights in a Triangular Region .

wI· Error

2 a (1/3, 1/3) 112 ois,

a (112,0) 1/6
3 b (1/2, 112) 1/6 O(h 2)
c (0, 112) 116

a (113, 113) 0.11250


b (a, ~) 0.0661917
c (~ , ~) 0.0661917
d (~, a) 0.0661917 O(h 6)
6
e (y, y) 0.0629696
f (0, y) 0.0629696
g (y,o) 0.0629696

In the above table, a = 0.0597159, f3 = 0.470142, 'Y = 0.101287, 8


0.797427, and (6, 6) denote the triangular coordinates, and h is the step size.

EXAMPLE F .3. Compute (a) I= [11 [11 6x y dy dx; 2

(b) 1= iot' ir":


o 6~~ d6 d6 ·
427

SOLUTION . (a) For N = 2. we have

2 2 2 2
1= L L WiWj! (~i)! (~j) = LL WiWj 6~;TJJ
i = l j=l i=l j=l

= 6 . 2 [(0.5773502692)2 + (0.5773502692)2] = 2.666666663.

The exact value is 8/3 = 2.666666667 .


(b) For N = 6, we have

I =(0.1125)G·~) + 0.0661917 [(0.0597159)(0.470142)2 + (0.470142) 3


+ (0.470142)(0.0597159)2) + 0.0629696 [(0.101287)3
+ (0.797427)(0.101287)2 + (0.101287)(0.797427)2]
= 0.0166661. •
G
Gradient-Based Methods

In this appendix we will show some gradient-based numerical methods for finding
minimizers of functions from R N to R with no constraints.

G.!. Method of Steepest Descent

The simplest of the gradient-based methods is the method of steepest descent,


which is based on the fact that if f : R N -4 R has all first-order derivatives, then
its value decreases the fastest along the direction determined by the negative of

the gradient, i.e., the fastest descent direction is -1I~j~:~ II at any point x in R N .
Suppose that f has a minimizer x* at which \7f(x*) = 0 holds necessarily. To
find x" , we take an initial guess x(1) and calculate \7f(x(1)) . If\7f(x(l)) i=- 0,
t \7f(X(l))
let t denote any real number. Then x(t) = x(l) - II\7f(x)11 is a parametric
equation of the straight line passing through X(l) in the direction determined by
\7f(x(l )) Th
-11\7f(x) II'
..
e restnction 0
i ] on thi1S I'me 1S
, 'I . bl f .
a sing e vana e unction

_ (1) t \7f(x(l))
get) - f(x ) - II\7f(x(l))II'

Suppose that t(l) is a real number at which g(t(l)) = ming(t) . Let


tER

(2) _ (1) t(l)\7f(x(l))


x - x - 11\7f(x(l))11 '
430 C. GRADIENT-BASED METHODS

Then, by definition, j(x{1)) > j(X(2)), since g(t{1)) < g(O) . If we repeat this
process with a new 'initial guess' x(2) and if we again get V j(x(2)) =I- 0, then
. generates a sequence 0 f pom
It . R N : x (1) , x (2) , ... ,x (k) , ... ,whiICh satiISfy
. ts in
j(X(l)) > j(X(2)) > ... > j(x(k)) > ... , Under appropriate conditions on I,
we will have lirnk_oo x(k) = x*.
EXAMPLE G .1. Solve the unconstrained program

Minimize j(X1,X2)=2xi+x~-2x1X2-X1-X2, (Xl ,X2)ER2, (G.1)

by using the method of steepest descent. Here we have

For simplicity, let the initial guess be x(O) = [0 OIT. Then

Vj(O, 0) = [-1 _l]T,


Vj~,~ T
- II\7j(O,O)1I = [-0.7071 0.7071] ,
1
g(t) = j(O + O. 707lt, 0 + 0.7071 t) = '2 t 2 - 1.4142 t + 1.
Solving g'(t(O)) = t(O) - 1.4142 = 0, we get t(O) = 1.4142. Thus,

x(l) = [0 O]T + 1.4142 [0.7071 0.7071t = [1.0 LOt.

Repeating this precess, we obtain

_ \7j(1.0,1.0) = [-0.7071 0.7071f


11\7 j(1.0, 1.0) II '
g(t) = j(1.0 - 0.7071 t, 1.0 + 0.7071 t) = 2.5 t 2 - 1.4142 t,
g'(t) = 5t -1.41421.
Solving g'(t{1)) = 0, we get ttl) = 0.2828, and

X(2) = [1.0 1.ojT + 0.2828 [-0.7080 0.70631 T = [0.8 1.2]T.

Similarly,

X(3) = [0.8 1.2jT + 0.2828 [0.7071 0.7071jT = [1.0 1.4]T.

The exact solution is found by directly solving


G .2. CONJUGATE GRADIENT METHOD 431

which gives [xi x;]T = [1.0 1.5]T. It can be easily checked that after one
more iteration, we have X(4) = x* .•

G.2. Conjugate Gradient Method

A faster way of solving an equation of the type (G.l) is the generalized conjugate
gradient method. It can be stated as follows. Let x(O) be an arbitrary initial guess,
let r(O) = - \1 f(x(O)), and y(O) = r(O) . Let t(O) be the solution of min f(x(O) +
tER
ty(O)), and let

X(l) = x(O) + t(O)y(O),


(0) _ [r(1) - r(O)]T r(l)
r(l) = -\1 f(x(1)), 7 - [r(O)]T r(O) ,

y(l) = r(1) + 7(0)y(0).

Repeat this process with t(1) as the solution of min f(x(1) + t y(1)). When this
tER
method is applied to aquadraticfunction f(x) = ~xT A x-xTb, with the gradient
\1 f(x) = Ax - b, the method is called the linear conjugate gradient method, or
simply the conjugate gradient method .
EXAMPLE G .2. Solve the unconstrained program

Minimize f(XI,X2) = 2xi + x~ - 2XIX2 - Xl - X2, (XI,X2)


2
E R , (G.2)

by using the conjugate gradient method . The first step is the same as in the method
of steepest descent. We take the same initial guess x(O) = [0 O]T as in the
previous example. Then

r(O) = y(O) = _ \1 f(O, 0) = [07071 07071]T


II\1f(O,O)II' . ,

and

X(l) = [0 O]T + 1.4142 [0.7071 0.7071]T = [1.0 1.0]T,

r(l) = _ nf(x(1)) = _ \1 f(1.0, 1.0) = [-0.7071 0 7071]T


v II\1f(1.0,1.0)11 .,
[r(1) r(O)JT r(l)
7 (0) - - -10
- [r(O)]T r(O) -.,

y(l) = r(l) + 7(0)y(0) = [0 1.4142]T,


g(t) = f(1.0, 1.0 + 1.4142 t) = 2t 2 - 1.4142 t - 1.
432 c. GRADIENT-BASED METHODS

Solving g'(t(1)) = 0, we get t(1) = 0.3535, and

X(2 ) = [1.0 1.0f + 0.3535 [0 1.4142]T = [1.0 1.~9991T.

It can be easily seenthat after two iterations, this methodproduces a more accurate
approximation of the exact solution as comparedwith the previous method. •
Bibliography

M. Abramowitz and I. A. Stegun (eds.), Handbook of Mathematical Functions with For-


mulas, Graphs and Mathematical Tables, National Bureau of Standards, Appl. Math.
Series 55, U.S. Govt. Printing Office, Washington, DC; reprinted by Dover , New
York, 1965.
E. E. Agur and J. Vlachopoulos, Heat transfer to molten polymer flow in tubes, J . Appl.
Polm . Sci., 26 (19S1), 765-773.
J. Ed Akin , Finite Element Analysis for Undergraduates , Academic Press, New York,
19S6.
O. Axelsson and V. A. Barker, Finite Element Solution of Boundary Value Problems ,
Academic Press, Orlando, FL , 19S4.
G . K. Batchelor, An Introduction to Fluid Dynamics, Cambridge University Press, Cam-
bridge, 1967.
R. Bellman, Introduction to Matrix Analysis, 2nd ed ., McGraw-Hill, New York, 1970.
E. Bernhardt , G. Bertacchi, and A. Moroni , Modeling of flow in extruder dies-Funda-
mentals and Applications of the TMconcept-faBest finite element flow analysis , in
Applications of Computer Modeling of Extrusion and Other Continuous Polymer
Processes (Keith T . O'Brien, eds .), Oxford Un iversity Press, Munich, 1992.
R. B. Bird, R. Armstrong and O. Hassager, Dynamics of Polymeric Fluids, Vol. 1. Fluid
Mechanics, Wiley, New York , 1976.
D. V. Boger , A. Cabelli, and A. L. Halmos, The behavior of a power-law fluid flowing
through a sudden expansion, AIChE Journal, 21 (1975) , 540-549.
E . K. Bruch, The Boundary Element Method for Groundwater Flow, Lecture Notes in
Engg., vol. 70, Springer-Verlag, Berlin, 1991.
R. 1. Burden and J . Douglas Faires, Numerical Analysis, Academic Press, New York ,
1997.
C. R. Ca lladine, Plasticity For Engineers, Wiley, New York, 19S5.
B. Carnaham, H. A. Luther, and J . O. Wilkes , Applied Numerical Methods , Wiley , New
York ,1969 .
H. S. Carslaw and J . C. Jaeger, Conduction of Heat in Solids, 2nd ed., Oxford University
Press, New York , 1959.
Y . A. Cengel, Heat Transfer, WCB McGraw-Hill, Boston, MA, 1995.
R. P. Chhabra and J . F. Richardson, Non-Newtonian Flow in the Process Industries-
434 BIBLIOGRAPHY

Fundamentals and Engineering Applications, Butterworth-Heinemann, London, 1999.


R. W. Clough , The finite element method in plane stress analysis, Proc. 2nd ASCE
Conf. Electronic Comput. (Sept. 1960), Pittsburgh, PA, 345-378.
___ , The finite element method after twenty-five years: A Personal View, Computers
and Structures, 12 (1980), 361-370.
J . J . Connor and C. A. Brebbia, Finite Element Techniques for Structural Engin eers,
Butterworth, London, 1973.
___ , Finite Element Techniques for Fluid Flow, Butterworth, London , 1976.
R. Courant , Variational methods for the solution of problems of equilibrium and vibra-
tions , Bull. Amer. Math. Soc., 49 (1943), 1-23.
___ , Differential and Integral Calculus, vol. 1, 2, Interscience, New York , 1964, 1965.
___ and D. Hilbert, Methods of Mathematical Physics , vol. 1, Interscience, New York ,
1952.
Y. H. Dai and Y. Yuan , Nonlinear Conjugate Gradient Method s, Shanghai Scientific and
Technology Publisher, 1999.
_ __ a nd Y. Yuan , A nonlinear conjugate gradient with a strong global convergence,
SIAM J . Optim., 10 (2000) , 177-182.
Y. H. Dai et aI., Testing different nonlinear conjugate gradient method s, Research Re-
port , Institute of Computational Mathematics and Scientific/Engineering Comput-
ing, Chines e Acad emy of Sciences, 1998.
___ and Y. Yuan , A nonlinear conjugate gradient with a strong global convergence,
SIAM J . of Optim. , 10 (2000) , 177-182.
A. J. Davies, Th e Finite Elem ent Method , Clarendon Press, Oxford , 1980.
A. M. J . Davis and T .-Z. Mai, Steady pressure-driven non-Newtonian flow in a partially
filled pipe, J . Non-Newtonian Fluid Mech., 41 (1991),81-100.
A. 1. Deak and T . H. Pian, Applications of the smooth surface interpolation to finite
element analysis , AlA Aerospace J . (Jan. 1967), 187-189 .
B. F . de Veubeke, Upper and lower bounds in matrix structural analysis, in Matrix
Methods of Structural Analysis (B. F . de Veubeke, 00.), Pergamon, New York, 1964,
pp . 165-201.
P. England and J . Jackson , Activ e deformation of the continents, Ann . Rev. Earth
Planet . ScL, 11 (1989) , 197-226.
___ a nd D. McKenzie, A thin viscous sheet model for continental deformation , Gee-
phys . J . R. Astr. Soc., 10 (1983), 295-321.
___ and D. McKenzie, Correction to: A thin viscous sheet model for contin ental
deformation, Geophys . J. R. Astr. Soc., 13 (1983), 523-532.
J . L. Fastook, The finite-el ement method for solving conservat ion equation s in glaciology,
Computatio nal Science and Engineering, 1 (1993) , 55-67.
B. A. Finlayson , The Method of Weighted Residual s and Variational Principl es, Aca-
dem ic Press, New York, 1972.
A. F . Flores, J . C. Gottifredi, G.V. Morales, and O. D. Quiroga, Heat transfer to power-
law fluids flo wing in tubes and flat ducts with viscous heat generation, Chemical
Engineering Science, 46 (1991), 1385-139 2.
Flu ent , Inc., Fidap Theory Manual, Fluent, Inc., 1998.
K. O. Friedrichs, A fini te difference scheme for the Neumann and the Diri chlet problems,
NYO-9760 , Courant Institute of Mathematical Science, New York University, New
York, 1962.
C. E. Frob erg, Introduction to Num erical Analysis, Addison-Wesley, Reading, MA, 1969.
P. Garrity, A finit e elem ent model for vortex-flow temperature separation of a compress-
ible fluid, Final Homework Project Math 6230, University of New Orleans (Private
Communicati on) (2000), 1-7 .
BIBLIOGRAPHY 435

P. N. Godbole, S. Nakazawa, and O. C. Zienkiewicz, Blood flow analysis by the fin ite
element method, International Conference on Finite Elements in Biomechanics, Feb-
ruary 18-20, 1980.
G . L. Goudreau and R. L. Taylor, Evaluation of numerical integration m ethods in elas-
todynamics, J . Computer Methods in Appl. Mech . Engg. , 2 (1973),69-97.
J . Greenstadt, On the reduction of continuous problems to discrete form , IBM J . Res .
Div . 3 (1959), 355-363.
P. C. Hammer, O. P. Marlowe, and A. H. Stroud, Numerical Integration over Simplexes
and Cones , in Mathematical Tables and Aids to Computation, National Research
Council, Washington, DC, vol. 10, 1956, pp. 130-137.
J. Hersch, Equations differentielles et functions de cellules, C. R. Acad. Sci. Paris, 240
(1955), 1602-1604.
M. A. Hussain, S. Kar, and R.R. Puniyani, Relationship between power law coefficients
and major blood constituents affecting the whole blood viscosity, Journal of Bio-
Sciences, 24 (1999) , 329-337.
M. Iga and J . N. Reddy, Penalty finite element analysis of free surface flows of power-law
fluids. The mathematics of finite elements and applications" VI, Uxbridge (1987),
Academic Press, London , 423-433(1988) , 76-80.
E . Isaacson and H. B. Keller , Analysis of Numerical Methods , Wiley , New York, 1966.
W . Janna, Introduction to Fluid Mechanics , PWS, Boston, 1993.
___ , Design of Fluid Thermal Systems, PWS, Boston , 1993.
R. E. Jones, A generalization of the direct-stiffness method of structural analysis, AIAA
J., 2 (1964) , 821-826.
S. Kakac and Y. Yener, Heat Conduction, Srd ed., Taylor and Francis, Washington, D.
C ., 1993.
L. V. Kantorovich and V . L. Krylov, Approximate Methods of Higher Analysis, Inter-
science, New York, 1958.
H. Kardestuncer and D. H. Norrie (eds .), Finite Element Handbook, McGraw-Hill , New
York , 1987.
W . M. Khairy , Partially filled pipe flow, Final Homework Project Math 6230 , University
of New Orleans (Private Communication) (April 1998), 1-29.
___ , Flow past a cylinder in viscous flow, Final Exam Project Math 6230, Universi ty
of New Orleans (Private Communication) (May 1998) , 1-91.
K. Khellaf and G . Lauriat , A new analyti cal solution for heat transfer in the entrance
region of ducts : Hydrodynamically developed flows of power-law fluids with constant
wall temperature, Int. J . Heat Mass Transfer, 40 (1996), 3443-3447 .
R. D. Krieg, Unconditional stability in numerical time integration methods, Trans.
ASME, J. Appl. Mech . 40 (1973), 417-421.
N. M. Newmark, A method of com putation for structural dynamics , J . Engg. Mech .
Division, ASCE 85 (1959) , 67-94.
M. Krizek and P. Neittaanrnaki, Finite Element Approximation of Variational Problems
and Applications, Longman Scientific & Tecnical, Harlow, U.K. , 1990.
A. Kumar and M. Bhattacharya, Numerical analysis of aseptic processing of a n on-
Newtonian liquid food in a tubular heat exchanger, Chern. Eng. Comm., 103 (1991) ,
27-51.
P. K. Kythe, An Introduction to Boundary Element Methods, CRC Press, Boca Raton ,
FL , 1995.
___ , Fundamental Solutions for Differential Operators and Applications, Birkhauser ,
Boston, 1996.
___ , P. Puri, and M. R. Schaferkotter, Partial Differential Equations and Boundary
Value Problems with Mathematica, CRC Press, Boca Raton, 2002.
436 BIBLIOGRAPHY

J . E. Lay, An experimental and analytical study oj vortex-flow temperature separation


by superposition oj spiral and axial flow, Part 2, ASME J. Heat Transfer, 81 (1959) ,
213-222.
N. N. Lebedev, I. P. Skalskaya, and Y. S. Uflyand , Worked Problems in Applied Mathe-
matics, Dover, New York, 1965.
L. Lefton and D. Wei, Finite element solutions [or pressure driven flow oj power law
fluid in pipes oj noncircular cross section, private communication, presented at Soc.
of Engg ., 32nd Annual Tech. Meeting, SES '95, New Orleans, LA, 29 Oct - 2 Nov.,
1995.
M. J . Lighthill, Mathematical Biofluiddynamics, SIAM, Philadelphia, PA, 1975.
T. J. Liu, H. M. Lin , and C. N. Hong , Comparison oj two numerical methods [or the
solution oj non-Newtonian flow in ducts, Int . J . Numer. Methods Fluids, 8 (1988),
845-861.
___ , S. Wen , and J. Tsou , Three-dimensional finite element analysis oj polymeric
fluid flow in an extrusion die. Part I: Entrance effect, Polymer Engineering Science ,
34 (1994), 827-834.
D. L. Logan , A First Course in the Finite Element Method, PWS, Boston, 1997.
B. C. Lychet and R. B. Bird, The Graetz-Nusselt problem [or a power-law non-newtonian
fluid , Chemical Engineering Science, 6 (1956), 35-41.
J. McMahon, Lower bounds [or the electrostatic capacity oj a cube, Proc. Royal Irish
Acad. , 55(A) (1953), 133-167.
R. J . Melosh, Basis [or the derivation oj matrices [or the direct stiffness method, AIAA
J ., 1 (1963) , 1631-1637.
S. G. Nash and A. Sofer, Linear and Nonlinear Programming, McGraw-Hill, 1996.
M. Petyt, Introduct ion to Finite Element Vibration Analysis, Cambridge University
Press, Cambridge, 1998.
E. Polak and G. Ribiere, Note sur la convergence de directions conjuqee», Rev. Francaise
Infomat Recherche Opertionelle, 3e Annee, 16 (1969), 35-43.
P. Ya. Polubarinova-Kochina, Theory oj Groundwater Movement, Translated from Rus-
sian by J . M. R. de Wiest , Princeton Univ . Press, Princton, NJ, 1962.
G. P6lya, Sur une interpretation de la methode des differences finies qui peut [ournir
des bornes superieures ou inJerieures, C . R. Acad. Sci. Paris, 235 (1952) , 995-997.
___ , Estimates [or Eigenvalues, in Studies in Mathematics and Mechanics Presented
to Richard von Mises, Academic Press, New York, 1954, pp . 200-207.
B. T. Polyak, The conjugate gradient method in extremem problems, USSR CompoMath.
and Math. Phys., 9 (1969), 94-112.
W . Prager and J . L. Synge , Approximations in elasticity based on the concept oJJunction
space, Quart. Appl. Math. , 5 (1947), 241-269 .
C. T . Reddy and D. J. Shippy, Alternative integration Jormulae [or triangular finite
elements, Intern. J . Numer. Methods in Engg ., 17 (1981),133-139.
J . N. Reddy , An Introduction to the Finite Element Method, McGraw-Hill, New York,
1984; 2nd ed ., McGraw-Hill, New York, 1993.
V. Sahai, Effects oj upwinding on the solution oj a I-D advection-diffusion problem,
Lawrence Livermore National Laboratory, Report UCRL-ID-109126 (December 1991),
1-15.
I. J. Schoenberg, Contributions to the problem oj approximation oj equidistant data by
analytic Junctions : Part A. On the problem oj smoothing or graduation . A first class
oj analytic approximation [ormulae , Quart. Appl. Math ., 4 (1948), 45-99.
L. J . Segerlind, Applied Finite Element Analysis, 2nd ed., Wiley , New York, 1984.
J . F. Shackelford , Introduction to Materials Science for Engineers , 5th ed., Prentice-Hall,
2000.
BIBLIOGRAPHY 437

V . P. Shih, C . C. Huang, and S.Y. Tsay, Extended leveque solution for laminar heat
transfer to power-Law fluids in pipes with wall slip, Int . J. Heat Mass Transfer, 38
(1995), 403-408 .
I. S. Sokolnikoff, Mathematical Theory of Elasticity, McGraw-Hill, New York , 1956.
L. J . Sonder and P. C. England, Vertical averages of rheology of the continental litho-
sphere" Earth Planet. Sci. Lett., 77 (1986) , 81-90.
E . M. Sparrow and R. D. Cess, Radiation Heat Transfer, Augmented Edition , McGraw-
Hill-Hemisphere, New York, 1978.
J . F . Steffe, Rheological Methods in Food Process Engineering, Freeman Press, San Fran-
cisco, CA , 1992.
K. Stephan, S. Lin, M. Durst, F. Huang, and D. Seher, A similarity relation for energy
separation in a vortex tube, Int . J . Heat Mass Transfer, 27 (1984), 911-920.
J. L. Synge, Triangulation in the hypercircle method for plane problems , Proc. Royal
Irish Acad., 54(A) (1952), 341-367.
___ , The Hypercircle in Mathematical Physics, Cambridge University Press, New
York, 1957.
B. Szabo and I. Babuska, Finite Element Analysis, Wiley , New York, 1991.
C. Taylor and T. G. Hughes, Finite Element Programming of the Navier-Stokes Equa-
tions, Pineridge Press Limited, Swansea, UK, 1981.
S. P. Timoshenko and N. Godier, Theory of Elasticity, McGraw-Hill, New York , 1951.
___ , D.H. Young, and W. Weaver, Jr., Vibration Problems in Engineering, Wiley,
New York, 1974.
P. Tong and J. N. Rossettos, Finite-Element Method, The MIT Press, Cambridge, MA,
1977.
D. Touati-Ahrned and C. Storey, Efficient hybrid conjugate gradient techniques, J . Op-
timization Theory Appl., 64 (1990), 379-397.
A. C. Ugural, Stresses in Plates and Shells, McGraw-Hill, New York, 1981.
J . V. Uspensky, Theory of Equations, McGraw-Hill, 1948.
K. Washizu, Variational Methods in Elasticity and Plasticity, 2nd ed., Pergamon Press,
New York, 1975.
W . Weaver, Jr. and P. R. Johnston, Finite Elements for Structural Analysis, Prentice-
Hall, Englewood Cliffs, NJ, 1984.
D. Wei, Penalty approximations to the stationary power-law Navier-Stokes problem,
Numer. Funet. Anal. Optim., 22 (2001) , 749-765.
___ , Existence and uniqueness of solutions to the stationary power-law Navier-Stokes
problem in bounded convex domains, Proceedings of Dynamic Systems and Applica-
tions III (2001) ,611-618.
___ and Z. Zhang, Decay estimates of heat transfer to molten polymer flow in pipes
with visous dissipation, Electronic Journal of Differential Equations. 2001 (2001) ,
1-14.
___ and L. Lefton, A penalty method for approximations of the stationary power-law
Stokes problem, Electron. J. Diff. Eqns., 2001 (2001), 1-12.
___ and H. .Luo, Finite element solutions of heat transfer in molten polymer flow in
tubes with viscous dissipation, International Journal of Heat and Mass Transfer 46
(2003), 3097-3108.
___ and L. Lefton, Penalty finite element solutions of the stationary power-law Stokes
problem, to appear in, J . of Numer. Math.
H. F . Weinberger, Upper and lower bounds for eigenvalues by finite difference methods,
Commun. Pure Appl. Math., 9 (1956) , 613-623.
___ , Lower bounds for higher eigenvalues by finite difference methods, Pacific J .
Math., 8 (1958), 339-368.
438 BIBLIOGRAPHY

G. N. White, Difference equations for plane thermal elasticity , LAMS-2745, Los Alamos
Scientific Laboratories, Los Alamos , NM, 1962.
J. R. Whiteman, A Bibliography for Finite Elements, Academic Press, New York, 1975.
W . Wilkinson, Non-Newtonian Fluids, Pergamon Press, New York, 1960.
J . C. Wu and J. F. Thompson, Numerical solutions of tim e dependent incompressible
Navier-Stokes equation using integra-differential formulation , Comput. Fluids, 1
(1973), 197-215 .
O. C. Zienkiewicz and Y. K. Cheung, Finite elements in the solution of field problems,
The Engineer, 220 (1965), 507-510.
___ and R. L. Taylor, The Finite Element Method, Vol. 1: Linear Problems , McGraw-
Hill, New York, 1989.
Index

A boundary condition s, 3, 6ff, 9, 13ff, 17ff,


acceleration due to gravity, 173 20,40,45,47, 55, 58,63, 66ff,
adiabatic isentropic pressure, 180 69,72,77, 80ff, 108, 110, 117
advection-diffusion analysis, 239ff 126,135 ,146,159,172,179,
air-like gas, 204 200, 202ff, 205, 207, 214, 229,
algorithm, 318ff 231,236,250,255,258,260,
angle of twist, 144 270, 281, 313,326, 34 1, 344,
angular momentum, 180 348
Ansys codes, 363ff convection, 169
programming, 225 Dirichlet, 3, 10, 15, 172
apple, 205ff essential, 5ff, 8, 55, 76ff, 159,270
approximation(s) homogeneous, 13
semidiscrete, 213, 228 homogeneous essential , 4, 13
semidiscrete Galerkin , 206 impermeable , 146
semidiscrete Rayleigh-Ritz, 208ff insulated, 164
temporal , 405ff mixed, 3, 223, 235
e-,406 natural, 3, 5ff, 24, 76ff, 136, 159,
aquifer, unconfined , 178ff 261,270,326
area coordinates, 97, 186 Neumann, 3
integrals, 186, 188 nonhomogeneou s, 13
of an element , 90, 98, 110 of the first kind, 3
of a triangle, 97 of the second kind, 3
aspect ratio, 285 of the third kind, 3
assembly of element matrices, 112 Robin, 3
axial elastic rod, 324 boundary integral(s), 111 ff, 138
axial vibrations of a plastic rod, 342ff boundary value problem, 9, 12,25,37,
axisymmetric heat transfer, 175ff 72,107,244
Poisson's, 20
B two-point, 39, 253, 268
backtracking, 315 nonlinear two-point, 264
beam, 78, 83, 85, 210 blood,320
cantilever, 294ff
Euler-Bernoulli, 33, 323ff, 332, 348 C
overlapping, 81 cable, 66
beam problem, 87ff can of mushroom soup, 189ff
bending moment, 76, 80ff, 82 Cauchy conditions , 3
boundary cavity, unit square, 307, 319ff
impermeable, 146 circular bar, 72
insulated, 140 cylinder, 169
440 INDEX

pipe, 158, 259 elastic constants , 270


cladding, 70ff plate, 282, 286
coefficient(s) rod, 47
consistency, 256, 320 solid plane, 231
convection, 166, 189, 266ff elasticity, plane, 269ff
convective heat transfer, 137,203 electrodes, 124, 126
film, 50, 64, 70ff, 135 electrostatic field, 124
of permeability, 146, 149, 178 potential , 126
thermal expansion, 54 element
computer codes, Ansys, 363ff area of an, 90, 98
Fortran, 374ff length, 98
Mathematica, 351ff volume, 98
Matlab, 350, 368ff element s
concrete slab, 169 bilinear rectangular, 9lff, 93, 107ff,
coolant, 70 111, 126, 131,134, 170, I82ff,
coordinate(s) 187ff, 195, 203, 205, 225, 284,
area, 97 288, 350, 398ff, 402
curvilinear, 415ff constant-strain triangular, 270ff
cylindrical polar, 347 CST, 270ff, 280, 276, 294, 296,
global, 147 cubic, 66
local , 147 elastic , 272
oblique, 96 isoparametric, 409ff
of nodes , 160ff linear for heat transfer in fluids,
parametric, 31 194ff
polar, 23, 175, 202, 347 linear triangular, 89ff, 91, 101ff,
rectangular Cartesian, 97 107ff, 119, 125, 133, 144, 153,
triangular, 96ff, 197 159ff, 170, 179, 18lff, 184ff,
Cramer's rule, 29 195,199,203 ,215,233,275,
cubic element, 66 308, 336, 397ff, 399, 402
current, 126 one-dimensional, 37ff
cylinder penalty bilinear rectangular Stokes,
elliptic, 152 309ff,320
solid circular, 169 penalty linear triangle power-law
cylindrical sleeve, 67 Stokes, 31Iff
penalty linear triangular Stokes,
D 30 Iff
dc current , 126ff quadratic, 35, 45ff, 49
deflection of a beam, 81, 83, 85 quadrilateral, 45ff, 108
transverse, 66 two-dimensional, 89ff
degree(s) of freedom, 94, 269, 284, 289 energy
density, 181, 231 elastic potential, 325ff, 327
discretization, 107 kinetic, 325ff, 327
displacement, 87ff potential, 2, 342
longitudinal, 254 total, 325ff, 328, 336, 342
vector, 335 equality constrained programs, 297ff
divergence theorem, 293, 394 equation(s)
Bessel's 13
E deformation of a bar, 18
eigenfunction(s), 347ff finite element, 50, 54, 59, 77, 79,
eigenvalue(s), 11, 282,328, 347ff 109,146,176, 202ff, 218ff,
eigenvector(s), 328 228,232
INDEX 441

fourth-order 14, 37 laminar ideal, 37


fourth -order (beam), 31, 37, 86 laminar power-law, 195
Eule r-Bernoulli, 37, 75ff Newtonian Stokes , 319
Euler-Lagrange, 2ff, 6ff, 24, 58, 74, non-Newtonian, 267
136, 261, 300 , 311 of ideal fluids, 152
generalized second-order, 107 power-law non-Newtonian, 319
heat, 194 power-law pressure driven, 256ff
Helmholtz, 347 partially filled pipe , 158ff
hyperbolic, 344ff shear thinning, 320
Laplace, 124, 131ff, 133, 135, 146, turbulent, 260ff
152 viscous on a moving plate , 238
linear interpolation, 245 fluids
linear Stokes, 299ff food , 256
mixing-length, 259ff non-Newtonian, 196,267,316
nonlinear elliptic, 197 power-law, 256ff, 267ff, 319
Navier-Stokes, 8 viscous incompressible, 229
normalized, 173 flux , 49, 52
ordinary differential, 344 food products, 257
parabolic, 346 form
partial differential, 7 basis , 9, 12
Poisson 's, 4, IOff, 15, 22, 107ff, bilinear,5ff, 109, 177,202
116, 129ff, 131, 144,167,172, linear, 5ff, 109, 177, 202
205 quasi-weak, 207ff
polynomial, 346 variation al, 76ff
power-law Stokes, 311 weak ,3, 17ff,49, 53,58,66,75
Rayleigh-Ritz, 16 weak variational, 1, 3ff, 49, 108ff,
second-order (bar), 24, 37, 175 177
Stokes, 297ff formula
wave, 344 ff Dai-Yuan , 264
error, 9, 39 Hestenes-Stiefel, 264
approximation, 107 integration by parts , 7, 38, 183,244,
discretization, 107, 116 393ff
roundoff, 200, Rodrigues' , 423
vector, 38, 242 formulation
Euler-Bernoulli beam theory, 75ff penalty, 299ff
experimental data, 182 local weak , 108ff
semidiscrete, 208, 212
F semidiscrete weak, 229
fuel element, 70ff weak, 3, 17ff, 86
fin weak variational , I , 3ff, 49 , 108ff,
circular pipe, 222 177
cooling, 243, 248,250 Fortran codes, 374ff
parabolic, 71 Fourier's law, 182
rectangular cooling, 68 Fourier sine series , 237
tapered, 56ff forward time -marching process, 406
two-dimensional circular, 164 free axial vibrations of an elastic rod,
flexural rigidity , 83 327ff
flow discharge , 162ff free in-plane vibrations of an elastic
fluid flows , 152ff plate , 334ff, 348
Couette, 256 free vibrations of a Euler elastic beam,
laminar, 259 330ff
442 INDEX

free vibration mode, 328 linear, 2


frequency,328,348 quadratic, 17
Froude number, 173 furnace, 170
function(s)
admissible, 3, 300 G
basis, 22 Galerkin finite element method, 37ff
Bessel,I70 Gauss integration scheme, 198
bilinear " 91 points, 198, 423
bilinear interpolation, 309 Gaussian quadrature, 181,202, 423ff
characteristic, 27 weight, 198
cubic shape, 36 gradient local element , 304
deflection, 76 vector, 144ff
Dirac delta, 83 gradient-based methods, 429ff
discrete energy, 346 gravitational force, 44, 46
first-order continuity, 412ff Green's identity, 1, 173
gamma, 393 identities , 421ff
global, 94 grid of heating cables, 169
global cubic shape, 28ff groundwater seepage, 146
global piecewise linear shape, 36
global shape, 94,96, 106,262,313 H
Heaviside unit, 87 half-beam, 216
Hermite cubic, 31ff half-cylinder, 170
Hermite interpolation, 76 Hamiltonian principle, 323ft, 325ff,
Hermite shape, 31ff, 37ff, 330 347ff
interpolation, 76, 89, 91, 350 head, constant, 146
Lagrange, 297ff, 410 heat conduction, 18ff, 51, 172, 182
Lagrange shape, 25, 34, 37, 270 flux, 50, 68
linear, 412 time-dependent, 217ff
local cubic shape, 30 unsteady, 234ff
local linear interpolation shape, 26, heat diffusion , 239ff
local quadratic shape, 29, 34 heat transfer, 137ff
local shape, 26ff, 92 axisymmetric, 175ff
linear global shape, 28ff convection, 239ff
linear interpolation, 26 generation, 137, 142, 166
linear shape, 178, 30 I nonlinear, 196ff
linear triangular shape, 312 steady-state, 137ff
nonlinear, 250 supply, 142
one-dimensional shape, 25ff heat treated 2024 aluminum alloy, 253
quadratic, 59,413 heating cables, 169
quadratic shape, 29ff Hooke's law, generalized, 252ff
shape, 28ff, 36, 95, 98, 101, 105, human blood, 258
109,244 hydraulic conductivity, 149
stream, 152, 157
stress, 134, 144, 167ff I
tent-shaped, 27, 38 industrial oven, 267ff
test, 9, 14,49,53,76,108, 138, initial condition(s), 211ff, 214, 234, 238
212,215,228,231,273,344 initial guess , 241
weight, 423ff interface continuity, 114
functional(s), 2, 7, 12ff, 17,58,74, 324ff integrals
bilinear, 3 one-dimensional, 416
energy, 74, 324ff two-dimensional, 412ff
I NDE X 443

integration tridiagonal, 40
by parts, 7, 38, 183,244, 393ff membrane, elliptical , 168
formulas , 393ff rectangular, 134
Gauss-Legendre, 198ff methodes)
numerical time, 22lff classical, 207ff
inter-element quantities , 247 conjugate gradient, 58ff, 265ff,
internal energy generation, 72 313ff, 318ff, 43 Iff
interpolation, 273 Galerkin, 1, 3, 9ff, 10, 17,21 , 37ff,
bilinear, 93 72, 208ff, 211, 233, 241, 244
finite element, 220 Galerkin finite element , 37ff
linear, 245 Gauss elimination, 156
Hermite , 330 Gauss -Legendre integration, 198ff
semidiscrete , 230 gradient-based, 429ff
iteration(s), 60ff, 198ff, 241, 264 Laplace transform , 208ff
McCauly's, 81
J Newton's, 197ff, 24Iff, 248ff, 250,
Jacobian matrix, 197ff, 416ff, 418 252
nonlinear conjugate gradient, 26Iff,
K 318ff
Kronecker delta, 91 numerical time integration, 221ff
of separation of variables, 226, 236
L of steepest descent, 59, 263ff, 266,
Lagrange multipliers , 297ff 297,429ff
Laplacian, 398 penalty, 297ff, 309
length of an element, 28 Rayleigh-Ritz , 1,3, II , 13, 15, 17,
linear triangular elements , 89ff, 91, 21,241,26lff
lOIff, 107ff, 119, 125, 133, a-scheme, 235ff
144,153, 159ff, 170, 179, trapezoidal, 233
18lff, 184ff, 195, 199,203, weighted residual, 1, 3,207
215,233,275 , 308, 336, 397ff, minimizers, 298
399,402 modulus of elasticity, 54, 75
load (force) vector, 39, 50ff, 57, 66, 77ff, moments , 87ff
79, 83ff, 86, 110ff, 117ff, 125, mushroom can, 235
144, 178,275
low carbon steel (annealed), 253 N
Newtonian model, 256
M Newton's law of cooling, 137
magnetic field, 238 nodal value, 28
matrix boundary, 308, 313
coefficient , 39, 270 node(s)
connectivity, 79, 113ff, 119, 117, global, 25, 48, 78, 89, 129ff, 169,
119,125,140,147,154,189, 277, 296, 343
308,313,338 local, 26, 94, 129ff, 277
global stiffness, 39, 42, 50, 68, 83ff, no-slip wall condition, 259
85, 117ff, 148, 153ff, 288, 291, nuclear reactor, 70
333,340 numerical time integration, 22lff
Jacobian, 197ff, 416ff, 418
local stiffness, 4Iff, 51, 57, 66, 77, o
85ff, 110ff, 134, 140, 144, 147, open square cavity, 307
153, 16 ~ 178,275,350 operator, II
strain, 273 differential, 272
444 INDEX

p radially symmetric , 175ff


parametric coordinates, 31 second-order linear, 45ff
Pascal triangle, 418ff transient, 207ff
penalty formulati on, 299ff, 311ff two-dimensional, 107ff, 137ff
penalty parameter, 309 two-dimensional transient, 228ff
permeability coefficient, 178 program
piezometric head, 178 equalit y-constrained, 297ff
piston flow, 194 linear, 299
plane elasticity, 269ff nonlinear, 2563, 313
plastic rod, 252 Matiab , 319ff
polar coordinates, 23, 175, 202, 347 Mathematica, 320
moment , 73 unconstrained, 263ff, 313ff, 430
polyethelene melt, 199 pumping , 178ff
polynomial
Legendre, 423 Q
potential lines, 150ff quadrant , 180
power-law fluids, 256, 267, 319 quadrilateral elements, 109
Prandtl theory of torsion , 144, 168 quadrilaterals, 107
pressure , 8, 72, 173, 229, 299ff, 311
gradient, 180, 257 R
principle of virtual work, 325ff radial distance, 180
prismatic bar, 73 radian frequency of vibrations , 73
rod,37, 44, 46,343 radiation, 250
problem heat transfer, 243ff
axisymmetric, 183,203,293 recharge , 178
cavity Stokes, 307ff, 314ff, 319ff rectangle, 139
Dirichlet , I rectangular bar, 165, 167
eigen value, 73, 344ff, 347 residual, 9,17, 211
equality-constrained , 297ff Reynolds number, 173, 259ff
heat conduction, 239 river, 146
heat transfer, 202, 346, 398 Rodrigue s' formula, 423
homogeneous, 328
membrane, 345ff S
minimizat ion, 261 scheme
Neumann, I backward difference, 349, 405
Newtonian linear Stokes, 313ff conjugate gradient, 58, 265ff, 313ff,
nonlinear heat conduction, 71 318ff, 43lff
optimization, 297 Crank-Nicols on, 209, 405
plane strain , 265ff Euler, 344
plane stress , 269ff finite difference, 221
potential flow, 146 forward difference, 405
power-law Stokes, 311ff, 318 Galerkin , 209, 222, 405
Stokes, 309 Gauss integration, 198
Sturm -Liouville , 77 Newmark, 211, 217, 407
two-point boundary value, 39, 253, Runge-Kutta , 344
268 0-, 208ff, 235
unsteady heat conduction, 235ff search,319
problems seepage, 146ff
eigenvalue, 222, 328, 347ff shear force, 76ff, 80
nonlinear one-dimensional , 241ff modulus , 73, 144
one-dimensional, 37ff stress, 144
I NDEX 445

silica brick , 267 tractions surface, 28 1


soup , mushroom, 189ff transient problem s, 207ff
specific heat, 180 trapezoidal rule, 221
square cavity, 307, 319ff triangle , 99ff
plate, 279, 337 triangular regions, 426
region , 131 twisting of a square bar, 144
unit, 134
stainless steel bar, 253 U
rod, 44, 46 unit square, 120
stationary point, 298ff square cavity, 307, 319ff
sterlizing process, 189 upper echelon form, 156
strain , 76, 252
strain-displacement relations, 269ff V
stress analysis, 252ff variables
stress -strain relations, 229ff, 325 primary, 6, 78, 116
support conditions of a beam, 78 secondary, 8, 67, 78, 149
system two dependent, 63ff
finite element, 246 vector
Galerkin finite element , 197 force (load), 39, 50ff, 57, 66, 77ff ,
global , 182,251,260 79, 83ff, 86, IlOff, 117ff, 125,
nonlinear, 197 144,178, 275
oblique, 96 local load, 4 1
rectangular Cartesian, 97 velocity
mean flow, 196
T potential, I49ff, 152, 157
temperature vectors, 150, 299
ambient, 66, 137, 164,243 vibration analysis, 323ff
change, 54 of elastic rods, 329ff, 348
distribution, 56, 64, 67,1 38,1 64, of a membr ane, 343
170, 200ff, 218, 222, 225, 227, virtual displacement, 272, 274
239, 244, 400 work , 274, 325ff
gradient, 164, 167,202 viscosity, 299
temporal approximations, 405ff von Karman constant, 259
theorem, vortex core flow, 179
divergence, 293, 384 vortex-flow temper ature separation , 179
gradient, 394 vortex tubes , 179
thermal conductivity, 19,51,63 , 70ff,
142, 165ff, 167, 18lff, 189, W
203,244f weak form , 3, 17f[, 137,273
thin-film lubrication, 72 weak formul ation , 86
time-dependent heat conduction, 217ff weak variational form , I, 3ff, 49, 108ff,
time integration, numerical, 22lff 177
time- step, 215 weight function , 423
tolerance, 241 weight s, 77, 405 , 423ff
tomato paste , 266 wiggle effect , 240
torsion , 144ff work, 324ff, 328
of a hollow square membrane, 167
Prandtl theory, 144 y
torsional vibrations, 73 Young ' s modulus, 47, 63, 279, 324, 333

You might also like